You are on page 1of 278

Edexcel Maths Core 4

Mark Scheme Pack

2006-2013

PhysicsAndMathsTutor.com
PhysicsAndMathsTutor.com

GCSE
Edexcel GCE
Core Mathematics C4 (6666)

Summer 2005

Mark Scheme (Results)


Core Mathematics C4 (6666)
Edexcel GCE
PhysicsAndMathsTutor.com

Final Version
June 2005
6666 Core C4
Mark Scheme
Question
Scheme Marks
Number

⎛ 9x ⎞2
( 4 − 9x)
1
1. 2
= 2 ⎜1 − ⎟ B1
⎝ 4 ⎠
⎛ 12 ⎛ 9 x ⎞ 12 ( − 12 ) ⎛ 9 x ⎞ 2 1
( − 12 ) ( − 32 ) ⎛ 9x ⎞
3

= 2 ⎜1 + ⎜ − ⎟ + ⎜− ⎟ +
2
⎜ − ⎟ + ... ⎟⎟ M1
⎜ 1⎝ 4 ⎠ 1.2 ⎝ 4 ⎠ 1.2.3 ⎝ 4 ⎠
⎝ ⎠
⎛ 9 81 2 729 3 ⎞
= 2 ⎜1 − x − x − x + ... ⎟
⎝ 8 128 1024 ⎠
9 81 729 3
= 2 − x, − x 2 , − x + ... A1, A1, A1
4 64 512
[5]

1
( − 12 ) 1
( − 12 ) ( − 23 )
( ... ) ( ... )
2 2 2 3
Note The M1 is gained for or
1.2 1.2.3

Special Case
⎛ 9 81 2 729 3 ⎞
If the candidate reaches = 2 ⎜1 − x − x − x + ... ⎟ and goes no further
⎝ 8 128 1024 ⎠
allow A1 A0 A0

6666 Core C4
June 2005 Advanced Subsidiary/Advanced Level in GCE Mathematics
PhysicsAndMathsTutor.com

Question
Scheme Marks
Number

⎛ dy ⎞ dy
2. 2x + ⎜ 2x + 2 y ⎟ − 6 y =0 M1 (A1) A1
⎝ dx ⎠ dx
dy
=0 ⇒ x+ y =0 or equivalent M1
dx

Eliminating either variable and solving for at least one value of x or y. M1


y 2 − 2 y 2 − 3 y 2 + 16 = 0 or the same equation in x
y = ±2 or x = ± 2 A1
( 2, − 2 ) , ( −2, 2 ) A1
[7]
dy x + y
Note: =
dx 3 y − x

Alternative
3 y 2 − 2 xy − ( x 2 + 16 ) = 0
2 x ± √ (16 x 2 + 192 )
y=
6
dy 1 1 8x
= ± .
dx 3 3 √ (16 x 2 + 192 ) M1 A1± A1

dy 8x
=0 ⇒ =±1
√ (16 x + 192 )
dx 2 M1

64 x 2 = 16 x 2 + 192
x=± 2 M1 A1
( 2, − 2 ) , ( −2, 2 ) A1
[7]

6666 Core C4
June 2005 Advanced Subsidiary/Advanced Level in GCE Mathematics
PhysicsAndMathsTutor.com

Question
Scheme Marks
Number

5x + 3 A B
(a) = +
3.
( 2 x − 3)( x + 2 ) 2 x − 3 x + 2

5 x + 3 = A ( x + 2 ) + B ( 2 x − 3)

Substituting x = −2 or x = 32 and obtaining A or B; or equating coefficients and


M1
solving a pair of simultaneous equations to obtain A or B.

A = 3, B = 1 A1, A1
(3)
If the cover-up rule is used, give M1 A1 for the first of A or B found, A1 for the
second.


5x + 3 3
(b) dx = ln ( 2 x − 3) + ln ( x + 2 ) M1 A1ft
( 2 x − 3)( x + 2 ) 2

⎡ ... ⎤ = 3 ln 9 + ln 2
6
M1 A1
⎣ ⎦2 2
= ln 54 cao A1 (5)
[8]

6666 Core C4
June 2005 Advanced Subsidiary/Advanced Level in GCE Mathematics
PhysicsAndMathsTutor.com

Question
Scheme Marks
Number

∫( ∫(
1 1 dx
dx = cos θ dθ Use of x = sin θ and = cos θ
1 − x2 ) 1 − sin 2 θ )
4. 1
2
3
2 dθ M1


1
= dθ M1 A1
cos 2 θ
= ∫ sec2 θ dθ = tan θ M1 A1

π
Using the limits 0 and 6 to evaluate integral M1
1 ⎛ √3⎞
[ tan θ ] 0
π
6
= ⎜= ⎟ cao A1
√3 ⎝ 3 ⎠
[7]

Alternative for final M1 A1

1
Returning to the variable x and using the limits 0 and 2 to evaluate integral M1

⎡ x ⎤2 1 ⎛ √3⎞
⎢ ⎥ = ⎜= ⎟ cao A1
⎢⎣ √ (1 − x ) ⎥⎦ 0 √ 3 ⎝ 3 ⎠
2

6666 Core C4
June 2005 Advanced Subsidiary/Advanced Level in GCE Mathematics
PhysicsAndMathsTutor.com

Question
Scheme Marks
Number

∫ xe ∫
1 1
dx = x e −
2x 2x
5. (a) e 2 x dx Attempting parts in the right direction M1 A1
2 2
1 1
= x e2 x − e2 x A1
2 4

1
⎡ 1 2x 1 2x ⎤ 1 1 2
⎢⎣ 2 x e − 4 e ⎥⎦ = 4 + 4 e M1 A1
0

(5)
(b) x = 0.4 ⇒ y ≈ 0.890 22
x = 0.8 ⇒ y ≈ 3.962 43 Both are required to 5 d.p B1
. (1)

I ≈ × 0.2 × [ ... ]
1
(c) B1
2
≈ ... × ⎡⎣ 0+7.389 06+2 ( 0.29836+.890 22+1.992 07+3.962 43) ⎤⎦ M1 A1ft
ft their answers to (b)
≈ 0.1× 21.675 22
≈ 2.168 cao A1 (4)
[10]
1 1 2
Note + e ≈ 2.097 …
4 4

6666 Core C4
June 2005 Advanced Subsidiary/Advanced Level in GCE Mathematics
PhysicsAndMathsTutor.com

Question
Scheme Marks
Number

dx dy
6. (a) = −2 cosec 2 t , = 4sin t cos t both M1 A1
dt dt
d y −2sin t cos t
dx
=
cosec 2 t
( = −2sin 3 t cos t ) M1 A1
(4)
(b) At t = π4 , x = 2, y = 1 both x and y B1
dy ⎛ 1⎞
Substitutes t = π4 into an attempt at to obtain gradient ⎜ − ⎟ M1
dx ⎝ 2⎠
Equation of tangent is y − 1 = −
1
2
( x − 2) M1 A1
Accept x + 2 y = 4 or any correct equivalent (4)

(c) Uses 1 + cot 2 t = cosec2 t , or equivalent, to eliminate t M1


2
⎛ x⎞ 2
1+ ⎜ ⎟ = correctly eliminates t A1
⎝2⎠ y
8
y= cao A1
4 + x2

The domain is x …0 B1 (4)


[12]

An alternative in (c)
1 1

⎛ y ⎞2 x x ⎛ y ⎞2
sin t = ⎜ ⎟ ; cos t = sin t = ⎜ ⎟
⎝2⎠ 2 2⎝ 2⎠
y x2 y
sin 2 t + cos 2 t = 1 ⇒ + × =1 M1 A1
2 4 2
8
Leading to y = A1
4 + x2

6666 Core C4
June 2005 Advanced Subsidiary/Advanced Level in GCE Mathematics
PhysicsAndMathsTutor.com

Question
Scheme Marks
Number

7. (a) k component 2 + 4λ = −2 ⇒ λ = −1 M1 A1
Note µ = 2
Substituting their λ (or µ ) into equation of line and obtaining B M1

B: ( 2, 2, − 2 ) Accept vector forms A1


(4)
⎛1⎞ ⎛1⎞
⎜ ⎟ ⎜ ⎟
(b) ⎜ −1⎟ = √ 18; ⎜ −1⎟ = √ 2 both B1
⎜4⎟ ⎜0⎟
⎝ ⎠ ⎝ ⎠

⎛1⎞ ⎛1⎞
⎜ ⎟ ⎜ ⎟
⎜ −1⎟ ⋅ ⎜ −1⎟ = 1 + 1 + 0 ( = 2 ) B1
⎜4⎟ ⎜0⎟
⎝ ⎠ ⎝ ⎠
2 1
cos θ = = cao M1 A1
√ 18 √ 2 3
(4)
uuur uuur 2 uuur
(c) AB = −i + j − 4k ⇒ AB = 18 or AB = √ 18 ignore direction of vector M1
uuur uuur 2 uuur
BC = 3i − 3 j ⇒ BC = 18 or BC = √ 18 ignore direction of vector M1
uuur uuur
Hence AB = BC ¿ A1 (3)

uuur
(d) OD = 6i − 2 j + 2k Allow first B1 for any two correct B1 B1
Accept column form or coordinates (2)
[13]

6666 Core C4
June 2005 Advanced Subsidiary/Advanced Level in GCE Mathematics
PhysicsAndMathsTutor.com

Question
Scheme Marks
Number

dV
8. (a) is the rate of increase of volume (with respect to time) B1
dt
− kV : k is constant of proportionality and the negative shows decrease (or loss)
dV B1
giving = 20 − kV ¿ These Bs are to be awarded independently
dt
(2)


1
(b) dV = ∫ 1dt separating variables M1
20 − kV
1
− ln ( 20 − kV ) = t ( +C ) M1 A1
k
Using V = 0, t = 0 to evaluate the constant of integration M1
1
c = − ln 20
k
1 ⎛ 20 ⎞
t = ln ⎜ ⎟
k ⎝ 20 − kV ⎠
Obtaining answer in the form V = A + B e − kt M1
V=
20 20 − kt
k
− e
k
Accept
20
k
(1 − e− kt ) A1 (6)

dV
(c) = 20 e − kt Can be implied M1
dt
dV 1
= 10, t = 5 ⇒ 10 = 20 e− kt ⇒ k = ln 2 ≈ 0.139 M1 A1
dt 5
75
At t = 10, V = awrt 108 M1 A1 (5)
ln 2
[13]
Alternative to (b)

dV
Using printed answer and differentiating = − kB e − kt M1
dt
Substituting into differential equation
− kB e − kt = 20 − kA − kB e − kt M1
20
A= M1 A1
k
Using V = 0, t = 0 in printed answer to obtain A + B = 0 M1
20
B=− A1 (6)
k

6666 Core C4
June 2005 Advanced Subsidiary/Advanced Level in GCE Mathematics
PhysicsAndMathsTutor.com

GCE
Edexcel GCE
Core Mathematics C4 (6666)

January 2006

Mark Scheme (Results)


Core Mathematics C4 (6666)
Edexcel GCE
PhysicsAndMathsTutor.com

January 2006
6666 Core Mathematics C4
Mark Scheme

Question Scheme Marks


Number

1. Differentiates M1

A1,
6 x + 8 y dy
dx − 2,
to obtain :
....................... + (6 x dy
dx + 6 y ) = 0
+(B1)
⎡ dy 2 − 6 x − 6 y ⎤
⎢ = ⎥
⎣ dx 6x + 8 y ⎦

Substitutes x = 1, y = – 2 into expression involving dy


dx , to give dy
dx = − 108 M1, A1

Uses line equation with numerical ‘gradient’ y – (– 2) = (their gradient)(x – 1) M1


or finds c and uses y = (their gradient ) x + " c"

To give 5 y + 4 x + 6 = 0 (or equivalent = 0) A1√ [7]

2. π π 3π π
(a) x 0
16 8 16 4

1 1.01959 1.08239 1.20269 1.41421 M1 A1


y
(2)
M1 for one correct, A1 for all correct

1 π
× × {1 + 1.4142 + 2(1.01959 + ... + 1.20269)}
(b) M1 A1√
Integral =
2 16
⎛ π ⎞
⎜= x 9.02355 ⎟ = 0.8859 A1 cao
⎝ 32 ⎠ (3)

(c) approx − 0.88137


Percentage error = ×100 = 0.51 % (allow 0.5% to 0.54% for A1) M1 A1 (2)
0.88137
[7]
approx − ln (1 + 2 )
M1 gained for (± )
ln (1 + 2 )
PhysicsAndMathsTutor.com

Question Scheme Marks


Number

⎡ u 2 + 1⎤ M1
3. Uses substitution to obtain x = f(u) ⎢ ⎥ ,
⎣ 2 ⎦
du
and to obtain u = const. or equiv. M1
dx

3(u 2 + 1) A1
Reaches ∫ 2u udu or equivalent
⎛ 3⎞ M1
∫ ⎜⎝ 3u + ⎟ du or equiv.
2
Simplifies integrand to
2⎠
M1 A1√
Integrates to 1
2 u 3 + 23 u

A1√ dependent on all previous Ms

Uses new limits 3 and 1 substituting and subtracting (or returning to function of x
with old limits)
M1

To give 16 cso A1
[8]

“By Parts”
Attempt at “ right direction” by parts M1
⎛ 1
⎞ 1
[ 3 x ⎜ 2 x − 1) ⎟
⎟ – { ∫ 3 (2 x − 1) dx } ] M1{M1A1}
2 2

⎝ ⎠
(2 x − 1)2
3
……………. – M1A1√

Uses limits 5 and 1 correctly; [42 – 26] 16 M1A1


PhysicsAndMathsTutor.com

4.

Attempts V = π x 2 e 2 x dx M1

⎡ x 2e2 x ⎤
=π ⎢ − ∫ xe 2 x dx ⎥ (M1 needs parts in the correct direction) M1 A1
⎣ 2 ⎦

x 2 e 2 x ⎛ xe 2 x e 2x ⎞
= π[ ⎜
−⎜ −∫ dx ⎟⎟ ] (M1 needs second application of parts) M1 A1√
2 ⎝ 2 2 ⎠

∫ xe
2x
M1A1√ refers to candidates dx , but dependent on prev. M1

2
e 2 x ⎛ xe 2 x e 2 x ⎞
= π [x − ⎜⎜ − ⎟]
A1 cao
2 ⎝ 2 4 ⎟⎠

Substitutes limits 3 and 1 and subtracts to give… [dep. on second and third Ms] dM1

= π ⎡⎣ 134 e6 − 14 e 2 ⎤⎦ or any correct exact equivalent.


A1
[8]
[Omission of π loses first and last marks only]
PhysicsAndMathsTutor.com

Question Scheme Marks


Number

5. (a) Considers 3x 2 + 16 = A(2 + x) 2 + B (1 − 3 x)(2 + x) + C (1 − 3 x)

and substitutes x = –2 , or x = 1/3 , M1

or compares coefficients and solves simultaneous equations

A1, A1
To obtain A = 3, and C = 4

Compares coefficients or uses simultaneous equation to show B = 0. B1


(4)

M1
(b) Writes 3(1 − 3 x) −1 + 4(2 + x) −2
(M1, A1)
= 3(1 + 3 x, +9 x + 27 x + ......) +
2 3

(−2) ⎛ x ⎞ (−2)(−3) ⎛ x ⎞ (−2)(−3)(−4) ⎛ x ⎞


2 3
4
(1 + ⎜ ⎟+ ⎜ ⎟ + ⎜ ⎟ +….) ( M1 A1 )
4 1 ⎝2⎠ 1.2 ⎝ 2 ⎠ 1.2.3 ⎝ 2⎠

= 4 + 8 x, + 27 34 x 2 + 80 12 x 3 + ... A1, A1
(7)

Or uses (3 x 2 + 16)(1 − 3 x) −1 (2 + x) −2 M1

(3x 2 + 16) (1 + 3x, +9 x 2 + 27 x3 +) ×


(M1A1)×

(−2) ⎛ x ⎞ (−2)(−3) ⎛ x ⎞ (−2)(−3)(−4) ⎛ x ⎞


2 3

¼ (1 + ⎜ ⎟+ ⎜ ⎟ + ⎜ ⎟ ) (M1A1)
1 ⎝ 2⎠ 1.2 ⎝ 2 ⎠ 1.2.3 ⎝ 2⎠

= 4 + 8 x, + 27 34 x 2 + 80 12 x 3 + ... A1, A1
(7)

[11]
PhysicsAndMathsTutor.com

6. (a) λ = −4 → a = 18, µ =1→ b = 9 M1 A1, A1


(3)

⎛8 + λ ⎞ ⎛ 1⎞
⎜ ⎟ ⎜ ⎟
(b) ⎜12 + λ ⎟ • ⎜ 1 ⎟ =0 M1
⎜14 − λ ⎟ ⎜ −1⎟
⎝ ⎠ ⎝ ⎠

∴ 8 + λ + 12 + λ − 14 + λ = 0 A1

Solves to obtain λ ( λ = −2 ) dM1

Then substitutes value for λ to give P at the point (6, 10, 16) (any form) M1, A1
(5)

(c) OP = 36 + 100 + 256 M1

(= 392 ) = 14 2 A1 cao
(2)
[10]

dV
7. (a) = 4π r 2 B1
dr (1)

dr dV dr 1000 M1,A1
(b) Uses = . in any form, =
dt dt dV 4π r 2 (2t + 1) 2 (2)

(c) −2
V = ∫ 1000(2t + 1) dt and integrate to p (2 t + 1) − 1 , = −500(2t + 1) −1 (+ c) M1, A1

Using V=0 when t=0 to find c , (c = 500 , or equivalent) M1

1
∴V = 500(1 − ) (any form) A1
2t + 1 (4)
(d) (i) Substitute t = 5 to give V, M1,
⎛ 3V ⎞
then use r = 3 ⎜ ⎟ to give r , = 4.77 M1, A1
⎝ 4π ⎠ (3)

(ii) Substitutes t = 5 and r = ‘their value’ into ‘their’ part (b) M1

dr
= 0.0289 (≈ 2.90 x10 − 2 ) ( cm/s) ∗ AG A1
dt (2)
[12]
PhysicsAndMathsTutor.com

π 5π
8. (a) Solves y = 0 ⇒ cos t = 1
2 to obtain t = or (need both for A1) M1 A1
3 3
(2)
Or substitutes both values of t and shows that y = 0

dx
(b) = 1 − 2 cos t M1 A1
dt
5π 5π
3 3

∫ ydx = ∫ (1 − 2 cos t )(1 − 2 cos t )dt ∫ (1 − 2 cos t ) dt ∗


2
Area= = AG B1
π
3
π
3
(3)

(c) M1
Area ∫
= 1 − 4 cos t + 4 cos 2 tdt 3 terms


= 1 − 4 cos t + 2(cos 2t + 1)dt (use of correct double angle formula) M1

= ∫ 3 − 4 cos t + 2 cos 2tdt


= [3t − 4sin t + sin 2t ] M1 A1

5π π
Substitutes the two correct limits t = and and subtracts. M1
3 3

= 4π + 3 3 A1A1
(7)

[12]
PhysicsAndMathsTutor.com

GCE
Edexcel GCE
Core Mathematics C4 (6666)

June 2006

Mark Scheme
(Final)
Core Mathematics C4 (6666)
Edexcel GCE
PhysicsAndMathsTutor.com

June 2006
6666 Pure Mathematics C4
Mark Scheme

Question
Scheme Marks
Number

Differentiates implicitly to include either


⎧ dy ⎫
dy dy ⎛ dy ⎞ M1
1. dy dy ±ky or ±3 . (Ignore ⎜ = ⎟ .)
⎨ =⎬ 6x − 4y +2−3 =0 dx dx ⎝ dx ⎠
⎩ dx ⎭ dx dx A1
Correct equation.

⎧ dy 6x + 2 ⎫
⎨ = ⎬ not necessarily required.
⎩ dx 4y + 3 ⎭

Substituting x = 0 & y = 1 into an equation


dy 0 + 2 2 involving dy
At (0, 1), = = dx
; dM1;
dx 4 + 3 7
to give 72 or −−72 A1 cso

7 −1 Uses m(T) to ‘correctly’ find m(N). Can be


Hence m(N) = − or 2 A1 oe.
2 7
ft from “their tangent gradient”.

Either N: y − 1 = − 72 (x − 0) y − 1 = m(x − 0) with


‘their tangent or normal gradient’;
M1;
or uses y = mx + 1 with ‘their tangent or
or N: y = − x + 1
7
2
normal gradient’ ;

Correct equation in the form


N: 7x + 2y – 2 = 0 ' ax + by + c = 0 ' , A1 oe cso
where a, b and c are integers.
[7]

7 marks

dy 2
Beware: = does not necessarily imply the award of all the first four marks in this question.
dx 7
So please ensure that you check candidates’ initial differentiation before awarding the first A1 mark.

Beware: The final accuracy mark is for completely correct solutions. If a candidate flukes the final line then
they must be awarded A0.

Beware: A candidate finding an m(T) = 0 can obtain A1ft for m(N) = ∞ , but obtains M0 if they write
y − 1 = ∞(x − 0) . If they write, however, N: x = 0, then can score M1.

Beware: A candidate finding an m(T) = ∞ can obtain A1ft for m(N) = 0, and also obtains M1 if they write
y − 1 = 0(x − 0) or y = 1.

Beware: The final cso refers to the whole question.


6666/01 Core Maths C4 2
June 2006 Advanced Subsidiary/Advanced Level in GCE Mathematics
PhysicsAndMathsTutor.com

Question
Scheme Marks
Number
Aliter
Differentiates implicitly to include either
⎧⎪ d x ⎫⎪ dx dx ⎛ dx ⎞ M1
1. dx dx ±kx or ±2 . (Ignore ⎜ = ⎟ .)
⎨ =⎬ 6x − 4y + 2 −3=0
dy dy
dy dy ⎝ dy ⎠ A1
⎩⎪ d y ⎪⎭ Correct equation.
Way 2
⎧ dx 4y + 3 ⎫
⎨ = ⎬ not necessarily required.
⎩ dy 6x + 2 ⎭

Substituting x = 0 & y = 1 into an


dx 4 + 3 7
At (0, 1), = = equation involving dxdy
; dM1;
dy 0 + 2 2
to give 72 A1 cso

−1
dx
7 Uses m(T) or dy
to ‘correctly’ find m(N).
Hence m(N) = − or 2 A1 oe.
2 7 Can be ft using “ −1 . dx
dy
”.

y − 1 = m(x − 0) with
Either N: y − 1 = − 72 (x − 0) ‘their tangent, dx
or normal gradient’;
dy
M1;
or uses y = mx + 1 with ‘their tangent,
or N: y = − 72 x + 1
dx
dy
or normal gradient’ ;

Correct equation in the form


N: 7x + 2y – 2 = 0 ' ax + by + c = 0 ' , A1 oe cso
where a, b and c are integers.

7 marks

6666/01 Core Maths C4 3


June 2006 Advanced Subsidiary/Advanced Level in GCE Mathematics
PhysicsAndMathsTutor.com

Question
Scheme Marks
Number
Aliter
1. 2y 2 + 3y − 3x 2 − 2x − 5 = 0
Way 3
( y + 34 )
2
− 169 = 3x 2
2
+x + 5
2

y= ( 3 x2
2
+x+ 49
16 )− 3
4

Differentiates using the chain rule; M1;


( )
dy 1 − 21
= 3x 2
2
+x+ 49
16 ( 3x + 1) Correct expression for
dy
.
dx 2
dx A1 oe

At (0, 1), Substituting x = 0 into an equation involving


− 21 dy
dy 1 ⎛ 49 ⎞ 1⎛ 4⎞ 2 dx
; dM1
= = =
dx 2 ⎜⎝ 16 ⎟⎠ ⎜
2⎝7⎠⎟ 7 7
2
to give or −2
−7
A1 cso

7 Uses m(T) to ‘correctly’ find m(N).


Hence m(N) = − A1
2 Can be ft from “their tangent gradient”.

Either N: y − 1 = − 72 (x − 0) y − 1 = m(x − 0) with


‘their tangent or normal gradient’;
M1
or uses y = mx + 1 with ‘their tangent or
or N: y = − 72 x + 1
normal gradient’

Correct equation in the form ' ax + by + c = 0 ' ,


N: 7x + 2y – 2 = 0 A1 oe
where a, b and c are integers.
[7]

7 marks

6666/01 Core Maths C4 4


June 2006 Advanced Subsidiary/Advanced Level in GCE Mathematics
PhysicsAndMathsTutor.com

Question
Scheme Marks
Number

Considers this identity complete


and either substitutes
x = 21 , equates M1
2. (a) 3x − 1 ≡ A(1 − 2x) + B
coefficients or solves
simultaneous
equations
Let x = 21 ; 3
2
−1 = B ⇒ B= 1
2

Equate x terms; 3 = − 2A ⇒ A = − 32 A = − 32 ; B = 1
2 A1;A1

(No working seen, but A and B correctly stated ⇒ award all


three marks. If one of A or B correctly stated give two out of [3]
the three marks available for this part.)

Moving powers to top


(b) f(x) = − 32 (1 − 2x)−1 + 21 (1 − 2x)−2 on any one of the two M1
expressions

⎧ ( −1)( −2) ( −1)( −2)( −3) ⎫ Either 1 ± 2x or 1 ± 4x


= − 32 ⎨1 + ( −1)( −2x); + ( −2x)2 + ( −2x)3 + ...⎬ from either first or
⎩ 2! 3! ⎭ dM1;
second expansions
respectively

Ignoring − 32 and 21 ,
⎧ ( −2)( −3) ( −2)( −3)( −4) ⎫ any one correct
+ 21 ⎨1 + ( −2)( −2x); + ( −2x)2 + ( −2x)3 + ...⎬
⎩ 2! 3! ⎭ {..........} expansion. A1
Both {..........} correct. A1

{
= − 32 1 + 2x + 4x 2 + 8x 3 + ... + } 1
2 {1 + 4x + 12x 2
}
+ 32x 3 + ...

= −1 − x ; + 0x 2 + 4x 3 −1 − x ; (0x 2 ) + 4x 3 A1; A1
[6]

9 marks

Beware: In part (a) take care to spot that A = − 32 and B = 1


2
are the right way around.

Beware: In ePEN, make sure you aware the marks correctly in part (a). The first A1 is for A = − 32 and the
second A1 is for B = 1
2
.

Beware: If a candidate uses a method of long division please escalate this to you team leader.

6666/01 Core Maths C4 5


June 2006 Advanced Subsidiary/Advanced Level in GCE Mathematics
PhysicsAndMathsTutor.com

Question
Scheme Marks
Number
Aliter
2. (b) f(x) = (3x − 1)(1 − 2x)−2 Moving power to top M1
Way 2
⎛ ( −2)( −3) ⎞
⎜ 1 + ( −2)( −2x) ; + ( −2x)2 + ⎟ 1 ± 4x ; dM1;
= ( 3x − 1) × ⎜ Ignoring (3x − 1) , correct
2! ⎟
⎜ ( −2)( −3)( −4)
⎜ ( −2x) + ... ⎟⎟
3
(...........) expansion A1
⎝ 3! ⎠

= (3x − 1)(1 + 4x + 12x 2 + 32x 3 + ...)

= 3x + 12x 2 + 36x3 − 1 − 4x − 12x 2 − 32x3 + ... Correct expansion A1

= −1 − x ; + 0x 2 + 4x 3 −1 − x ; (0x 2 ) + 4x 3 A1; A1
[6]

Aliter
2. (b) Maclaurin expansion
Way 3
f(x) = − 32 (1 − 2x)−1 + 21 (1 − 2x)−2 Bringing both
M1
powers to top

Differentiates to give
f ′(x) = − 3(1 − 2x)−2 + 2(1 − 2x)−3 a(1 − 2x) −2 ± b(1 − 2x)−3 ; M1;
A1 oe
−3(1 − 2x)−2 + 2(1 − 2x)−3

f ′′(x) = − 12(1 − 2x)−3 + 12(1 − 2x)−4

f ′′′(x) = − 72(1 − 2x)−4 + 96(1 − 2x)−5 Correct f ′′(x) and f ′′′(x) A1

∴ f(0) = − 1 , f ′(0) = − 1 , f ′′(0) = 0 and f ′′′(0) = 24

gives f(x) = − 1 − x; + 0x 2 + 4x 3 + ... −1 − x ; (0x 2 ) + 4x 3 A1; A1


[6]

6666/01 Core Maths C4 6


June 2006 Advanced Subsidiary/Advanced Level in GCE Mathematics
PhysicsAndMathsTutor.com

Question
Scheme Marks
Number
Aliter
Moving powers to top
2. (b) f(x) = − 3(2 − 4x)−1 + 21 (1 − 2x)−2 on any one of the two M1
expressions
Way 4
⎧ −1 −2 ( −1)( −2) −3 ⎫ Either 21 ± x or 1 ± 4x
⎪(2) + ( −1)(2) ( −4x); + (2) ( −4x)2 ⎪
⎪ 2! ⎪
= −3 ⎨ ⎬ from either first or dM1;
⎪ ( −1)( −2)( −3) −4 ⎪ second expansions
+ (2) ( −4x) + ...
3
⎪⎩ 3! ⎪⎭ respectively

Ignoring −3 and 21 ,
⎧ ( −2)( −3) ( −2)( −3)( −4) ⎫ any one correct
+ 21 ⎨1 + ( −2)( −2x); + ( −2x)2 + ( −2x)3 + ...⎬
⎩ 2! 3! ⎭ {..........} expansion.
A1

Both {..........} correct. A1

= −3 { 1
2 }
+ x + 2x 2 + 4x 3 + ... + 1
2 {1 + 4x + 12x 2
}
+ 32x 3 + ...

= −1 − x ; + 0x 2 + 4x 3 −1 − x ; (0x 2 ) + 4x 3 A1; A1
[6]

6666/01 Core Maths C4 7


June 2006 Advanced Subsidiary/Advanced Level in GCE Mathematics
PhysicsAndMathsTutor.com

Question
Scheme Marks
Number

3. (a) Area Shaded =


∫ 3 sin (
0
x
2 ) dx

Integrating 3 sin ( 2x ) to give


⎡ −3 cos ( 2x ) ⎤

=⎢ 1 ⎥ k cos ( 2x ) with k ≠ 1 . M1
⎣ 2 ⎦0
Ignore limits.

= ⎡⎣ − 6 cos ( 2x ) ⎤⎦ −6 cos ( 2x ) or cos ( 2x )


2π −3
0
1
2
A1 oe.

= [ −6( −1)] − [ −6(1)] = 6 + 6 = 12 12 A1 cao


[3]
(Answer of 12 with no working scores M0A0A0.)


2π 2π
Use of V = π y 2 dx .
∫ (3 sin ( )) ∫
dx = 9π sin2 ( 2x ) dx
2
(b) Volume = π x
2 M1
0 0 Can be implied. Ignore limits.

Consideration of the Half Angle


⎡NB : cos 2x = ±1 ± 2 sin2 x gives sin2 x = 1 − cos2x ⎤ Formula for sin2 ( 2x ) or the
⎣ 2 ⎦
M1 ∗
⎡NB : cos x = ±1 ± 2 sin ( x ) gives sin ( x ) =
2 2 1 − cos x⎤ Double Angle Formula
⎣ 2 2 2 ⎦
for sin2 x


⎛ 1 − cos x ⎞
∫ ⎜⎝
Correct expression for Volume
∴ Volume = 9( π) ⎟ dx A1
2 ⎠ Ignore limits and π .
0

9 ( π)

=
2 ∫ (1 − cos x) dx
0

9 ( π) Integrating to give ±ax ± b sin x ; depM1 ∗ ;


[ x − sin x ] 0

= Correct integration
2
k − k cos x → kx − k sin x A1


=
2
[(2π − 0) − (0 − 0)]

9π Use of limits to give


= (2π) = 9 π2 or 88.8264… A1 cso
2 either 9 π2 or awrt 88.8
Solution must be completely [6]
correct. No flukes allowed.

9 marks

6666/01 Core Maths C4 8


June 2006 Advanced Subsidiary/Advanced Level in GCE Mathematics
PhysicsAndMathsTutor.com

Question 3

Note: π is not needed for the middle four marks of question 3(b).

Beware: Owing to the symmetry of the curve between x = 0 and x = 2π candidates can find:



Area = 2 3 sin ( 2x ) dx
0
in part (a).

∫ (3 sin ( ))
2
• Volume = 2 π x
2
dx
0

Beware: If a candidate gives the correct answer to part (b) with no working please escalate this response up
to your team leader.

6666/01 Core Maths C4 9


June 2006 Advanced Subsidiary/Advanced Level in GCE Mathematics
PhysicsAndMathsTutor.com

Question
Scheme Marks
Number

4. (a) x = sin t , y = sin ( t + π


6 )
Attempt to differentiate both x and M1
dx dy
= cos t , = cos ( t + π
6 ) y wrt t to give two terms in cos
dt dt Correct dx and dy A1
dt dt

Divides in correct way and


π
When t = , substitutes for t to give any of the
6 four underlined oe:
dy
=
cos ( 6 + 6π ) =
π 1
2
=
1
= awrt 0.58
Ignore the double negative if
A1
dx cos ( 6π ) 3
3 candidate has differentiated
sin → − cos
2

When t =
π
6
, x=
1
2
, y=
2
3 The point ( 1
2
, 2
3
) or ( 1
2
, awrt 0.87 ) B1

Finding an equation of a tangent


with their point and their tangent
T: y − 2
3
= 1
3
( x − 21 ) gradient or finds c and uses dM1
y = (their gradient)x + " c " .
Correct EXACT equation of A1 oe
tangent oe.

or 2
3
= 1
3
( 21 ) + c ⇒ c= 2
3
− 6
3
= 3
3

or T: ⎡ y = x+ ⎤
3 3
⎣ 3 3

[6]

(b) y = sin ( t + π
6 ) = sin t cos 6π + cos t sin 6π Use of compound angle formula
M1
for sine.

Nb : sin2 t + cos2 t ≡ 1 ⇒ cos2 t ≡ 1 − sin2 t

(1 − x )
Use of trig identity to find cos t in
∴ x = sin t gives cos t = 2
M1
terms of x or cos2 t in terms of x.

∴y = sin t + 21 cos t
3
2

Substitutes for
gives y= 2
3
x+ 1
2 (1 − x )2
AG sin t , cos 6π , cos t and sin 6π to A1 cso
give y in terms of x.
[3]

9 marks

6666/01 Core Maths C4 10


June 2006 Advanced Subsidiary/Advanced Level in GCE Mathematics
PhysicsAndMathsTutor.com

Question
Scheme Marks
Number
Aliter
4. (a) x = sin t , y = sin ( t + π
6 ) = sin t cos 6π + cos t sin 6π (Do not give this for part (b))
Way 2 Attempt to differentiate x and y
wrt t to give dx
dt
in terms of cos
M1
and dy
dt
in the
form ±a cos t ± b sin t
dx dy
= cos t , = cos t cos 6π − sin t sin 6π dx dy A1
dt dt Correct dt
and dt

π dy cos 6π cos 6π − sin 6π sin 6π


When t = , =
6 dx cos ( 6π )
Divides in correct way and
substitutes for t to give any of the A1
3
− 1 1
1 four underlined oe:
= 4
3
4
= 2
3
= = awrt 0.58
2 2
3

π 1 3
The point ( 1
2
, 2
3
)
When t = , x= , y= B1
6 2 2 or ( 21 , awrt 0.87 )

Finding an equation of a tangent


T: y − 2
3
= 1
3
(x − ) 1
2
with their point and their tangent
gradient or finds c and uses dM1
y = (their gradient)x + " c " .
Correct EXACT equation of A1 oe
tangent oe.

or 2
3
= 1
3
( 21 ) + c ⇒ c= 2
3
− 6
3
= 3
3

or T: ⎡ y = x+ ⎤
3 3
⎣ 3 3

[6]

6666/01 Core Maths C4 11


June 2006 Advanced Subsidiary/Advanced Level in GCE Mathematics
PhysicsAndMathsTutor.com

Question
Scheme Marks
Number
Aliter
4. (a) y= 2
3
x+ 1
2 (1 − x ) 2

Way 3 Attempt to differentiate two terms


3 ⎛ 1⎞⎛ 1⎞ using the chain rule for the second M1
dy
( ) ( −2x )
− 21
= + 1 − x2
dx 2 ⎜⎝ 2 ⎟⎠ ⎜⎝ 2 ⎟⎠ term.
A1
Correct dy
dx

Correct substitution of x = 1
3 ⎛ 1⎞⎛ 1⎞
( ) ( −2(0.5)) =
2
dy − 21 1
= + ⎜ ⎟ ⎜ ⎟ 1 − (0.5)2 dy A1
dx 2 ⎝ 2 ⎠⎝ 2 ⎠ 3 into a correct
dx

When t =
π
6
, x=
1
2
, y=
2
3 The point ( 1
2
, 2
3
) or ( 1
2
, awrt 0.87 ) B1

Finding an equation of a tangent


with their point and their tangent
T: y − 2
3
= 1
3
( x − 21 ) gradient or finds c and uses dM1
y = (their gradient)x + " c " .
Correct EXACT equation of A1 oe
tangent oe.

or 2
3
= 1
3
( 21 ) + c ⇒ c= 2
3
− 6
3
= 3
3

or T: ⎡ y = x+ ⎤
3 3
⎣ 3 3

[6]
Aliter
Substitutes x = sin t into the
4. (b) x = sin t gives y = 2
3
sin t + 1
2 (1 − sin t ) 2
equation give in y.
M1
Way 2
Nb : sin2 t + cos2 t ≡ 1 ⇒ cos2 t ≡ 1 − sin2 t

Use of trig identity to deduce that


cos t = (
1 − sin2 t ) cos t = (1 − sin t ) .
2 M1

gives y = sin t + 21 cos t


3
2

Using the compound angle


Hence y = sin t cos 6π + cos t sin 6π = sin ( t + π
)
6 formula to prove y = sin ( t + 6π ) A1 cso
[3]

9 marks

6666/01 Core Maths C4 12


June 2006 Advanced Subsidiary/Advanced Level in GCE Mathematics
PhysicsAndMathsTutor.com

Question
Scheme Marks
Number

5. (a) Equating i ; 0=6+λ ⇒ λ = −6 λ = −6 B1 ⇒ d


Can be implied
Using λ = − 6 and

For inserting their stated λ into


equating j ; a = 19 + 4( −6) = − 5 either a correct j or k component M1 ⇒ d
Can be implied.

equating k ; b = −1 − 2( −6) = 11 a = −5 and b = 11 A1


[3]
With no working…
… only one of a or b stated correctly gains the
first 2 marks.
… both a and b stated correctly gains 3 marks.
uuur
(b) OP = ( 6 + λ ) i + (19 + 4λ ) j + ( −1 − 2λ ) k

direction vector or l1 = d = i + 4 j − 2k

uuur uuur Allow this statement for M1


OP ⊥ l1 ⇒ OP • d = 0 uuur
if OP and d are defined as above.

⎛ 6+λ ⎞ ⎛ 1⎞
⎜ ⎟ ⎜ ⎟
ie. ⎜ 19 + 4λ ⎟ •⎜ 4 ⎟ = 0 ( or x + 4y − 2z = 0 ) Allow either of these two
underlined statements
M1
⎜ −1 − 2λ ⎟ ⎜ −2 ⎟
⎝ ⎠ ⎝ ⎠

∴ 6 + λ + 4(19 + 4λ ) − 2( −1 − 2λ ) = 0 Correct equation A1 oe

6 + λ + 76 + 16λ + 2 + 4λ = 0 Attempt to solve the equation in λ dM1

21λ + 84 = 0 ⇒ λ = −4 λ = −4 A1

uuur Substitutes their λ into an


OP = ( 6 − 4 ) i + (19 + 4( −4)) j + ( −1 − 2( −4)) k uuur M1
expression for OP
uuur
OP = 2 i + 3 j + 7 k 2 i + 3 j + 7 k or P(2, 3, 7) A1
[6]
uuur
Note: A similar method may be used by using OP = ( 0 + λ ) i + ( −5 + 4λ ) j + (11 − 2λ ) k and d = i + 4 j − 2k
uuur
OP • d = 0 yields 6 + λ + 4( −5 + 4λ ) − 2(11 − 2λ ) = 0
This simplifies to 21λ − 42 = 0 ⇒ λ = 2 .
uuur
OP = ( 0 + 2 ) i + ( −5 + 4(2)) j + (11 − 2(2) ) k
uuur
OP = 2 i + 3 j + 7 k

6666/01 Core Maths C4 13


June 2006 Advanced Subsidiary/Advanced Level in GCE Mathematics
PhysicsAndMathsTutor.com

Question
Scheme Marks
Number
Aliter uuur
(b) OP = ( 6 + λ ) i + (19 + 4λ ) j + ( −1 − 2λ ) k
Way 2 uuur
AP = ( 6 + λ − 0 ) i + (19 + 4λ + 5 ) j + ( −1 − 2λ − 11) k

direction vector or l1 = d = i + 4 j − 2k

uuur uuur uuur uuur Allow this


uuurstatement
uuur
AP ⊥ OP ⇒ AP • OP = 0 for M1 if AP and OP
are defined as above.

⎛ 6+λ ⎞ ⎛ 6+λ ⎞
⎜ ⎟ ⎜ ⎟
ie. ⎜ 24 + 4λ ⎟ • ⎜ 19 + 4λ ⎟ = 0 underlined statement M1
⎜ −12 − 2λ ⎟ ⎜ −1 − 2λ ⎟
⎝ ⎠ ⎝ ⎠

∴ ( 6 + λ )(6 + λ ) + (24 + 4λ )(19 + 4λ ) + ( −12 − 2λ )( −1 − 2λ ) = 0 Correct equation A1 oe

Attempt to solve the dM1


36 + 12λ + λ 2 + 456 + 96λ + 76λ + 16λ 2 + 12 + 24λ + 2λ + 4λ 2 = 0 equation in λ

21λ 2 + 210λ + 504 = 0

λ 2 + 10λ + 24 = 0 ⇒ ( λ = −6 ) λ = − 4 λ = −4 A1

uuur Substitutes their λ


OP = ( 6 − 4 ) i + (19 + 4( −4) ) j + ( −1 − 2( −4) ) k into an expression for M1
uuur
OP
uuur
OP = 2 i + 3 j + 7 k 2 i + 3 j + 7 k or A1
P(2, 3, 7)
[6]

uuur
Note: A similar method to way 2 may be used by using OP = ( 5 + λ ) i + (15 + 4λ ) j + (1 − 2λ ) k
uuur
and AP = ( 5 + λ − 0 ) i + (15 + 4λ + 5 ) j + (1 − 2λ − 11) k
uuur uuur
AP • OP = 0 yields ( 5 + λ )(5 + λ ) + (20 + 4λ )(15 + 4λ ) + ( −10 − 2λ )(1 − 2λ ) = 0
This simplifies to 21λ 2 + 168λ + 315 = 0 . λ 2 + 8λ + 15 = 0 ⇒ ( λ = −5 ) λ = − 3
uuur
OP = ( 5 − 3 ) i + (15 + 4( −3)) j + (1 − 2( −3)) k
uuur
OP = 2 i + 3 j + 7 k

6666/01 Core Maths C4 14


June 2006 Advanced Subsidiary/Advanced Level in GCE Mathematics
PhysicsAndMathsTutor.com

Question
Scheme Marks
Number
uuur
5. (c) OP = 2 i + 3 j + 7 k
uuur uuur
OA = 0 i − 5 j + 11k and OB = 5 i + 15 j + k

Subtracting
uuur uuuvectors
r uuurto find any two
uuur uuur
AP = ± ( 2 i + 8 j − 4 k ) , PB = ± ( 3 i + 12 j − 6k ) of AP , PB or AB ; and both are M1;
uuur
AB = ± ( 5 i + 20 j − 10 k ) uuur correctly
uuur ft using candidate’s A1 ±
OA and OP found in parts (a) and
(b) respectively.
uuur uuur uuur uuur
As AP = 2
3 (3 i + 12 j − 6k ) = 2
3
PB AP = 2
3
PB
uuur uuur uuur uuur
2(
or AB = 5
2i + 8 j − 4k ) = 52 AP or AB = 5
2
AP
uuur uuur uuur uuur
3(
or AB = 5
3i + 12 j − 6k ) = 53 PB or AB = 5
3
PB
uuur uuur uuur uuur
2(
or PB = 3
2i + 8 j − 4k ) = 32 AP or PB = 3
2
AP
uuur uuur uuur uuur
5(
or AP = 2
5i + 20 j − 10k ) = 52 AB or AP = 2
5
AB
uuur uuur uuur uuur
5(
or PB = 3
5i + 20 j − 10 k ) = 35 AB etc… or PB = 3
5
AB

alternatively candidates could say for example


that
uuur uuur
AP = 2 ( i + 4 j − 2k ) PB = 3 ( i + 4 j − 2k )

A, P and B are collinear


then the points A, P and B are collinear. A1
Completely correct proof.
uuur uuur
∴ AP : PB = 2 : 3 2:3 or 1: 32 or 84 : 189 aef B1 oe
allow SC 32 [4]

Aliter
At B; 5 = 6 + λ , 15 = 19 + 4λ or 1 = −1 − 2λ Writing down any of the three
5. (c) M1
or at B; λ = − 1 underlined equations.
Way 2
gives λ = − 1 for all three equations. λ = − 1 for all three equations
A1
or when λ = − 1 , this gives r = 5 i + 15 j + k or λ = − 1 gives r = 5 i + 15 j + k

Hence B lies on l1. As stated in the question both Must state B lies on l1 ⇒
A1
A and P lie on l1. ∴ A, P and B are collinear. A, P and B are collinear
uuur uuur
∴ AP : PB = 2 : 3 2:3 or aef B1 oe

[4]

13 marks

Beware of candidates who will try to fudge that one vector is multiple of another for the final A mark in part (c).
6666/01 Core Maths C4 15
June 2006 Advanced Subsidiary/Advanced Level in GCE Mathematics
PhysicsAndMathsTutor.com

Question
Scheme Marks
Number

6. (a)
x 1 1.5 2 2.5 3
y 0 0.5 ln 1.5 ln 2 1.5 ln 2.5 2 ln 3
or y 0 0.2027325541… ln2 1.374436098… 2 ln 3

Either 0.5 ln 1.5 and 1.5 ln 2.5


or awrt 0.20 and 1.37 B1
(or mixture of decimals and ln’s) [1]

{
For structure of trapezium
I1 ≈ × 1× 0 + 2 ( ln 2 ) + 2ln 3}
1
(b)(i) 2 rule {.............} ; M1;

1 A1
= × 3.583518938... = 1.791759... = 1.792 (4sf) 1.792
2 cao

(ii) 1
Outside brackets × 0.5 B1;
{
2
× 0.5 ; × 0 + 2 ( 0.5 ln1.5 + ln 2 + 1.5 ln 2.5 ) + 2ln 3}
1
I2 ≈ For structure of trapezium
2
rule {.............} ;
M1

1
= × 6.737856242... = 1.684464... awrt 1.684 A1
4
[5]

With increasing ordinates, the line segments at the Reason or an appropriate diagram
(c) B1
top of the trapezia are closer to the curve. elaborating the correct reason.
[1]

Beware: In part (b) candidate can add up the individual trapezia:

(b)(i) I1 ≈ 1
2 ( 0 + ln 2 ) + 21 (ln 2 + ln 3 )

(ii) I2 ≈ 21 . 21 ( 0 + 0.5ln1.5 ) + 21 . 21 ( 0.5ln1.5 + ln 2 ) + 21 . 21 ( ln 2 + 1.5 ln 2.5 ) + 21 . 21 (1.5ln 2.5 + 2ln 3 )

6666/01 Core Maths C4 16


June 2006 Advanced Subsidiary/Advanced Level in GCE Mathematics
PhysicsAndMathsTutor.com

Question
Scheme Marks
Number

⎧⎪u = ln x ⇒ du dx
= 1
x ⎫⎪ Use of ‘integration by
6. (d) ⎨ dv ⎬ parts’ formula in the M1
⎪⎩ dx = x − 1 ⇒ v = − x ⎪⎭
2
x
2 correct direction

⎛ x2 ⎞ 1 ⎛ x2 ⎞
I=⎜
⎝ 2
− x ⎟ ln x −
⎠ ∫ ⎜
x⎝ 2
− x ⎟ dx

Correct expression A1

An attempt to multiply at
⎛ x2 ⎞ ⎛x ⎞
=⎜
⎝ 2
− x ⎟ ln x −
⎠ ∫ ⎜ 2 − 1⎟ dx
⎝ ⎠
least one term through by
1
and an attempt to ...
x

⎛ x2 ⎞ ⎛ x2 ⎞ … integrate; M1;
=⎜ − x ⎟ ln x − ⎜ − x ⎟ (+c)
⎝ 2 ⎠ ⎝ 4 ⎠ correct integration A1

3
⎡ ⎛ x2 ⎞ x2 ⎤
∴I = ⎢ ⎜ − x ⎟ ln x − + x⎥
⎣⎝ 2 ⎠ 4 ⎦1

= ( 32 ln3 − 9
4
+ 3 ) − ( − 21 ln1 − 1
4
+ 1) Substitutes limits of 3 and
ddM1
1 and subtracts.

= 32 ln 3 + 3
4
+0− 3
4
= 32 ln3 AG 3
2
ln 3 A1 cso

[6]

Aliter
6. (d) ∫ (x − 1)ln x dx = ∫ x ln x dx − ∫ ln x dx
Way 2
x2 x2 ⎛ 1 ⎞
∫ ∫
x ln x dx = ln x − . dx Correct application of ‘by
2 ⎜⎝ x ⎟⎠
M1
2 parts’

x2 x2
= ln x − (+ c) Correct integration A1
2 4

⎛ 1⎞
∫ ∫
ln x dx = x ln x − x. ⎜ ⎟ dx Correct application of ‘by
M1
⎝x⎠ parts’

= x ln x − x (+ c) Correct integration A1

3 Substitutes limits of 3 and ddM1




1
( x − 1) ln x dx = ( 9
2
ln3 − 2 ) − ( 3ln3 − 2 ) = 32 ln 3 AG 1 into both integrands and
subtracts.
3
2
ln 3 A1 cso
[6]

6666/01 Core Maths C4 17


June 2006 Advanced Subsidiary/Advanced Level in GCE Mathematics
PhysicsAndMathsTutor.com

Question
Scheme Marks
Number
Aliter
⎧⎪u = ln x ⇒ du
dx
= 1x ⎫⎪ Use of ‘integration by
6. (d) ⎨ dv ( x − 1)2 ⎬
parts’ formula in the M1
⎪⎩ dx = ( x − 1) ⇒ v = 2 ⎪⎭ correct direction
Way 3
( x − 1) ( x − 1)
2 2

I=
2
ln x −
∫ 2x
dx Correct expression A1

( x − 1) Candidate multiplies out


2
x 2 − 2x + 1
=
2
ln x −
∫ 2x
dx numerator to obtain three
terms…

… multiplies at least one


( x − 1)
2
⎛1 1 ⎞
=
2
ln x −
∫ ⎜ 2 x − 1 + 2x ⎟ dx
⎝ ⎠
term through by 1x and
then attempts to ...

( x − 1) … integrate the result; M1;


2
⎛ x2 1 ⎞
= ln x − ⎜ − x + ln x ⎟ (+c)
2 ⎝ 4 2 ⎠ correct integration A1

3
⎡ ( x − 1)2 x2 1 ⎤
∴I = ⎢ ln x − + x − ln x ⎥
⎢⎣ 2 4 2 ⎥⎦
1

= ( 2ln3 − 9
4
+ 3 − 21 ln3 ) − ( 0 − 1
4
+ 1− 0) Substitutes limits of 3 and
ddM1
1 and subtracts.

= 2ln 3 − 21 ln 3 + 3
4
+ 1
4
−1 = 32 ln3 AG 3
2
ln 3 A1 cso

[6]

∫ 2x dx can also integrate to 2 ln 2x


1 1
Beware:

Beware: If you are marking using WAY 2 please make sure that you allocate the marks in the order they
appear on the mark scheme. For example if a candidate only integrated lnx correctly then they would be
awarded M0A0M1A1M0A0 on ePEN.

6666/01 Core Maths C4 18


June 2006 Advanced Subsidiary/Advanced Level in GCE Mathematics
PhysicsAndMathsTutor.com

Question
Scheme Marks
Number
Aliter By substitution
6. (d) u = ln x ⇒ du
dx
= 1
x

Way 4
I=
∫ ( e − 1) .ue
u u
du Correct expression

Use of ‘integration by
=
∫ (
u e2u − eu du ) parts’ formula in the M1
correct direction

⎛1 ⎞ ⎛1 ⎞
= u ⎜ e2u − eu ⎟ −
∫ ⎜⎝ 2 e − eu ⎟ dx
2u
Correct expression A1
⎝2 ⎠ ⎠

⎛1 ⎞ ⎛1 ⎞ Attempt to integrate; M1;


= u ⎜ e2u − eu ⎟ − ⎜ e2u − eu ⎟ (+c)
⎝2 ⎠ ⎝4 ⎠
correct integration A1

ln3
⎡1 1 ⎤
∴ I = ⎢ ue2u − ueu − e2u + eu ⎥
⎣2 4 ⎦ ln1

= ( 92 ln3 − 3ln3 − 94 + 3 ) − ( 0 − 0 − 41 + 1) Substitutes limits of ln3


ddM1
and ln1 and subtracts.

= 32 ln 3 + 3
4
+ 1
4
−1 = 32 ln3 AG 3
2
ln 3 A1 cso

[6]

13 marks

6666/01 Core Maths C4 19


June 2006 Advanced Subsidiary/Advanced Level in GCE Mathematics
PhysicsAndMathsTutor.com

Question
Scheme Marks
Number

dS dS
7. (a) From question, =8 =8 B1
dt dt

dS dS
S = 6x 2 ⇒ = 12x = 12x B1
dx dx

2
dS dS 8
dx dS dS
dt
=
dt
÷
dx
=
8
12x
;= 3
x
⇒ (k = 32 ) Candidate’s
dt
÷ ;
dx 12x
M1; A1oe

[4]

dV dV
(b) V = x3 ⇒ = 3x 2 = 3x 2 B1
dx dx

dV dV dx ⎛ 2 ⎞ dV dx M1;
= × = 3x 2 . ⎜ ⎟ ; = 2x Candidate’s × ; λx
dt dx dt ⎝ 3x ⎠ dx dt A1

1 dV 1 1 dV 1
As x = V 3 , then = 2V 3 AG Use of x = V 3 , to give = 2V 3 A1
dt dt
[4]

Separates the variables with

∫V ∫
dV −1

∫V ∫ 2 dt
dV or V 3 dV on one side and
(c) 1
= 1
3 B1
3

∫ 2 dt on the other side.


integral signs not necessary.

∫V ∫ 2 dt
− 31
dV =
Attempts to integrate and …
2
3
2
V = 2t (+c)
3 … must see V 3 and 2t; M1;
2
Correct equation with/without + c. A1

Use of V = 8 and t = 0 in a
changed equation containing c ; M1 ∗ ; A1
2
3
2
(8) 3 = 2(0) + c ⇒ c = 6
c=6

2
Hence: 3
2
V 3 = 2t + 6
Having found their “c” candidate …
(16 2 ) … substitutes V = 16 2 into an depM1 ∗
2

= 2t + 6 ⇒ 12 = 2t + 6
3
3
2
equation involving V, t and “c”.

giving t = 3. t = 3 A1 cao
[7]

15 marks

6666/01 Core Maths C4 20


June 2006 Advanced Subsidiary/Advanced Level in GCE Mathematics
PhysicsAndMathsTutor.com

Question
Scheme Marks
Number
Aliter
1 2 2
7. (b) x = V 3 & S = 6x 2 ⇒ S = 6V 3 S = 6V 3 B1
Way 2
dS −1 dV 1 31 dS −1 dV 1 31
= 4V 3 or = V = 4V 3 or = V B1
dV dS 4 dV dS 4

dV dS dV ⎛ 1 ⎞ 2 1 dS dV
= × = 8. ⎜ ;= = 2V 3 AG
1

− 31 ⎟ − 31
Candidate’s × ; 2V 3 M1; A1
dt dt dS ⎝ 4V ⎠ V dt dS

In ePEN, award Marks for


Way 2 in the order they appear
on this mark scheme.
[4]

Aliter
Separates the variables with

∫ ∫
dV 1 − 31
∫ 2V ∫ 1 dt
dV or V dV oe on one
7. (c) 1
= 2V
1
3 2 B1
3

side and
∫ 1 dt on the other side.
Way 2 integral signs not necessary.

∫ ∫ 1 dt
1 − 31
V dV =
2
Attempts to integrate and …
2

( 21 ) ( 32 ) V … must see V 3 and t; M1;


2
3
= t (+c)
Correct equation with/without + c. A1

Use of V = 8 and t = 0 in a
changed equation containing c ; M1 ∗ ; A1
2
3
4
(8) = (0) + c ⇒ c = 3
3

c=3

2
Hence: 3
4
V3 = t + 3
Having found their “c”
candidate …
(16 2 ) … substitutes V = 16 2 into an depM1 ∗
2

=t+3 ⇒ 6=t+3
3
3
4
equation involving V, t and “c”.

giving t = 3. t = 3 A1 cao
[7]

Beware: On ePEN award the marks in part (c) in the order they appear on the mark scheme.

6666/01 Core Maths C4 21


June 2006 Advanced Subsidiary/Advanced Level in GCE Mathematics
PhysicsAndMathsTutor.com

Question
Scheme Marks
Number
Aliter similar to way 1.
dV dV
(b) V = x3 ⇒ = 3x 2 = 3x 2 B1
dx dx
Way 3
dV dV dS dx ⎛ 1 ⎞ dV dS dx M1;
= × × = 3x 2 .8. ⎜ ⎟ ; = 2x Candidate’s × × ; λx
dt dx dt dS ⎝ 12x ⎠ dx dt dS A1

1 dV 1 1 dV 1
As x = V 3 , then = 2V 3 AG Use of x = V 3 , to give = 2V 3 A1
dt dt
[4]
Aliter
Separates the variables with

∫V ∫
dV −1

∫V ∫ 2 dt
dV or V 3 dV on one side and
(c) 1
= 1
3 B1
3

∫ 2 dt on the other side.


Way 3 integral signs not necessary.

∫V ∫ 2 dt
− 31
dV =
Attempts to integrate and …
2
2 … must see V 3 and 34 t; M1;
V =
3 4
t (+c)
Correct equation with/without + c. A1
3

Use of V = 8 and t = 0 in a
changed equation containing c ; M1 ∗ ; A1
2
(8) = (0) + c ⇒ c = 4
3 4
3
c=4

2
Hence: V 3 = 4
3
t+4
Having found their “c” candidate …
(16 2 ) … substitutes V = 16 2 into an depM1 ∗
2

= t+6 ⇒ 8= t+4
3
4 4
3 3
equation involving V, t and “c”.

giving t = 3. t = 3 A1 cao
[7]

• Beware when marking question 7(c). There are a variety of valid ways that a candidate can use to find the
constant “c”.
• In questions 7(b) and 7(c) there may be “Ways” that I have not listed. Please use the mark scheme as a
guide of how the mark the students’ responses.
• In 7(c), if a candidate instead tries to solve the differential equation in part (a) escalate the response to
your team leader.
• IF YOU ARE UNSURE ON HOW TO APPLY THE MARK SCHEME PLEASE ESCALATE THE
RESPONSE UP TO YOUR TEAM LEADER VIA THE REVIEW SYSTEM.
• Note: dM1 denotes a method mark which is dependent upon the award of the previous method mark.
ddM1 denotes a method mark which is dependent upon the award of the previous two method marks.
depM1 ∗ denotes a method mark which is dependent upon the award of M1∗ .
6666/01 Core Maths C4 22
June 2006 Advanced Subsidiary/Advanced Level in GCE Mathematics
PhysicsAndMathsTutor.com

Mark Scheme (Results)


January 2007

GCE

GCE Mathematics

Core Mathematics C4 (6666)

Edexcel Limited. Registered in England and Wales No. 4496750


Registered Office: One90 High Holborn, London WC1V 7BH
PhysicsAndMathsTutor.com

January 2007
6666 Core Mathematics C4
Mark Scheme
Question
Scheme Marks
Number
** represents a constant
−2 −2 Takes 2 outside the
⎛ 5x ⎞ 1 ⎛ 5x ⎞
= ( 2)
−2 −2
1. f(x) = (2 − 5x) ⎜1 − 2 ⎟ = ⎜1 − bracket to give any of B1
⎝ ⎠ 4⎝ 2 ⎟⎠ (2)-2 or 41 .

Expands (1 + * * x )−2 to
give an
M1
unsimplified
⎧ ( −2)( −3) ( −2)( −3)( −4) ⎫ 1 + ( −2)(* * x) ;
= 41 ⎨1 + ( −2)(* * x); + (* * x)2 + (* * x)3 + ...
⎩ 2! 3! ⎭
A correct unsimplified
{..........} expansion A1
with candidate’s
(* * x )

⎧ ( −2)( −3) − 5x 2 ( −2)( −3)( −4) − 5x 3 ⎫


= 41 ⎨1 + ( −2)( − 25x ); + ( 2 ) + ( 2 ) + ...⎬
⎩ 2! 3! ⎭

⎧ 75x 2 125x 3 ⎫
= 41 ⎨1 + 5x; + + + ...⎬
⎩ 4 2 ⎭

Anything that
1 5x 75x 2 125x 3 cancels to 1 + 5x ; A1;
= + ;+ + + ... 4 4
4 4 16 8
Simplified 75x 2
16
+ 125x 3
8 A1

1 1 11 2 5
= + 1 x; + 4 x + 15 x 3 + ...
4 4 16 8
[5]

5 marks

1
PhysicsAndMathsTutor.com

Question
Scheme Marks
Number
Aliter
1. f(x) = (2 − 5x)−2
Way 2
1
4
or (2)−2 B1
Expands (2 − 5x)−2 to
give an M1
⎧ −2 −3 ( −2)( −3) −4 ⎫ unsimplifed
⎪(2) + ( −2)(2) (* * x); + (2) (* * x)2 ⎪
⎪ 2! ⎪ (2) + ( −2)(2)−3 (* * x) ;
−2
=⎨ ⎬
⎪ ( −2)( −3)( −4) ⎪
+ (2)−5 (* * x)3 + ...
⎪⎩ 3! ⎭⎪ A correct unsimplified
{..........} expansion A1
with candidate’s
(* * x )

⎧ −2 −3 ( −2)( −3) −4 ⎫
⎪⎪(2) + ( −2)(2) ( −5x); + 2!
(2) ( −5x)2 ⎪

=⎨ ⎬
⎪ ( −2)( −3)( −4) ⎪
+ (2)−5 ( −5x)3 + ...
⎩⎪ 3! ⎭⎪

⎧⎪ 41 + ( −2)( 81 )( −5x); + (3)( 161 )(25x 2 )⎫⎪


=⎨ ⎬
⎪⎩ + ( −4)( 161 )( −125x 3 ) + ... ⎪⎭

Anything that
1 5x 75x 2 125x 3 cancels to 1 + 5x ; A1;
= + ;+ + + ... 4 4
4 4 16 8
Simplified 75x 2
16
+ 125x 3
8 A1

1 1 11 2 5
= + 1 x; + 4 x + 15 x 3 + ...
4 4 16 8
[5]

5 marks

Attempts using Maclaurin expansions need to be referred to your team leader.

2
PhysicsAndMathsTutor.com

Question
Scheme Marks
Number


1 1
2
⎛ ⎞ π Use of V = π y 2 dx .
2 2

∫ ∫
1 1
2. (a) Volume = π ⎜⎜ ⎟⎟ dx = dx B1
− 41 ⎝
3 (1 + 2x ) ⎠ 9
− 41
(1 + 2x )
2

Can be implied. Ignore limits.

1 Moving their power to the top.


⎛π⎞
2

∫ (1 + 2x )
−2 (Do not allow power of -1.)
=⎜ ⎟ dx
⎝9⎠ Can be implied. M1
− 41
Ignore limits and 9π

⎛ π ⎞ ⎡ (1 + 2x) ⎤
−1 2 Integrating to give ±p(1 + 2x)−1 M1
= ⎜ ⎟⎢ ⎥
⎝ 9 ⎠ ⎣⎢ ( −1)(2) ⎦⎥ − 1 − 21 (1 + 2x)−1 A1
4

⎛π⎞ 1

= ⎜ ⎟ ⎡ − 21 (1 + 2x)−1 ⎤ 1
2

⎝9⎠⎣ ⎦ −4

⎛ π ⎞ ⎡⎛ −1 ⎞ ⎛ −1 ⎞ ⎤
= ⎜ ⎟ ⎢⎜ ⎟−⎜ ⎟⎥
⎝ 9 ⎠ ⎣⎢⎝ 2(2) ⎠ ⎝ 2( 21 ) ⎠ ⎦⎥

⎛π⎞
= ⎜ ⎟ ⎡⎣ − 41 − ( −1)⎤⎦
⎝9⎠

Use of limits to give exact


π
= values of A1 aef
12 π
or 36 or 224π or aef

12

[5]
(b) From Fig.1, AB = 1
2
− (− 1
4 )= 3
4
units

As 3
4
units ≡ 3cm

3
then scale factor k = = 4.
( 34 )

⎛ π ⎞
Hence Volume of paperweight = ( 4 ) ⎜ ⎟ ( 4 ) × (their answer to part (a)) M1
3 3

⎝ 12 ⎠

16π 16 π
or awrt 16.8
V= cm3 = 16.75516... cm3 3
A1
3 or 6412π or aef
[2]

7 marks
π
Note: 9
(or implied) is not needed for the middle three marks of question 2(a).

3
PhysicsAndMathsTutor.com

Question
Scheme Marks
Number
Aliter


1 1
2
⎛ ⎞ Use of V = π y 2 dx .
2 2

∫ ∫
1 1
2. (a) Volume = π ⎜⎜ ⎟⎟ dx = π dx B1
− 41 ⎝
3 (1 + 2x ) ⎠ − 41
( 3 + 6x )
2

Can be implied. Ignore limits.


Way 2
1
2
Moving their power to the top.
= ( π)
∫ (3 + 6x )
−2
dx (Do not allow power of -1.)
M1
Can be implied.
− 41
Ignore limits and π

Integrating to give ±p(3 + 6x)−1


1

⎡ (3 + 6x)−1 ⎤ 2 M1
= ( π) ⎢ ⎥ −1
⎣ ( −1)(6) ⎦ − 1 − 61 (3 + 6x) A1
4

= ( π ) ⎡ − 61 (3 + 6x)−1 ⎤ 1
2

⎣ ⎦ −4

⎡⎛ −1 ⎞ ⎛ −1 ⎞ ⎤
= ( π ) ⎢⎜ ⎟ − ⎜ 3 ⎟⎥
⎣⎢⎝ 6(6) ⎠ ⎝ 6( 2 ) ⎠ ⎦⎥

= ( π ) ⎡⎣ − 361 − ( − 91 )⎤⎦

Use of limits to give exact


π
= values of A1 aef
12 π
or 36 or 224π or aef

12

[5]

Note: π is not needed for the middle three marks of question 2(a).

4
PhysicsAndMathsTutor.com

Question
Scheme Marks
Number
3. (a) x = 7 cos t − cos 7t , y = 7 sin t − sin7t ,
Attempt to differentiate x and y
with respect to t to give
dx dy dx
in the form ± A sin t ± B sin7t M1
= − 7 sin t + 7 sin 7t , = 7 cos t − 7 cos 7t dt
dt dt dy
in the form ±C cos t ± D cos 7t
dt
dx dy
Correct dt
and dt A1

dy 7 cos t − 7 cos 7t dy
∴ = Candidate’s dt
B1
dx −7 sin t + 7 sin 7t dx
dt

[3]

(b) π dy 7 cos 6π − 7 cos 76π Substitutes t = π


or 30o into their
When t = , m(T) = = ; 6
6 dx −7 sin 6π + 7 sin 76π dy M1
dx
expression;

( )
to give any of the four underlined
7 3
− − 7 23 7 3
=
2
= = − 3 = awrt − 1.73 expressions oe A1 cso
− 72 − 72 −7 (must be correct solution only)

−1 1 Uses m(T) to ‘correctly’ find


Hence m(N) = or = awrt 0.58 m(N). Can be ft from “their A1 oe.
− 3 3 tangent gradient”.

When t = 6π ,
(
The point 4 3, 4 )
x = 7 cos 6π − cos 76π = 7 3
− − ( )= 3 8 3
=4 3 B1
or ( awrt 6.9, 4 )
2 2 2

y = 7 sin − sin π
6

6
= 7
2
− (− 1
2 )= 8
2
=4

Finding an equation of a normal


N: y − 4 = 1
3 (x − 4 3 ) with their point and their normal M1
gradient or finds c by using
y = (their gradient)x + " c " .

Correct simplified
EXACT equation of normal. A1 oe
N: y = 1
3
x or y= 3
3
x or 3y = 3x This is dependent on candidate
(
using correct 4 3 , 4 )
or 4 = 1
3 (4 3 ) + c ⇒ c=4−4 = 0

Hence N: y = 1
3
x or y = 3
3
x or 3y = 3x
[6]
9 marks

5
PhysicsAndMathsTutor.com

Question
Scheme Marks
Number
Aliter
3. (a) x = 7 cos t − cos 7t , y = 7 sin t − sin7t ,
Way 2 Attempt to differentiate x and y
with respect to t to give dx
dt
in the
dx
= − 7 sin t + 7 sin 7t ,
dy
= 7 cos t − 7 cos 7t form ± A sin t ± B sin7t M1
dt dt dy
dt
in theform ±C cos t ± D cos 7t
Correct dxdt
and dy
dt
A1
dy 7cos t − 7 cos7t −7( −2 sin 4t sin3t)
dy
= = = tan 4t Candidate’s dt
B1
dx −7 sin t + 7 sin7t −7(2cos 4t sin3t) dx
dt

[3]

(b) Substitutes t = π
or 30o into their
π dy 6
When t = , m(T) = = tan 46π ;
6 dx dy
dx
expression; M1

=
2 ( ) (1)
2
3

= − 3 = awrt − 1.73
to give any of the three underlined
2 ( − 21 ) (1) expressions oe A1 cso
(must be correct solution only)

−1 1 Uses m(T) to ‘correctly’ find


Hence m(N) = or = awrt 0.58 m(N). Can be ft from “their A1 oe.
− 3 3 tangent gradient”.

When t = 6π ,
The point 4 3, 4 ( )
x = 7 cos 6π − cos 76π = 7 3
− − ( )= 3 8 3
=4 3 B1
or ( awrt 6.9, 4 )
2 2 2

y = 7 sin 6π − sin 76π = 7


2
− ( − 21 ) = 8
2
=4

Finding an equation of a normal


N: y − 4 = 1
3 (x − 4 3 ) with their point and their normal M1
gradient or finds c by using
y = (their gradient)x + " c " .

Correct simplified
EXACT equation of normal. A1 oe
N: y = 1
3
x or y= 3
3
x or 3y = 3x This is dependent on candidate
(
using correct 4 3 , 4 )
or 4 = 1
3 (4 3 ) + c ⇒ c=4−4 = 0

Hence N: y = 1
3
x or y = 3
3
x or 3y = 3x
[6]
9 marks

6
PhysicsAndMathsTutor.com

Beware: A candidate finding an m(T) = 0 can obtain A1ft for m(N) → ∞ , but
obtains M0 if they write y − 4 = ∞(x − 4 3 ) . If they write, however, N: x = 4 3 ,
then they can score M1.

Beware: A candidate finding an m(T) = ∞ can obtain A1ft for m(N) = 0, and also
obtains M1 if they write y − 4 = 0(x − 4 3 ) or y = 4.

7
PhysicsAndMathsTutor.com

Question
Scheme Marks
Number

2x − 1 A B
4. (a) ≡ +
(x − 1)(2x − 3) (x − 1) (2x − 3)

Forming this identity.


2x − 1 ≡ A(2x − 3) + B(x − 1) NB: A & B are not assigned in M1
this question
Let x = 32 , 2 = B ( 21 ) ⇒ B=4

either one of A = − 1 or B = 4 . A1
Let x = 1, 1 = A ( −1) ⇒ A = −1
both correct for their A, B. A1

−1 4
giving +
(x − 1) (2x − 3)
[3]

(2x − 1)
∫ ∫
dy Separates variables as shown
(b) & (c) = dx B1
y (2x − 3)(x − 1) Can be implied

−1
∫ (x − 1) + (2x − 3) dx
4 Replaces RHS with their partial
=
fraction to be integrated. M1

At least two terms in ln’s M1


∴ ln y = − ln(x − 1) + 2ln(2x − 3) + c At least two ln terms correct A1
All three terms correct and ‘+ c’ A1
[5]

y = 10, x = 2 gives c = ln10 c = ln10 B1

∴ ln y = − ln(x − 1) + 2ln(2x − 3) + ln10

ln y = − ln(x − 1) + ln(2x − 3)2 + ln10 Using the power law for


M1
logarithms

⎛ (2x − 3)2 ⎞
ln y = ln ⎜ ⎟ + ln10 or Using the product and/or quotient
⎝ (x − 1) ⎠ laws for logarithms to obtain a
M1
⎛ 10(2x − 3)2 ⎞ single RHS logarithmic term
ln y = ln ⎜ ⎟ with/without constant c.
⎝ (x − 1) ⎠

10(2x − 3)2 10(2x − 3)2


y= y= or aef. isw A1 aef
(x − 1) (x − 1)
[4]

12 marks

8
PhysicsAndMathsTutor.com

Question
Scheme Marks
Number
Aliter
(2x − 1)
∫y ∫ (2x − 3)(x − 1) dx
4. dy Separates variables as shown
= B1
(b) & (c) Can be implied
Way 2
−1

4 Replaces RHS with their partial
= + dx M1
(x − 1) (2x − 3) fraction to be integrated.

At least two terms in ln’s M1


∴ ln y = − ln(x − 1) + 2ln(2x − 3) + c At least two ln terms correct A1
All three terms correct and ‘+ c’ A1

See below for the award of B1 decide to award B1 here!! B1

Using the power law for


ln y = − ln(x − 1) + ln(2x − 3)2 + c M1
logarithms

Using the product and/or quotient


⎛ (2x − 3)2 ⎞ laws for logarithms to obtain a
ln y = ln ⎜ ⎟+c M1
⎝ x −1 ⎠ single RHS logarithmic term
with/without constant c.

⎛ A(2x − 3)2 ⎞
ln y = ln ⎜ ⎟ where c = ln A
⎝ x −1 ⎠

⎛ (2x − 3)2 ⎞ ⎛ (2x − 3)2 ⎞


ln⎜ +c
⎜ x − 1 ⎟⎟
ln⎜
⎜ x − 1 ⎟⎟
or e ln y
=e ⎝ ⎠
=e ⎝ ⎠
ec

A(2x − 3)2
y =
(x − 1)

y = 10, x = 2 gives A = 10
award
A = 10 for B1
above

10(2x − 3)2 10(2x − 3)2


y= y= or aef & isw A1 aef
(x − 1) (x − 1)
[5] & [4]

Note: The B1 mark (part (c)) should be awarded in the same place on ePEN as in the
Way 1 approach.

9
PhysicsAndMathsTutor.com

Question
Scheme Marks
Number
Aliter
(2x − 1)
∫y ∫ (2x − 3)(x − 1) dx
dy Separates variables as shown Can
(b) & (c) = B1
be implied
Way 3
−1

2 Replaces RHS with their partial
= + dx M1
(x − 1) (x − 32 ) fraction to be integrated.

At least two terms in ln’s M1


∴ ln y = − ln(x − 1) + 2ln(x − 32 ) + c At least two ln terms correct A1
All three terms correct and ‘+ c’ A1
[5]

y = 10, x = 2 gives c = ln10 − 2ln ( 21 ) = ln 40 c = ln10 − 2ln ( 21 ) or c = ln 40 B1 oe

∴ ln y = − ln(x − 1) + 2ln(x − 32 ) + ln 40

ln y = − ln(x − 1) + ln(x − 32 )2 + ln10 Using the power law for


M1
logarithms

⎛ (x − 32 )2 ⎞
ln y = ln ⎜ ⎟ + ln 40 or Using the product and/or quotient
⎝ (x − 1) ⎠ laws for logarithms to obtain a
M1
⎛ 40(x − 32 )2 ⎞ single RHS logarithmic term
ln y = ln ⎜ ⎟ with/without constant c.
⎝ (x − 1) ⎠

40(x − 32 )2 40(x − 32 )2
y= y= or aef. isw A1 aef
(x − 1) (x − 1)
[4]

Note: Please mark parts (b) and (c) together for any of the three ways.

10
PhysicsAndMathsTutor.com

Question
Scheme Marks
Number

5. (a) sin x + cos y = 0.5 ( eqn ∗ )

⎧ dy ⎫ Differentiates implicitly to include


dy
=⎬ cos x − sin y =0

⎩ dx ⎭ dx
( eqn # )
± sin y
dy
dx
. (Ignore ( dy
dx )
= .)
M1

dy cos x cos x
= A1 cso
dx sin y sin y
[2]

Candidate realises that they need


to solve ‘their numerator’ = 0
dy cos x
(b) =0 ⇒ = 0 ⇒ cos x = 0 …or candidate sets ddyx = 0 in their M1
dx sin y
(eqn #) and attempts to solve the
resulting equation.

both x = − 2π , π
or x = ±90o or
giving x = − 2π or x = π
2
2
A1
awrt x = ± 1.57 required here

When x = − 2π , sin ( − 2π ) + cos y = 0.5 Substitutes either their


When x = , sin (
π
2
π
2 ) + cos y = 0.5 x= π
2
or x = − 2π into eqn ∗ M1

⇒ cos y = 1.5 ⇒ y has no solutions Only one of y = 2π


3
or − 23π or 120o
A1
⇒ cos y = − 0.5 ⇒ y = 23π or − 23π or −120° or awrt -2.09 or awrt 2.09

Only exact coordinates of


In specified range ( x, y ) = ( 2π , 23π ) and ( 2π , − 23π ) ( 2π , 23π ) and ( 2π , − 23π ) A1

Do not award this mark if


candidate states other
coordinates inside
the required range.
[5]

7 marks

11
PhysicsAndMathsTutor.com

Question
Scheme Marks
Number

6. y = 2x = e x ln 2

dy dy
(a) = ln 2.e x ln2 = ln 2.e x ln2 M1
dx dx
Way 1
dy
Hence = ln 2.(2x ) = 2x ln 2 AG 2x ln 2 AG A1 cso
dx
[2]
Aliter
(a) ln y = ln 2x ( ) leads to ln y = x ln 2 Takes logs of both sides, then uses
the power law of logarithms…
Way 2 M1
1 dy … and differentiates implicitly to
= ln 2
y dx
dy
give 1y dx = ln 2

dy
Hence = y ln 2 = 2x ln 2 AG 2x ln 2 AG A1 cso
dx
[2]
2
Ax 2 ( x )
M1
dy
y=2
2 2
(b) (x )
⇒ = 2x. 2 ( x ).ln 2 2
2x. 2 ( x ).ln 2
dx A1
or 2x. y.ln 2 if y is defined

dy
Substitutes x = 2 into their
dy dx
When x = 2, = 2(2) 2 4 ln 2 M1
dx which is of the form ± k 2( x2 )

2
or Ax 2 ( x )

dy
= 64 ln 2 = 44.3614... 64ln 2 or awrt 44.4 A1
dx
[4]

6 marks

12
PhysicsAndMathsTutor.com

Question
Scheme Marks
Number
Aliter
6. (b) ( )
ln y = ln 2x
2
leads to ln y = x 2 ln 2
Way 2
1 dy
= Ax.ln 2 M1
1 dy y dx
= 2x.ln 2
y dx 1 dy
= 2x.ln 2 A1
y dx

dy
Substitutes x = 2 into their
dy dx
When x = 2, = 2(2) 2 4 ln 2 M1
dx which is of the form ± k 2( x2 )

2
or Ax 2 ( x )

dy
= 64 ln 2 = 44.3614... 64ln 2 or awrt 44.4 A1
dx

[4]

13
PhysicsAndMathsTutor.com

Question
Scheme Marks
Number
uuur uuur
7. a = OA = 2i + 2 j + k ⇒ OA = 3
uuur uuur
b = OB = i + j − 4 k ⇒ OB = 18
uuur uuur
BC = ± ( 2i + 2 j + k ) ⇒ BC = 3
uuur uuur
AC = ± ( i + j − 4 k ) ⇒ AC = 18

uuur
(a) c = OC = 3i + 3 j − 3k 3i + 3 j − 3k B1 cao
[1]

(b)
⎛ 2⎞ ⎛ 1 ⎞
uuur uuur ⎜ ⎟ ⎜ ⎟
OA • OB = ⎜ 2 ⎟ • ⎜ 1 ⎟ = 2 + 2 − 4 = 0 or…
⎜ 1 ⎟ ⎜ −4 ⎟
⎝ ⎠ ⎝ ⎠
⎛ −1⎞ ⎛ 2 ⎞ An attempt to take the dot product
uuur uuur ⎜ ⎟ ⎜ ⎟ uuur uuur
BO • BC = ⎜ −1⎟ • ⎜ 2 ⎟ = −2 − 2 + 4 = 0 or… between either OA and OB
⎜ 4 ⎟ ⎜ 1⎟ uuur uuur uuur uuur M1
⎝ ⎠ ⎝ ⎠ OA and AC , AC and BC
uuur uuur
⎛ 1 ⎞ ⎛ 2⎞ or OB and BC
uuur uuur ⎜ ⎟ ⎜ ⎟
AC • BC = ⎜ 1 ⎟ • ⎜ 2 ⎟ = 2 + 2 − 4 = 0 or…
⎜ −4 ⎟ ⎜ 1 ⎟ Showing the result is equal to zero. A1
⎝ ⎠ ⎝ ⎠
⎛ −2 ⎞ ⎛ 1⎞
uuur uuur ⎜ ⎟ ⎜ ⎟
AO • AC = ⎜ −2 ⎟ • ⎜ 1 ⎟ = −2 − 2 + 4 = 0
⎜ −1⎟ ⎜ −4 ⎟
⎝ ⎠ ⎝ ⎠

and therefore OA is perpendicular to OB and perpendicular and


A1 cso
hence OACB is a rectangle. OACB is a rectangle

Using distance formula to find


M1
either the correct height or width.
Multiplying the rectangle’s
Area = 3 × 18 = 3 18 = 9 2 M1
height by its width.
exact value of
A1
3 18 , 9 2 , 162 or aef
[6]
uuur
(c) OD = d = 1
2 ( 3i + 3 j − 3k ) 1
2 ( 3i + 3 j − 3k ) B1
[1]

14
PhysicsAndMathsTutor.com

Question
Scheme Marks
Number
using dot product formula
uuur uuur
DA = ± ( 21 i + 21 j + 52 k ) & DC = ± ( 32 i + 32 j − 32 k ) Identifies a set of two M1
(d) uuur uuur relevant vectors
or BA = ± (i + j + 5k ) & OC = ± ( 3i + 3 j − 3k ) Correct vectors ± A1
Way 1
⎛ 0.5 ⎞ ⎛ 1.5 ⎞ Applies dot product
⎜ ⎟ ⎜ ⎟ formula on multiples
⎜ 0.5 ⎟ • ⎜ 1.5 ⎟ 3 3 15 of these vectors.
⎜ 2.5 ⎟ ⎜ −1.5 ⎟ + −
cos D = ( ± ) ⎝ ⎠ ⎝ ⎠ = (±) 4 4 4 = (±) 1 Correct ft.
27 27 27 3 application of dot
.
2 2 4 product formula

Attempts to find the


⎛ 1⎞
D = cos−1 ⎜ − ⎟ correct angle D ddM1
⎝ 3⎠ rather than 180° − D .

109.5° or
D = 109.47122...o A1
awrt 109° or 1.91c
Aliter using dot product formula and direction vectors [6]
uuur uuur Identifies a set of two M1
(d) d BA = ± (i + j + 5k ) & d OC = ± (i + j − k ) direction vectors
Correct vectors ± A1
Way 2
Applies dot product
⎛ 1 ⎞ ⎛ 1⎞
⎜ ⎟ ⎜ ⎟ formula on multiples dM1
⎜ 1 ⎟ • ⎜ 1⎟ of these vectors.
⎜ −1⎟ ⎜ 5 ⎟ 1+ 1− 5
cos D = ( ± ) ⎝ ⎠ ⎝ ⎠ = ( ± )
1 Correct ft.
= (±)
3 . 27 3 . 27 3 application of dot A1
product formula.

Attempts to find the


⎛ 1⎞
D = cos−1 ⎜ − ⎟ correct angle D ddM1
⎝ 3⎠ rather than 180° − D .

109.5° or
D = 109.47122...o A1
awrt 109° or 1.91c
[6]

15
PhysicsAndMathsTutor.com

Question
Scheme Marks
Number
Aliter using dot product formula and similar triangles
uuur uuur Identifies a set of two M1
(d) dOA = ( 2i + 2 j + k ) & d OC = (i + j − k ) direction vectors
Correct vectors A1
Way 3
Applies dot product
⎛ 2⎞ ⎛ 1 ⎞
⎜ ⎟ ⎜ ⎟ formula on multiples dM1
⎜ 2⎟ •⎜ 1 ⎟ of these vectors.
⎜ 1 ⎟ ⎜ −1⎟ 2+ 2−1
cos ( 21 D ) = ⎝ ⎠ ⎝ ⎠ =
1 Correct ft.
=
9. 3 9. 3 3 application of dot A1
product formula.

Attempts to find the


⎛ 1 ⎞ correct angle D by
D = 2 cos−1 ⎜ ⎟
⎝ 3⎠ doubling their angle ddM1
for 21 D .

109.5° or
D = 109.47122...o A1
awrt 109° or 1.91c
[6]

Aliter using
uuur cosine rule uuur uuur
(d) DA = 21 i + 1
2
j + 52 k , DC = 32 i + 3
2
j − 32 k , AC = i + j − 4 k
Way 4
Attempts to find all
uuur uuur uuur the lengths of all
27 27 M1
DA = , DC = , AC = 18 three edges of
2 2 ∆ ADC
All Correct A1

⎛ 27 ⎞ ⎛ 27 ⎞
2 2
Using the cosine rule
( )
2
⎜⎜ ⎟ +⎜ ⎟ − 18 formula with correct dM1
⎝ 2 ⎟⎠ ⎜⎝ 2 ⎟⎠ 1
cos D = =− ‘subtraction’.
⎛ 27 ⎞ ⎛ 27 ⎞ 3 Correct ft application
2 ⎜⎜ ⎟⎟ ⎜⎜ ⎟⎟
⎝ 2 ⎠⎝ 2 ⎠ of the cosine rule A1
formula

Attempts to find the


⎛ 1⎞ −1
D = cos ⎜ − ⎟ correct angle D ddM1
⎝ 3⎠ rather than 180° − D .

109.5° or
D = 109.47122...o ° A1
awrt 109 or 1.91c
[6]

16
PhysicsAndMathsTutor.com

Question
Scheme Marks
Number
Aliter using
uuur
trigonometry on auuur
right angled triangle
uuur
(d) DA = 2 i + 2 j + 2 k OA = 2i + 2 j + k AC = i + j − 4 k
1 1 5

Way 5
Attempts to find two
Let X be the midpoint of AC
out of the three M1
uuur uuur uuur uuur uuur lengths in ∆ ADX
27 3
DA = , DX = 1
2
OA = , AX = 1
2
AC = 1
2
18
2 2
(hypotenuse), (adjacent) , (opposite)
Any two correct A1

Uses correct
18 18 dM1
3
sohcahtoa to find 21 D
sin( 21 D) = 2
27
, cos( 21 D) = 2
27
or tan( 21 D) = 2
3
2 Correct ft application
2 2 A1
of sohcahtoa

Attempts to find the


⎛ 18 ⎞ correct angle D by
eg. D = 2 tan−1 ⎜ 2

⎜ 3 ⎟ doubling their angle ddM1
⎝ 2 ⎠
for 21 D .

109.5° or
D = 109.47122...o A1
awrt 109° or 1.91c
[6]
Aliter using
uuur
trigonometry on auuur
right angled similar triangle OAC
uuur
(d) OC = 3i + 3 j − 3k OA = 2i + 2 j + k AC = i + j − 4 k
Way 6 Attempts to find two
uuur uuur uuur out of the three
OC = 27 , OA = 3 , AC = 18 M1
lengths in ∆ OAC
(hypotenuse), (adjacent), (opposite)
Any two correct A1

Uses correct dM1


18 3 18 sohcahtoa to find 21 D
sin( 21 D) = , cos( 21 D) = or tan( 21 D) =
27 27 3 Correct ft application
of sohcahtoa A1

Attempts to find the


⎛ 18 ⎞ correct angle D by
eg. D = 2 tan−1 ⎜⎜ ⎟⎟
⎝ 3 ⎠
doubling their angle ddM1
for 21 D .

109.5° or
D = 109.47122...o A1
awrt 109° or 1.91c
[6]

17
PhysicsAndMathsTutor.com

Question Marks
Scheme
Number
Aliter
uuur
7. (b) (i) c = OC = ± ( 3i + 3 j − 3k )
uuur
AB = ± ( −i − j − 5 k )
Way 2
uuur uuur A complete method M1
OC = (3) + (3) + ( −3) =
2 2 2
(1) + (1) + ( −5) = AB
2 2 2
of proving that the
diagonals are equal.
uuur uuur
OC = AB = 27 Correct result. A1
As

diagonals are equal A1 cso


then the diagonals are equal, and OACB is a rectangle. and
OACB is a rectangle
[3]
uuur uuur
a = OA = 2i + 2 j + k ⇒ OA = 3
uuur uuur
b = OB = i + j − 4 k ⇒ OB = 18
uuur uuur
BC = ± ( 2i + 2 j + k ) ⇒ BC = 3
uuur uuur
AC = ± ( i + j − 4 k ) ⇒ AC = 18
uuur uuur
c = OC = ± ( 3i + 3 j − 3k ) ⇒ OC = 27
uuur uuur
AB = ± ( −i − j − 5 k ) ⇒ AB = 27
Aliter
7. (b) (i) (OA)2 + ( AC )2 = (OC )2
or (BC )2 + (OB )2 = (OC )2
or (OA)2 + (OB )2 = ( AB )2
or (BC )2 + ( AC )2 = ( AB )2 or equivalent
Way 3
A complete method M1
of proving that
( )
2
⇒ (3)2 + ( 18)2 = 27 Pythagoras holds
using their values.
Correct result
A1

and therefore OA is perpendicular to OB A1 cso


perpendicular and
or AC is perpendicular to BC
OACB is a rectangle
and hence OACB is a rectangle.
[3]
14marks

18
PhysicsAndMathsTutor.com

Question
Scheme Marks
Number

8. (a)
x 0 1 2 3 4 5
7 10 13
y e1 e2 e e e e4
or y 2.71828… 7.38906… 14.09403… 23.62434… 36.80197… 54.59815…
Either e 7 , e 10 and e 13
or awrt 14.1, 23.6 and 36.8
or e to the power
awrt 2.65, 3.16, 3.61
(or mixture of decimals and e’s)
At least two correct B1
All three correct B1
[2]

(b) 1
Outside brackets × 1 B1;
1
2 { (
I ≈ × 1 ; × e1 + 2 e2 + e 7
+e 10
+e 13
)+e } 4 2
For structure of trapezium
rule {.............} ; M1

1 A1
= × 221.1352227... = 110.5676113... = 110.6 (4sf) 110.6
2 cao
[3]

Beware: In part (b) candidates can add up the individual trapezia:

(
(b) I ≈ 21 .1( e1 + e2 ) + 21 .1 e2 + e 7
) + .1( e
1
2
7
+e 10
) + .1( e
1
2
10
+e 13
) + .1( e
1
2
13
+ e4 )

19
PhysicsAndMathsTutor.com

Question
Scheme Marks
Number
1 dt − 21 − 21 dt
t = (3x + 1) 2 ⇒ = 1
2
.3.(3x + 1) A(3x + 1) or t =A M1
dx dx
(c) dt dt
− 21
… or t 2 = 3 x + 1 ⇒ 2t =3 3
2
(3x + 1) or 2t =3 A1
dx dx

dt 3 3 dx 2t Candidate obtains either


so = 1
= ⇒ =
dx 2.(3x + 1) 2 2t dt 3 dt
dx
or dx
dt
in terms of t …

… and moves on to dM1

∫ ∫ ∫
(3 x +1) dx 2t substitute this into I to
∴I = e dx = et . dt = e t . .dt
dt 3 convert an integral wrt x
to an integral wrt t.

∴I=
∫ 2
3
t et dt
∫ 2
3
t et A1
change limits: changes limits x → t so
B1
when x = 0, t = 1 & when x = 5, t = 4 that 0 → 1 and 5 → 4

Hence I =
∫1
2
3
tet dt ; where a = 1, b = 4, k = 2
3

[5]
Let k be any constant for
⎪⎧u = t ⇒ dt = 1 ⎪⎫
du

(d) ⎨ dv t⎬
the first three marks of this
⎩⎪ dt = e ⇒ v = e ⎭⎪
t
part.

Use of ‘integration by

( )
parts’ formula in the M1

k t et dt = k t e t − e t .1 dt
∫ correct direction.
Correct expression with a A1
constant factor k.

Correct integration
(
= k t et − et ) +c with/without A1
a constant factor k

4
Substitutes their changed

∫ 2
3
tet dt =
2
3
{(
4e4 − e4 − e1 − e1 ) ( )} limits into the integrand dM1 oe
1 and subtracts oe.

= 32 (3e4 ) = 2e 4 = 109.1963... either 2e4 or awrt 109.2 A1


[5]
15 marks
• Note: dM1 denotes a method mark which is dependent upon the award of the
previous method mark
• ddM1 denotes a method mark which is dependent upon the award of the previous
two method marks.

20
PhysicsAndMathsTutor.com

Mark Scheme (Results)


Summer 2007

GCE

GCE Mathematics

Core Mathematics C4 (6666)

Edexcel Limited. Registered in England and Wales No. 4496750


Registered Office: One90 High Holborn, London WC1V 7BH
PhysicsAndMathsTutor.com

June 2007
6666 Core Mathematics C4
Mark Scheme

Question
Scheme Marks
Number
** represents a constant
−3 −3 Takes 3 outside the bracket
⎛ 2x ⎞ 1 ⎛ 2x ⎞
= (3)
−3
f( x ) = (3 + 2 x ) −3
⎜1 + 3 ⎟ = ⎜ 1+ to give any of (3)−3 or 27
1
.
3 ⎟⎠
1. (a) B1
⎝ ⎠ 27 ⎝ See note below.

Expands (1 + * * x )−3 to
give a simplified or an un-
simplified M1;
⎧ ( −3)( −4) ( −3)( −4)( −5) ⎫ 1 + ( −3)(* * x) ;
= 1
27 ⎨1 + ( −3)(* * x); + (* * x)2 + (* * x)3 + ...⎬
⎩ 2! 3! ⎭ A correct simplified or an
un-simplified
with * * ≠ 1
{..........} expansion with A1
candidate’s followed thro’
(* * x )

⎧ ( −3)( −4) 2x 2 ( −3)( −4)( −5) 2x 3 ⎫


= 1
27 ⎨1 + ( −3)( 3 ) +
2x
(3) + ( 3 ) + ...⎬
⎩ 2! 3! ⎭

⎧ 8x 2 80 3 ⎫
= 1
27 ⎨1 − 2x + − x + ...⎬
⎩ 3 27 ⎭

Anything that
1 2x 8x 2 80x 3 cancels to 1 − 2x ; A1;
= − ;+ − + ... 27 27
27 27 81 729
Simplified 8x 2
81
− 80x3
729 A1
[5]

5 marks

Note: You would award: B1M1A0 for


Special Case: If you see the
⎧⎪ constant 271 in a candidate’s final
( −3)( −4) ( −3)( −4)( −5) ⎫
= 1
27 ⎨1 + ( −3)( 3 ) +
2x
(2x)2 + (2x)3 + ...⎬ binomial expression, then you can
⎩⎪ 2! 3! ⎭
award B1
because * * is not consistent.

6666/01 Core Maths C4 2 25th June 2007


June 2007 Advanced Subsidiary/Advanced Level in GCE Mathematics Version 8: THE FINAL VERSION
PhysicsAndMathsTutor.com

Question
Scheme Marks
Number
Aliter
1. f(x) = (3 + 2x)−3
Way 2
1
27
or (3)−3 (See note ↓ ) B1
−3
Expands (3 + 2x) to give
⎧ −3 −4 ( −3)( −4) −5 ⎫ an un-simplified or
⎪(3) + ( −3)(3) (* * x); + (3) (* * x)2 ⎪ simplified M1
⎪ 2! ⎪
=⎨ ⎬ (3)−3 + ( −3)(3)−4 (* * x) ;
⎪ ( −3)( −4)( −5) −6 ⎪
+ (3) (* * x) + ...
3
⎪⎩ 3! ⎭⎪ A correct un-simplified or
simplified
with * * ≠ 1 {..........} expansion with A1
candidate’s followed
thro’ ( * * x )

⎧ −3 −4 ( −3)( −4) −5 ⎫
⎪(3) + ( −3)(3) (2x); + (3) (2x)2 ⎪
⎪ 2! ⎪
=⎨ ⎬
⎪ ( −3)( −4)( − 5) ⎪
+ (3)−6 (2x)3 + ...
⎪⎩ 3! ⎪⎭

⎧⎪ 271 + ( −3)( 81
1
)(2x); + (6)( 243
1
)(4x 2 )⎫⎪
=⎨ ⎬
⎪⎩ + ( −10)( 729
1
)(8x 3 ) + ... ⎪⎭

Anything that
1 2x 8x 2 80x 3 cancels to 1 − 2x ; A1;
= − ;+ − + ... 27 27
27 27 81 729
Simplified 8x 2
81
− 80x3
729 A1

[5]

5 marks

Attempts using Maclaurin expansions need to be escalated up to your team leader.

If you feel the mark scheme does not apply fairly to a candidate please escalate the response up to your team leader.

Special Case: If you see the


constant 271 in a candidate’s
final binomial expression, then
you can award B1

6666/01 Core Maths C4 3 25th June 2007


June 2007 Advanced Subsidiary/Advanced Level in GCE Mathematics Version 8: THE FINAL VERSION
PhysicsAndMathsTutor.com

Question
Scheme Marks
Number

1
2x
2.

0
(2 + 1)
x 2
dx , with substitution u = 2 x

du dx 1 du
= 2x.ln 2 or du
= u.ln 2
= 2 x.ln 2 ⇒ = x dx dx
B1
dx du 2 .ln 2 or ( ) 1
u
du
dx
= ln 2

⎛ 1 ⎞
∫ (u + 1)
2x
∫ ∫ (u + 1)
1 1
dx = ⎜ ⎟ du k du
M1 ∗
(2 + 1)
2
⎝ ln 2 ⎠
x 2 2

where k is constant

⎛ 1 ⎞ ⎛ −1 ⎞ (u + 1)−2 → a(u + 1)−1 M1


=⎜ ⎟⎜ ⎟+c
⎝ ln 2 ⎠ ⎝ (u + 1) ⎠ (u + 1)−2 → − 1.(u + 1)−1 A1

change limits: when x = 0 & x = 1 then u = 1 & u = 2

1 2
2x 1 ⎡ −1 ⎤

0
(2 + 1)
x 2
dx =
ln 2 ⎢⎣ (u + 1) ⎥⎦ 1

1 ⎡⎛ 1 ⎞ ⎛ 1 ⎞ ⎤ Correct use of limits


= ⎢⎜ − 3 ⎟ − ⎜ − 2 ⎟ ⎥ depM1 ∗
ln 2 ⎣⎝ ⎠ ⎝ ⎠⎦ u = 1 and u = 2

1 − −
=
1 1 1 1 1
6ln2 or ln4 ln8 or 2ln2 3n2 A1 aef
6 ln 2
Exact value only! [6]
Alternatively candidate can revert back to x …

1 1
2x 1 ⎡ −1 ⎤

0
(2 + 1)
x 2
dx =
ln 2 ⎢⎣ (2 x + 1) ⎥⎦ 0

1 ⎡⎛ 1 ⎞ ⎛ 1 ⎞ ⎤ Correct use of limits


= ⎢⎜ − 3 ⎟ − ⎜ − 2 ⎟ ⎥ depM1 ∗
ln 2 ⎣⎝ ⎠ ⎝ ⎠⎦ x = 0 and x = 1

1
= 1
6ln2 or 1
ln4
− 1
ln8 or 1
2ln2
− 1
3ln2 A1 aef
6 ln 2
Exact value only!
6 marks

If you see this integration There are other acceptable


applied anywhere in a 1
answers for A1, eg: 2ln8 or 1
ln64
candidate’s working then you can
NB: Use your calculator to check
award M1, A1
eg. 0.240449…

6666/01 Core Maths C4 4 25th June 2007


June 2007 Advanced Subsidiary/Advanced Level in GCE Mathematics Version 8: THE FINAL VERSION
PhysicsAndMathsTutor.com

Question
Scheme Marks
Number

⎧⎪u = x ⇒ ddux = 1 ⎫⎪
3. (a) ⎨ dv ⎬
⎩⎪ dx = cos 2 x ⇒ v = 2 sin 2 x ⎭⎪
1

(see note below)


Use of ‘integration by parts’
∫ ∫
M1
Int = x cos 2 x dx = 1
2
x sin 2 x − 1
2
sin 2 x.1 dx formula in the correct direction.
Correct expression.
A1

sin 2 x → − 21 cos 2 x
= 1
2
x sin 2x − 1
2 (− 1
2
cos 2x ) + c or sin kx → − k1 cos kx dM1
with k ≠ 1 , k > 0

= 1
2
x sin 2 x + 1
4
cos 2 x + c Correct expression with +c A1
[4]

Substitutes correctly
∫ ∫ ( ) dx
cos2 x + 1
(b) x cos2 x dx = x 2 for cos2 x in the M1
given integral

∫ ∫
1 1
= x cos 2 x dx + x dx
2 2

1
1⎛ 1 ⎞ 1 ( their answer to (a)) ;

1
= ⎜ x sin 2 x + cos 2 x ⎟ ; + x dx 2 A1;
2⎝2 4 ⎠ 2
or underlined expression

1 1 1 Completely correct expression


= x sin 2 x + cos 2 x + x 2 ( + c ) A1
4 8 4 with/without +c
[3]

7 marks

Notes:

(b) Int =
∫ x cos 2x dx = 1
2
x sin 2 x ±
∫ 1
2
sin 2 x.1 dx This is acceptable for M1 M1

⎪⎧u = x ⇒ ddux = 1 ⎪⎫
⎨ dv ⎬
⎩⎪ dx = cos 2 x ⇒ v = λ sin 2 x ⎭⎪

∫ x cos 2x dx ∫
Int = = λ x sin 2 x ± λ sin 2 x.1 dx This is also
M1
acceptable for M1

6666/01 Core Maths C4 5 25th June 2007


June 2007 Advanced Subsidiary/Advanced Level in GCE Mathematics Version 8: THE FINAL VERSION
PhysicsAndMathsTutor.com

Question
Scheme Marks
Number

Aliter Substitutes correctly


∫ ∫ ( ) dx
cos2 x + 1
3. (b) x cos2 x dx = x 2 for cos2 x in the
Way 2 given integral …
M1

⎧⎪u = x ⇒ ddux = 1 ⎫⎪ … or
⎨ dv 1 1 ⎬
u = x and dv
= 21 cos 2 x + 1

⎩⎪ dx = 2 cos 2 x + 2 ⇒ v = 4 sin 2 x + x⎪
1 1 dx 2
2 ⎭

= 1
4
x sin 2 x + 1
2
x2 −
∫( 1
4
sin 2 x + 1
2
x ) dx

1
= 1
x sin 2 x + 1
x 2 + 81 cos 2 x − 41 x 2 + c ( their answer to (a)) ; A1
4 2 2
or underlined expression

1 1 1 Completely correct expression


= x sin 2 x + cos 2 x + x 2 ( + c ) A1
4 8 4 with/without +c
[3]

Substitutes correctly
∫ ∫ ( )
Aliter (b) x cos 2 x dx = x 2 cos2 x − 1 dx
Way 3 for cos 2x in
∫ x cos 2x dx M1


⇒ 2 x cos2 x dx −
∫ x dx = 1
2
x sin 2 x + 1
4
cos 2 x + c

1
1⎛ 1 ⎞ 1 ( their answer to (a)) ;
∫ ∫
1
⇒ x cos x dx = ⎜ x sin 2 x + cos 2 x ⎟ ; +
2
x dx 2 A1;
2⎝2 4 ⎠ 2 or underlined expression

1 1 1 Completely correct expression


= x sin 2 x + cos 2 x + x 2 ( + c ) A1
4 8 4 with/without +c
[3]

7 marks

6666/01 Core Maths C4 6 25th June 2007


June 2007 Advanced Subsidiary/Advanced Level in GCE Mathematics Version 8: THE FINAL VERSION
PhysicsAndMathsTutor.com

Question
Scheme Marks
Number

4. (a) A method of long division gives,


Way 1
2(4 x 2 + 1) 4
≡ 2+ A=2 B1
(2x + 1)(2 x − 1) (2 x + 1)(2x − 1)

4 B C
≡ +
(2 x + 1)(2 x − 1) (2 x + 1) (2 x − 1)

4 ≡ B(2 x − 1) + C (2 x + 1) Forming any one of these two


M1
or their remainder, Dx + E ≡ B(2 x − 1) + C (2 x + 1) identities. Can be implied.

Let x = − 21 , 4 = − 2B ⇒ B = −2
See note below
either one of B = − 2 or C = 2 A1
Let x = 21 , 4 = 2C ⇒ C =2
both B and C correct A1
[4]

Aliter
2(4 x 2 + 1) B C
4. (a) ≡ A+ +
(2 x + 1)(2 x − 1) (2 x + 1) (2 x − 1)
Way 2
decide to award B1 here!! …
See below for the award of B1 … for A = 2 B1

2(4 x 2 + 1) ≡ A(2 x + 1)(2 x − 1) + B(2 x − 1) + C (2 x + 1) Forming this identity.


M1
Can be implied.

Equate x2, 8 = 4A ⇒ A=2

Let x = − 21 , 4 = − 2B ⇒ B = −2
See note below
either one of B = − 2 or C = 2 A1
Let x = 21 , 4 = 2C ⇒ C =2
both B and C correct A1
[4]

If a candidate states one of either


B or C correctly then the method
mark M1 can be implied.

6666/01 Core Maths C4 7 25th June 2007


June 2007 Advanced Subsidiary/Advanced Level in GCE Mathematics Version 8: THE FINAL VERSION
PhysicsAndMathsTutor.com

Question
Scheme Marks
Number

2(4 x 2 + 1)
∫ ∫ 2 − (2x + 1) + (2x − 1) dx
2 2
4. (b) dx =
(2 x + 1)(2 x − 1)

= 2 x − 22 ln(2 x + 1) + 22 ln(2 x − 1) ( + c ) Either p ln(2 x + 1) or q ln(2 x − 1)


M1 ∗
or either p ln 2 x + 1 or q ln 2 x − 1
A → Ax B1
− ln(2 x + 1) + ln(2 x − 1)
2
2
2
2
A1
or − ln(2 x + 1) + ln(2 x − 1) cso & aef
See note below.

2
2(4 x 2 + 1)
∫ dx = [ 2 x − ln(2 x + 1) + ln(2 x − 1)] 1
2

(2 x + 1)(2 x − 1)
1

Substitutes limits of 2 and 1


= ( 4 − ln 5 + ln 3 ) − ( 2 − ln 3 + ln1) and subtracts the correct way round. depM1 ∗
(Invisible brackets okay.)

= 2 + ln 3 + ln 3 − ln 5

⎛ 3(3) ⎞ Use of correct product (or


= 2 + ln ⎜ ⎟ power) and/or quotient laws for
⎝ 5 ⎠ logarithms to obtain a single M1
logarithmic term for their numerical
expression.

⎛9⎞ ⎛9⎞
= 2 + ln ⎜ ⎟ 2 + ln ⎜ ⎟ A1
⎝5⎠ ⎝5⎠
Or 2 − ln ( 59 ) and k stated as 9
5
. [6]

10 marks

Some candidates may find rational values for B To award this M1 mark, the candidate
and C. They may combine the denominator of must use the appropriate law(s) of
their B or C with (2x +1) or (2x – 1). Hence: logarithms for their ln terms to give a
Either b(2ax −1) → k ln(b(2 x − 1)) or one single logarithmic term. Any error
a
b (2 x +1)
→ k ln(b(2 x + 1)) is okay for M1. in applying the laws of logarithms
would then earn M0.

Candidates are not allowed to fluke


− ln(2 x + 1) + ln(2 x − 1) for A1. Hence cso. If they
do fluke this, however, they can gain the final A1 Note: This is not a
mark for this part of the question. dependent method mark.

6666/01 Core Maths C4 8 25th June 2007


June 2007 Advanced Subsidiary/Advanced Level in GCE Mathematics Version 8: THE FINAL VERSION
PhysicsAndMathsTutor.com

Question
Scheme Marks
Number

5. (a) If l1 and l2 intersect then:

⎛1⎞ ⎛ 1⎞ ⎛ 1⎞ ⎛2⎞
⎜ ⎟ ⎜ ⎟ ⎜ ⎟ ⎜ ⎟
⎜ 0 ⎟ + λ ⎜ 1⎟ = ⎜ 3 ⎟ + µ ⎜ 1 ⎟
⎜ −1⎟ ⎜0⎟ ⎜6⎟ ⎜ −1⎟
⎝ ⎠ ⎝ ⎠ ⎝ ⎠ ⎝ ⎠

i : 1 + λ = 1 + 2µ (1)
Writes down any two of these equations
Any two of j : λ=3+ µ (2) M1
correctly.
k: −1 = 6 − µ (3)

(1) & (2) yields λ = 6, µ = 3 Solves two of the above equations to


find …
(1) & (3) yields λ = 14, µ =7 either one of λ or µ correct A1
(2) & (3) yields λ = 10, µ =7 both λ and µ correct A1

checking eqn (3), -1 ≠ 3 Complete method of putting their values


Either checking eqn (2), 14 ≠ 10 of λ and µ into a third equation to B1
checking eqn (1), 11 ≠ 15 show a contradiction.

or for example:
this type of explanation is also allowed
checking eqn (3), LHS = -1 , RHS = 3 for B1 .
⇒ Lines l1 and l2 do not intersect
[4]

Aliter
5. (a) k : −1 = 6 − µ ⇒ µ=7
Uses the k component to find µ
Way 2
i: 1 + λ = 1 + 2µ ⇒ 1 + λ = 1 + 2(7) and substitutes their value of µ
into either one of the i or j component. M1
j: λ=3+ µ ⇒ λ = 3 + (7)

i: λ = 14 either one of the λ ’s correct A1


j: λ = 10 both of the λ ’s correct A1

Either: These equations are then inconsistent


Or: 14 ≠ 10 Complete method giving rise to any one
of these three explanations. B1
Or: Lines l1 and l2 do not intersect
[4]

6666/01 Core Maths C4 9 25th June 2007


June 2007 Advanced Subsidiary/Advanced Level in GCE Mathematics Version 8: THE FINAL VERSION
PhysicsAndMathsTutor.com

Question
Scheme Marks
Number
Aliter
5. (a) If l1 and l2 intersect then:
Way 3
⎛1⎞ ⎛ 1⎞ ⎛ 1⎞ ⎛2⎞
⎜ ⎟ ⎜ ⎟ ⎜ ⎟ ⎜ ⎟
⎜ 0 ⎟ + λ ⎜ 1⎟ = ⎜ 3 ⎟ + µ ⎜ 1 ⎟
⎜ −1⎟ ⎜0⎟ ⎜6⎟ ⎜ −1⎟
⎝ ⎠ ⎝ ⎠ ⎝ ⎠ ⎝ ⎠

i : 1 + λ = 1 + 2µ (1)
Any two of j : λ=3+ µ (2) Writes down any two of these equations M1
k: −1 = 6 − µ (3)

(1) & (2) yields µ = 3 either one of the µ ’s correct A1


(3) yields µ = 7 both of the µ ’s correct A1

Either: These equations are then inconsistent


Or: 3≠ 7 Complete method giving rise to any one
of these three explanations. B1
Or: Lines l1 and l2 do not intersect
[4]

Aliter
i : 1 + λ = 1 + 2µ (1)
5. (a) Any two of j : λ=3+ µ (2) Writes down any two of these equations M1
Way 4 k: −1 = 6 − µ (3)

(1) & (2) yields µ = 3 µ =3 A1


(3) RHS = 6 − 3 = 3 RHS of (3) = 3 A1

(3) yields −1≠ 3 Complete method giving rise to this


explanation. B1
[4]

6666/01 Core Maths C4 10 25th June 2007


June 2007 Advanced Subsidiary/Advanced Level in GCE Mathematics Version 8: THE FINAL VERSION
PhysicsAndMathsTutor.com

Question
Scheme Marks
Number

Only one of either


⎛2⎞ ⎛5⎞
⎛2⎞ ⎛5⎞ uuur ⎜ ⎟ uuur ⎜ ⎟
uuur ⎜ ⎟ uuur ⎜ ⎟ OA = ⎜ 1 ⎟ or OB = ⎜ 5 ⎟ or
5. (b) λ = 1 ⇒ OA = ⎜ 1 ⎟ & µ = 2 ⇒ OB = ⎜ 5 ⎟ ⎜ ⎟ ⎜ 4⎟ B1
⎜ −1⎟ ⎜ 4⎟ ⎝ −1⎠ ⎝ ⎠
⎝ ⎠ ⎝ ⎠
A(2,1, − 1) or B(5,5, 4) .
(can be implied)

⎛5⎞ ⎛ 2 ⎞ ⎛3⎞ ⎛ −3 ⎞
uuur uuur uuur ⎜ ⎟ ⎜ ⎟ ⎜ ⎟ uuur ⎜ ⎟ Finding the difference between their
uuur uuur
AB = OB − OA = ⎜ 5 ⎟ − ⎜ 1 ⎟ = ⎜ 4 ⎟ or BA = ⎜ −4 ⎟ OB and OA . M1
⎜ 4 ⎟ ⎜ −1⎟ ⎜ 5 ⎟ ⎜ ⎟
⎝ ⎠ ⎝ ⎠ ⎝ ⎠ ⎝ −5 ⎠ (can be implied)

Applying the dot product formula


between “allowable” vectors. See M1
notes below.
uuur
AB = 3 i + 4 j + 5k , d 1 = i + j + 0k & θ is angle

uuur Applies dot product formula between


AB • d 1 ⎛3 + 4 + 0⎞ uuur
cos θ = uuur = ± ⎜⎜ ⎟⎟ d 1 and their ± AB. M1
AB . d 1 ⎝ 50 . 2 ⎠
Correct expression. A1

cos θ = 7
10
7
10 or 0.7 or 7
100 A1 cao
but not 7 [6]
50 2

10 marks

Candidates can score this mark if there is a complete method for finding the
dot product between their vectors in the following cases:
uuur
Case 1: their ft ± AB = ± ( 3 i + 4 j + 5k ) Case 2: d 1 = i + j + 0k Case 3: d 1 = i + j + 0k
and d 1 = i + j + 0k and d 2 = 2i + j − 1k and d 2 = 2(2i + j − 1k )
⎛3 + 4 + 0⎞ 2 + 1+ 0 4+2+0
⇒ cos θ = ± ⎜⎜ ⇒ cos θ = ⇒ cos θ =
⎟⎟ 2. 6 2 . 24
⎝ 50 . 2 ⎠

uuur uuur
Case 4: their ft ± AB = ± ( 3 i + 4 j + 5k ) Case 5: their ft OA = 2 i + 1j − 1k
uuur
and d 2 = 2i + j − k and their ft OB = 5 i + 5 j + 4k
⎛6 + 4 − 5⎞ ⎛ 10 + 5 − 4 ⎞
⇒ cos θ = ± ⎜⎜ ⎟⎟ ⇒ cos θ = ± ⎜⎜ ⎟⎟
⎝ 50 . 6 ⎠ ⎝ 6 . 66 ⎠
Note: If candidate use cases 2, 3, 4 and 5 they cannot gain the final three marks for this part.
Note: Candidate can only gain some/all of the final three marks if they use case 1.

6666/01 Core Maths C4 11 25th June 2007


June 2007 Advanced Subsidiary/Advanced Level in GCE Mathematics Version 8: THE FINAL VERSION
PhysicsAndMathsTutor.com

Examples of awarding of marks M1M1A1 in 5.(b)

Example Marks

50 . 2 cos θ = ± ( 3 + 4 + 0 ) M1M1A1
(Case 1)

2 . 6 cos θ = 3 M1M0A0
(Case 2)

2 . 24 cos θ = 4 + 2 M1M0A0
(Case 3)

6666/01 Core Maths C4 12 25th June 2007


June 2007 Advanced Subsidiary/Advanced Level in GCE Mathematics Version 8: THE FINAL VERSION
PhysicsAndMathsTutor.com

Question
Scheme Marks
Number

6. (a) x = tan2 t , y = sin t

dx dy dx dy
= 2(tan t ) sec 2 t , = cos t Correct and B1
dt dt dt dt

± cos t
M1
dy cos t ⎛ cos t ⎞ 4
their ddxt
∴ = ⎜= ⎟
dx 2 tan t sec 2 t ⎝ 2 sin t ⎠ + cos t
A1
their ddxt
[3]

(b) When t = π4 , x =1, y = 1


2
(need values) (
The point 1, 1
2 ) or (1, awrt 0.71) B1, B1
These coordinates can be implied.
( y = sin ( π4 ) is not sufficient for B1)
π dy cos π4
When t = , m(T) = =
4 dx 2 tan π4 sec 2 π
4

1 1 1
1 2
= 2
= 2
= 2
= =
( ) ()
2 any of the five underlined expressions or
2.(1) 1 2.(1)(2) 4 2 8 B1 aef
2.(1) 1
1
1
2
awrt 0.18
2

Finding an equation of a tangent with


( x − 1)
their point and their tangent gradient or
T: y − 1
2
= 1
4 2 finds c by using
M1 aef
y = (their gradient)x + " c " .

T: y = 1
4 2
x+ 3
4 2
or y= 8
2
x + 3 82 Correct simplified
A1 aef cso
EXACT equation of tangent

or 1
2
= 1
4 2
(1) + c ⇒ c = 1
2
− 1
4 2
= 3
4 2

Hence T: y = 1
4 2
x+ 3
4 2
or y= 8
2
x+ 3 2
8

[5]

A candidate who incorrectly differentiates tan2 t to give


Note: The x and y coordinates
must be the right way round.
dx
dt
= 2 sec 2 t or ddxt = sec 4 t is then able to fluke the
correct answer in part (b). Such candidates can
potentially get: (a) B0M1A1 (b) B1B1B1M1A0 cso.
Note: cso means “correct solution only”.
Note: part (a) not fully correct implies candidate can
achieve a maximum of 4 out of 5 marks in part (b).

6666/01 Core Maths C4 13 25th June 2007


June 2007 Advanced Subsidiary/Advanced Level in GCE Mathematics Version 8: THE FINAL VERSION
PhysicsAndMathsTutor.com

Question
Scheme Marks
Number

sin2 t
6. (c) x = tan2 t = y = sin t
cos2 t
Way 1
sin2 t
x= Uses cos2 t = 1 − sin2 t M1
1 − sin2 t

y2 Eliminates ‘t’ to write an equation


x= M1
1− y 2 involving x and y.

x (1 − y 2 ) = y 2 ⇒ x − xy 2 = y 2

Rearranging and factorising with an


x = y 2 + xy 2 ⇒ x = y 2 (1 + x ) ddM1
attempt to make y 2 the subject.

x x
y2 = A1
1+ x 1+ x
[4]
Aliter
6. (c) 1 + cot 2 t = co sec 2 t Uses 1 + cot 2 t = co sec 2 t M1
Way 2
1 1
= Uses cos ec 2 t = M1 implied
sin2 t sin2 t

1 1 Eliminates ‘t’ to write an equation


Hence, 1+ = 2 ddM1
x y involving x and y.

1 x 1 x
Hence, y 2 = 1 − or 1− or A1
(1 + x ) 1+ x (1 + x ) 1+ x
[4]

1
is an acceptable response for the final accuracy A1 mark.
1+ x1

6666/01 Core Maths C4 14 25th June 2007


June 2007 Advanced Subsidiary/Advanced Level in GCE Mathematics Version 8: THE FINAL VERSION
PhysicsAndMathsTutor.com

Question
Scheme Marks
Number
Aliter
6. (c) x = tan2 t y = sin t
Way 3
1 + tan2 t = sec 2 t Uses 1 + tan2 t = sec 2 t M1

1 1
= Uses sec 2 t = M1
cos2 t cos2 t

1
=
1 − sin2 t

1 Eliminates ‘t’ to write an equation


Hence, 1+ x = ddM1
1− y 2 involving x and y.

1 x 1 x
Hence, y 2 = 1 − or 1− or A1
(1 + x ) 1+ x (1 + x ) 1+ x

[4]
Aliter
6. (c) y 2 = sin2 t = 1 − cos 2 t Uses sin2 t = 1 − cos2 t M1
Way 4
1 1
= 1− Uses cos2 t = M1
sec 2 t sec 2 t

1
= 1− then uses sec 2 t = 1 + tan2 t ddM1
(1 + tan2 t )

1 x 1 x
Hence, y 2 = 1 − or 1− or A1
(1 + x ) 1+ x (1 + x ) 1+ x
[4]

1
is an acceptable response for the final accuracy A1 mark.
1+ x1

6666/01 Core Maths C4 15 25th June 2007


June 2007 Advanced Subsidiary/Advanced Level in GCE Mathematics Version 8: THE FINAL VERSION
PhysicsAndMathsTutor.com

Question
Scheme Marks
Number
Aliter
6. (c) x = tan2 t y = sin t
Way 5
x = tan2 t ⇒ tan t = x

M1
Draws a right-angled triangle and places
(1 + x ) both x and 1 on the triangle
x

t Uses Pythagoras to deduce the


1 hypotenuse
M1

x Eliminates ‘t’ to write an equation


Hence, y = sin t = ddM1
1+ x involving x and y.

x x
Hence, y 2 = A1
1+ x 1+ x
[4]

12 marks

1
is an acceptable response for the final accuracy A1 mark.
1+ x1

There are so many ways that a candidate can proceed with part (c). If a candidate produces a correct
solution then please award all four marks. If they use a method commensurate with the five ways as
detailed on the mark scheme then award the marks appropriately. If you are unsure of how to apply the
scheme please escalate your response up to your team leader.

6666/01 Core Maths C4 16 25th June 2007


June 2007 Advanced Subsidiary/Advanced Level in GCE Mathematics Version 8: THE FINAL VERSION
PhysicsAndMathsTutor.com

Question
Scheme Marks
Number

π π 3π π
x 0 16 8 16 4
7. (a)
y 0 0.445995927… 0.643594252… 0.817421946… 1

Enter marks into ePEN in the correct order. 0.446 or awrt 0.44600 B1
awrt 0.64359 B1
awrt 0.81742 B1
0 can be [3]
implied
Outside brackets
1
2
× 16π or 32
π B1
For structure of trapezium
1 π
(b)
Way 1
Area ≈ ×
2 16
{
; × 0 + 2 ( 0.44600 + 0.64359 + 0.81742 ) + 1} { }
rule ............. ;
M1
Correct expression
inside brackets which all must
be multiplied by h2 . A1

π
= × 4.81402... = 0.472615308... = 0.4726 (4dp) for seeing 0.4726 A1 cao
32
[4]

π B1
and a divisor of 2 on all
{ }
0 + 0.44600 0.44600 + 0.64359 0.64359 + 0.81742 0.81742 + 1 16
Area ≈ π
16
× 2
+ 2
+ 2
+ 2 terms inside brackets.
One of first and last ordinates,
Aliter
two of the middle ordinates
(b) M1
which is equivalent to: inside brackets ignoring the 2.
Way 2
Correct expression inside
1 π brackets if 21 was to be
Area ≈ ×
2 16
{
; × 0 + 2 ( 0.44600 + 0.64359 + 0.81742 ) + 1} factorised out.
A1

π
= × 2.40701... = 0.472615308... = 0.4726 0.4726 A1 cao
16
[4]

Area = 1
2
× π
20
× {0 + 2(0.44600 + 0.64359 + 0.81742) + 1} = 0.3781, gains B0M1A1A0

In (a) for x = π
16
writing 0.4459959… then 0.45600 gains B1 for awrt 0.44600 even though 0.45600 is incorrect.

In (b) you can follow though a candidate’s values from part (a) to award M1 ft, A1 ft

Question
Scheme Marks
Number

6666/01 Core Maths C4 17 25th June 2007


June 2007 Advanced Subsidiary/Advanced Level in GCE Mathematics Version 8: THE FINAL VERSION
PhysicsAndMathsTutor.com

∫( ) ∫ tan x dx
2
π π
tan x dx or

∫( )
4 4

Volume = (π ) (π ) ∫ tan x dx
2
7. (c) tan x dx = Can be implied. M1
0 0
Ignore limits and (π )

tan x → ln sec x
= (π ) [ln sec x ] 04 or = (π ) [ − ln cos x ] 04
π π
A1
or tan x → − ln cos x

The correct use of limits on a


= (π ) ⎡⎣( ln sec π4 ) − ( ln sec 0 ) ⎤⎦ function other than tan x; ie
x = π4 ‘minus’ x = 0 .
or dM1
= (π ) ⎡⎣( − ln cos 4 ) − ( ln cos 0 ) ⎤⎦
π ln(sec 0) = 0 may be
implied. Ignore (π )

= π ⎡ln
⎢⎣ ( ) − ln ( )⎤⎥⎦ = π ⎡⎣ln
1
1
2
1
1
2 − ln1⎤

or
= π ⎡ − ln
⎣ ( ) − ln (1)⎤⎦
1
2

π ln 2 or π ln 2
2

= π ln 2 or π ln 2
2
or 1
2
π ln 2 or −π ln ( ) or
1
2
π
2
ln ( 21 ) or 1
2
π ln 2 or −π ln ( )1
2
A1 aef

or π
2
ln ( 21 )
must be exact. [4]

11 marks

If a candidate gives the correct exact answer and then writes 1.088779…, then such a candidate can
be awarded A1 (aef). The subsequent working would then be ignored. (isw)

Beware: In part (c) the factor of π is not needed for the first three marks.

Beware: In part (b) a candidate can also add up individual trapezia in this way:

( )
Area ≈ 21 . 16π ( 0 + 0.44600 ) + 21 . 16π ( 0.44600 + 0.64359 ) + 21 . 16π 0.64359 + 0.81742 + 21 . 16π ( 0.81742 + 1)

6666/01 Core Maths C4 18 25th June 2007


June 2007 Advanced Subsidiary/Advanced Level in GCE Mathematics Version 8: THE FINAL VERSION
PhysicsAndMathsTutor.com

Question
Scheme Marks
Number

dP
8. (a) = kP and t = 0, P = P0 (1)
dt

Separates the variables with

∫P ∫ k dt on either
dP
∫ ∫
dP and
= k dt M1
P
side with integral signs not
necessary.

Must see ln P and kt ;


ln P = kt ; ( + c ) Correct equation with/without + A1
c.

When t = 0, P = P0 ⇒ ln P0 = c Use of boundary condition (1)

( or )
to attempt to find the constant of M1
P = Ae kt ⇒ P0 = A integration.

ln P = kt + ln P0 ⇒ eln P = e kt + ln P0 = e kt .eln P0

Hence, P = P0 e kt P = P0 e kt A1
[4]

Substitutes P = 2P0 into an


(b) P = 2P0 & k = 2.5 ⇒ 2P0 = P0 e 2.5 t M1
expression involving P

e 2.5t = 2 ⇒ ln e 2.5t = ln 2 or 2.5t = ln 2 Eliminates P0 and takes


M1
…or e kt
= 2 ⇒ ln e kt
= ln 2 or kt = ln 2 ln of both sides

⇒ t = 1
2.5
ln 2 = 0.277258872... days

t = 0.277258872... × 24 × 60 = 399.252776... minutes

awrt t = 399 or
t = 399min or t = 6 hr 39 mins (to nearest minute) A1
6 hr 39 mins
[3]

P = P0 e kt written down without the first M1 mark given scores all four marks in part (a).

6666/01 Core Maths C4 19 25th June 2007


June 2007 Advanced Subsidiary/Advanced Level in GCE Mathematics Version 8: THE FINAL VERSION
PhysicsAndMathsTutor.com

Question
Scheme Marks
Number

dP
8. (c) = λP cos λ t and t = 0, P = P0 (1)
dt

Separates the variables with

∫P ∫ λ cos λt dt on
dP
∫ ∫
dP and
= λ cos λ t dt M1
P
either side with integral signs
not necessary.

Must see ln P and sin λ t ;


ln P = sin λt ; ( + c ) Correct equation with/without + A1
c.

When t = 0, P = P0 ⇒ ln P0 = c Use of boundary condition (1)

( or )
to attempt to find the constant of M1
P = Ae sin λt ⇒ P0 = A integration.

ln P = sin λ t + ln P0 ⇒ eln P = e sin λt + ln P0 = e sin λt .eln P0

Hence, P = P0 e sin λt P = P0 e sin λt A1


[4]

(d) P = 2P0 & λ = 2.5 ⇒ 2P0 = P0 e sin2.5 t

e sin2.5t = 2 ⇒ sin 2.5t = ln 2 Eliminates P0 and makes


sin λ t or sin 2.5t the subject M1
…or … e λt = 2 ⇒ sin λ t = ln 2
by taking ln’s

t = 1
2.5
sin−1 ( ln 2 ) Then rearranges
dM1
to make t the subject.
(must use sin-1)
t = 0.306338477...

t = 0.306338477... × 24 × 60 = 441.1274082... minutes

awrt t = 441 or
t = 441min or t = 7 hr 21 mins (to nearest minute) A1
7 hr 21 mins
[3]

14 marks

P = P0 e sin λt written down without the first M1 mark given scores all four marks in part (c).

6666/01 Core Maths C4 20 25th June 2007


June 2007 Advanced Subsidiary/Advanced Level in GCE Mathematics Version 8: THE FINAL VERSION
PhysicsAndMathsTutor.com

Question
Scheme Marks
Number

dP
= kP and t = 0, P = P0 (1)
dt

Separates the variables with

∫ kP ∫ dt on either side
Aliter dP
∫ ∫
dP and
8. (a) = 1 dt M1
Way 2 kP
with integral signs not
necessary.

1
Must see ln P and t ;
ln P = t ; ( + c )
k
1
k Correct equation with/without + A1
c.

When t = 0, P = P0 ⇒ 1
k
ln P0 = c Use of boundary condition (1)

( or )
to attempt to find the constant of M1
P = Ae kt ⇒ P0 = A integration.

1
k
ln P = t + k1 ln P0 ⇒ ln P = kt + ln P0
⇒ eln P = e kt + ln P0 = e kt .eln P0

Hence, P = P0 e kt P = P0 e kt A1
[4]

Separates the variables with

∫ ∫ dt on either side
Aliter dP
∫ ∫ 1 dt
dP and
8. (a) = kP M1
Way 3 kP
with integral signs not
necessary.

Must see 1
ln ( kP ) and t ;
ln ( kP ) = t ; ( + c )
k
1
k Correct equation with/without + A1
c.

t = 0, P = P0 ⇒ k1 ln ( kP0 ) = c Use of boundary condition (1)


When
( or )
to attempt to find the constant of M1
kP = Ae kt ⇒ kP0 = A integration.

1
k
ln ( kP ) = t + k1 ln ( kP0 ) ⇒ ln ( kP ) = kt + ln ( kP0 )
ln( kP ) kt + ln( kP0 ) ln( kP0 )
⇒e =e = e kt .e
⇒ kP = e kt . ( kP0 ) ⇒ kP = kP0 e kt
( or kP = kP0 e kt )
Hence, P = P0 e kt P = P0 e kt A1
[4]

Question
Scheme Marks
Number

6666/01 Core Maths C4 21 25th June 2007


June 2007 Advanced Subsidiary/Advanced Level in GCE Mathematics Version 8: THE FINAL VERSION
PhysicsAndMathsTutor.com

dP
= λ P cos λ t and t = 0, P = P0 (1)
dt

Separates the variables with

∫ λP and ∫ cos λt dt on
Aliter dP
∫ ∫
dP
8. (c) = cos λ t dt M1
Way 2 λP
either side with integral signs
not necessary.

Must see λ
1
ln P and 1
λ sin λ t ;
λ ln P = λ sin λ t ; ( + c )
1 1
Correct equation with/without + A1
c.

When t = 0, P = P0 ⇒ 1
λ ln P0 = c Use of boundary condition (1)

( or )
sin λ t to attempt to find the constant of M1
P = Ae ⇒ P0 = A integration.

1
λ ln P = 1
λ sin λ t + λ1 ln P0 ⇒ ln P = sin λ t + ln P0

⇒ eln P = e sin λt + ln P0 = e sin λt .eln P0

Hence, P = P0 e sin λt P = P0 e sin λt A1


[4]

P = P0 e kt written down without the first M1 mark given scores all four marks in part (a).

P = P0 e sin λt written down without the first M1 mark given scores all four marks in part (c).

6666/01 Core Maths C4 22 25th June 2007


June 2007 Advanced Subsidiary/Advanced Level in GCE Mathematics Version 8: THE FINAL VERSION
PhysicsAndMathsTutor.com

Question
Scheme Marks
Number

dP
= λ P cos λ t and t = 0, P = P0 (1)
dt

Separates the variables with

∫ λP and ∫ cos λt dt on
Aliter dP
∫ ∫
dP
8. (c) = cos λ t dt M1
Way 3 λP
either side with integral signs
not necessary.

Must see 1
λ ln ( λ P ) and
1
ln ( λ P ) = 1
sin λ t ; ( + c )
1
λ sin λ t ; A1
λ λ
Correct equation with/without +
c.

When t = 0, P = P0 ⇒ 1
λ ln ( λ P0 ) = c Use of boundary condition (1)

( or λ P = Ae sin λt ⇒ λP0 = A )
to attempt to find the constant of M1
integration.

1
λ ln ( λ P ) = λ
1
sin λ t + λ1 ln ( λ P0 )

⇒ ln ( λ P ) = sin λ t + ln ( λ P0 )

ln( λ P ) sin λ t + ln( λ P0 ) ln( λ P0 )


⇒e =e = e sin λt .e

⇒ λ P = e sin λt . ( λ P0 )
( or λ P = λ P0 e sin λt )

Hence, P = P0 e sin λt P = P0 e sin λt A1


[4]

• Note: dM1 denotes a method mark which is dependent upon the award of the previous method mark.
ddM1 denotes a method mark which is dependent upon the award of the previous two method marks.
depM1 ∗ denotes a method mark which is dependent upon the award of M1∗ .
ft denotes “follow through”
cao denotes “correct answer only”
aef denotes “any equivalent form”

6666/01 Core Maths C4 23 25th June 2007


June 2007 Advanced Subsidiary/Advanced Level in GCE Mathematics Version 8: THE FINAL VERSION
PhysicsAndMathsTutor.com

Mark Scheme (Results)


January 2008

GCE

GCE Mathematics (6666/01)

Edexcel Limited. Registered in England and Wales No. 4496750


1
Registered Office: One90 High Holborn, London WC1V 7BH
PhysicsAndMathsTutor.com

January 2008
6666 Core Mathematics C4
Mark Scheme
Question
Number
Scheme Marks

x 0 π
4
π
2

4 π
1. (a)
y 0 1.844321332… 4.810477381… 8.87207 0

awrt 1.84432 B1
awrt 4.81048 or 4.81047 B1
0 can be [2]
implied
Outside brackets
awrt 0.39 or 12 × awrt 0.79 B1
1
2
× π4 or π8
1 π For structure of trapezium
(b)
2 4
{
Area ≈ × ;× 0 + 2 (1.84432 + 4.81048 + 8.87207 ) + 0} rule {.............} ; M1
Way 1
Correct expression
inside brackets which all must
be multiplied by their “outside A1
constant”.

π
= × 31.05374... = 12.19477518... = 12.1948 (4dp) 12.1948 A1 cao
8
[4]

π
(or awrt 0.79 ) and a divisor
{ }
4
0 + 1.84432 1.84432 + 4.81048 4.81048 + 8.87207 8.87207 + 0
Area ≈ π4 × 2
+ 2
+ 2
+ 2 of 2 on all terms inside B1
brackets.
Aliter One of first and last ordinates,
(b) which is equivalent to: two of the middle ordinates M1
Way 2 inside brackets ignoring the 2.
1 π Correct expression inside
2 4
{
Area ≈ × ;× 0 + 2 (1.84432 + 4.81048 + 8.87207 ) + 0} brackets if 12 was to be A1
factorised out.

π
= × 15.52687... = 12.19477518... = 12.1948 (4dp) 12.1948 A1 cao
4
[4]

6 marks

1 π
Note an expression like Area ≈ × + 2 (1.84432 + 4.81048 + 8.87207 ) would score B1M1A0A0
2 4

2
PhysicsAndMathsTutor.com

Question
Scheme Marks
Number
** represents a constant
(which must be consistent for first accuracy mark)
1 1 Takes 8 outside the
⎛ 3x ⎞ 3 ⎛ 3x ⎞ 3
(8 − 3x ) = ( 8 ) ⎜1 − ⎟ = 2 ⎜1 − ⎟ bracket to give any of B1
1 1
2. (a) 3 3

⎝ 8 ⎠ ⎝ 8 ⎠ 1
(8) 3 or 2 .

1
Expands (1 + ** x) 3 to
give a simplified or an M1;
un-simplified
1 + ( 13 )(** x) ;
⎧ ( 1 )(− 23 ) ( 1 )( − 32 )( − 53 ) ⎫
= 2 ⎨1 + ( 13 )(** x); + 3 (** x) 2 + 3 (** x)3 + ...⎬ A correct simplified or an
⎩ 2! 3! ⎭ un-simplified
{..........} expansion with A1
with ** ≠ 1
candidate’s followed
through (** x )

⎧ ( 1 )(− 23 ) 3 x 2 ( 13 )(− 23 )(− 53 ) 3 x 3 ⎫ Award SC M1 if you see


= 2 ⎨1 + ( 13 )(− 38x ) + 3 (− 8 ) + (− 8 ) + ...⎬ ( 13 )(− 23 ) ( 1 )(− 32 )(− 53 )
(** x) 2 + 3 (** x)3
⎩ 2! 3! ⎭ 2! 3!

= 2{1 − 18 x ; − 1
64 x2 − 5
1536 x3 − ...} Either 2{1 − 18 x ........} or
anything that A1;
1 1 2 5 3 cancels to 2 − 1 x ;
= 2 − x; − x − x − ... 4
4 32 768
Simplified − 1
32
x −
2 5
768
x3 A1
[5]

Attempt to substitute
1 1 1 5
(b) (7.7) 3 ≈ 2 − (0.1) − (0.1) 2 − (0.1)3 − ... x = 0.1 into a candidate’s M1
4 32 768 binomial expansion.

= 2 − 0.025 − 0.0003125 − 0.0000065104166...

= 1.97468099... awrt 1.9746810 A1


[2]

7 marks

You would award B1M1A0 for


If you see the constant term “2” in a
⎧ ( 1 )(− 2 ) (1 )(− 2 )(− 5 ) ⎫
= 2⎨1 +(13)(− 38x ) + 3 3 (− 38x )2 + 3 3 3 (−3x)3 + ...⎬ candidate’s final binomial expansion,
⎩ 2! 3! ⎭ then you can award B1.
because ** is not consistent.

Be wary of calculator value of ( 7.7 ) 3 = 1.974680822...


1

3
PhysicsAndMathsTutor.com

Question
Scheme Marks
Number
Aliter
1
2. (a) (8 − 3 x) 3
Way 2
1
2 or (8) 3 (See note ↓ ) B1
1
Expands (8 − 3 x) 3 to give
⎧ 1 −2 ( 1 )(− 23 ) − 35 ⎫ an un-simplified or
⎪(8) 3 + ( 13 )(8) 3 (** x); + 3 (8) (** x) 2 ⎪ M1;
⎪ 2! ⎪ simplified
=⎨ ⎬ −2
( 1 )( − 32 )(− 53 ) − 83
1

⎪ (8) 3 + ( 13 )(8) 3 (** x);


+ 3 (8) (** x)3 + ...⎪
⎪⎩ 3! ⎪⎭ A correct un-simplified
or simplified
with ** ≠ 1 {..........} expansion with A1
candidate’s followed
through (** x )

⎧ 1 −2 ( 1 )(− 23 ) − 53 ⎫
⎪(8) 3 + ( 13 )(8) 3 (−3x); + 3 (8) ( −3x) 2 ⎪ Award SC M1 if you see
⎪ 2! ⎪ ( 13 )(− 23 ) − 53
=⎨ ⎬ 2!
(8) (** x) 2

⎪ ( 3 )(− 3 )(− 3 ) − 3
1 2 5
(8) (−3 x) + ...⎪
8
( 1 )(− 23 )(− 53 ) − 83
+ 3
+ 3 (8) (** x)3
⎪⎩ 3! ⎪⎭ 3!

= {2 + ( 13 )( 14 )(−3x) + (− 91 )( 321 )(9 x 2 ) + ( 815 )( 256


1
)(−27 x 3 ) + ...}

Anything that
cancels to 2 − 1 x ;
1 1 2 5 3 4 A1;
=2− x; − x − x − ...
4 32 768 or 2{1 − x ........}
1
8

Simplified − 321 x 2 − 5
768
x3 A1
[5]

Attempts using Maclaurin expansion should be escalated up to your team leader.

If you see the constant term “2” in a


Be wary of calculator value of ( 7.7 ) 3 = 1.974680822...
1
candidate’s final binomial expansion,
then you can award B1.

4
PhysicsAndMathsTutor.com

Question
Scheme Marks
Number

Use of V = π ∫ y 2 dx .
2
⎛ 1 ⎞
∫ ∫
b b
1
3. Volume = π ⎜ ⎟ dx = π dx B1
a ⎝ 2x + 1 ⎠ ( 2 x + 1)
2
a
Can be implied. Ignore limits.

∫ ( 2 x + 1)
b
−2
=π dx
a

b
⎡ (2 x + 1) −1 ⎤
= (π ) ⎢ ⎥
⎣ (−1)(2) ⎦ a

Integrating to give ± p (2 x + 1) −1 M1
= (π ) ⎡⎣ − 12 (2 x + 1) −1 ⎤⎦
b

a − 12 (2 x + 1) −1 A1

⎡⎛ −1 ⎞ ⎛ −1 ⎞ ⎤
= ( π ) ⎢⎜ ⎟−⎜ ⎟⎥
Substitutes limits of b and a and
dM1
⎣⎝ 2(2b + 1) ⎠ ⎝ 2(2a + 1) ⎠ ⎦ subtracts the correct way round.

π ⎡ −2a − 1 + 2b + 1 ⎤
=
2 ⎢⎣ (2a + 1)(2b + 1) ⎥⎦

π⎡ 2(b − a ) ⎤
=
2 ⎣ (2a + 1)(2b + 1) ⎥⎦

π (b − a) π (b − a )
= A1 aef
(2a + 1)(2b + 1) (2a + 1)(2b + 1)
[5]

5 marks

Allow other equivalent forms such as

πb −πa −π (a − b) π (b − a) πb −πa
or or or .
(2a + 1)(2b + 1) (2a + 1)(2b + 1) 4ab + 2a + 2b + 1 4ab + 2a + 2b + 1

Note that π is not required for the middle


three marks of this question.

5
PhysicsAndMathsTutor.com

Question
Scheme Marks
Number

Aliter Use of V = π ∫ y 2 dx .
2
⎛ 1 ⎞
∫ ∫
b b
1
3. Volume = π ⎜ ⎟ dx = π dx B1
a ⎝ 2x + 1 ⎠ ( 2 x + 1)
2
a
Way 2 Can be implied. Ignore limits.

∫ ( 2 x + 1)
b
−2
= π dx
a

Applying substitution u = 2 x + 1 ⇒ ddux = 2 and changing


limits x → u so that a → 2a + 1 and b → 2b + 1 , gives

2 b +1
u −2
= (π )
∫ 2 a +1 2
du

2b + 1
⎡ u −1 ⎤
= (π ) ⎢ ⎥
⎣ (−1)(2) ⎦ 2 a + 1

2b + 1 Integrating to give ± p u −1 M1
= (π ) ⎡⎣ − u 1 −1

⎦ 2a + 1 −1
− u
2 1
2 A1

⎡⎛ −1 ⎞ ⎛ −1 ⎞ ⎤ Substitutes limits of 2b + 1 and


= ( π ) ⎢⎜ ⎟−⎜ ⎟⎥ 2a + 1 and subtracts the correct dM1
⎣⎝ 2(2b + 1) ⎠ ⎝ 2(2a + 1) ⎠ ⎦ way round.

π ⎡ −2a − 1 + 2b + 1 ⎤
=
2 ⎢⎣ (2a + 1)(2b + 1) ⎥⎦

π⎡ 2(b − a ) ⎤
=
2 ⎣ (2a + 1)(2b + 1) ⎥⎦

π (b − a) π (b − a )
= A1 aef
(2a + 1)(2b + 1) (2a + 1)(2b + 1)
[5]

5 marks

Note that π is not required for the middle


three marks of this question.

Allow other equivalent forms such as

πb −πa −π (a − b) π (b − a) πb −πa
or or or .
(2a + 1)(2b + 1) (2a + 1)(2b + 1) 4ab + 2a + 2b + 1 4ab + 2a + 2b + 1

6
PhysicsAndMathsTutor.com

Question
Scheme Marks
Number

⎧ du 1

⎪⎪u = ln ( 2 ) ⇒ = = 1x ⎪
x 2


∫ ln ( ∫
x
dx
4. (i) x
2 ) dx = 1.ln ( 2x ) dx ⇒ ⎨ 2

⎪ dv = 1 ⇒ v =x ⎪
⎪⎩ dx ⎪⎭

Use of ‘integration by parts’

∫ ln ( ) dx = x ln ( ) − ∫ x.
x x 1 formula in the correct M1
2 2 x dx
direction.
Correct expression. A1


= x ln ( 2x ) − 1 dx
An attempt to multiply x by a
candidate’s ax or bx1 or 1x . dM1

= x ln ( 2x ) − x + c Correct integration with + c A1 aef


[4]
π


2

(ii) sin 2 x dx
π
4

⎡ NB : cos 2 x = ±1 ± 2sin 2 x or sin 2 x =



1
2 ( ±1 ± cos 2 x ) ⎤

Consideration of double angle
formula for cos 2x
M1

π π
1 − cos 2 x
∫ ∫
1
(1 − cos 2 x ) dx
2 2
= dx =
π
4
2 2 π
4

Integrating to give
1⎡ π
± ax ± b sin 2 x ; a , b ≠ 0 dM1
= x − 12 sin 2 x ⎤⎦ π
2

2 ⎣ 4
Correct result of anything
equivalent to 12 x − 14 sin 2 x A1

= 12 ⎡
⎢⎣ ( π
2 )
− sin(2π ) − ( π
4

sin
2 )⎥⎦
( π2 ) ⎤
Substitutes limits of π2 and π4
and subtracts the correct way ddM1
round.
= 12 ⎡⎣ ( π2 − 0) − ( π4 − 12 ) ⎤⎦

= 1
2 ( π4 + 12 ) = π8 + 14 1
2 ( π4 + 12 ) or π
8
+ 14 or π
8
+ 82 A1 aef ,
cso
Candidate must collect their [5]
π term and constant term
together for A1
No fluked answers, hence cso.
9 marks

+ 14 = 0.64269...
∫ ∫ ( their v ).( their ddux ) dx
π
Note: ln ( 2x ) dx = (their v)ln ( 2x ) − for M1 in part (i). Note 8

7
PhysicsAndMathsTutor.com

Question
Scheme Marks
Number
Aliter

∫ ln ( ) dx ∫ ( ln x − ln 2) dx = ∫ ln x dx − ∫ ln 2 dx
4. (i) x
=
2
Way 2

⎧ du 1 ⎫
⎪⎪u = ln x ⇒ =
dx x ⎪⎪
∫ ln x dx = ∫ 1.ln x dx ⇒ ⎨
⎪ dv = 1 ⇒ v =x ⎪

⎩⎪ dx ⎭⎪

∫ ln x dx = x ln x − ∫ x. 1 Use of ‘integration by parts’


x dx
formula in the correct M1
direction.

= x ln x − x + c Correct integration of ln x
A1
with or without + c

∫ ln 2 dx = x ln 2 + c
Correct integration of ln 2
M1
with or without + c

Hence,
∫ ln ( x
2 ) dx = x ln x − x − x ln 2 + c Correct integration with + c A1 aef
[4]

Note:
∫ ln x dx = ( their v ) ln x − ∫ ( their v ).( their du
dx ) dx for M1 in part (i).

8
PhysicsAndMathsTutor.com

Question
Scheme Marks
Number
Aliter

∫ ln ( ) dx
4. (i) x
2
Way 3

du 1
u= x
2
⇒ =
dx 2

Applying substitution
correctly to give

∫ ln ( x
2 ) dx ∫
= 2 ln u du
∫ ln ( x
2 ) dx ∫
= 2 ln u du
Decide to award
2nd M1 here!

∫ ln u dx = ∫1.ln u du
Use of ‘integration by parts’
∫ ln u dx = u ln u − u. u1 du
∫ formula in the correct M1
direction.

= u ln u − u + c Correct integration of ln u
A1
with or without + c

Decide to award
M1
2nd M1 here!

∫ ln ( x
2 ) dx = 2 ( u ln u − u ) + c

Hence,
∫ ln ( x
2 ) dx = x ln ( 2x ) − x + c Correct integration with + c A1 aef
[4]

9
PhysicsAndMathsTutor.com

Question
Scheme Marks
Number
Aliter
π π

∫ ∫ ∫
4. (ii) 2
sin x dx = 2
2
sin x.sin x dx and I = sin 2 x dx
Way 2 π
4
π
4

⎪⎧u = sin x ⇒ dx = cos x ⎪⎫


du

⎨ dv ⎬
⎪⎩ dx = sin x ⇒ v = − cos x ⎪⎭

{
∴ I = − sin x cos x + cos 2 x dx
∫ } An attempt to use the correct
by parts formula.
M1

∴ I = − sin x cos x +{ ∫ (1 − sin


2
x ) dx}
∫ sin
2
{ ∫
x dx = − sin x cos x + 1 dx − sin 2 x dx
∫ }
∫ {
2 sin 2 x dx = − sin x cos x + 1 dx
∫ } For the LHS becoming 2I dM1


2 sin 2 x dx = {− sin x cos x + x}

∫ sin
2
{
x dx = − 12 sin x cos x + x
2 } Correct integration A1

( ) − (− )⎤⎥⎦
π

∫ sin 2 x dx = ⎡ − 12 sin( π2 )cos( π2 ) + Substitutes limits of π2 and π4


2 ( π2 ) ( π4 )
∴ 1
sin( π4 ) cos( π4 ) +
π ⎢⎣ 2 2 2
4 and subtracts the correct way ddM1
= ⎡⎣(0 + π4 ) − (− 14 + π8 ) ⎤⎦ round.

= π8 + 14 1
2 ( π4 + 12 ) or π
8
+ 14 or π
8
+ 82 A1 aef
cso
Candidate must collect their [5]
π term and constant term
together for A1
No fluked answers, hence cso.

Note π
8
+ 14 = 0.64269...

10
PhysicsAndMathsTutor.com

Question
Scheme Marks
Number

5. (a) x 3 − 4 y 2 = 12 xy ( eqn ∗ )

x = −8 ⇒ − 512 − 4 y 2 = 12(−8) y Substitutes x = −8 (at least once) into * to


obtain a three term quadratic in y . M1
− 512 − 4 y 2 = − 96 y
Condone the loss of = 0.

4 y 2 − 96 y + 512 = 0
y 2 − 24 y + 128 = 0

( y − 16)( y − 8) = 0
An attempt to solve the quadratic in y by
either factorising or by the formula or by dM1
24 ± 576 − 4(128)
y= completing the square.
2

Both y = 16 and y = 8.
y = 16 or y = 8. A1
or ( −8, 8 ) and ( −8, 16 ).
[3]

Differentiates implicitly to include either


M1
⎧ dy ⎫ dy ⎛ dy ⎞ ± ky ddyx or 12 x ddyx . Ignore ddyx = ...
(b) ⎨ = ⎬ 3x 2 − 8 y ; = ⎜ 12 y + 12 x ⎟
⎩ dx ⎭ dx ⎝ dx ⎠ Correct LHS equation; A1;
Correct application of product rule (B1)

⎧ dy 3x 2 − 12 y ⎫
⎨ = ⎬ not necessarily required.
⎩ dx 12 x + 8 y ⎭

dy 3(64) − 12(8) 96 Substitutes x = −8 and at least one of their


@ ( −8, 8 ) , = = = − 3, dM1
dx 12(−8) + 8(8) −32 y-values to attempt to find any one of ddyx .
dy 3(64) − 12(16) 0
@ ( −8, 16 ) ,
One gradient found. A1
= = = 0.
dx 12(−8) + 8(16) 32 Both gradients of -3 and 0 correctly found. A1 cso
[6]

9 marks

11
PhysicsAndMathsTutor.com

Question
Scheme Marks
Number

Differentiates implicitly to include either


Aliter ⎧⎪ dx ⎫⎪ dx ⎛ dx ⎞ ± kx 2 ddyx or 12 y ddyx . Ignore ddxy = ... M1
5. (b) ⎨ = ⎬ 3x 2 − 8 y ; = ⎜12 y + 12 x ⎟
⎪⎩ dy ⎪⎭ dy ⎜ dy ⎟ Correct LHS equation A1;
Way 2 ⎝ ⎠
Correct application of product rule (B1)

⎧ dy 3x 2 − 12 y ⎫
⎨ = ⎬ not necessarily required.
⎩ dx 12 x + 8 y ⎭

Substitutes x = −8 and at least one of their


dy 3(64) − 12(8) 96
@ ( −8, 8 ) , = = = − 3, y-values to attempt to find any dM1
dx 12(−8) + 8(8) −32 one of ddyx or ddxy .
dy 3(64) − 12(16) 0
@ ( −8, 16 ) , = = = 0. One gradient found. A1
dx 12(−8) + 8(16) 32
Both gradients of -3 and 0 correctly found. A1 cso
[6]

12
PhysicsAndMathsTutor.com

Question
Scheme Marks
Number
Aliter
5. (b) x 3 − 4 y 2 = 12 xy ( eqn ∗ )
Way 3
4 y 2 +12 xy − x 3 = 0

−12 x ± 144 x 2 − 4(4)(− x 3 )


y=
8

−12 x ± 144 x 2 + 16 x 3
y=
8

−12 x ± 4 9 x 2 + x 3
y=
8

(9x + x3 )
1

y = − 32 x ± 2
1 2
2

A credible attempt to make y the subject


and an attempt to differentiate either − 32 x
( 12 ) ( 9 x 2 + x3 ) ; (18 x + 3x 2 )
dy − 12 M1
= − 32 ± 1

(9x )
1

+x
2 1 2 3 2
dx or 2 .

= − 32 ± k ( 9 x 2 + x 3 ) ( g( x) )
dy − 12
dy 3 18 x + 3x 2 A1
=− ± 1 dx
dx 2 4(9 x 2 + x3 ) 2
( 12 ) ( 9 x 2 + x3 ) ; (18 x + 3x 2 )
dy − 12
= − 32 ± 1
2 A1
dx

dy 3 18(−8) + 3(64)
@ x = −8 =− ± Substitutes x = −8 find any one of dy
. dM1
2 4(9(64) + ( −512)) 2
1
dx
dx

3 48 3 48
=− ± =− ±
2 4 (64) 2 32

dy 3 3 One gradient correctly found. A1


∴ = − ± = −3, 0.
dx 2 2 Both gradients of -3 and 0 correctly found. A1
[6]

13
PhysicsAndMathsTutor.com

Question
Scheme Marks
Number

⎛2⎞ ⎛ 3⎞
uuur ⎜ ⎟ uuur ⎜ ⎟
6. (a) OA = ⎜ 6 ⎟ & OB = ⎜ 4 ⎟
⎜ −1⎟ ⎜1⎟
⎝ ⎠ ⎝ ⎠

⎛ 3⎞ ⎛ 2 ⎞ ⎛ 1 ⎞ Finding the difference


uuur uuur M1 ±
uuur uuur uuur ⎜ ⎟ ⎜ ⎟ ⎜ ⎟
AB = OB − OA = ⎜ 4 ⎟ − ⎜ 6 ⎟ = ⎜ −2 ⎟ between OB and OA .
⎜ 1 ⎟ ⎜ −1⎟ ⎜ 2 ⎟
⎝ ⎠ ⎝ ⎠ ⎝ ⎠ Correct answer. A1
[2]
An expression of the form
⎛2⎞ ⎛1⎞ ⎛ 3⎞ ⎛1⎞ ( vector ) ± λ ( vector )
M1
⎜ ⎟ ⎜ ⎟ ⎜ ⎟ ⎜ ⎟ uuur uuur
l1 : r = ⎜ 6 ⎟ + λ ⎜ −2 ⎟ or
⎜ −1⎟ ⎜ 2⎟
r = ⎜ 4 ⎟ + λ ⎜ −2 ⎟
⎜1⎟ ⎜ 2⎟
(
r = OA ± λ their AB or )
uuur uuur
(b)
⎝ ⎠ ⎝ ⎠ ⎝ ⎠ ⎝ ⎠
(
r = OB ± λ their AB or )
uuur uuur
( )
A1
⎛2⎞ ⎛ −1 ⎞ ⎛ 3⎞ ⎛ −1 ⎞ r = OA ± λ their BA or aef
⎜ ⎟ ⎜ ⎟ ⎜ ⎟ ⎜ ⎟ uuur uuur
l1 : r = ⎜ 6 ⎟ + λ ⎜ 2 ⎟ or
⎜ −1⎟ ⎜ −2 ⎟
r = ⎜ 4⎟ +λ ⎜ 2 ⎟
⎜1⎟ ⎜ −2 ⎟ (
r = OB ± λ their BA )
⎝ ⎠ ⎝ ⎠ ⎝ ⎠ ⎝ ⎠ ( r is needed.)
[2]
⎛ 0⎞ ⎛1⎞ ⎛1⎞
⎜ ⎟ ⎜ ⎟ ⎜ ⎟
(c) l2 : r = ⎜ 0 ⎟ + µ ⎜ 0 ⎟ ⇒ r = µ ⎜0⎟
⎜ 0⎟ ⎜1⎟ ⎜1⎟
⎝ ⎠ ⎝ ⎠ ⎝ ⎠
uuur
AB = d 1 = i − 2 j + 2k , d 2 = i + 0 j + k & θ is angle

⎛ 1 ⎞ ⎛1⎞
⎜ ⎟ ⎜ ⎟
uuur ⎜ −2 ⎟ • ⎜ 0 ⎟ Considers dot product
uuur
AB • d 2 ⎜ 2 ⎟ ⎜1⎟
⎝ ⎠ ⎝ ⎠ between d 2 and their AB. M1
cos θ = =
( ) ( )
uuur
AB . d 2 (1) 2 + (−2) 2 + (2) 2 . (1) 2 + (0) 2 + (1) 2

1+ 0 + 2
cos θ = Correct followed through
(1) + (−2) + (2) 2 . (1) 2 + (0) 2 + (1) 2
2 2
expression or equation. A1

3
cos θ = ⇒ θ = 45o or π
4
or awrt 0.79. θ = 45o or π
4
or awrt 0.79 A1 cao
3. 2
[3]

This means that cosθ does not


necessarily have to be the subject of
the equation. It could be of the form
3 2 cos θ = 3.

14
PhysicsAndMathsTutor.com

Question
Scheme Marks
Number

⎛2⎞ ⎛1⎞ ⎛1⎞


⎜ ⎟ ⎜ ⎟ ⎜ ⎟
6. (d) If l1 and l2 intersect then: ⎜ 6 ⎟ + λ ⎜ −2 ⎟ = µ ⎜ 0 ⎟
⎜ −1⎟ ⎜ 2⎟ ⎜1⎟
⎝ ⎠ ⎝ ⎠ ⎝ ⎠

i: 2 + λ =µ (1) Either seeing equation (2) written


j : 6 − 2λ = 0 (2) down correctly with or without any
other equation or seeing equations M1
k : − 1 + 2λ = µ (3) (1) and (3) written down correctly.

Attempt to solve either equation (2)


(2) yields λ = 3 or simultaneously solve any two of dM1
the three equations to find …
Any two yields λ = 3, µ = 5
either one of λ or µ correct. A1

⎛ 5⎞
⎛2⎞ ⎛ 1 ⎞ ⎛ 5⎞ ⎛1⎞ ⎛ 5⎞ ⎜ ⎟
⎜ ⎟ ⎜ ⎟ ⎜ ⎟ ⎜ ⎟ ⎜ ⎟ ⎜ 0 ⎟ or 5i + 5k
l1 : r = ⎜ 6 ⎟ + 3 ⎜ −2 ⎟ = ⎜ 0 ⎟ or r = 5 ⎜ 0 ⎟ = ⎜ 0 ⎟ ⎜ 5⎟
⎝ ⎠ A1 cso
⎜ −1⎟ ⎜ 2 ⎟ ⎜ 5⎟ ⎜1⎟ ⎜ 5⎟
⎝ ⎠ ⎝ ⎠ ⎝ ⎠ ⎝ ⎠ ⎝ ⎠ Fully correct solution & no incorrect
values of λ or µ seen earlier.
[4]

Aliter ⎛ 3⎞ ⎛1⎞ ⎛1⎞


⎜ ⎟ ⎜ ⎟ ⎜ ⎟
6. (d) If l1 and l2 intersect then: ⎜ 4 ⎟ + λ ⎜ −2 ⎟ = µ ⎜ 0 ⎟
Way 2 ⎜1⎟ ⎜ 2⎟ ⎜1⎟
⎝ ⎠ ⎝ ⎠ ⎝ ⎠

i: 3 + λ =µ (1) Either seeing equation (2) written


j : 4 − 2λ = 0 (2) down correctly with or without any
other equation or seeing equations M1
k : 1 + 2λ = µ (3) (1) and (3) written down correctly.

(2) yields λ = 2 Attempt to solve either equation (2)


or simultaneously solve any two of dM1
Any two yields λ = 2, µ = 5 the three equations to find …
either one of λ or µ correct. A1

⎛ 5⎞
⎛ 3⎞ ⎛ 1 ⎞ ⎛5⎞ ⎛1⎞ ⎛ 5⎞ ⎜ ⎟
⎜ ⎟ ⎜ ⎟ ⎜ ⎟ ⎜ ⎟ ⎜ ⎟ ⎜ 0 ⎟ or 5i + 5k
l1 : r = ⎜ 4 ⎟ + 2 ⎜ −2 ⎟ = ⎜ 0 ⎟ or r = 5 ⎜ 0 ⎟ = ⎜ 0 ⎟ ⎜ 5⎟
⎝ ⎠ A1 cso
⎜1⎟ ⎜ 2 ⎟ ⎜5⎟ ⎜1⎟ ⎜ 5⎟
⎝ ⎠ ⎝ ⎠ ⎝ ⎠ ⎝ ⎠ ⎝ ⎠ Fully correct solution & no incorrect
values of λ or µ seen earlier.
[4]

11 marks
Note: Be careful! λ and µ are not defined in the question, so a candidate could interchange these or use different scalar
parameters.

15
PhysicsAndMathsTutor.com

Question
Scheme Marks
Number

Aliter ⎛2⎞ ⎛ −1 ⎞ ⎛1⎞


⎜ ⎟ ⎜ ⎟ ⎜ ⎟
6. (d) If l1 and l2 intersect then: ⎜ 6 ⎟ + λ ⎜ 2 ⎟ = µ ⎜ 0 ⎟
Way 3 ⎜ −1⎟ ⎜ −2 ⎟ ⎜1⎟
⎝ ⎠ ⎝ ⎠ ⎝ ⎠

i: 2 − λ =µ (1) Either seeing equation (2) written


j : 6 + 2λ = 0 (2) down correctly with or without any
other equation or seeing equations M1
k : − 1 − 2λ = µ (3) (1) and (3) written down correctly.

Attempt to solve either equation (2)


(2) yields λ = −3 or simultaneously solve any two of dM1
the three equations to find …
Any two yields λ = −3, µ = 5
either one of λ or µ correct. A1

⎛ 5⎞
⎛2⎞ ⎛ −1 ⎞ ⎛ 5 ⎞ ⎛1⎞ ⎛ 5⎞ ⎜ ⎟
⎜ ⎟ ⎜ ⎟ ⎜ ⎟ ⎜ ⎟ ⎜ ⎟ ⎜ 0 ⎟ or 5i + 5k
l1 : r = ⎜ 6 ⎟ − 3 ⎜ 2 ⎟ = ⎜ 0 ⎟ or r = 5 ⎜ 0 ⎟ = ⎜ 0 ⎟ ⎜ 5⎟
⎝ ⎠ A1 cso
⎜ −1⎟ ⎜ −2 ⎟ ⎜ 5 ⎟ ⎜1⎟ ⎜ 5⎟
⎝ ⎠ ⎝ ⎠ ⎝ ⎠ ⎝ ⎠ ⎝ ⎠ Fully correct solution & no incorrect
values of λ or µ seen earlier.
[4]

Aliter ⎛ 3⎞ ⎛ −1 ⎞ ⎛1⎞
⎜ ⎟ ⎜ ⎟ ⎜ ⎟
6. (d) If l1 and l2 intersect then: ⎜ 4 ⎟ + λ ⎜ 2 ⎟ = µ ⎜ 0 ⎟
Way 4 ⎜1⎟ ⎜ −2 ⎟ ⎜1⎟
⎝ ⎠ ⎝ ⎠ ⎝ ⎠

i: 3 − λ =µ (1) Either seeing equation (2) written


j : 4 + 2λ = 0 (2) down correctly with or without any
other equation or seeing equations M1
k : 1 − 2λ = µ (3) (1) and (3) written down correctly.

(2) yields λ = −2 Attempt to solve either equation (2)


or simultaneously solve any two of dM1
Any two yields λ = −2, µ = 5 the three equations to find …
either one of λ or µ correct. A1

⎛ 5⎞
⎛ 3⎞ ⎛ −1 ⎞ ⎛ 5 ⎞ ⎛ 1⎞ ⎛ 5⎞ ⎜ ⎟
⎜ ⎟ ⎜ ⎟ ⎜ ⎟ ⎜ ⎟ ⎜ ⎟ ⎜ 0 ⎟ or 5i + 5k
l1 : r = ⎜ 4 ⎟ − 2 ⎜ 2 ⎟ = ⎜ 0 ⎟ or r = 5 ⎜ 0 ⎟ = ⎜ 0 ⎟ ⎜ 5⎟
⎝ ⎠ A1 cso
⎜1⎟ ⎜ −2 ⎟ ⎜ 5 ⎟ ⎜ 1⎟ ⎜ 5⎟
⎝ ⎠ ⎝ ⎠ ⎝ ⎠ ⎝ ⎠ ⎝ ⎠ Fully correct solution & no incorrect
values of λ or µ seen earlier.
[4]

11 marks

16
PhysicsAndMathsTutor.com

Question
Scheme Marks
Number

⎡ 1 ⎤ dx 1 dx 1
7. (a) ⎢ x = ln ( t + 2 ) , y = t + 1 ⎥ , ⇒ =
dt t + 2
Must state =
dt t + 2
B1
⎣ ⎦

∫ t + 1 dx .
1
Area =
M1;
⎛ 1 ⎞⎛ 1 ⎞
∫ ∫
ln 4 2
1
Area( R) = dx ; = ⎜ ⎟⎜ ⎟ dt Ignore limits.
t +1 ⎝ t + 1 ⎠⎝ t + 2 ⎠ ⎛ 1 ⎞ ⎛ 1 ⎞
ln 2 0

∫ ⎜⎝ t + 1 ⎟⎠ × ⎜⎝ t + 2 ⎟⎠ dt . Ignore limits. A1 AG

Changing limits, when:


x = ln 2 ⇒ ln 2 = ln(t + 2) ⇒ 2 = t + 2 ⇒ t = 0 changes limits x → t
B1
so that ln 2 → 0 and ln 4 → 2
x = ln 4 ⇒ ln 4 = ln(t + 2) ⇒ 4 = t + 2 ⇒ t = 2


2
1
Hence, Area( R ) = dt
0 (t + 1)(t + 2)
[4]

⎛ 1 ⎞ A B A B
(b) ⎜ ⎟= + + with A and B found M1
⎝ (t + 1)(t + 2) ⎠ (t + 1) (t + 2) (t + 1) (t + 2)

1 = A(t + 2) + B(t + 1)

Let t = −1, 1 = A (1) ⇒ A =1 Finds both A and B correctly.


Can be implied. A1
Let t = −2, 1 = B ( −1) ⇒ B = − 1 (See note below)

∫ ∫
2 2
1 1 1
dt = − dt
0 (t + 1)(t + 2) 0 (t + 1) (t + 2)

Either ± a ln(t + 1) or ± b ln(t + 2) dM1


= [ ln(t + 1) − ln(t + 2) ] 0
2

Both ln terms correctly ft. A1

= ( ln 3 − ln 4 ) − ( ln1 − ln 2 ) Substitutes both limits of 2 and 0


ddM1
and subtracts the correct way round.

ln 3 − ln 4 + ln 2 or ln ( 34 ) − ln ( 12 )
= ln 3 − ln 4 + ln 2 = ln 3 − ln 2 = ln ( 3
) A1 aef isw
or ln 3 − ln 2 or ln ( 32 )
2

(must deal with ln 1) [6]

1 1 1
Takes out brackets. Writing down = + means first M1A0 in (b).
(t + 1)(t + 2) (t + 1) (t + 2)

1 1 1
Writing down = − means first M1A1 in (b).
(t + 1)(t + 2) (t + 1) (t + 2)

17
PhysicsAndMathsTutor.com

Question
Scheme Marks
Number

1
x = ln ( t + 2 ) , y=
t +1

Attempt to make t =… the subject M1


7. (c) ex = t + 2 ⇒ t = ex − 2
giving t = e x − 2 A1

Eliminates t by substituting in y dM1


1 1
y= ⇒ y= x 1
e − 2 +1
x
e −1 giving y = x A1
e −1
[4]

1 1 1− y
t +1 = ⇒ t = −1 or t = Attempt to make t =… the subject M1
Aliter y y y
7. (c)
1− y 1 1− y
Way 2 y (t + 1) = 1 ⇒ yt + y = 1 ⇒ yt = 1 − y ⇒ t = Giving either t = −1 or t = A1
y y y

⎛1 ⎞ ⎛1− y ⎞
x = ln ⎜ − 1 + 2 ⎟ or x = ln ⎜ + 2⎟ Eliminates t by substituting in x dM1
⎝y ⎠ ⎝ y ⎠

⎛1 ⎞
x = ln ⎜ + 1⎟
⎝y ⎠

1
ex = +1
y

1
ex − 1 =
y

1 1
y= giving y = A1
e −1
x
e −1
x

[4]

(d) Domain : x > 0 x > 0 or just > 0 B1


[1]

15 marks

18
PhysicsAndMathsTutor.com

Question
Scheme Marks
Number

Aliter Attempt to make t + 1 = … the subject M1


7. (c) e x = t + 2 ⇒ t + 1 = e x −1
giving t + 1 = e x −1 A1
Way 3

Eliminates t by substituting in y dM1


1 1
y= ⇒ y= x 1
t +1 e −1 giving y = x A1
e −1
[4]

Aliter Attempt to make t + 2 =… the subject M1


1 1 1+ y
7. (c) t +1 = ⇒ t + 2 = + 1 or t + 2 = 1 1+ y
y y y Either t + 2 = + 1 or t + 2 = A1
Way 4 y y

⎛1 ⎞ ⎛1+ y ⎞
x = ln ⎜ + 1⎟ or x = ln ⎜ ⎟ Eliminates t by substituting in x dM1
⎝y ⎠ ⎝ y ⎠

⎛1 ⎞
x = ln ⎜ + 1⎟
⎝y ⎠

1 1
ex = +1 ⇒ ex − 1 =
y y

1 1
y= giving y = A1
e −1
x
e −1
x

[4]

19
PhysicsAndMathsTutor.com

Question
Scheme Marks
Number

dV dV
8. (a) = 1600 − c h or = 1600 − k h , Either of these statements M1
dt dt

dV dV dh 1
(V = 4000h ⇒) = 4000 = 4000 or = M1
dh dh dV 4000
dV
dh dh dV
= × = dt
dV
dt dV dt dh

dh 1600 − c h 1600 c h
Either, = = − = 0.4 − k h
dt 4000 4000 4000
dh
Convincing proof of A1 AG
dt
dh 1600 − k h 1600 k h
or = = − = 0.4 − k h
dt 4000 4000 4000
[3]

dV
(b) When h = 25 water leaks out such that = 400
dt

400 = c h ⇒ 400 = c 25 ⇒ 400 = c(5) ⇒ c = 80

c 80
From above; k = = = 0.02 as required Proof that k = 0.02 B1 AG
4000 4000
[1]
Aliter
(b) 400 = 4000k h
Way 2
⇒ 400 = 4000k 25
Using 400, 4000 and h = 25
⇒ 400 = k (20000) ⇒ k = 400
20000
= 0.02 or h = 5 . Proof that k = 0.02 B1 AG
[1]

Separates the variables with

∫ 0.4 − k ∫ ∫ ∫
dh dh dh
(c) = 0.4 − k h ⇒ = dt and dt on either side M1 oe
dt h 0.4 − k h
with integral signs not necessary.

÷ 0.02

100
1
∴ time required = dh
0 0.4 − 0.02 h ÷ 0.02


100
50
time required = dh Correct proof A1 AG
0 20 − h
[2]

20
PhysicsAndMathsTutor.com

Question
Scheme Marks
Number


100
50
8. (d) dh with substitution h = (20 − x) 2
0 20 − h

dh dh dh
= 2(20 − x)( −1) or = − 2(20 − x) Correct B1 aef
dx dx dx

h = (20 − x) 2 ⇒ h = 20 − x ⇒ x = 20 − h
20 − x
±λ
∫ dx or
∫ ∫
50 50 x
dh = . − 2(20 − x) dx M1
20 − h 20 − x

x
±λ dx
20 − (20 − x)
where λ is a constant
x − 20
= 100
∫ x
dx

⎛ 20 ⎞
⎝ ∫
= 100 ⎜ 1 − ⎟ dx
x ⎠

± α x ± β ln x ; α , β ≠ 0
= 100 ( x − 20ln x ) ( + c )
M1
100 x − 2000ln x A1

change limits: when h = 0 then x = 20


and when h = 100 then x = 10


100
50
dh = [100 x − 2000ln x ] 20
10

0 20 − h

∫ ( ) ( )
100
50 100
or dh = ⎡100 20 − h − 2000ln 20 − h ⎤
0 20 − h ⎣ ⎦0 Correct use of limits, ie. putting
them in the correct way round
Either x = 10 and x = 20
= (1000 − 2000ln10 ) − ( 2000 − 2000ln 20 ) ddM1
or h = 100 and h = 0

= 2000ln 20 − 2000ln10 − 1000 Combining logs to give...


2000ln 2 − 1000
or −2000ln ( 12 ) − 1000
A1 aef
= 2000ln 2 − 1000
[6]

(e) Time required = 2000ln 2 − 1000 = 386.2943611... sec

= 386 seconds (nearest second)

= 6 minutes and 26 seconds (nearest second) 6 minutes, 26 seconds B1


[1]

13 marks

21
PhysicsAndMathsTutor.com

Mark Scheme (Results)


Summer 2008

GCE

GCE Mathematics (6666/01)

1
Edexcel Limited. Registered in England and Wales No. 4496750
Registered Office: One90 High Holborn, London WC1V 7BH
PhysicsAndMathsTutor.com

June 2008
6666 Core Mathematics C4
Mark Scheme
Question Scheme Marks

x 0 0.4 0.8 1.2 1.6 2


1. (a)
y e 0
e 0.08
e 0.32
e 0.72
e1.28
e2
1.08329
or y 1 1.37713… 2.05443… 3.59664… 7.38906…

Either e0.32 and e1.28 or


awrt 1.38 and 3.60 B1
(or a mixture of e’s and
decimals)
[1]

Outside brackets
B1;
1
2
× 0.4 or 0.2
(b)
Area ≈ × 0.4 ; ×⎡⎣ e0 + 2 ( e0.08 + e0.32 + e0.72 + e1.28 ) + e 2 ⎤⎦
1 For structure of
Way 1 2 trapezium
M1
rule [ ............. ] ;

= 0.2 × 24.61203164... = 4.922406... = 4.922 (4sf) 4.922 A1 cao


[3]

Aliter ⎤ 0.4 and a divisor of 2 on B1


Area ≈ 0.4 × ⎡ e + e0.08 e0.08 + e0.32 e0.32 + e0.72 e0.72 + e1.28 e1.28 + e 2
0
+ + + +
(b) ⎣ 2 2 2 2 2 ⎦ all terms inside brackets.
Way 2
One of first and last
which is equivalent to: ordinates, two of the
middle ordinates inside M1
brackets ignoring the 2.
Area ≈ × 0.4 ; ×⎡⎣ e0 + 2 ( e0.08 + e0.32 + e0.72 + e1.28 ) + e 2 ⎤⎦
1
2

= 0.2 × 24.61203164... = 4.922406... = 4.922 (4sf) 4.922 A1 cao


[3]

4 marks

Note an expression like Area ≈ × 0.4 + e0 + 2 ( e0.08 + e0.32 + e0.72 + e1.28 ) + e 2 would score B1M1A0
1
2

Allow one term missing (slip!) in the ( ) brackets for

The M1 mark for structure is for the material found in the curly brackets ie
⎡⎣ first y ordinate + 2 ( intermediate
2 ft y ordinate ) + final y ordinate ⎤⎦
PhysicsAndMathsTutor.com

Question
Scheme Marks
Number

⎧⎪u = x ⇒ ddux = 1⎫⎪


2. (a) ⎨ dv x⎬
⎪⎩ dx = e ⇒ v = e ⎪⎭
x

Use of ‘integration by parts’


formula in the correct direction. M1
∫ xe dx = x e x − ∫ e x .1 dx
x
(See note.)
Correct expression. (Ignore dx) A1

= x e x − ∫ e x dx

= x ex − ex ( + c ) Correct integration with/without + c A1


[3]

(b) ⎧⎪u = x 2 ⇒ du
= 2 x ⎫⎪
dx
⎨ dv ⎬
⎩⎪ dx = e ⇒ v = e x ⎭⎪
x

Use of ‘integration by parts’ formula


M1
∫ x e dx = x e − ∫ e .2 x dx
2 x 2 x x
in the correct direction.
Correct expression. (Ignore dx) A1

= x 2 e x − 2 ∫ x e x dx

Correct expression including + c.


= x2e x − 2 ( x e x − e x ) + c (seen at any stage! in part (b)) A1 ISW
You can ignore subsequent working.
[3]
⎧⎪= x e − 2 x e + 2e + c ⎫⎪
2 x x x

⎨ x 2 ⎬
⎩⎪= e ( x − 2 x + 2 ) + c ⎭⎪
Ignore subsequent working

6 marks

Note integration by parts in the correct direction means that u


and ddvx must be assigned/used as u = x and ddvx = e x in part (a)
for example

+ c is not required in part


(a).
i i di t (b)

3
PhysicsAndMathsTutor.com

Question
Scheme Marks
Number

dA
3. (a) dA = 0.032 seen
From question, = 0.032 dt B1
dt
or implied from working.

⎧ dA ⎫ 2π x by itself seen
⎨A = π x ⇒ = ⎬ 2π x B1
2

⎩ dx ⎭ or implied from working

dx dA dA 1 ⎧ 0.016 ⎫
= ÷ = ( 0.032 ) ; ⎨= ⎬ 0.032 ÷ Candidate's
dA
; M1;
dt dt dx 2π x ⎩ π x ⎭ dx

dx 0.016
When x = 2cm , =
dt 2π

dx
Hence, = 0.002546479... (cm s-1) awrt 0.00255 A1 cso
dt
[4]

(b) V = π x 2 (5 x) = 5 π x3 V = π x 2 (5 x) or 5π x 3 B1

dV
= 15 π x 2
dV dx
= 15π x 2 B1
dx or ft from candidate’s V
in one variable

dV dV d x ⎛ 0.016 ⎞
⎟ ; {= 0.24 x}
dV dx
= × = 15π x 2 . ⎜ Candidate’s × ; M1
dt dx dt ⎝ πx ⎠ dx dt

dV
When x = 2cm , = 0.24(2) = 0.48 (cm3 s −1 ) 0.48 or awrt 0.48 A1 cso
dt
[4]

8 marks

4
PhysicsAndMathsTutor.com

Question
Marks
Number Scheme

4. (a) 3 x 2 − y 2 + xy = 4 ( eqn ∗ )

Differentiates implicitly to include either


± ky ddyx or x ddxy . (Ignore ( dy
dx )
= )
M1
⎧ dy ⎫ dy ⎛ dy ⎞
⎨ = ⎬ 6x − 2 y + ⎜y + x ⎟=0 Correct application ( ) of product rule B1
⎩ dx ⎭ dx ⎜⎝ dx ⎟⎠
⎛ dy ⎞
( 3x 2
− y2 ) → ⎜ 6x − 2 y ⎟ and ( 4 → 0) A1
⎝ dx ⎠

⎧ dy −6 x − y ⎫ ⎧ dy 6x + y ⎫
⎨ = ⎬ or ⎨ = ⎬ not necessarily required.
⎩ dx x − 2y ⎭ ⎩ dx 2y − x ⎭

−6 x − y dy 8
dy 8
= ⇒ =
8 Substituting = into their
dx 3 M1 ∗
dx 3 x − 2y 3
equation.

giving −18 x − 3 y = 8 x − 16 y

Attempt to combine either terms in x


giving 13 y = 26 x or terms in y together to give either dM1 ∗
ax or by.

Hence, y = 2 x ⇒ y − 2 x = 0 simplifying to give y − 2 x = 0 AG A1 cso


[6]

(b) At P & Q, y = 2 x . Substituting into eqn ∗

Attempt replacing y by 2x
gives 3x 2 − (2 x) 2 + x(2 x) = 4 M1
in at least one of the y terms in eqn ∗

Simplifying gives, x 2 = 4 ⇒ x = ± 2 Either x = 2 or x = −2 A1

y = 2x ⇒ y = ± 4

Hence coordinates are (2, 4) and (−2, − 4) Both (2, 4) and (−2, − 4) A1
[3]

9 marks

5
PhysicsAndMathsTutor.com

Question
Scheme Marks
Number
** represents a constant (which must be consistent for first accuracy mark)
− 12 − 12
5. (a) 1 −1 ⎛ 3x ⎞ 1 ⎛ 3x ⎞
= (4 − 3 x) 2 = ( 4 ) 2 ⎜1 − ⎟
−1 − 12
= ⎜1 − ⎟ (4) or 1
2 outside brackets B1
(4 − 3 x) ⎝ 4 ⎠ 2⎝ 4 ⎠

−1
Expands (1 + ** x) 2 to give a
simplified or an un-simplified M1;
1 + (− 12 )(** x) ;
⎡ (− 1 )( − 23 ) ⎤ A correct simplified or an un-
= 12 ⎢ 1 + (− 12 )(** x); + 2 (** x) 2 + ... ⎥
⎣ 2! ⎦ simplified [ .......... ] expansion
with candidate’s followed A1
with ** ≠ 1 through (** x )

1⎡ (− 1 )(− 23 ) 3 x 2 ⎤ Award SC M1 if you see


= ⎢ 1 + (− 12 )(− 34x ) + 2 (− 4 ) + ... ⎥ (− 12 )(− 32 )
2⎣ (− 12 )(** x) + (** x) 2
2! ⎦ 2!

⎣⎡ 1 + 8 x ; ... ⎦⎤
1 3
2
A1 isw
= 12 ⎡⎣ 1 + 83 x ; + 27
128 x 2 + ... ⎤⎦ SC: K ⎡⎣ 1 + 83 x + 27
128 x 2 + ... ⎤⎦
1
2
⎡⎣ ........; 128 x ⎤⎦
27 2
A1 isw

⎧ 1 3 27 2 ⎫
⎨= + x; + x + ...⎬ Ignore subsequent working
⎩ 2 16 256 ⎭

[5]
⎛1 3 27 2 ⎞ Writing ( x + 8) multiplied by
(b) ( x + 8) ⎜ + x + x + ... ⎟
⎝ 2 16 256 ⎠ candidate’s part (a) M1
expansion.

= 1
x+ 3
x 2 + ..... Multiply out brackets to find
2 16
a constant term, two x terms M1
+ 4 + 32 x + 27
32 x 2 + ..... and two x 2 terms.

33 2 Anything that cancels to


= 4 + 2 x; + x + ...
32 33 2 A1; A1
4 + 2 x; x
32

[4]

9 marks

6
PhysicsAndMathsTutor.com

Question Scheme Marks


Number

6. (a) Lines meet where:

⎛ −9 ⎞ ⎛2⎞ ⎛3⎞ ⎛3⎞


⎜ ⎟ ⎜ ⎟ ⎜ ⎟ ⎜ ⎟
⎜ 0 ⎟ + λ ⎜ 1 ⎟ = ⎜ 1 ⎟ + µ ⎜ −1⎟
⎜ 10 ⎟ ⎜ −1⎟ ⎜ 17 ⎟ ⎜5⎟
⎝ ⎠ ⎝ ⎠ ⎝ ⎠ ⎝ ⎠

i : − 9 + 2λ = 3 + 3µ (1) Need any two of these correct


Any two of j: λ =1 − µ (2) equations seen anywhere in part M1
k : 10 − λ = 17 + 5µ (3) (a).

Attempts to solve simultaneous


(1) – 2(2) gives: −9 = 1 + 5µ ⇒ µ = −2 equations to find one of dM1
either λ or µ

(2) gives: λ =1−− 2 = 3 Both λ = 3 & µ = − 2 A1

Substitutes their value of either


⎛ −9 ⎞ ⎛2⎞ ⎛3⎞ ⎛3⎞ λ or µ into the line l1 or l2
⎜ ⎟ ⎜ ⎟ ⎜ ⎟ ⎜ ⎟
r = ⎜ 0 ⎟ + 3⎜ 1 ⎟ or r = ⎜ 1 ⎟ − 2 ⎜ −1⎟ respectively. This mark can be ddM1
⎜ 10 ⎟ ⎜ −1⎟ ⎜ 17 ⎟ ⎜5⎟ implied by any two correct
⎝ ⎠ ⎝ ⎠ ⎝ ⎠ ⎝ ⎠
components of ( −3, 3, 7 ) .

⎛ −3 ⎞
⎛ −3 ⎞ ⎜ ⎟
⎜ ⎟
Intersect at r = ⎜ 3 ⎟ or r = −3i + 3 j + 7k ⎜ 3 ⎟ or −3i + 3 j + 7k
⎜7⎟ A1
⎜7⎟ ⎝ ⎠
⎝ ⎠
or ( −3, 3, 7 )

Either check that λ = 3 , µ = − 2


Either check k: in a third equation or check
λ = 3 : LHS = 10 − λ = 10 − 3 = 7 that λ = 3 ,
B1
µ = − 2 give the same
µ = − 2 : RHS = 17 + 5µ = 17 − 10 = 7
coordinates on the other line.
Conclusion not needed.
(As LHS = RHS then the lines intersect.) [6]

(b) d1 = 2i + j − k , d 2 = 3i − j + 5k
Dot product calculation between the
⎛2⎞ ⎛3⎞ two direction vectors:
⎜ ⎟ ⎜ ⎟ (2 × 3) + (1 ×− 1) + (−1 × 5) M1
As d1 • d 2 = ⎜ 1 ⎟ • ⎜ −1⎟ = (2 × 3) + (1 ×− 1) + (−1 × 5) = 0
⎜ −1⎟ ⎜ 5 ⎟ or 6 − 1 − 5
⎝ ⎠ ⎝ ⎠
Then l1 is perpendicular to l2. Result ‘=0’ and
A1
appropriate conclusion
[2]

7
PhysicsAndMathsTutor.com

Question
Scheme Marks
Number

6. (c) Equating i ; − 9 + 2λ = 5 ⇒ λ=7

⎛ −9 ⎞ ⎛ 2 ⎞ ⎛ 5⎞ Substitutes candidate’s λ = 7 into


⎜ ⎟ ⎜ ⎟ ⎜ ⎟
r = ⎜ 0 ⎟ + 7⎜ 1 ⎟ = ⎜ 7⎟ the line l1 and finds 5 i + 7 j + 3k .
⎜ 10 ⎟ ⎜ −1⎟ ⎜ 3 ⎟ B1
⎝ ⎠ ⎝ ⎠ ⎝ ⎠ The conclusion on this occasion is
uuur not needed.
( = OA. Hence the point A lies on l1.)
[1]

uuur
(d) Let OX = − 3i + 3 j + 7k be point of intersection

Finding the difference between


uuur
their OX (can be implied) and
⎛ −3 ⎞ ⎛ 5 ⎞ ⎛ −8 ⎞ uuur
uuur uuur uuur ⎜ ⎟ ⎜ ⎟ ⎜ ⎟ OA .
AX = OX − OA = ⎜ 3 ⎟ − ⎜ 7 ⎟ = ⎜ −4 ⎟ ±
⎛ ⎛ −3 ⎞ ⎛ 5 ⎞ ⎞ M1
⎜ 7 ⎟ ⎜ 3⎟ ⎜ 4 ⎟ uuur ⎜⎜ ⎟ ⎜ ⎟⎟
⎝ ⎠ ⎝ ⎠ ⎝ ⎠ AX = ± ⎜ ⎜ 3 ⎟ − ⎜ 7 ⎟ ⎟
⎜⎜ 7 ⎟ ⎜ 3⎟⎟
⎝⎝ ⎠ ⎝ ⎠⎠
uuur uuur uuur uuur uuur
OB = OA + AB = OA + 2 AX

⎛ 5⎞ ⎛ −8 ⎞ ⎛5⎞ ⎛ ⎞
uuur ⎜ ⎟ ⎜ ⎟ ⎜ ⎟ ⎜ uuur ⎟
OB = ⎜ 7 ⎟ + 2 ⎜ −4 ⎟ ⎜7⎟ + 2 ⎜ their AX ⎟ dM1
⎜ 3⎟ ⎜ 4⎟ ⎜ 3⎟ ⎜ ⎟
⎝ ⎠ ⎝ ⎠ ⎝ ⎠ ⎝ ⎠

⎛ −11⎞
⎛ −11⎞ ⎜ ⎟
uuur ⎜ ⎟ uuur
Hence, OB = ⎜ −1 ⎟ or OB = −11i − j + 11k ⎜ −1 ⎟ or −11i − j + 11k
⎜ 11 ⎟ A1
⎜ 11 ⎟ ⎝ ⎠
⎝ ⎠
or ( −11, − 1, 11)
[3]

12 marks

8
PhysicsAndMathsTutor.com

Question
Scheme Marks
Number

7. (a) 2

2

A
+
B
4− y 2
(2 − y )(2 + y ) (2 − y ) (2 + y )

Forming this identity.


2 ≡ A(2 + y ) + B(2 − y ) NB: A & B are not assigned in M1
this question
Let y = −2, 2 = B ( 4) ⇒ B = 1
2

Let y = 2, 2 = A( 4) ⇒ A = 1
2
Either one of A = 1
2 or B = 1
2 A1

1 1 1 1
giving 2
+ 2 2
+ 2
, aef A1 cao
(2 − y ) (2 + y ) (2 − y ) (2 + y )

(If no working seen, but candidate writes down [3]


correct partial fraction then award all three marks. If
no working is seen but one of A or B is incorrect then
M0A0A0.)

9
PhysicsAndMathsTutor.com

Question
Marks
Number Scheme

Separates variables as shown.


∫ ∫
7. (b) 2 1
dy = dx Can be implied. Ignore the B1
4 − y2 cot x integral signs, and the ‘2’.

∫ ∫
1 1
2
+ 2
dy = tan x dx
(2 − y ) (2 + y )

ln(sec x) or − ln(cos x) B1
Either ± a ln(λ − y ) or ± b ln(λ + y ) M1;
∴ − 12 ln(2 − y ) + 12 ln(2 + y ) = ln(sec x) + ( c ) their ∫ cot1 x dx = LHS correct with ft
for their A and B and no error A1
with the “2” with or without + c

Use of y = 0 and x = π3 in an
y = 0, x = π
3
⇒ − ln 2 + 1
2
1
2 ln 2 = ln ( ( )) + c
1
cos π3 integrated equation containing c M1*
;

{0 = ln 2 + c ⇒ c = − ln 2 }
− 12 ln(2 − y ) + 12 ln(2 + y ) = ln(sec x) − ln 2

1 ⎛2+ y⎞ Using either the quotient (or


⎛ sec x ⎞
ln ⎜ ⎟ = ln ⎜ ⎟ product) or power laws for M1
2 ⎝2− y⎠ ⎝ 2 ⎠ logarithms CORRECTLY.

⎛2+ y⎞ ⎛ sec x ⎞
ln ⎜ ⎟ = 2ln ⎜ ⎟
⎝2− y⎠ ⎝ 2 ⎠

Using the log laws correctly to


⎛2+ y⎞
2
⎛ sec x ⎞
ln ⎜ ⎟ = ln ⎜ ⎟ obtain a single log term on both dM1*
⎝ 2− y⎠ ⎝ 2 ⎠ sides of the equation.

2 + y sec 2 x
=
2− y 4

8 + 4y 8 + 4y
Hence, sec 2 x = sec 2 x = A1 aef
2− y 2− y
[8]

11 marks

10
PhysicsAndMathsTutor.com

Question
Scheme Marks
Number

8. (a) At P (4, 2 3) either 4 = 8cos t or 2 3 = 4sin 2t 4 = 8cos t or 2 3 = 4sin 2t M1

t = π3 or awrt 1.05 (radians) only


⇒ only solution is t = π3 where 0 „ t „ π
2 A1
π
stated in the range 0 „ t „ 2
[2]

(b) x = 8cos t , y = 4sin 2t


Attempt to differentiate both x and y
dx dy wrt t to give ± p sin t and M1
= − 8sin t , = 8cos 2t ± q cos 2t respectively
dt dt
Correct dx
dt
and dy
dt
A1

Divides in correct way round


and attempts to substitute their
dy 8cos ( 23π )
At P, = value of t (in degrees or M1*
dx −8sin ( π3 )
radians) into their ddyx
expression.

You may need to check


⎧ ⎫ candidate’s substitutions for
⎪ 8 ( − 12 ) 1 ⎪
⎨ = = = ⎬ M1*
( )
awrt 0.58
⎪ ( −8 ) 2 ⎪
3
3 Note the next two method
⎩ ⎭
marks are dependent on M1*

−1 1
Hence m(N) = − 3 or 1
Uses m(N) = − . dM1*
3
their m(T)

Uses y − 2 3 = ( their mN )( x − 4 )
or finds c using x = 4 and
N: y − 2 3 = − 3 ( x − 4 ) dM1*
y = 2 3 and uses
y = (their m N ) x + " c " .

N: y = − 3 x + 6 3 AG y = − 3x + 6 3 A1 cso
AG

or 2 3 = − 3 ( 4) + c ⇒ c = 2 3 + 4 3 = 6 3
so N: ⎡⎣ y = − 3x + 6 3 ⎤⎦
[6]

11
PhysicsAndMathsTutor.com

Question Scheme Marks


dx
4
π
3 attempt at A = y dt M1
8. (c) A = ∫ y dx = ∫ 4sin 2t. ( −8sin t ) dt dt
π correct expression
0 2 A1
(ignore limits and dt )
π π

Seeing sin 2t = 2sin t cos t


3 3

A = ∫ −32sin 2t.sin t dt =
π
∫ −32 ( 2sin t cos t ) .sin t dt
π anywhere in PART (c).
M1
2 2

π
3

A= ∫ −64.sin
2
t cos t dt Correct proof. Appreciation
π
2 of how the negative sign
affects the limits. A1 AG
π
2 Note that the answer is
A= ∫ 64.sin given in the question.
2
t cos t dt
π
3

[4]

(d) {Using substitution u = sin t ⇒ ddut = cos t }


{change limits:
when t = π3 , u = 23 & when t = π2 , u = 1 }

⎡ sin 3 t ⎤ 2
π
⎡ u3 ⎤
1 k sin 3 t or ku 3 with u = sin t M1
A = 64 ⎢ ⎥ or A = 64 ⎢ ⎥ Correct integration
⎣ 3 ⎦ π3 ⎣3⎦ 2
3
ignoring limits.
A1

Substitutes limits of either


( t = π2 and t = π3 ) or
⎡ 1 ⎛ 1 3 3 3 ⎞⎤
A = 64 ⎢ − ⎜⎜ . . .
( u = 1 and u = )2
3
and dM1
⎟⎟ ⎥
⎣⎢ 3 ⎝ 3 2 2 2 ⎠ ⎥⎦ subtracts the correct way
round.

⎛1 1 ⎞ 64 64
−8 3 A1 aef
A = 64 ⎜ − 3⎟ = −8 3
⎝3 8 ⎠ 3 3 isw
Aef in the form a + b 3 , [4]
with awrt 21.3 and anything
that cancels to a = 643 and
b = − 8.
(Note that a = 64
3
, b = − 8)

16
marks

12
PhysicsAndMathsTutor.com

Mark Scheme (Final)


January 2009

GCE

GCE Core Mathematics C4 (6666/01)

Edexcel Limited. Registered in England and Wales No. 4496750


Registered Office: One90 High Holborn, London WC1V 7BH
PhysicsAndMathsTutor.com

January 2009
6666 Core Mathematics C4
Mark Scheme
Question
Scheme Marks
Number

1. (a) C: y 2 − 3 y = x3 + 8
Differentiates implicitly to include either
⎧ dy ⎫ dy dy dy dy ⎛ dy ⎞ M1
⎨ =⎬ 2y − 3 = 3x 2 ± ky or ± 3 . (Ignore ⎜ = ⎟ .)
⎩ dx ⎭ dx dx dx dx ⎝ dx ⎠
Correct equation. A1

A correct (condoning sign error) attempt to


dy dy dy
( 2 y − 3) = 3 x 2 combine or factorise their ‘ 2 y − 3 ’. M1
dx dx dx
Can be implied.

dy 3x 2 3x 2
= A1 oe
dx 2 y − 3 2y −3
[4]

(b) y = 3 ⇒ 9 − 3(3) = x3 + 8 Substitutes y = 3 into C. M1

x3 = − 8 ⇒ x = − 2 Only x = − 2 A1

dy
= 4 from correct working.
dx
dy 3(4) dy Also can be ft using their ‘x’ value and y = 3 in the A1
(−2,3) ⇒ = ⇒ =4
dx 6 − 3 dx
dy 3x 2
correct part (a) of =
dx 2 y − 3
[3]

7 marks

1(b) final A1 . Note if the candidate inserts their x value and y = 3


2
dy 3x dy
into = , then an answer of = their x 2 , may indicate a
dx 2 y − 3 dx
correct follow through.
PhysicsAndMathsTutor.com

Question
Scheme Marks
Number

2 2
3
∫ dx = ∫ 3(1 + 4 x) 2 dx
−1
2. (a) Area(R) =
0 (1 + 4 x) 0

− 12
Integrating 3(1 + 4 x) to give
2 M1
⎡ 3(1 + 4 x) ⎤
1
± k (1 + 4 x) 2 .
1
2

=⎢ ⎥
⎢⎣ ⎥⎦ 0
1
2 .4 Correct integration.
A1
Ignore limits.
2
= ⎡ 32 (1 + 4 x) 2 ⎤
1

⎣ ⎦0

Substitutes limits of 2 and 0 into a


= ( 3
2 )
9 − ( 32 (1) ) changed function and subtracts the M1
correct way round.

= 9
2
− 3
2
= 3 (units) 2 3 A1
[4]
(Answer of 3 with no working scores M0A0M0A0.)

Use of V = π ∫ y 2 dx .
2
⎛ ⎞
2


3
Volume = π ⎜
⎜ (1 + 4 x) ⎟⎟
(b) dx B1
0 ⎝ ⎠ Can be implied. Ignore limits and dx .

2
9
= (π ) ∫ dx
0
1 + 4x

± k ln 1 + 4 x M1
= (π ) ⎡⎣ 94 ln 1 + 4x ⎤⎦ 0
2
9
4
ln 1 + 4x A1

= (π ) ⎡⎣( 94 ln 9 ) − ( 94 ln1) ⎤⎦
Substitutes limits of 2 and 0
dM1
and subtracts the correct way round.

So Volume = 9
4
π ln 9 9
4
π ln 9 or 92 π ln 3 or 18
4
π ln 3 A1 oe isw
[5]

9 marks

Note the answer must be a one term exact Note that ln1 can be implied as equal to 0.
value. Note, also you can ignore
subsequent working here.
Note that = 9
4
π ln 9 + c (oe.) would be awarded the final A0.
PhysicsAndMathsTutor.com

Question
Scheme Marks
Number

3. (a) 27 x 2 + 32 x + 16 ≡ A(3 x + 2)(1 − x) + B (1 − x) + C (3 x + 2) 2 Forming this identity M1

Substitutes either x = − 23 or x = 1
x = − 23 , 12 − 64
3
+ 16 = ( 53 ) B ⇒ 20
3
= ( 53 ) B ⇒ B = 4 into their identity or equates 3
terms or substitutes in values to M1
write down three simultaneous
x = 1, 27 + 32 + 16 = 25 C ⇒ 75 = 25 C ⇒ C = 3 equations.
Both B = 4 and C = 3 A1
(Note the A1 is dependent on
both method marks in this part.)

27 = − 3 A + 9C ⇒ 27 = − 3 A + 27 ⇒ 0 = − 3 A
Equate x2: Compares coefficients or
⇒ A=0
substitutes in a third x-value or
B1
uses simultaneous equations to
x = 0, 16 = 2 A + B + 4C
show A = 0.
⇒ 16 = 2 A + 4 + 12 ⇒ 0 = 2 A ⇒ A = 0
[4]

4 3
(b) f ( x) = +
(3 x + 2) 2
(1 − x)

Moving powers to top on any one


= 4(3 x + 2) −2 + 3(1 − x) −1 M1
of the two expressions

= 4 ⎡ 2 (1 + 32 x ) ⎤ + 3(1 − x) −1
−2

⎣ ⎦

= 1(1 + 32 x ) + 3(1 − x) −1
−2

⎧ Either 1 ± (− 2)( 32x ) or


(−2)(−3) 3 x 2 ⎫
= 1 ⎨1 + (− 2)( 32x ); + ( 2 ) + ...⎬ 1 ± (−1)( − x) from either first or dM1;
⎩ 2! ⎭
second expansions respectively
Ignoring 1 and 3, any one
⎧ (−1)(−2) ⎫ correct {..........} expansion.
A1
+ 3 ⎨1 + (−1)(− x); + (− x) 2 + ...⎬
⎩ ⎭
Both {..........} correct. A1
2!

{ {
= 1 − 3 x + 274 x 2 + ...} + 3 1 + x + x 2 + ...}

= 4 + 0 x ; + 394 x 2 4 + (0 x) ; 39
4
x2 A1; A1
[6]
PhysicsAndMathsTutor.com

Question
Scheme Marks
Number

3. (c) 1.08 + 6.4 + 16


Actual = f (0.2) = Attempt to find the
(6.76)(0.8) actual value of f(0.2)
23.48 2935 or seeing awrt 4.3 and believing it
= = 4.341715976... =
5.408 676 is candidate’s actual f(0.2).

Or Candidates can also attempt to find M1


4 3 the actual value by using
Actual = f (0.2) = + A B C
(3(0.2) + 2) 2
(1 − 0.2) + +
(3x + 2) (3x + 2) 2
(1 − x)
4 2935
= + 3.75 = 4.341715976... = with their A, B and C.
6.76 676

Estimate = f (0.2) = 4 + 39
4
(0.2) 2 Attempt to find an estimate for
= 4 + 0.39 = 4.39 f(0.2) using their answer to (b) M1

4.39 − 4.341715976... their estimate - actual


%age error = × 100 × 100 M1
4.341715976... actual

= 1.112095408... = 1.1 % (2sf ) 1.1% A1 cao


[4]

14 marks
PhysicsAndMathsTutor.com

Question
Scheme Marks
Number

4. (a) d 1 = − 2 i + j − 4k , d 2 = q i + 2 j + 2 k

As
⎧ ⎛ −2⎞ ⎛ q ⎞ ⎫ Apply dot product calculation between
⎪ ⎜ ⎟ ⎜ ⎟ ⎪ two direction vectors, ie. M1
⎨d1 • d 2 = ⎜ 1 ⎟ • ⎜ 2 ⎟ ⎬ = (−2 × q) + (1 × 2) + (−4 × 2) (−2 × q ) + (1 × 2) + (−4 × 2)
⎪ ⎜ −4⎟ ⎜ 2⎟ ⎪
⎩ ⎝ ⎠ ⎝ ⎠ ⎭

d1 • d 2 = 0 ⇒ − 2q + 2 − 8 = 0 Sets d1 • d 2 = 0
A1 cso
− 2q = 6 ⇒ q = − 3 AG and solves to find q = − 3
[2]

(b) Lines meet where:

⎛ 11 ⎞ ⎛ −2 ⎞ ⎛ −5 ⎞ ⎛q⎞
⎜ ⎟ ⎜ ⎟ ⎜ ⎟ ⎜ ⎟
⎜ 2 ⎟ + λ ⎜ 1 ⎟ = ⎜ 11 ⎟ + µ ⎜ 2 ⎟
⎜ 17 ⎟ ⎜ −4 ⎟ ⎜ p⎟ ⎜ 2⎟
⎝ ⎠ ⎝ ⎠ ⎝ ⎠ ⎝ ⎠

i : 11 − 2λ = − 5 + q µ (1) Need to see equations


(1) and (2).
First two of j : 2 + λ = 11 + 2 µ (2)
Condone one slip. M1
k : 17 − 4 λ = p + 2µ (3) (Note that q = −3 .)

Attempts to solve (1) and (2) to find


(1) + 2(2) gives: 15 = 17 + µ ⇒ µ = −2
one of either λ or µ dM1
Any one of λ = 5 or µ = − 2 A1
(2) gives: 2 + λ = 11 − 4 ⇒ λ = 5
Both λ = 5 and µ = − 2 A1

Attempt to substitute their λ and µ


(3) ⇒ 17 − 4(5) = p + 2(−2) into their k component to give an ddM1
equation in p alone.

⇒ p = 17 − 20 + 4 ⇒ p = 1 p =1 A1 cso
[6]

⎛ 11 ⎞ ⎛ − 2⎞ ⎛ −5⎞ ⎛ − 3⎞
⎜ ⎟ ⎜ ⎟ ⎜ ⎟ ⎜ ⎟ Substitutes their value of λ or µ into
(c) r = ⎜ 2 ⎟ + 5⎜ 1 ⎟ or r = ⎜ 11 ⎟ − 2 ⎜ 2 ⎟ M1
⎜ 17 ⎟ ⎜ − 4⎟ ⎜ 1 ⎟ ⎜ 2⎟ the correct line l1 or l2 .
⎝ ⎠ ⎝ ⎠ ⎝ ⎠ ⎝ ⎠

⎛ 1 ⎞ ⎛ 1 ⎞
⎜ ⎟ ⎜ ⎟
Intersect at r = ⎜ 7 ⎟ or (1, 7, − 3) ⎜ 7 ⎟ or (1, 7, − 3) A1
⎜ − 3⎟ ⎜ − 3⎟
⎝ ⎠ ⎝ ⎠
[2]
PhysicsAndMathsTutor.com

Question
Scheme Marks
Number
uuur
(d) Let OX = i + 7 j − 3k be point of intersection
uuur
⎛ 1 ⎞ ⎛ 9 ⎞ ⎛ −8 ⎞ Finding vector AX by finding the
uuur uuur uuur ⎜ ⎟ ⎜ ⎟ ⎜ ⎟ uuur uuur
AX = OX − OA = ⎜ 7 ⎟ − ⎜ 3 ⎟ = ⎜ 4 ⎟ difference between OX and OA . Can M1 ±
⎜ − 3 ⎟ ⎜13 ⎟ ⎜ −16 ⎟ uuur
⎝ ⎠ ⎝ ⎠ ⎝ ⎠ be ft using candidate’s OX .
uuur uuur uuur uuur uuur
OB = OA + AB = OA + 2 AX

⎛9⎞ ⎛ −8 ⎞ ⎛9⎞ ⎛ ⎞
uuur ⎜ ⎟ ⎜ ⎟ ⎜ ⎟ ⎜ uuur ⎟
OB = ⎜ 3 ⎟ + 2⎜ 4 ⎟ ⎜3⎟+ 2 ⎜ their AX ⎟ dM1
⎜ 13 ⎟ ⎜ −16 ⎟ ⎜ 13 ⎟ ⎜ ⎟
⎝ ⎠ ⎝ ⎠ ⎝ ⎠ ⎝ ⎠

⎛ −7 ⎞
⎛ −7 ⎞ ⎜ ⎟
uuur ⎜ ⎟ uuur
Hence, OB = ⎜ 11 ⎟ or OB = −7 i + 11 j − 19 k ⎜ 11 ⎟ or −7 i + 11 j − 19 k
⎜ −19 ⎟ A1
⎜ −19 ⎟ ⎝ ⎠
⎝ ⎠
or ( −7, 11, − 19 )
[3]

13 marks
PhysicsAndMathsTutor.com

Question
Scheme Marks
Number

r 16 2h Uses similar triangles, ratios or


5. (a) Similar triangles ⇒ = ⇒ r= trigonometry to find either one of these M1
h 24 3 two expressions oe.

4 π h3 Substitutes r =
2
1 1 ⎛ 2h ⎞ 2h
into the formula for the
V = π r 2h = π ⎜ ⎟ h = AG 3
A1
3 3 ⎝ 3 ⎠ 27 volume of water V.
[2]

dV dV
(b) From the question, =8 =8 B1
dt dt

dV 12 π h 2 4 π h 2 dV 12 π h 2 4π h2
= = = or B1
dh 27 9 dh 27 9

dV dV
Candidate’s ÷ ; M1;
dh dV dV 9 18 dt dh
= ÷ = 8× =
dt dt dh 4π h 2
π h2 ⎛ 12 π h 2 ⎞ 9 18
8÷⎜ ⎟ or 8 × or oe A1
⎝ 27 ⎠ 4π h 2
π h2

dh 18 1 18 1
When h = 12, = = or 8π A1 oe isw
dt 144 π 8π 144 π
[5]

7 marks

Note the answer must be a one term exact value.


18
Note, also you can ignore subsequent working after .
144 π
PhysicsAndMathsTutor.com

Question
Scheme Marks
Number

∫ tan
2
6. (a) x dx

⎡ NB : sec 2 A = 1+ tan 2 A gives tan 2 A = sec2 A − 1 ⎤ The correct underlined identity. M1 oe


⎣ ⎦

= ∫ sec 2 x − 1 dx

= tan x − x ( + c ) Correct integration


A1
with/without + c
[2]

∫ x ln x dx
1
(b) 3

⎧⎪u = ln x ⇒ ddux = 1x ⎫⎪
⎨ dv −3 −2 −1 ⎬
⎪⎩ dx = x ⇒ v = x−2 = 2 x2 ⎪⎭


1 1 1 Use of ‘integration by parts’ formula
=− 2
ln x − − 2 . dx in the correct direction.
M1
2x 2x x
Correct expression. A1


1 1 1 An attempt to multiply through
=− ln x + dx
2x 2
2 x3 k
, n ∈ , n … 2 by 1x and an
xn
1 1⎛ 1 ⎞ attempt to ...
=− ln x + ⎜ − 2 ⎟ ( + c )
2x 2
2 ⎝ 2x ⎠
… “integrate”(process the result); M1

correct solution with/without + c A1 oe


[4]

Correct direction means that u = ln x .


PhysicsAndMathsTutor.com

Question
Scheme Marks
Number

e3 x
(c)
∫ 1 + ex
dx

⎧ du dx 1 dx 1 ⎫ Differentiating to find any one of the


⎨u = 1 + e ⇒ = ex , = x , =
x
⎬ B1
⎩ dx du e du u − 1⎭ three underlined

Attempt to substitute for e 2 x = f (u ) ,


e 2 x .e x (u − 1) 2 .e x 1
=
∫ 1 + ex
dx =
∫ u
. x du
e their
dx 1
= x and u = 1 + e x
du e
M1*
dx 1
(u − 1)3 or e3 x = f (u ) , their = and

1
or = . du du u − 1
u (u − 1)
u = 1 + ex .

(u − 1) 2 (u − 1) 2
=
∫ u
du ∫ u
du A1

u 2 − 2u + 1
=
∫ u
du An attempt to
multiply out their numerator
to give at least three terms

1
= u−2+ du and divide through each term by u dM1*
u

u2 Correct integration
= − 2u + ln u ( + c ) with/without +c A1
2

(1 + e x ) 2 Substitutes u = 1 + e x back into their


= − 2(1 + e x ) + ln(1 + e x ) + c integrated expression with at least dM1*
2 two terms.

= 12 + e x + 12 e 2 x − 2 − 2e x + ln(1 + e x ) + c

= 12 + e x + 12 e 2 x − 2 − 2e x + ln(1 + e x ) + c

= 12 e 2 x − e x + ln(1 + e x ) − 32 + c
1
e 2 x − e x + ln(1 + e x ) + k
= 12 e 2 x − e x + ln(1 + e x ) + k
2
AG
must use a + c and " − 32 " combined. A1 cso
[7]

13 marks
PhysicsAndMathsTutor.com

Question
Scheme Marks
Number

7. (a) At A, x = −1 + 8 = 7 & y = (−1) 2 = 1 ⇒ A(7,1) A(7,1) B1


[1]

(b) x = t 3 − 8t , y = t2 ,

dx dy
= 3t 2 − 8 , = 2t
dt dt

dy dx
dy 2t Their divided by their M1
∴ = 2
dt dt

dx 3t − 8 Correct dy
dx A1

2( −1) −2 −2 2
At A, m(T) = = = = Substitutes for t to give any of the
A1
3(−1) − 8
2
3−8 −5 5 four underlined oe:

T : y − ( their 1) = mT ( x − ( their 7 ) ) Finding an equation of a tangent


with their point and their tangent
gradient
or 1 = 2
5 (7) + c ⇒ c =1− 14
5
= − 95 or finds c and uses dM1
y = (their gradient) x + " c " .
Hence T : y = 52 x − 95

gives T : 2 x − 5 y − 9 = 0 AG 2x − 5 y − 9 = 0 A1 cso
[5]

Substitution of both x = t 3 − 8t and


(c) 2(t 3 − 8t ) − 5t 2 − 9 = 0 M1
y = t 2 into T

2t 3 − 5t 2 − 16t − 9 = 0

(t + 1) {(2t 2 − 7t − 9) = 0} A realisation that


(t + 1) {(t + 1)(2t − 9) = 0} ( t + 1) is a factor. dM1

{t = −1 (at A)} t= 9
2
at B t= 9
2 A1

Candidate uses their value of t to


x = ( 92 ) − 8 ( 92 ) =
2 729
8 − 36 = 441
8 = 55.125 or awrt 55.1 find either the x or y coordinate
ddM1

y = ( 92 ) = 814 = 20.25 or awrt 20.3


2
One of either x or y correct. A1
Both x and y correct. A1
Hence B ( 441
8
, 814 ) awrt [6]

12 marks
PhysicsAndMathsTutor.com

• Note: dM1 denotes a method mark which is dependent upon the award of the previous method mark.
ddM1 denotes a method mark which is dependent upon the award of the previous two method marks.
Oe or equivalent.
PhysicsAndMathsTutor.com

January 2009
6666 Core Mathematics C4
Appendix
Question 1
Question
Scheme Marks
Number

Aliter
1. (a) C: y 2 − 3 y = x3 + 8
Way 2
Differentiates implicitly to include either
⎪⎧ dx ⎪⎫ dx dx ⎛ dx ⎞ M1
⎨ =⎬ 2 y − 3 = 3x 2 ± kx 2 . (Ignore ⎜ = ⎟ .)
⎩⎪ dy ⎪⎭ dy dy ⎝ d y ⎠
Correct equation. A1

1 dx 1
2 y − 3 = 3x 2 Applies = dy
( )
dy
dx
dy dx ( ) dM1

dy 3x 2 3x 2
= A1 oe
dx 2 y − 3 2y −3
[4]

Aliter
1. (a) C: y 2 − 3 y = x3 + 8
Way 3

gives x 3 = y 2 − 3 y − 8
⇒ x = ( y 2 − 3 y − 8)
1
3

( f ( y ) ) ( f ′( y ) ) .
− 23
= ( y 2 − 3 y − 8 ) ( 2 y − 3)
dx 1 − 23 Differentiates in the form 1
3
M1
dy 3 Correct differentiation. A1

dx 2y − 3
=
3 ( y 2 − 3 y − 8)
2
dy 3

3 ( y 2 − 3 y − 8)
2
3
dy 1
dy Applies =
dx
=
2y − 3 dx d
dy
x
( ) dM1

3 ( x3 ) 3 ( x3 )
2 2
3 3
dy dy 3x 2 3x 2
= ⇒ = or A1 oe
dx 2y − 3 dx 2 y − 3 2y − 3 2y −3
[4]
PhysicsAndMathsTutor.com

Question 2
Question
Scheme Marks
Number

Aliter 2
3
2

∫ dx = ∫ 3(1 + 4 x)
− 12
2. (a) Area(R) = dx
Way 2 0 (1 + 4 x) 0

{Using substitution u = 1 + 4 x ⇒ ddux = 4 }


{change limits:
When x = 0 , u = 1 & when x = 2 , u = 9 }
9

∫ 3u
− 12 1
So, Area(R) = 4 du
1
− 12
Integrating ± λ u
2
to give ± k u 2 . M1
1
⎡ 3 u 12 ⎤
=⎢ ⎥
⎢⎣ 4 ( 12 ) ⎥⎦ Correct integration.
A1
0 Ignore limits.
9
= ⎡ 32 u 2 ⎤
1

⎣ ⎦1
Substitutes limits of either
( u = 9 and u = 1) or
= ( 3
2 )
9 − ( 32 (1) ) in x, ( x = 2 and x = 0 ) into a changed M1
function and subtracts the correct way
round .
= 9
2
− 3
2
= 3 (units) 2 3 A1
[4]
Aliter 2
3
2

∫ dx = ∫ 3(1 + 4 x) 2 dx
−1
2. (a) Area(R) =
Way 3 0 (1 + 4 x) 0

{Using substitution
u 2 = 1 + 4 x ⇒ 2u ddux = 4 ⇒ 12 udu = dx }
{change limits:
When x = 0 , u = 1 & when x = 2 , u = 3 }
3 3
So, Area(R) = ∫
1
3 1
u 2 u du = ∫
1
3
2 du

3 Integrating ± λ to give ± k u . M1
⎡3 ⎤
=⎢ u⎥ Correct integration.
⎣2 ⎦1 A1
Ignore limits.

Substitutes limits of either


( u = 3 and u = 1) or
= ( 32 (3) ) − ( 32 (1) ) in x, ( x = 2 and x = 0 ) into a changed M1
function and subtracts the correct way
round .

= 9
2
− 3
2
= 3 (units) 2 3 A1
[4]
PhysicsAndMathsTutor.com

Question 3
Question
Scheme Marks
Number

Aliter
3. (a) 27 x 2 + 32 x + 16 ≡ A(3x + 2)(1 − x) + B(1 − x) + C (3 x + 2) 2 Forming this identity M1
Way 2

x 2 terms : 27 = − 3 A + 9C (1)
x terms : 32 = A − B + 12C (2) equates 3 terms. M1
constants: 16 = 2 A + B + 4C (3)

(2) + (3) gives 48 = 3 A + 16C (4)

(1) + (4) gives 75 = 25C ⇒ C = 3

(1) gives 27 = − 3 A + 27 ⇒ 0 = − 3 A ⇒ A = 0

(2) gives 32 = − B + 36 ⇒ B = 36 − 32 = 4 Both B = 4 and C = 3 A1


Decide to award B1 for A = 0 B1
[4]

3. (a) If the candidate assumes A = 0 and writes the identity 27 x 2 + 32 x + 16 ≡ B(1 − x) + C (3x + 2) 2
and goes on to find B = 4 and C = 3 then the candidate is awarded M0M1A0B0.

3. (a) If the candidate has the incorrect identity 27 x 2 + 32 x + 16 ≡ A(3 x + 2) + B(1 − x) + C (3 x + 2) 2 and
goes on to find B = 4, C = 3 and A = 0 then the candidate is awarded M0M1A0B1.

3. (a) If the candidate has the incorrect identity


27 x 2 + 32 x + 16 ≡ A(3 x + 2) 2 (1 − x) + B (1 − x) + C (3 x + 2) 2 and goes on to find B = 4, C = 3 and
A = 0 then the candidate is awarded M0M1A0B1.
PhysicsAndMathsTutor.com

Question
Scheme Marks
Number

Aliter 4 3
3. (b) f ( x) = +
(3 x + 2) 2 (1 − x)
Way 2

Moving powers to top on any one


= 4(3 x + 2) −2 + 3(1 − x) −1 M1
of the two expressions

= 4(2 + 3 x) −2 + 3(1 − x) −1

Either (2) − 2 ± (− 2)(2) −3 (3x) or


⎧ (− 2)( − 3) − 4 ⎫
= 4 ⎨(2) − 2 + (− 2)(2) −3 (3 x); + (2) (3x) 2 + ⎬ 1 ± (−1)(− x) from either first or dM1;
⎩ 2! ⎭
second expansions respectively
Ignoring 1 and 3, any one
⎧ (−1)(−2) ⎫ correct {..........} expansion.
A1
+ 3 ⎨1 + (−1)(− x); + (− x) 2 + ...⎬
⎩ ⎭
Both {..........} correct. A1
2!

=4 { 1
4 − 43 x + 16
27 2
{
x + ...} + 3 1 + x + x 2 + ...}

= 4 + 0 x ; + 394 x 2 4 + (0 x) ; 39
4
x2 A1; A1
[6]
PhysicsAndMathsTutor.com

Question
Scheme Marks
Number

Aliter 1.08 + 6.4 + 16


3. (c) Actual = f (0.2) =
(6.76)(0.8) Attempt to find the
Way 2 M1
23.48 2935 actual value of f(0.2)
= = 4.341715976... =
5.408 676

Estimate = f (0.2) = 4 + 39
4
(0.2) 2 Attempt to find an estimate for f(0.2)
= 4 + 0.39 = 4.39 using their answer to (b) M1

⎛ 4.39 ⎞ ⎛ ⎛ their estimate ⎞ ⎞


%age error = 100 − ⎜ × 100 ⎟ 100 − ⎜ ⎜ ⎟ × 100 ⎟ M1
⎝ 4.341715976... ⎠ ⎝⎝ actual ⎠ ⎠

= 100 − 101.1120954

= −1.112095408... = 1.1 % (2sf ) 1.1% A1 cao


[4]

3. (c) Note that:


4.39 − 4.341715976...
%age error = × 100
4.39 Should be awarded the final marks of
M0A0
= 1.0998638... = 1.1 % (2sf )

3. (c) Also note that:


⎛ 4.341715976... ⎞
%age error = 100 − ⎜ × 100 ⎟
⎝ 4.39 ⎠ Should be awarded the final marks of
M0A0
= 1.0998638... = 1.1 % (2sf )

…so be wary of 1.0998638…


PhysicsAndMathsTutor.com

Question 4
Question
Scheme Marks
Number

4. (a) − 2q + 2 − 8 is sufficient for M1.

Aliter
Only apply Way 2 if candidate does not find both λ and
4. (b) µ.
Way 2
Lines meet where:

⎛ 11 ⎞ ⎛ −2 ⎞ ⎛ −5 ⎞ ⎛q⎞
⎜ ⎟ ⎜ ⎟ ⎜ ⎟ ⎜ ⎟
⎜ 2 ⎟ + λ ⎜ 1 ⎟ = ⎜ 11 ⎟ + µ ⎜ 2 ⎟
⎜ 17 ⎟ ⎜ −4 ⎟ ⎜ p⎟ ⎜ 2⎟
⎝ ⎠ ⎝ ⎠ ⎝ ⎠ ⎝ ⎠

i : 11 − 2λ = − 5 + q µ (1) Need to see equations


(2) and (2).
First two of j : 2 + λ = 11 + 2 µ (2)
Condone one slip. M1
k : 17 − 4 λ = p + 2µ (3) (Note that q = −3 .)

(2) gives λ = 9 + 2µ

Attempts to solve (1) and (2) to find


(1) gives 11 − 2(9 + 2 µ ) = − 5 − 3µ
one of either λ or µ dM1

11 − 18 − 4µ = − 5 − 3µ

gives: 11 − 18 + 5 = µ ⇒ µ = −2 Any one of λ = 5 or µ = − 2 A1

Candidate writes down a correct


(3) gives 17 − 4(9 + 2 µ ) = p + 2µ equation containing p and one of either A1
λ or µ which has already been found.

Attempt to substitute their value for


λ ( = 9 + 2 µ ) and µ into their
(3) ⇒ 17 − 4(9 + 2(−2)) = p + 2(−2) ddM1
k component to give an equation in
p alone.

⇒ 17 − 20 = p − 4 ⇒ p = 1 p =1 A1 cso
[6]

If no working is shown then any two out of the three


4. (c) M1
coordinates can imply the first M1 mark.

⎛ 1 ⎞ ⎛ 1 ⎞
⎜ ⎟ ⎜ ⎟
Intersect at r = ⎜ 7 ⎟ or (1, 7, − 3) ⎜ 7 ⎟ or (1, 7, − 3) A1
⎜ − 3⎟ ⎜ − 3⎟
⎝ ⎠ ⎝ ⎠
[2]
PhysicsAndMathsTutor.com

Question
Scheme Marks
Number
Aliter
uuur
4. (d) Let OX = i + 7 j − 3k be point of intersection
Way 2
Finding the difference between their
uuur uuur
⎛ 1 ⎞ ⎛ 9 ⎞ ⎛ −8 ⎞ OX (can be implied) and OA .
uuur uuur uuur ⎜ ⎟ ⎜ ⎟ ⎜ ⎟
AX = OX − OA = ⎜ 7 ⎟ − ⎜ 3 ⎟ = ⎜ 4 ⎟ ⎛ ⎛ 1 ⎞ ⎛ 9 ⎞ ⎞ M1 ±
uuur ⎜⎜ ⎟ ⎜ ⎟⎟
⎜ − 3 ⎟ ⎜ 13 ⎟ ⎜ −16 ⎟ AX = ± ⎜ ⎜ 7 ⎟ − ⎜ 3 ⎟ ⎟
⎝ ⎠ ⎝ ⎠ ⎝ ⎠
⎜ ⎜ − 3 ⎟ ⎜ 13 ⎟ ⎟
⎝⎝ ⎠ ⎝ ⎠⎠
uuur uuur uuur uuur uuur
OB = OX + XB = OX + AX

⎛ 1 ⎞ ⎛ −8 ⎞ ⎛ ⎞ ⎛ ⎞
uuur ⎜ ⎟ ⎜ ⎟ ⎜ uuur ⎟ ⎜ uuur ⎟
OB = ⎜ 7 ⎟ + ⎜ 4 ⎟ ⎜ their OX ⎟ + ⎜ their AX ⎟ dM1
⎜ −3 ⎟ ⎜ −16 ⎟ ⎜ ⎟ ⎜ ⎟
⎝ ⎠ ⎝ ⎠ ⎝ ⎠ ⎝ ⎠

⎛ −7 ⎞
⎛ −7 ⎞ ⎜ ⎟
uuur ⎜ ⎟ uuur
Hence, OB = ⎜ 11 ⎟ or OB = −7 i + 11 j − 19 k ⎜ 11 ⎟ or −7 i + 11 j − 19 k
⎜ −19 ⎟ A1
⎜ −19 ⎟ ⎝ ⎠
⎝ ⎠
or ( −7, 11, − 19 )
[3]

Aliter At A, λ = 1. At X, λ = 5.
4. (d)
Way 3 λB = ( their λX ) + ( their λX − their λA )
Hence at B, λ = 5 + (5 − 1) = 9 M1
λB = 2 ( their λX ) − ( their λA )

⎛ 11 ⎞ ⎛ − 2⎞
uuur ⎜ ⎟ ⎜ ⎟ Substitutes their value of λ into the
OB = ⎜ 2 ⎟ + 9 ⎜ 1 ⎟
⎜17 ⎟ ⎜ − 4⎟ line l1. dM1
⎝ ⎠ ⎝ ⎠

⎛ −7 ⎞
⎛ −7 ⎞ ⎜ ⎟
uuur ⎜ ⎟ uuur
Hence, OB = ⎜ 11 ⎟ or OB = −7 i + 11 j − 19 k ⎜ 11 ⎟ or −7 i + 11 j − 19 k
⎜ −19 ⎟ A1
⎜ −19 ⎟ ⎝ ⎠
⎝ ⎠
or ( −7, 11, − 19 )
[3]
PhysicsAndMathsTutor.com

Question
Scheme Marks
Number
uuur
Aliter OA = 9i + 3j + 13k
uuur
4. (d) and the point of intersection OX = i + 7 j − 3k
Way 4
Finding the difference
uuur
between their OX (can be
⎛ 9 ⎞ ⎛ Minus 8 ⎞ ⎛ 1 ⎞ uuur
⎜ ⎟ ⎜ ⎟ ⎜ ⎟ implied) and OA .
⎜ 3 ⎟ → ⎜ Plus 4 ⎟ → ⎜ 7 ⎟ ⎛ ⎛ 1 ⎞ ⎛ 9 ⎞ ⎞ M1 ±
⎜ ⎟ ⎜ ⎟ ⎜ ⎟ uuur ⎜⎜ ⎟ ⎜ ⎟⎟
⎝ 13 ⎠ ⎝ Minus 16 ⎠ ⎝ − 3 ⎠ ( )
AX = ± ⎜ ⎜ 7 ⎟ − ⎜ 3 ⎟ ⎟
⎜ ⎜ − 3 ⎟ ⎜13 ⎟ ⎟
⎝⎝ ⎠ ⎝ ⎠⎠

⎛ 1 ⎞ ⎛ Minus 8 ⎞ ⎛ − 7 ⎞ ⎛ ⎞ ⎛ ⎞
⎜ ⎟ ⎜ ⎟ ⎜ ⎟ ⎜ uuur ⎟ ⎜ uuur ⎟
⎜ 7 ⎟ → ⎜ Plus 4 ⎟ → ⎜ 11 ⎟ ⎜ their OX ⎟ + ⎜ their AX ⎟ dM1
⎜ − 3 ⎟ ⎜ Minus 16 ⎟ ⎜ −19 ⎟ ⎜ ⎟ ⎜ ⎟
⎝ ⎠ ⎝ ⎠ ⎝ ⎠ ⎝ ⎠ ⎝ ⎠

⎛ −7 ⎞
⎛ −7 ⎞ ⎜ ⎟
uuur ⎜ ⎟ uuur
Hence, OB = ⎜ 11 ⎟ or OB = −7 i + 11 j − 19 k ⎜ 11 ⎟ or −7 i + 11 j − 19 k
⎜ −19 ⎟ A1
⎜ −19 ⎟ ⎝ ⎠
⎝ ⎠
or ( −7, 11, − 19 )
[3]
uuur uuur
Aliter OA = 9i + 3j + 13k and OB = ai + bj + ck
uuur
4. (d) and the point of intersection OX = i + 7 j − 3k
Way 5
As X is the midpoint of AB, then

9 + a 3 + b 13 + c ⎞
(1, 7, − 3) = ⎛⎜ , , ⎟
Writing down any two of
these “equations” correctly. M1
⎝ 2 2 2 ⎠

a = 2(1) − 9 = − 7
b = 2(7) − 3 = 11 An attempt to find at least
two of a, b or c. dM1
c = 2(− 3) − 13 = −19

⎛ −7 ⎞
⎜ ⎟
⎛ −7 ⎞
uuur ⎜ uuur ⎜ 11 ⎟ or −7 i + 11 j − 19 k
⎟ ⎜ −19 ⎟
Hence, OB = ⎜ 11 ⎟ or OB = −7 i + 11 j − 19 k ⎝ ⎠ A1
⎜ −19 ⎟
⎝ ⎠ or ( −7, 11, − 19 ) or
a = − 7, b = 11, c = −19
[3]
PhysicsAndMathsTutor.com

Question
Scheme Marks
Number
Aliter
uuur
4. (d) Let OX = i + 7 j − 3k be point of intersection
Way 6
Finding the difference
uuur
between their OX (can be
⎛ 1 ⎞ ⎛ 9 ⎞ ⎛ −8 ⎞ uuur
uuur uuur uuur ⎜ ⎟ ⎜ ⎟ ⎜ ⎟ implied) and OA .
AX = OX − OA = ⎜ 7 ⎟ − ⎜ 3 ⎟ = ⎜ 4 ⎟
⎛⎛ 1 ⎞ ⎛ 9 ⎞⎞
⎜ − 3 ⎟ ⎜13 ⎟ ⎜ −16 ⎟ uuur ⎜⎜ ⎟ ⎜ ⎟⎟
⎝ ⎠ ⎝ ⎠ ⎝ ⎠ AX = ± ⎜ ⎜ 7 ⎟ − ⎜ 3 ⎟ ⎟ M1 ±
⎜ ⎜ − 3 ⎟ ⎜ 13 ⎟ ⎟
uuur ⎝⎝ ⎠ ⎝ ⎠⎠
and AX = 64 + 16 + 256 = 336 = 4 21 uuur
Note AX = 336 would
imply M1.

⎛ 1 ⎞ ⎛ 11 − 2λ ⎞ ⎛ −10 + 2λ ⎞
uuur uuur uuur ⎜ ⎟ ⎜ ⎟ ⎜ ⎟
BX = OX − OB = ⎜ 7 ⎟ − ⎜ 2 + λ ⎟ = ⎜ 5 − λ ⎟
⎜ − 3 ⎟ ⎜17 − 4λ ⎟ ⎜ − 20 + 4λ ⎟
⎝ ⎠ ⎝ ⎠ ⎝ ⎠
uuur uuur
Hence BX = AX = 336 gives
Writes distance equation
uuur 2
of BX = 336 where
uuur uuur uuur
BX = OX − OB and
( −10 + 2λ ) + ( 5 − λ ) + ( −20 + 4λ ) = 336
2 2 2
dM1
⎛ 11 − 2λ ⎞
uuur ⎜ ⎟
OB = ⎜ 2 + λ ⎟
⎜17 − 4λ ⎟
⎝ ⎠

100 − 40λ + 4λ 2 + 25 − 10λ + λ 2 + 400 − 160λ + 16λ 2 = 336


21λ 2 − 210λ + 525 = 336
21λ 2 − 210λ + 189 = 0
λ 2 − 10λ + 9 = 0
(λ − 1)(λ − 9) = 0

⎛ 11 − 2(9) ⎞
uuur ⎜ ⎟
At A, λ = 1 and at B λ = 9, so, OB = ⎜ 2 + 9 ⎟
⎜ 17 − 4(9) ⎟
⎝ ⎠
⎛ −7 ⎞
⎛ −7 ⎞ ⎜ ⎟
uuur ⎜ ⎟ uuur
Hence, OB = ⎜ 11 ⎟ or OB = −7 i + 11 j − 19 k ⎜ 11 ⎟ or −7 i + 11 j − 19 k
⎜ −19 ⎟ A1
⎜ −19 ⎟ ⎝ ⎠
⎝ ⎠
or ( −7, 11, − 19 )
[3]
PhysicsAndMathsTutor.com

Question 5
Question
Scheme Marks
Number

5. (a) Similar shapes ⇒ either

π (16) 2 24 ⎛ 24 ⎞
3 3
1
V ⎛ h ⎞
3
=⎜ ⎟ or =⎜ ⎟
⎝ h ⎠ 3 π (16) 24 ⎝ 24 ⎠
1 2
V
Uses similar shapes to find either one of
M1
these two expressions oe.
π r 2 (24) ⎛ 24 ⎞
3 3
1
V ⎛ h ⎞
3
=⎜ ⎟ or =⎜ ⎟
⎝ h ⎠ 3 π r (24) ⎝ 24 ⎠
1 2
V

Substitutes their equation to give the


4 π h3
3
⎛ h ⎞
V = 2048π × ⎜ ⎟ = AG correct formula for the volume A1
⎝ 24 ⎠ 27 of water V.
[2]

5. (a) Candidates simply writing:

2
4 1 3 1 ⎛ 16 ⎞
V = × πh or V = π ⎜ ⎟ h3 would be awarded M0A0.
9 3 3 ⎝ 24 ⎠

dV dV
(b) From question, = 8 ⇒ V = 8t ( + c ) = 8 or V = 8t B1
dt dt
1 1 1 1 1 1 1

⎛ 27V ⎞ 3 ⎛ 27(8t ) ⎞ 3 ⎛ 54t ⎞ 3 ⎛ 2t ⎞ 3 ⎛ 27(8t ) ⎞ 3 ⎛ 54t ⎞ 3 ⎛ 2t ⎞ 3


h=⎜ ⎟ ⇒ h=⎜ ⎟ = ⎜ ⎟ = 3⎜ ⎟ ⎜ ⎟ or ⎜ ⎟ or 3 ⎜ ⎟ B1
⎝ 4π ⎠ ⎝ 4π ⎠ ⎝ π ⎠ ⎝π ⎠ ⎝ 4π ⎠ ⎝ π ⎠ ⎝π ⎠

dh −2
1
= ± k t 3 ; M1;
dh ⎛ 2 ⎞ 1 −2
3 dt
= 3⎜ ⎟ t 3
⎝π ⎠ 3
1
dt dh ⎛ 2 ⎞3 1 −2
= 3⎜ ⎟ t 3 A1 oe
dt ⎝π ⎠ 3

⎛ 12 ⎞ π
3

When h = 12, t = ⎜ ⎟ × = 32π


⎝ 3⎠ 2

So when 1
1 2 1
dh ⎛ 2 ⎞3 ⎛ 1 ⎞3 ⎛ 2 ⎞3 1 8π A1 oe
h = 12, = ⎜ ⎟ ⎜ ⎟ = ⎜ 3 ⎟
=
dt ⎝ π ⎠ ⎝ 32π ⎠ ⎝ 1024π ⎠ 8π
[5]
PhysicsAndMathsTutor.com

Question 7
Question
Scheme Marks
Number

It is acceptable for a candidate to write x = 7, y = 1, to


7. (a) A(7,1) B1
gain B1.
[1]

Aliter
(c) x = t 3 − 8t = t (t 2 − 8) = t ( y − 8)
Way 2

So, x 2 = t 2 ( y − 8) 2 = y ( y − 8) 2

2 x − 5 y − 9 = 0 ⇒ 2 x = 5 y + 9 ⇒ 4 x 2 = (5 y + 9) 2

Forming an equation in terms of y


Hence, 4 y ( y − 8) 2 = (5 y + 9) 2 M1
only.

4 y ( y 2 − 16 y + 64) = 25 y 2 + 90 y + 81

4 y 3 − 64 y 2 + 256 y = 25 y 2 + 90 y + 81

4 y 3 − 89 y 2 + 166 y − 81 = 0

A realisation that
( y − 1)( y − 1)(4 y − 81) = 0 ( y − 1) is a factor. dM1

Correct factorisation A1

y= 81
4
= 20.25 (or awrt 20.3) Correct y-coordinate (see below!)

Candidate uses their y-coordinate


x 2 = 814 ( 814 − 8) 2 ddM1
to find their x-coordinate.
Decide to award A1 here for
A1
correct y-coordinate.
x= 441
8 = 55.125 (or awrt 55.1) Correct x-coordinate A1
Hence B ( 441
8
, 814 ) [6]
PhysicsAndMathsTutor.com

Question
Scheme Marks
Number

Aliter
7. (c) t= y
Way 3

( y) ( y)
3
So x = −8

2 x − 5 y − 9 = 0 yields

( y) ( y ) − 5y − 9 = 0
3
Forming an equation in terms of y
2 − 16 M1
only.

( y) ( y) − 9 = 0
3
⇒2 − 5 y − 16

( ){( ) }
A realisation that
y +1 2y − 7 y − 9 = 0
( )
y + 1 is a factor. dM1

( y +1 ){( )(
y +1 2 y − 9 = 0 ) } Correct factorisation. A1

y= 81
4
= 20.25 (or awrt 20.3) Correct y-coordinate (see below!)

( ) − 8( ) Candidate uses their y-coordinate


3
x= 81 81
ddM1
4 4 to find their x-coordinate.
Decide to award A1 here for
A1
correct y-coordinate.
x= 441
8
= 55.125 (or awrt 55.1) Correct x-coordinate A1
Hence B ( 441
8
, 814 ) [6]
PhysicsAndMathsTutor.com

Mark Scheme (Results)


Summer 2009

GCE

GCE Mathematics (6666/01)


PhysicsAndMathsTutor.com

June 2009
6666 Core Mathematics C4
Mark Scheme

Question
Scheme Marks
Number

Q1 1
f ( x) = = (4 + x) 2
−1
M1
√ (4 + x)
1 1
= ( 4) (1 + ) (1 + ... )
− 12 ... ...
... or B1
2 2 √ (1 + ...)
 1  x
( − 12 ) ( − 32 )  x 2 ( − 12 ) ( − 32 )( − 52 )  x 3 
= ...  1 + ( − 2 )   +   +   + ...  M1 A1ft
 4 2 4 3! 4 
 
x
ft their  
4
1 1 3 2 5 3
= − x, + x − x + ... A1, A1 (6)
2 16 256 2048

[6]

Alternative
1
f ( x) = = (4 + x) 2
−1
M1
√ (4 + x)
( − 12 ) ( − 23 ) ( − 12 ) ( − 32 )( − 52 )
= 4 2 + ( − 12 ) 4 2 x +
−1 −3 −5 −7
4 2 x2 + 4 2 x 3 + ... B1 M1 A1
1.2 1.2.3
1 1 3 2 5 3
= − x, + x − x + ... A1, A1 (6)
2 16 256 2048

6666/01 GCE Mathematics June 2009 2


PhysicsAndMathsTutor.com

Question
Scheme Marks
Number

Q2 (a) 1.14805 awrt 1.14805 B1 (1)

1 3π
(b) A ≈ × ( ... ) B1
2 8
= ... ( 3 + 2 ( 2.77164 + 2.12132 + 1.14805 ) + 0 ) 0 can be implied M1

=
16
( 3 + 2 ( 2.77164 + 2.12132 + 1.14805) ) ft their (a) A1ft


= × 15.08202 ... = 8.884 cao A1 (4)
16

x
3sin  
⌠  x 3
(c)  3cos   dx = M1 A1
⌡ 3 1
3
x
= 9sin  
3

  x  2
A = 9sin    = 9 − 0 = 9 cao A1 (3)
  3  0

[8]

6666/01 GCE Mathematics June 2009 3


PhysicsAndMathsTutor.com

Question
Scheme Marks
Number

4 − 2x A B C
Q3 (a) f ( x) = = + +
( 2 x + 1)( x + 1)( x + 3) 2 x + 1 x + 1 x + 3
4 − 2 x = A ( x + 1)( x + 3) + B ( 2 x + 1)( x + 3) + C ( 2 x + 1)( x + 1) M1
A method for evaluating one constant M1

x → − 12 , 5 = A ( 12 ) ( 52 ) ⇒ A = 4 any one correct constant A1


x → −1 , 6 = B ( −1)( 2 ) ⇒ B = −3
x → −3 , 10 = C ( −5 )( −2 ) ⇒ C = 1 all three constants correct A1 (4)

⌠ 4 3 1 
(b) (i)   − +  dx
⌡  2x +1 x +1 x + 3 
4
= ln ( 2 x + 1) − 3ln ( x + 1) + ln ( x + 3 ) + C A1 two ln terms correct M1 A1ft
2
All three ln terms correct and “+C” ; ft constants A1ft (3)

 2 ln ( 2 x + 1) − 3ln ( x + 1) + ln ( x + 3)  0
2
(ii)
= ( 2 ln 5 − 3ln 3 + ln 5) − ( 2 ln1 − 3ln1 + ln 3) M1
= 3ln 5 − 4 ln 3
 53 
= ln  4  M1
3 
 125 
= ln   A1 (3)
 81 

[10]

6666/01 GCE Mathematics June 2009 4


PhysicsAndMathsTutor.com

Question
Scheme Marks
Number

dy dy
Q4 (a) e −2 x − 2 y e −2 x = 2 + 2 y A1 correct RHS M1 A1
dx dx
d
dx
( y e −2 x ) = e −2 x
dy
dx
− 2 y e −2 x B1

( e−2 x − 2 y ) ddxy = 2 + 2 y e−2 x M1

d y 2 + 2 y e −2 x
= −2 x A1 (5)
dx e − 2y

d y 2 + 2 e0
(b) At P , = 0 = −4 M1
dx e −2
Using mm′ = −1
1
m′ = M1
4
1
y −1 = ( x − 0) M1
4
x − 4y + 4 = 0 or any integer multiple A1 (4)

[9]

Alternative for (a) differentiating implicitly with respect to y.


dx dx
e −2 x − 2 y e −2 x = 2 + 2y A1 correct RHS M1 A1
dy dy
d
dy
( y e −2 x ) = e −2 x − 2 y e −2 x
dx
dy
B1

( 2 + 2 y e−2 x ) ddxy = e−2 x − 2 y M1

dx e −2 x − 2 y
=
d y 2 + 2 y e −2 x
d y 2 + 2 y e −2 x
= −2 x A1 (5)
dx e − 2y

6666/01 GCE Mathematics June 2009 5


PhysicsAndMathsTutor.com

Question
Scheme Marks
Number

dx dy
Q5 (a) = −4sin 2t , = 6 cos t B1, B1
dt dt
dy 6 cos t  3 
=− = −  M1
dx 4sin 2t  4sin t 
π 3 √3
At t = , m=− =− accept equivalents, awrt −0.87 A1 (4)
3 4× 2√3
2

(b) Use of cos 2t = 1 − 2sin 2 t M1


x y
cos 2t = , sin t =
2 6
2
x  y
= 1− 2  M1
2 6
Leading to y = √ (18 − 9 x ) ( = 3 √ ( 2 − x )) cao A1
−2 ≤ x ≤ 2 k=2 B1 (4)

(c) 0 ≤ f ( x) ≤ 6 either 0 ≤ f ( x ) or f ( x ) ≤ 6 B1
Fully correct. Accept 0 ≤ y ≤ 6 , [ 0, 6] B1 (2)

[10]

Alternatives to (a) where the parameter is eliminated

y = (18 − 9 x ) 2
1
1
1
dy 1
= (18 − 9 x ) 2 × ( −9 )

B1
dx 2
π 2π
At t = , x = cos = −1 B1
3 3
dy 1 1 √3
= × × −9 = − M1 A1 (4)
dx 2 √ ( 27 ) 2

2 y 2 = 18 − 9 x
dy
2y = −9 B1
dx
π π
At t = , y = 6sin = 3√ 3 B1
3 3
dy 9 √3
=− =− M1 A1 (4)
dx 2×3√ 3 2

6666/01 GCE Mathematics June 2009 6


PhysicsAndMathsTutor.com

Question
Scheme Marks
Number

(5 − x )
3
1 2

∫ √ ( 5 − x ) dx = ∫ ( 5 − x ) dx = ( +C )
2
Q6 (a) M1 A1 (2)
− 32
 2 
 = − (5 − x ) + C 
3
2

 3 

2 2
∫ ( x − 1) √ ( 5 − x ) dx = − 3 ( x − 1)( 5 − x ) + ⌠ ( 5 − x ) 2 dx
3 3
(b) (i) 2
M1 A1ft
3⌡
2 (5 − x ) 2
5

= … + × ( +C ) M1
3 − 52
2 4
( x − 1)( 5 − x ) 2 − (5 − x ) 2 ( +C )
3 5
=− A1 (4)
3 15

5
 2 4   4 5 
(ii)  − ( x − 1)( 5 − x ) 2 − ( 5 − x ) 2  = ( 0 − 0 ) −  0 − × 4 2 
3 5

 3 15 1  15 
128  8 
=  = 8 ≈ 8.53  awrt 8.53 M1 A1 (2)
15  15 

[8]

Alternatives for ( b) and (c)


du  dx 
(b) u 2 = 5 − x ⇒ 2u = −1  ⇒ = −2u 
dx  du 


∫ ( x − 1) √ ( 5 − x ) dx = ( 4 − u ) u du du = ∫ ( 4 − u ) u ( −2u ) du
2 d x 2
M1 A1

= ∫ ( 2u 4 − 8u 2 ) du = u 5 − u 3 ( +C )
2 8
M1
5 3
2 8
= ( 5 − x ) 2 − ( 5 − x ) 2 ( +C )
5 3
A1
5 3

(c) x =1 ⇒ u = 2, x = 5 ⇒ u = 0
0
2 5 8 3  64 64 
 5 u − 3 u  = ( 0 − 0 ) −  5 − 3  M1
2

128  8 
=  = 8 ≈ 8.53  awrt 8.53 A1 (2)
15  15 

6666/01 GCE Mathematics June 2009 7


PhysicsAndMathsTutor.com

Question
Scheme Marks
Number

 10   8   2   −2 
uuur uuur uuur       uuur  
Q7 (a) AB = OB − OA =  14  −  13  =  1  or BA =  −1  M1
 −4   −2   −2   2
       
 8  2  10   2
       
r =  13  + λ  1  or r =  14  + λ  1  accept equivalents M1 A1ft (3)
 −2   −2   −4   −2 
       

 10   9   1 
uuur uuur uuur       −1 
 uuur  
(b) CB = OB − OC =  14  −  9  =  5  or BC =  −5 
 −4   6   −10   10 
       
(
CB = √ 12 + 52 + ( −10 )
2
) = √ (126) ( = 3 √ 14 ≈ 11.2 ) awrt 11.2 M1 A1 (2)

uuur uuur uuur uuur


(c) CB. AB = CB AB cos θ
( ± )( 2 + 5 + 20 ) = √ 126 √ 9 cos θ M1 A1
3
cos θ = ⇒ θ ≈ 36.7° awrt 36.7° A1 (3)
√ 14
B
(d)
θ d
X = sin θ M1 A1ft
√ 126
l
d √ 126 d = 3 √ 5 ( ≈ 6.7 ) awrt 6.7 A1 (3)

C
(e) BX = BC − d 2 = 126 − 45 = 81
2 2
M1
1 1 27 √ 5
! CBX = × BX × d = × 9 × 3 √ 5 = ( ≈ 30.2 ) awrt 30.1 or 30.2 M1 A1 (3)
2 2 2

[14]

Alternative for (e)


1
! CBX = × d × BC sin ∠XCB M1
2
1
= × 3 √ 5 × √ 126sin ( 90 − 36.7 ) ° sine of correct angle M1
2
27 √ 5
≈ 30.2 , awrt 30.1 or 30.2 A1 (3)
2

6666/01 GCE Mathematics June 2009 8


PhysicsAndMathsTutor.com

Question
Scheme Marks
Number

∫ sin θ dθ = 2 ∫ (1 − cos 2θ ) dθ = 2 θ − 4 sin 2θ ( +C )


2 1 1 1
Q8 (a) M1 A1 (2)

dx
(b) x = tan θ ⇒ = sec 2 θ


dx
π ∫ y 2 dx = π y 2 dθ = π ∫ ( 2sin 2θ ) sec 2θ dθ
2
M1 A1

( 2 × 2sin θ cos θ ) dθ
2

=π ∫ cos θ 2
M1

= 16π ∫ sin θ dθ
2
k = 16π A1
1 1 π
x = 0 ⇒ tan θ = 0 ⇒ θ = 0 , x = ⇒ tan θ = ⇒ θ= B1 (5)
√3 √3 6
π
 

6
 V = 16π sin 2 θ dθ 
 
 0

π
1 sin 2θ  6
(c) V = 16π  θ − M1
2 4  0
 π 1 π 
= 16π  − sin  − ( 0 − 0 )  Use of correct limits M1
 12 4 3 
 π √ 3  4 4
= 16π  −  = π 2 − 2π √ 3 p = , q = −2 A1 (3)
 12 8  3 3

[10]

6666/01 GCE Mathematics June 2009 9


PhysicsAndMathsTutor.com

Mark Scheme (Results)


January 2010

GCE

GCE Core Mathematics C4 (6666/01)

Edexcel Limited. Registered in England and Wales No. 4496750


Registered Office: One90 High Holborn, London WC1V 7BH
PhysicsAndMathsTutor.com

January 2010
6666 Core Mathematics C4
Mark Scheme

Question
Scheme Marks
Number

(a) (1 − 8 x ) 2 = 1 + ( 12 ) ( −8 x ) +
( 12 )( − 12 ) ( 12 )( − 12 ) ( − 32 )
( −8 x ) + ( −8 x )
1
Q1
2 3
+… M1 A1
2 3!
= 1 − 4 x − 8 x 2 ; − 32 x 3 − … A1; A1 (4)

⎛ 8 ⎞
(b) (1 − 8 x ) = ⎜1 − ⎟ M1
⎝ 100 ⎠
92 23 23
= = = ¿ cso A1 (2)
100 25 5

(c) 1 − 4 x − 8 x 2 − 32 x3 = 1 − 4 ( 0.01) − 8 ( 0.01) − 32 ( 0.01)


2 3

= 1 − 0.04 − 0.0008 − 0.000 032 = 0.959 168 M1

23 = 5 × 0.959 168 M1
= 4.795 84 cao A1 (3)
[9]

GCE Core Mathematics C4 (6666) January 2010 3


PhysicsAndMathsTutor.com

Question
Number Scheme Marks

Q2 (a) 1.386, 2.291 awrt 1.386, 2.291 B1 B1 (2)

1
(b) A ≈ × 0.5 ( ... ) B1
2
= ... ( 0 + 2 ( 0.608 + 1.386 + 2.291 + 3.296 + 4.385 ) + 5.545 ) M1
= 0.25 ( 0 + 2 ( 0.608 + 1.386 + 2.291 + 3.296 + 4.385 ) + 5.545 ) ft their (a) A1ft
= 0.25 × 29.477 ... ≈ 7.37 cao A1 (4)

ln x − ⌠
x2 x2 1
(c)(i) ∫ x ln x dx =2
⎮ × dx
⌡ 2 x
M1 A1
2
= ln x − ⌠
x x
⎮ dx
2 ⌡2
2
x x2
= ln x − ( +C ) M1 A1
2 4

4
⎡ x2 x2 ⎤ ⎛ 1⎞
(ii) ⎢ ln x − ⎥ = ( 8ln 4 − 4 ) − ⎜ − ⎟ M1
⎣2 4 ⎦1 ⎝ 4⎠
15
= 8ln 4 −
4
15
= 8 ( 2 ln 2 ) − ln 4 = 2 ln 2 seen or implied M1
4
1
= ( 64 ln 2 − 15 ) a = 64, b = −15 A1 (7)
4
[13]

GCE Core Mathematics C4 (6666) January 2010 4


PhysicsAndMathsTutor.com

Question
Scheme Marks
Number

dy
Q3 (a) −2sin 2 x − 3sin 3 y =0 M1 A1
dx
dy 2sin 2 x 2sin 2 x −2sin 2 x
=− Accept , A1 (3)
dx 3sin 3 y −3sin 3 y 3sin 3 y

π ⎛ 2π ⎞
(b) At x = , cos ⎜ ⎟ + cos 3 y = 1 M1
6 ⎝ 6 ⎠
1
cos 3 y = A1
2
π π
3y = ⇒y= awrt 0.349 A1 (3)
3 9

⎛π π ⎞ dy 2sin 2 ( π6 ) 2sin π3 2
(c) At ⎜ , ⎟ , =− = − =− M1
⎝6 9⎠ dx 3sin 3 ( 9 )
π π
3sin 3 3
π 2⎛ π⎞
y− = − ⎜x− ⎟ M1
9 3⎝ 6⎠
Leading to 6 x + 9 y − 2π = 0 A1 (3)
[9]

GCE Core Mathematics C4 (6666) January 2010 5


PhysicsAndMathsTutor.com

Question
Number Scheme Marks

Q4 (a) A: ( −6, 4, − 1) Accept vector forms B1 (1)

⎛ 4⎞ ⎛ 3⎞
⎜ ⎟⎜ ⎟
⎜ −1 ⎟ . ⎜ −4 ⎟ = 12 + 4 + 3 = 4 + ( −1) + 3 3 + ( −4 ) + 1 cos θ
2 2 2 2 2 2
(b) M1 A1
⎜ 3⎟ ⎜ 1⎟
⎝ ⎠⎝ ⎠
19
cos θ = awrt 0.73 A1 (3)
26

(c) X: (10, 0, 11) Accept vector forms B1 (1)

⎛ 10 ⎞ ⎛ −6 ⎞
⎜ ⎟ ⎜ ⎟
(d) AX = ⎜ 0 ⎟ − ⎜ 4 ⎟ Either order M1
⎜ 11 ⎟ ⎜ −1 ⎟
⎝ ⎠ ⎝ ⎠
⎛ 16 ⎞
⎜ ⎟
= ⎜ −4 ⎟ cao A1 (2)
⎜ 12 ⎟
⎝ ⎠

AX = 162 + ( −4 ) + 122
2
(e) M1

= 416 = 16 × 26 = 4 26 ¿ Do not penalise if consistent A1 (2)


incorrect signs in (d)

(f) l1
X
4 26 Use of correct right angled triangle M1
A AX
θ = cos θ M1
d l2 d
Y
4 26
d = 19 ≈ 27.9 awrt 27.9 A1 (3)
26
[12]

GCE Core Mathematics C4 (6666) January 2010 6


PhysicsAndMathsTutor.com

Question
Scheme Marks
Number

⌠ 9 x + 6 dx = ⌠ ⎛ 9 + 6 ⎞ dx
Q5 (a) ⎮ ⎮⎜ ⎟ M1
⌡ x ⌡⎝ x⎠
= 9 x + 6 ln x ( +C ) A1 (2)

⌠ 1 ⌠ 9 x + 6 dx
(b) ⎮ 13 d y = ⎮ ⌡ x
Integral signs not necessary B1
⌡y
⌠ 9x + 6
∫ y 3 d y = ⎮⌡ x dx
−1

2
y3
= 9 x + 6 ln x ( +C ) ± ky 3 = their ( a )
2
M1
2
3
3 23
y = 9 x + 6 ln x ( +C ) ft their ( a ) A1ft
2
y = 8, x =1
3 23
8 = 9 + 61n 1 + C M1
2
C = −3 A1
2
y 3 = ( 9 x + 6 ln x − 3)
2

3
y 2 = ( 6 x + 4 ln x − 2 )
3
( = 8 (3x + 2 ln x − 1) )
3
A1 (6)
[8]

GCE Core Mathematics C4 (6666) January 2010 7


PhysicsAndMathsTutor.com

Question
Scheme Marks
Number

dA
Q6 = 1.5 B1
dt
dA
A = π r2 ⇒ = 2π r B1
dr
When A = 2
2
2 = π r2 ⇒ r = ( = 0.797 884 ... ) M1
π

d A d A dr
= ×
dt dr dt
dr
1.5 = 2π r M1
dt

dr 1.5
= ≈ 0.299 awrt 0.299 A1
dt 2π π2
[5]

GCE Core Mathematics C4 (6666) January 2010 8


PhysicsAndMathsTutor.com

Question
Scheme Marks
Number

Q7 (a) y = 0 ⇒ t ( 9 − t 2 ) = t ( 3 − t )( 3 + t ) = 0
t = 0, 3, − 3 Any one correct value B1
At t = 0 , x = 5 ( 0 ) − 4 = −4
2
Method for finding one value of x M1

At t = 3 , x = 5 ( 3) − 4 = 41
2

( At t = −3, x = 5 ( −3) − 4 = 41
2
)
At A, x = −4 ; at B, x = 41 Both A1 (3)

dx
(b) = 10t Seen or implied B1
dt
⌠ ⌠ dx
⎮ y dx = ⎮ y dt = ∫ t ( 9 − t )10t dt
2
M1 A1
⌡ ⌡ dt
= ∫ ( 90t 2 − 10t 4 ) dt
90t 3 10t 5
=
3

5
( +C ) ( = 30t 3
− 2t 5 ( +C ) ) A1
3
⎡ 90t 3 10t 5 ⎤
⎢ 3 − 5 ⎥ = 30 × 3 − 2 × 3
3 5
( = 324 ) M1
⎣ ⎦0

A = 2∫ y dx = 648 ( units )
2
A1 (6)
[9]

GCE Core Mathematics C4 (6666) January 2010 9


PhysicsAndMathsTutor.com

Question
Scheme Marks
Number

dx
Q8 (a) = −2sin u B1
du
⌠ 1 ⌠ 1
⎮ 2 dx = ⎮ × −2sin u du M1
⌡ x 4− x ⌡ ( 2 cos u ) 4 − ( 2 cos u )
2 2 2

⌠ −2sin u
=⎮ du Use of 1 − cos 2 u = sin 2 u M1
⌡ 4 cos 2 u 4sin 2 u
=− ⌠ ±k ⌠
1 1 1
⎮ du ⎮ du M1
4 ⌡ cos 2 u ⌡ cos 2 u
1
= − tan u ( +C ) ± k tan u M1
4
π
x = 2 ⇒ 2 = 2 cos u ⇒ u =
4
π
x = 1 ⇒ 1 = 2 cos u ⇒ u = M1
3
π
⎡ 1 ⎤4 1⎛ π π⎞

⎢⎣ 4 tan u ⎥⎦ π = − 4 ⎝⎜ tan 4 − tan 3 ⎠⎟
3

⎛ 3 −1 ⎞
=−
1
4
(
1− 3 ) ⎜⎜ =
4 ⎟⎠
⎟ A1 (7)

2 2
⌠ ⎛ 4 ⎞
V =π⎮ ⎜ ⎟ dx M1
⎜ x ( 4 − x2 ) 4
(b) 1

⎮ ⎟
⌡1 ⎝ ⎠
2
⌠ 1
= 16π ⎮ dx 16π × integral in (a) M1
⌡1 x 2 4 − x 2
⎛ 3 −1 ⎞
= 16π ⎜⎜ ⎟⎟ 16π × their answer to part (a) A1ft (3)
⎝ 4 ⎠
[10]

GCE Core Mathematics C4 (6666) January 2010 10


PhysicsAndMathsTutor.com

Mark Scheme (Results)


Summer 2010

GCE

Core Mathematics C4 (6666)

Edexcel Limited. Registered in England and Wales No. 4496750


Registered Office: One90 High Holborn, London WC1V 7BH
PhysicsAndMathsTutor.com

June 2010
6666 Core Mathematics C4
Mark Scheme

Question
Scheme Marks
Number

⎛π ⎞ ⎛π ⎞
1. (a) y ⎜ ⎟ ≈ 1.2247, y ⎜ ⎟ = 1.1180 accept awrt 4 d.p. B1 B1 (2)
⎝6⎠ ⎝4⎠

⎛π ⎞ π
(b)(i) I ≈ ⎜ ⎟ (1.3229 + 2 ×1.2247 + 1) B1 for B1 M1
⎝ 12 ⎠ 12
≈ 1.249 cao A1

⎛π ⎞ π
(ii) I ≈ ⎜ ⎟ (1.3229 + 2 × (1.2973 + 1.2247 + 1.1180 ) + 1) B1 for B1 M1
⎝ 24 ⎠ 24
≈ 1.257 cao A1 (6)
[8]

GCE Core Mathematics C4 (6666) Summer 2010


PhysicsAndMathsTutor.com

Question
Scheme Marks
Number

du
2. = − sin x B1
dx
∫ sin x e dx = − ∫ eu du
cos x +1
M1 A1
= − eu ft sign error A1ft
= − ecos x+1
π
⎡⎣ − ecos x +1 ⎤⎦ 2 = − e1 − ( − e 2 ) or equivalent with u M1
0

= e ( e − 1) ¿ cso A1 (6)
[6]

GCE Core Mathematics C4 (6666) Summer 2010


PhysicsAndMathsTutor.com

Question
Scheme Marks
Number

3.
d x
dx
( 2 ) = ln 2.2 x B1

dy dy
ln 2.2 x + 2 y = 2 y + 2x M1 A1= A1
dx dx
Substituting ( 3, 2 )
dy dy
8ln 2 + 4 = 4+6 M1
dx dx
dy
= 4 ln 2 − 2 Accept exact equivalents M1 A1 (7)
dx
[7]

GCE Core Mathematics C4 (6666) Summer 2010


PhysicsAndMathsTutor.com

Question
Scheme Marks
Number

dx dy
4. (a) = 2sin t cos t , = 2sec 2 t B1 B1
dt dt
dy sec 2 t ⎛ 1 ⎞
= ⎜= 3 ⎟
or equivalent M1 A1 (4)
dx sin t cos t ⎝ sin t cos t ⎠

π 3
(b) At t = , x= , y = 2√3 B1
3 4
π
sec2
dy 3 = 16
= M1 A1
dx sin π cos π √ 3
3 3
16 ⎛ 3⎞
y − 2√3 = ⎜x− ⎟ M1
√3⎝ 4⎠
3
y=0 ⇒ x= M1 A1 (6)
8
[10]

GCE Core Mathematics C4 (6666) Summer 2010


PhysicsAndMathsTutor.com

Question
Scheme Marks
Number

5. (a) A=2 B1
2 x + 5 x − 10 = A ( x − 1)( x + 2 ) + B ( x + 2 ) + C ( x − 1)
2

x →1 −3 = 3B ⇒ B = −1 M1 A1
x → −2 −12 = −3C ⇒ C = 4 A1 (4)

−1
2 x 2 + 5 x − 10 ⎛ x⎞
= 2 + (1 − x ) + 2 ⎜ 1 + ⎟
−1
(b) M1
( x − 1)( x + 2 ) ⎝ 2⎠
(1 − x )
−1
= 1 + x + x 2 + ... B1
−1
⎛ x⎞ x x2
⎜ 1 + ⎟ = 1 − + + ... B1
⎝ 2⎠ 2 4
2 x 2 + 5 x − 10 ⎛ 1⎞
= ( 2 + 1 + 2 ) + (1 − 1) x + ⎜1 + ⎟ x 2 + ... M1
( x − 1)( x + 2 ) ⎝ 2⎠
= 5 + ... ft their A − B + 12 C A1 ft
3 2
= ... + x + ... 0 x stated or implied A1 A1 (7)
2
[11]

GCE Core Mathematics C4 (6666) Summer 2010


PhysicsAndMathsTutor.com

Question
Scheme Marks
Number

6. (a) f (θ ) = 4 cos 2 θ − 3sin 2 θ


⎛1 1 ⎞ ⎛1 1 ⎞
= 4 ⎜ + cos 2θ ⎟ − 3 ⎜ − cos 2θ ⎟ M1 M1
⎝2 2 ⎠ ⎝2 2 ⎠
1 7
= + cos 2θ ¿ cso A1 (3)
2 2

∫ θ cos 2θ dθ = 2 θ sin 2θ − 2 ∫ sin 2θ dθ


1 1
(b) M1 A1
1 1
= θ sin 2θ + cos 2θ A1
2 4
1 2 7 7
∫ θ f (θ ) dθ = 4 θ + 4 θ sin 2θ + 8 cos 2θ M1 A1
π
⎡π 2 7⎤
⎤ 2 = ⎢ + 0 − ⎥ − ⎡⎢0 + 0 + ⎤⎥
7
⎡ ... M1
⎣ ⎦0 8⎦ ⎣ 8⎦
⎣ 16
π 2
7
= − A1 (7)
16 4
[10]

GCE Core Mathematics C4 (6666) Summer 2010


PhysicsAndMathsTutor.com

Question
Scheme Marks
Number

7. (a) j components 3 + 2λ = 9 ⇒ λ = 3 ( µ = 1) M1 A1
Leading to C : ( 5, 9, − 1) accept vector forms A1 (3)
JJJG JJJG
(b) Choosing correct directions or finding AC and BC M1
⎛1⎞ ⎛5⎞
⎜ ⎟⎜ ⎟
⎜ 2 ⎟ . ⎜ 0 ⎟ = 5 + 2 = √ 6 √ 29 cos ∠ACB use of scalar product M1 A1
⎜1⎟ ⎜ 2⎟
⎝ ⎠⎝ ⎠
∠ACB = 57.95° awrt 57.95° A1 (4)

(c) A : ( 2, 3, − 4 ) B : ( −5, 9, − 5 )
⎛ 3⎞ ⎛ 10 ⎞
JJJG ⎜ ⎟ JJJG ⎜ ⎟
AC = ⎜ 6 ⎟ , BC = ⎜ 0 ⎟
⎜ 3⎟ ⎜4⎟
⎝ ⎠ ⎝ ⎠
AC = 3 + 6 + 3 ⇒ AC = 3 √ 6
2 2 2 2
M1 A1
BC 2 = 102 + 42 ⇒ BC = 2 √ 29 A1
1
 ABC = AC × BC sin ∠ACB
2
1
= 3 √ 6 × 2 √ 29sin ∠ACB ≈ 33.5 15 √ 5 , awrt 34 M1 A1 (5)
2
[12]

Alternative method for (b) and (c)


(b) A : ( 2, 3, − 4 ) B : ( −5, 9, − 5 ) C : ( 5, 9, − 1)
AB 2 = 7 2 + 62 + 12 = 86
AC 2 = 32 + 62 + 32 = 54
BC 2 = 102 + 02 + 42 = 116 Finding all three sides M1
116 + 54 − 86
cos ∠ACB = ( = 0.530 66 ...) M1 A1
2 √ 116 √ 54
∠ACB = 57.95° awrt 57.95° A1 (4)
If this method is used some of the working may gain credit in part (c) and
appropriate marks may be awarded if there is an attempt at part (c).

GCE Core Mathematics C4 (6666) Summer 2010


PhysicsAndMathsTutor.com

Question
Number Scheme Marks

dV
8. (a) = 0.48π − 0.6π h M1 A1
dt
dV dh
V = 9π h ⇒ = 9π B1
dt dt
dh
9π = 0.48π − 0.6π h M1
dt
dh
Leading to 75 = 4 − 5h ¿ cso A1 (5)
dt


75
(b) dh = ∫ 1dt separating variables M1
4 − 5h
−15ln ( 4 − 5h ) = t ( +C ) M1 A1
−15ln ( 4 − 5h ) = t + C
When t = 0 , h = 0.2
−15ln 3 = C M1
t = 15ln 3 − 15ln ( 4 − 5h )
When h = 0.5
⎛ 3 ⎞
t = 15ln 3 − 15ln1.5 = 15ln ⎜ ⎟ = 15ln 2 awrt 10.4 M1 A1
⎝ 1.5 ⎠

Alternative for last 3 marks


t = ⎡⎣ −15ln ( 4 − 5h ) ⎤⎦
0.5

0.2

= −15ln1.5 + 15ln 3 M1 M1
⎛ 3 ⎞
= 15ln ⎜ ⎟ = 15ln 2 awrt 10.4 A1 (6)
⎝ 1.5 ⎠

GCE Core Mathematics C4 (6666) Summer 2010


PhysicsAndMathsTutor.com

Mark Scheme (Results)


January 2011

GCE

GCE Core Mathematics C4 (6666) Paper 1

Edexcel Limited. Registered in England and Wales No. 4496750


Registered Office: One90 High Holborn, London WC1V 7BH
PhysicsAndMathsTutor.com

January 2011
Core Mathematics C4 6666
Mark Scheme

Question
Scheme Marks
Number


x cos 2 x cos 2 x
1. ∫ x sin 2 x dx = − 2
+
2
dx M1 A1 A1

sin 2 x
= ... + M1
4
π
π
[ ... ]02 = M1 A1
4

[6]

dI
2. = −16 ln ( 0.5 ) 0.5t M1 A1
dt
dI
At t = 3 = −16 ln ( 0.5 ) 0.53 M1
dt
= −2 ln 0.5 = ln 4 M1 A1

[5]

GCE Core Mathematics C4 (6666) January 2011 1


PhysicsAndMathsTutor.com

Question
Scheme Marks
Number
3.
5 A B
(a) = +
( x − 1)( 3x + 2 ) x − 1 3x + 2
5 = A ( 3 x + 2 ) + B ( x − 1)
x →1 5 = 5A ⇒ A = 1 M1 A1
2 5
x→− 5 = − B ⇒ B = −3 A1 (3)
3 3

∫ ⎛ 1 3 ⎞

5
(b) dx = ⎜ − ⎟ dx
( x − 1)( 3x + 2 ) ⎝ x − 1 3x + 2 ⎠
= ln ( x − 1) − ln ( 3 x + 2 ) ( +C ) ft constants M1 A1ft A1ft

(3)


⎛1⎞

5
(c) dx = ⎜ ⎟ dy M1
( x − 1)( 3x + 2 ) ⎝ y⎠
ln ( x − 1) − ln ( 3x + 2 ) = ln y ( +C ) M1 A1
K ( x − 1)
y= depends on first two Ms in (c) M1 dep
3x + 2
K
Using ( 2, 8 ) 8= depends on first two Ms in (c) M1 dep
8
64 ( x − 1)
y= A1 (6)
3x + 2

[12]

GCE Core Mathematics C4 (6666) January 2011 2


PhysicsAndMathsTutor.com

Question
Scheme Marks
Number
4.
JJJG
(a) AB = −2i + 2 j − k − ( i − 3 j + 2k ) = −3i + 5 j − 3k M1 A1 (2)

(b) r = i − 3 j + 2k + λ ( −3i + 5 j − 3k ) M1 A1ft (2)


or r = −2i + 2 j − k + λ ( −3i + 5 j − 3k )

JJJG
(c) AC = 2i + pj − 4k − ( i − 3 j + 2k )
JJJG
= i + ( p + 3) j − 6k or CA B1
⎛ 1 ⎞ ⎛ −3 ⎞
JJJG JJJG ⎜ ⎟⎜ ⎟
AC. AB = ⎜ p + 3 ⎟ . ⎜ 5 ⎟ = 0 M1
⎜ −6 ⎟ ⎜ −3 ⎟
⎝ ⎠⎝ ⎠
−3 + 5 p + 15 + 18 = 0
Leading to p = −6 M1 A1 (4)

AC 2 = ( 2 − 1) + ( −6 + 3) + ( −4 − 2 ) ( = 46 )
2 2 2
(d) M1
AC = √ 46 accept awrt 6.8 A1
(2)
[10]

GCE Core Mathematics C4 (6666) January 2011 3


PhysicsAndMathsTutor.com

Question
Scheme Marks
Number
5.
−2
⎛ 3 ⎞
( 2 − 3x )
−2 −2
(a) = 2 ⎜1 − x ⎟ B1
⎝ 2 ⎠

−2
⎛ 3 ⎞ −2. − 3 ⎛ 3 ⎞ −2. − 3. − 4 ⎛ 3 ⎞
2 3
⎛ 3 ⎞ M1 A1
⎜ 1 − x ⎟ = 1 + ( −2 ) ⎜ − x ⎟ + ⎜− x⎟ + ⎜ − x ⎟ + ...
⎝ 2 ⎠ ⎝ 2 ⎠ 1.2 ⎝ 2 ⎠ 1.2.3 ⎝ 2 ⎠
27 2 27 3
= 1 + 3x + x + x + ...
4 2
1 3 27 27
( 2 − 3x ) = + x + x 2 + x3 + ...
−2
M1 A1 (5)
4 4 16 8

⎛1 3 27 2 27 3 ⎞
(b) f ( x ) = ( a + bx ) ⎜ + x + x + x + ... ⎟
⎝4 4 16 8 ⎠
3a b
Coefficient of x; + =0 ( 3a + b = 0 ) M1
4 4
27a 3b 9
Coefficient of x 2 ; + = ( 9a + 4b = 3) A1 either correct M1 A1
16 4 16
Leading to a = −1, b = 3 M1 A1 (5)

27a 27b 27 27
(c) Coefficient of x 3 is + = × ( −1) + × 3 M1 A1ft
8 16 8 16
27
= cao A1 (3)
16

[13]

GCE Core Mathematics C4 (6666) January 2011 4


PhysicsAndMathsTutor.com

Question
Scheme Marks
Number
6.
dx 1 dy
(a) = , = 2t
dt t dt
dy
= 2t 2 M1 A1
dx
Using mm′ = −1 , at t = 3
1
m′ = − M1 A1
18
1
y − 7 = − ( x − ln 3) M1 A1 (6)
18

(b) x = ln t ⇒ t = e x B1
y =e −2 2x
M1 A1 (3)

∫ (e − 2 ) dx
2
(c) V =π 2x
M1

∫ (e − 2 ) dx = ∫ (e − 4 e 2 x + 4 ) dx
2
2x 4x
M1

e4 x 4 e2 x
= − + 4x M1 A1
4 2
ln 4
⎡ e4 x 4 e2 x ⎤
π⎢ − + 4 x ⎥ = π ⎡⎣( 64 − 32 + 4 ln 4 ) − ( 4 − 8 + 4 ln 2 ) ⎤⎦ M1
⎣ 4 2 ⎦ ln 2
= π ( 36 + 4 ln 2 ) A1
(6)
[15]

Alternative to (c) using parameters

∫ (t − 2)
2 dx
V =π 2
dt M1
dt

∫ ∫
⎛ 2 2 1⎞ ⎛ 3 4⎞
⎜ ( t − 2 ) × ⎟ dt = ⎜ t − 4t + ⎟ dt M1
⎝ t⎠ ⎝ t⎠
t4
− 2t 2 + 4 ln t = M1 A1
4
The limits are t = 2 and t = 4
4
⎡t4 ⎤
π ⎢ − 2t 2 + 4 ln t ⎥ = π ⎡⎣( 64 − 32 + 4 ln 4 ) − ( 4 − 8 + 4 ln 2 ) ⎤⎦ M1
⎣4 ⎦2
= π ( 36 + 4 ln 2 ) A1
(6)

GCE Core Mathematics C4 (6666) January 2011 5


PhysicsAndMathsTutor.com

Question
Scheme Marks
Number
7.
(a) x = 3 ⇒ y = 0.1847 awrt B1
x = 5 ⇒ y = 0.1667 awrt or 16 B1
(2)

1
(b) I≈ ⎡ 0.2 + 0.1667 + 2 ( 0.1847 + 0.1745 ) ⎤⎦ B1 M1 A1ft
2⎣
≈ 0.543 0.542 or 0.543 A1 (4)

dx
(c) = 2 (u − 4) B1
du

∫ 4 + √ ( x −1) ∫
1 1
dx = × 2 ( u − 4 ) du M1
u


⎛ 8⎞
=
⎜ 2 − ⎟ du A1
⎝ u⎠
= 2u − 8ln u M1 A1
x = 2 ⇒ u = 5, x = 5 ⇒ u = 6 B1
[ 2u − 8ln u ] 5 = (12 − 8ln 6 ) − (10 − 8ln 5)
6
M1

⎛5⎞
= 2 + 8ln ⎜ ⎟ A1
⎝6⎠
(8)
[14]

GCE Core Mathematics C4 (6666) January 2011 6


PhysicsAndMathsTutor.com

Mark Scheme (Results)

June 2011

GCE Core Mathematics C4 (6666) Paper 1


PhysicsAndMathsTutor.com

June 2011 FINAL


Core Mathematics C4 6666
Mark Scheme

Question
Scheme Marks
Number

9 x 2 = A ( x − 1)( 2 x + 1) + B ( 2 x + 1) + C ( x − 1)
2
1. B1

x →1 9 = 3B ⇒ B = 3 M1

2
1 9 ⎛ 3⎞
x→− = ⎜− ⎟ C ⇒ C =1 Any two of A, B, C A1
2 4 ⎝ 2⎠

x 2 terms 9 = 2A + C ⇒ A = 4 All three correct A1 (4)


[4]
Alternatives for finding A.

x terms 0 = − A + 2 B − 2C ⇒ A = 4
Constant terms 0 = − A + B + C ⇒ A = 4

GCE Core Mathematics C4 (6666) June 2011 4


PhysicsAndMathsTutor.com

Question
Scheme Marks
Number

f ( x ) = ( ... + ...)
− 12
2. M1
1 1
=9
− 12
( ... + ...) 3−1 ,
...
or 1 B1
3 92
(1 + kx )
2 n
= 1 + nkx 2 + ... n not a natural number, k ≠ 1 M1
( − 12 ) ( − 23 )
(1 + kx )
2 −2
( kx )
1
2 2
= ... + ft their k ≠ 1 A1 ft
2

− 12
⎛ 4 2⎞ 2 2 2 4
⎜1 + x ⎟ = 1− x + x A1
⎝ 9 ⎠ 9 27

1 2 2 2 4
f ( x) = − x + x A1 (6)
3 27 81
[6]

GCE Core Mathematics C4 (6666) June 2011 5


PhysicsAndMathsTutor.com

Question
Number Scheme Marks

dV 1
3. (a) = π h − π h2 or equivalent M1 A1
dh 2

dV 1 π
At h = 0.1 , = π ( 0.1) − π ( 0.1) = 0.04π
2
M1 A1 (4)
dh 2 25

dh dV dV π 1 π
(b) = ÷ = ×1 or ÷ their (a) M1
dt dt dh 800 2 π h − π h 2 800

dh π 25 1
At h = 0.1 , = × = awrt 0.031 A1 (2)
dt 800 π 32
[6]

GCE Core Mathematics C4 (6666) June 2011 6


PhysicsAndMathsTutor.com

Question
Scheme Marks
Number

(a) 0.0333 , 1.3596 awrt 0.0333 ,


4. B1 B1 (2)
1.3596

1 √2
(b) Area ( R ) ≈ × [ ...] B1
2 4
≈ ... ⎡⎣ 0 + 2 ( 0.0333 + 0.3240 + 1.3596 ) + 3.9210 ⎤⎦ M1
≈ 1.30 Accept
A1 (3)
1.3

du
(c) u = x2 + 2 ⇒ = 2x B1
dx
x3 ln ( x 2 + 2 ) dx
√2
Area ( R ) = ∫ B1
0

∫ x ln ( x + 2 ) dx = ∫ x 2 ln ( x 2 + 2 ) x dx = ∫ ( u − 2 )( ln u ) 12 du
3 2
M1

Area ( R ) = ∫ ( u − 2 ) ln u du
1 4
Hence À
2 2 A1 (4)
cso

⎛ u2 ⎞ ⌠ ⎛ u2 ⎞1
(d) ∫ ( u − 2 ) ln u du = ⎜ ⎝ 2
− 2u ⎟ ln u − ⎮ ⎜ − 2u ⎟ du
⎠ ⌡⎝ 2 ⎠u
M1 A1

⎛ u2 ⎞ ⌠⎛u ⎞
= ⎜ − 2u ⎟ ln u − ⎮ ⎜ − 2 ⎟ du
⎝ 2 ⎠ ⌡⎝2 ⎠
⎛u 2
⎞ ⎛u 2

= ⎜ − 2u ⎟ ln u − ⎜ − 2u ⎟ ( +C ) M1 A1
⎝ 2 ⎠ ⎝ 4 ⎠

4
1 ⎡⎛ u 2 ⎞ ⎛ u2 ⎞⎤
Area ( R ) = ⎢⎜ − 2u ⎟ ln u − ⎜ − 2u ⎟ ⎥
2 ⎣⎝ 2 ⎠ ⎝ 4 ⎠⎦ 2
= 12 ⎡⎣( 8 − 8 ) ln 4 − 4 + 8 − ( ( 2 − 4 ) ln 2 − 1 + 4 ) ⎤⎦ M1
= 1
2 ( 2 ln 2 + 1) ln 2 + 12 A1 (6)
[15]

GCE Core Mathematics C4 (6666) June 2011 7


PhysicsAndMathsTutor.com

Question
Number Scheme Marks

1 dy
5. = … B1
y dx
⎛1⎞
… = 2 ln x + 2 x ⎜ ⎟ M1 A1
⎝ x⎠
At x = 2 , ln y = 2(2) ln 2 M1
leading to y = 16 Accept y = e 4ln 2
A1

1 dy
At ( 2, 16 ) = 2 ln 2 + 2 M1
16 dx
dy
= 16 ( 2 + 2 ln 2 ) A1 (7)
dx
[7]
Alternative
y = e 2 x ln x B1

( 2 x ln x ) = 2 ln x + 2 x ⎛⎜ ⎞⎟
d 1
M1 A1
dx ⎝x⎠
dy ⎛ ⎛ 1 ⎞⎞
= ⎜ 2 ln x + 2 x ⎜ ⎟ ⎟ e 2 x ln x M1 A1
dx ⎝ ⎝ x ⎠⎠

dy
At x = 2 , = ( 2 ln 2 + 2 ) e 4ln 2 M1
dx

= 16 ( 2 + 2 ln 2 ) A1 (7)

GCE Core Mathematics C4 (6666) June 2011 8


PhysicsAndMathsTutor.com

Question
Number Scheme Marks

6. (a) i: 6 − λ = −5 + 2 µ
j: −3 + 2λ = 15 − 3µ Any two equations M1
leading to λ = 3 , µ = 4 M1 A1
⎛ 6 ⎞ ⎛ −1 ⎞ ⎛ 3 ⎞ ⎛ −5 ⎞ ⎛ 2 ⎞ ⎛ 3 ⎞
⎜ ⎟ ⎜ ⎟ ⎜ ⎟ ⎜ ⎟ ⎜ ⎟ ⎜ ⎟
r = ⎜ −3 ⎟ + 3 ⎜ 2 ⎟ = ⎜ 3 ⎟ or r = ⎜ 15 ⎟ + 4 ⎜ −3 ⎟ = ⎜ 3 ⎟ M1 A1
⎜ −2 ⎟ ⎜ 3 ⎟ ⎜ 7 ⎟ ⎜ 3⎟ ⎜ 1⎟ ⎜7⎟
⎝ ⎠ ⎝ ⎠ ⎝ ⎠ ⎝ ⎠ ⎝ ⎠ ⎝ ⎠
k: LHS = −2 + 3 ( 3) = 7 , RHS = 3 + 4 (1) = 7 B1 (6)
(As LHS = RHS, lines intersect)
⎛ 3⎞
⎜ ⎟
Alternatively for B1, showing that λ = 3 and µ = 4 both give ⎜ 3 ⎟
⎜7⎟
⎝ ⎠

⎛ −1 ⎞ ⎛ 2 ⎞
⎜ ⎟⎜ ⎟
(b) ⎜ 2 ⎟ . ⎜ −3 ⎟ = −2 − 6 + 3 = √ 14 √ 14 cos θ (θ ≈ 110.92° ) M1 A1
⎜ 3⎟ ⎜ 1⎟
⎝ ⎠⎝ ⎠
Acute angle is 69.1° awrt 69.1 A1 (3)

⎛ 6 ⎞ ⎛ −1 ⎞ ⎛ 5 ⎞
⎜ ⎟ ⎜ ⎟ ⎜ ⎟
(c) r = ⎜ −3 ⎟ + 1⎜ 2 ⎟ = ⎜ −1 ⎟ ( ⇒ B lies on l 1 ) B1 (1)
⎜ −2 ⎟ ⎜ 3 ⎟ ⎜ 1 ⎟
⎝ ⎠ ⎝ ⎠ ⎝ ⎠
l1
(d) Let d be shortest distance from B to l 2 B

⎛ 5⎞ ⎛ 3⎞ ⎛ 2⎞ d
uuur ⎜ ⎟ ⎜ ⎟ ⎜ ⎟
AB = ⎜ −1 ⎟ − ⎜ 3 ⎟ = ⎜ −4 ⎟
A θ M1
⎜ 1 ⎟ ⎜ 7 ⎟ ⎜ −6 ⎟ l2
⎝ ⎠ ⎝ ⎠ ⎝ ⎠

( )
uuur
AB = √ 22 + ( −4 ) + ( −6 ) = √ 56
2 2
awrt 7.5 A1

d
= sin θ M1
√ 56
d = √ 56sin 69.1° ≈ 6.99 awrt 6.99 A1 (4)
[14]

GCE Core Mathematics C4 (6666) June 2011 9


PhysicsAndMathsTutor.com

Question
Scheme Marks
Number

√3
7. (a) tan θ = √ 3 or sin θ = M1
2
π
θ= awrt 1.05 A1 (2)
3

dx dy
(b) = sec 2 θ , = cos θ
dθ dθ
d y cos θ
=
dx sec θ 2 ( = cos3 θ ) M1 A1

⎛π ⎞ 1
At P, m = cos3 ⎜ ⎟ = Can be implied A1
⎝3⎠ 8
Using mm′ = −1 , m′ = −8 M1
For normal y − 2 √ 3 = −8 ( x − √ 3)
1
M1
At Q, y = 0 − 12 √ 3 = −8 ( x − √ 3)
leading to x = 17
16 √ 3 ( k = 1716 ) 1.0625 A1 (6)

⌠ dx
∫y dx = ⎮ y 2 dθ = ∫ sin 2 θ sec2 θ dθ
2
(c) M1 A1
⌡ dθ
= ∫ tan 2θ dθ A1
= ∫ ( sec 2 θ − 1) dθ M1
= tan θ − θ ( +C ) A1

V = π ∫ y 2 dx = [ tan θ − θ ] 03 = π ⎡⎣( √ 3 − π3 ) − ( 0 − 0 ) ⎤⎦
π
3

0
M1
= √ 3π − 13 π 2 ( p = 1, q = − 13 ) A1 (7)
[15]

GCE Core Mathematics C4 (6666) June 2011 10


PhysicsAndMathsTutor.com

Question
Scheme Marks
Number

( 4 y + 3)
1
2
− 12

8.
(a) ∫ ( 4 y + 3) dx = ( 4 ) ( 12 ) ( +C ) M1 A1 (2)

(= ( 4 y + 3) )
1
1
2
2
+C


dy = ⌠
1 1
(b) ⎮ ⎮ 2 dx B1
⌡ √ ( 4 y + 3) ⌡x

∫ ( 4 y + 3)
− 12
d y = ∫ x −2 dx
1 1
( 4 y + 3) 2 = − ( +C )
1

M1
2 x

1 1
Using ( −2, 1.5) ( 4 ×1.5 + 3) 2 = − + C
1

M1
2 −2
leading to C =1 A1
1 1
( 4 y + 3) 2 = − + 1
1

2 x
2
( 4 y + 3) 2 = 2 −
1

M1
x
2
1⎛ 2⎞ 3
y = ⎜2− ⎟ − or equivalent A1 (6)
4⎝ x⎠ 4
[8]

GCE Core Mathematics C4 (6666) June 2011 11


PhysicsAndMathsTutor.com

Mark Scheme (Results)

January 2012

GCE Core Mathematics C4 (6666) Paper 1


PhysicsAndMathsTutor.com

January 2012
6666 Core Mathematics C4
Mark Scheme
Question
Scheme Marks
Number
 dy  dy  dy 
1. (a)  =  2 + 6y +  6 x y + 3x 2  = 8 x M1 A1 B1
 dx  dx  dx 
 dy 8 x − 2 − 6 xy 
 =  not necessarily required.
 dx 6 y + 3x 2 
dy 8(−1) − 2 − 6(−1)(1) 4
At P ( −1, 1) , m ( T ) = = = − dM1 A1 cso
dx 6(1) + 3(−1) 2
9
[5]
−1  9 
(b) So, m(N) = 4  =  M1
− 9  4
9
N: y − 1 = ( x + 1) M1
4
N: 9 x − 4 y + 13 = 0 A1
[3]
8
dy dy  dy 
(a) M1: Differentiates implicitly to include either ± ky or 3x 2 . (Ignore  =  ).
dx dx  dx 

A1: ( 2 x + 3 y ) →  2 + 6 y ddyx  and ( 4 x


2 2
→ 8 x ) . Note: If an extra “sixth” term appears then award A0.
 
d y
B1: 6 x y + 3 x 2 .
dx
dy
dM1: Substituting x = − 1 and y = 1 into an equation involving . Allow this mark if either the numerator
dx
dy 8 x − 2 − 6 xy
or denominator of = is substituted into or evaluated correctly.
dx 6 y + 3x 2
If it is clear, however, that the candidate is intending to substitute x = 1 and y = − 1, then award M0.
Candidates who substitute x = 1 and y = − 1, will usually achieve m ( T ) = − 4
Note that this mark is dependent on the previous method mark being awarded.
4 8 •
A1: For − or − or − 0.4 or awrt − 0.44
9 18
If the candidate’s solution is not completely correct, then do not give this mark.
1
(b) M1: Applies m(N) = − .
their m(T)
M1: Uses y − 1 = ( mN )( x − − 1) or finds c using x = − 1 and y = 1 and uses y = ( m N ) x + " c " ,
1 1
Where mN = − or mN = or mN = − their m(T) .
their m(T) their m(T)
A1: 9 x − 4 y + 13 = 0 or − 9 x + 4 y − 13 = 0 or 4 y − 9 x − 13 = 0 or 18 x − 8 y + 26 = 0 etc.
Must be “ = 0”. So do not allow 9 x + 13 = 4 y etc.
 6 y + 3x 2 
Note: mN = −   is M0M0 unless a numerical value is then found for mN .
 8 x − 2 − 6 xy 
PhysicsAndMathsTutor.com

Alternative method for part (a): Differentiating with respect to y


 dx  dx  dx  dx
 =  2 + 6 y +  6 xy + 3x 2  = 8 x
 d y  d y  d y  d y
 
dx dx dx  dx 
M1: Differentiates implicitly to include either 2 or 6 xy or ± kx . (Ignore  =  ).
dy dy dy  dy 
   2 dx 
A1: ( 2 x + 3 y ) →  2 ddyx + 6 y  and
2
 4 x → 8 x  . Note: If an extra “sixth” term appears then award A0.
dy 
  
dy
B1: 6 x y + 3 x 2 .
dx
dx dy
dM1: Substituting x = − 1 and y = 1 into an equation involving or . Allow this mark if either the
dy dx
dx 6 y + 3x 2
numerator or denominator of = is substituted into or evaluated correctly.
dy 8 x − 2 − 6 xy
If it is clear, however, that the candidate is intending to substitute x = 1 and y = − 1, then award M0.
Candidates who substitute x = 1 and y = − 1, will usually achieve m ( T ) = − 4
Note that this mark is dependent on the previous method mark being awarded.
4 8 •
A1: For − or − or − 0.4 or awrt − 0.44
9 18
If the candidate’s solution is not completely correct, then do not give this mark.
PhysicsAndMathsTutor.com

Question
Scheme Marks
Number
1 1
2. (a)
∫ x sin 3x dx = − 3 x cos3x − ∫ − 3 cos 3x {dx} M1 A1
1 1
= − x cos 3x + sin 3 x {+ c} A1
3 9
[3]
1 2
(b)
∫ x 2 cos3 x dx = x 2 sin 3 x −
3 ∫
3
x sin 3 x {dx} M1 A1

1 2 1 1 
= x 2 sin 3 x −  − x cos3 x + sin 3 x  {+ c} A1 isw
3 3 3 9 
 1 2 2 2 
= x sin 3x + x cos 3x − sin 3x {+ c} Ignore subsequent working [3]
 3 9 27 
6
(a) M1: Use of ‘integration by parts’ formula u v − ∫ v u ' (whether stated or not stated) in the correct direction,
where u = x → u ′ = 1 and v′ = sin 3 x → v = k cos3 x (seen or implied), where k is a positive or negative
constant. (Allow k = 1 ).
This means that the candidate must achieve x ( k cos3x ) −
∫ ( k cos 3x ) , where k is a consistent constant.
If x 2 appears after the integral, this would imply that the candidate is applying integration by parts in the wrong
direction, so M0.
1 1
3 ∫
A1: − x cos3 x − − cos3x {dx} . Can be un-simplified. Ignore the {dx}.
3
1 1
A1: − x cos3 x + sin 3x with/without + c. Can be un-simplified.
3 9
(b) M1: Use of ‘integration by parts’ formula u v − ∫ v u ' (whether stated or not stated) in the correct direction,
where u = x 2 → u ′ = 2 x or x and v′ = cos3 x → v = λ sin 3 x (seen or implied), where λ is a positive or
negative constant. (Allow λ = 1 ).
This means that the candidate must achieve x 2 (λ sin 3 x) −
∫ 2 x (λ sin 3x) , where u′ = 2 x
or x 2 (λ sin 3 x) −
∫ x (λ sin 3x) , where u′ = x.
If x3 appears after the integral, this would imply that the candidate is applying integration by parts in the wrong
direction, so M0.
1 2 2
A1:
3
x sin 3x −
3∫x sin 3x {dx} . Can be un-simplified. Ignore the {dx}.

1 2 2 1 1 
A1: x sin 3 x −  − x cos3 x + sin 3 x  with/without + c, can be un-simplified.
3 3 3 9 
You can ignore subsequent working here.
Special Case: If the candidate scores the first two marks of M1A1 in part (b), then you can award the final A1
1 2
as a follow through for x 2 sin 3 x − ( their follow through part(a) answer ) .
3 3
PhysicsAndMathsTutor.com

Question
Scheme Marks
Number
−2 −2
1 −2  5x  1  5x  1
3. (a) = (2 − 5 x)− 2 = ( 2 ) 1 −  = 1 −  (2) − 2 or B1
(2 − 5 x) 2
 2  4 2  4
1   (− 2)(− 3) 
=    1 + (− 2)(** x) + (** x) 2 + ...  see notes M1 A1ft
 
4 2! 
1    5 x  (− 2)(− 3)  5 x  
2

=    1 + (− 2)  −  +  −  + ... 
 4    2  2!  2  
1 75 2 
=  1 + 5 x; + x + ...  See notes below!
4 4 
1 5 75 2
= + x; + x + ... A1; A1
4 4 16
[5]
 2+kx  1 5  75  Can be implied by later work
(b)  2 
= (2 + kx)  + x +  x 2 + ...  M1
 (2 − 5 x)  4 4  16  even in part (c).
2(5 x) k x 7 x
x terms: + =
4 4 4
giving, 10 + k = 7 ⇒ k = − 3 k = −3 A1
[2]
2 2
150 x 5k x
(c) x 2 terms: + M1
16 4
75 5(− 3) 75 15 45 45 5
So, A = + = − = or 5 or 5.625 A1
8 4 8 4 8 8 8
[2]
9
1 1
(a) B1: (2) − 2 or outside brackets or as candidate’s constant term in their binomial expansion.
4 4
M1: Expands to give a simplified or an un-simplified,
(− 2)(− 3) (− 2)(− 3)
1 + (− 2)(** x) or (− 2) (** x ) + (** x ) or 1 + ...... + (** x ) , where ** ≠ 1 .
2 2

2! 2!
(− 2)(− 3)
A1: A correct simplified or an un-simplified 1 + (− 2)(** x) + (** x)2 expansion with candidate’s
2!
follow through (** x ) . Note that (** x ) must be consistent.
1  5 x  (− 2)(− 3) 
You would award B1M1A0 for =  1 + (− 2)  −  + (− 5 x)2 + ...  because ** is not consistent.
4   2  2! 
1    5 x  (− 2)(− 3)  5 x  
2
Invisible brackets    1 + (− 2)  −  +  −  + ...  is M1A0 unless recovered.
 4    2  2!  2  
1 5 1 1
A1: For + x (simplified fractions) or Also allow 0.25 + 1.25 x or + 1 x .
4 4 4 4
1  75 2 
Allow Special Case A1 for either SC: [ 1 + 5 x ; ... ] or SC: K  1 + 5 x + x + ...  .
4  4 
75 2 11 2
A1: Accept only x or 4 x or 4.6875 x 2
16 16
Alternative method: Candidates can apply an alternative form of the binomial expansion. (See next page).
PhysicsAndMathsTutor.com

3. (b) M1: Candidate writes down (2 + kx) ( their part (a) answer, at least up to the term in x.)
1 5  1 5 75 2 
(2 + kx)  + x + ...  or (2 + kx)  + x + x + ...  are fine.
4 4  4 4 16 
This mark can also be implied by candidate multiplying out to find two terms (or coefficients) in x.
A1: k = − 3
1 5 75 2 
M1: Multiplies out their (2 + kx)  + x + x + ...  to give exactly two terms (or coefficients) in x 2
(c) 4 4 16 
and attempts to find A using a numerical value of k.
45 5 45 2
A1: Either or 5 or 5.625 Note: x is A0.
8 8 8

Alternative method for part (a)


(− 2)( − 3) − 4
(2 − 5 x)− 2 = (2)− 2 + (− 2)(2) −3 (−5 x); + (2) (−5 x)2
2!
1
B1: or (2)− 2 ,
4
M1: Any two of three (un-simplified) terms correct.
A1: All three (un-simplified) terms correct.
1 5
A1: + x
4 4
75 2
A1: x
16
Note: The terms in C need to be evaluated, so −2C0 (2) − 2 + −2C1 (2) −3 (−5 x); + −2C2 (2) − 4 (−5 x) 2 without
further working is B0M0A0.

Alternative method for parts (b) and (c)


1 7 
(2 + kx) = (2 − 5 x)2  + x + A x 2 + ... 
2 4 
1 7 
(2 + kx) = (4 − 20 x + 25 x 2 )  + x + A x 2 + ... 
2 4 
 25 2 
(2 + kx) = 2 + (7 x − 10 x) +  4 Ax 2 − 35 x 2 + x 
 2 
Equate x terms: k = − 3
25 45 45
Equate x 2 terms: 0 = 4 A − 35 + ⇒ 4A = ⇒ A=
2 2 8
1 7 
(b) M1: For (2 + kx) = (4 ± λ x + 25 x 2 )  + x + A x 2 + ...  , where λ ≠ 0
2 4 
A1: k = − 3
(c) M1: Multiplies out to obtain three x 2 terms/coefficients, equates to 0 and attempts to find A.
45 5 45 2
A1: Either or 5 or 5.625 Note: x is A0.
8 8 8
PhysicsAndMathsTutor.com

Question
Scheme Marks
Number
2 2
  2x 
4. Volume = π
∫ 0
  2

 
3x +
  dx
4

Use of V = π ∫ y 2 dx . B1

2 ± k ln ( 3 x 2 + 4 ) M1
= ( π )  ln ( 3x 2 + 4 ) 
1 
ln ( 3x 2 + 4 )
1
 3 0 A1
3
 1  1 
= ( π )  ln16  −  ln 4  
Substitutes limits of 2 and 0
dM1
 3  3  and subtracts the correct way round.
1 1 2
So Volume = π ln 4 π ln 4 or π ln 2 A1 oe isw
3 3 3
[5]
5
NOTE: π is required for the B1 mark and the final A1 mark. It is not required for the 3 intermediate marks.
B1: For applying π ∫ y 2 . Ignore limits and dx . This can be implied by later working,
2x
but the pi and
∫ 3x 2
+4
must appear on one line somewhere in the candidate’s working.

( ∫ y ) would be B0.
2
B1 can also be implied by a correct final answer. Note: π
Working in x
 4
M1: For ± k ln ( 3 x 2 + 4 ) or ± k ln  x 2 +  where k is a constant and k can be 1.
 3
Note: M0 for ± k x ln ( 3 x + 4 ) .
2

( )
Note: M1 can also be given for ± k ln p ( 3 x 2 + 4 ) , where k and p are constants and k can be 1.

ln ( 3x 2 + 4 ) or ln  ( 3 x 2 + 4 )  or ln  x 2 +  or ln ( p (3 x 2 + 4) ) .
1 1 1  1  4 1
A1: For
3 3 3  3  3 3

You may allow M1 A1 for   ln ( 3x 2 + 4 ) or   ln ( 3 x 2 + 4 )


1 x  1  2x 
3 x  3  6x 
dM1: Substitutes limits of 2 and 0 and subtracts the correct way round. Working in decimals is fine for dM1.
1 2  16 
 
A1: For either π ln 4 , ln 4π , π ln 2, π ln 4 3 , π ln 2 3 , π ln   , 2 π ln  1  , etc.
1 1 2 1 16 16
3 3 3 3  4  46 
 
1
Note: π (ln16 − ln 4) would be A0.
3
Working in u: where u = 3 x 2 + 4 ,
M1: For ± k ln u where k is a constant and k can be 1.
Note: M1 can also be given for ± k ln ( pu ) , where k and p are constants and k can be 1.
1 1 1
A1: For ln u or ln 3u or ln pu .
3 3 3
dM1: Substitutes limits of 16 and 4 in u or limits of 2 and 0 in x and subtracts the correct way round.
A1: As above!
PhysicsAndMathsTutor.com

Question
Scheme Marks
Number
 π
5. x = 4sin  t +  , y = 3cos 2t , 0 „ t < 2π
 6
dx  π  dy
(a) = 4cos  t +  , = − 6sin 2t B1 B1
dt  6  dt
dy − 6sin 2t
So, =
dx  π B1 oe
4cos  t + 
 6
[3]
 dy 
(b)  = 0 ⇒  − 6sin 2t = 0 M1 oe
 dx 
π 
@ t = 0, x = 4sin   = 2 , y = 3cos 0 = 3 → (2, 3)
6 M1
π  2π  4 3
@t= , x = 4sin   = , y = 3cos π = − 3 → (2 3 , − 3)
2  3  2
 7π 
@ t = π , x = 4sin   = − 2 , y = 3cos 2π = 3 → ( − 2, 3)
 6 
3π  5π  4( − 3 )
@t= , x = 4sin   = , y = 3cos 3π = − 3 → (− 2 3 , − 3) A1A1A1
2  3  2
[5]
8
dx  π dy
(a) B1: Either one of = 4cos  t +  or = − 6sin 2t . They do not have to be simplified.
dt  6 dt
dx dy
B1: Both and correct. They do not have to be simplified.
dt dt
Any or both of the first two marks can be implied.
Don’t worry too much about their notation for the first two B1 marks.
dy dx dy 1
B1: Their divided by their or their × . Note: This is a follow through mark.
dt dt dt  dx 
their  
 dt 
Alternative differentiation in part (a)
dx
x = 2 3 sin t + 2cos t ⇒ = 2 3 cos t − 2sin t
dt
dy
y = 3(2cos 2 t − 1) ⇒ = 3 ( − 4cos t sin t )
dt
dy
or y = 3cos 2 t − 3sin 2 t ⇒ = − 6cos t sin t − 6sin t cos t
dt
dy
or y = 3(1 − 2sin 2 t ) ⇒ = 3 ( − 4cos t sin t )
dt
PhysicsAndMathsTutor.com

5. (b) dy
M1: Candidate sets their numerator from part (a) or their equal to 0.
dt
dx
Note that their numerator must be a trig function. Ignore equal to 0 at this stage.
dt
M1: Candidate substitutes a found value of t, to attempt to find either one of x or y.
The first two method marks can be implied by ONE correct set of coordinates for ( x , y ) or ( y , x ) interchanged.
A correct point coming from NO WORKING can be awarded M1M1.
A1: At least TWO sets of coordinates.
A1: At least THREE sets of coordinates.
A1: ONLY FOUR correct sets of coordinates. If there are more than 4 sets of coordinates then award A0.
Note: Candidate can use the diagram’s symmetry to write down some of their coordinates.
π 
Note: When x = 4sin   = 2 , y = 3cos 0 = 3 is acceptable for a pair of coordinates.
6
Also it is fine for candidates to display their coordinates on a table of values.
Note: The coordinates must be exact for the accuracy marks. Ie (3.46..., -3) or (-3.46..., -3) is A0.
dy
Note: = 0 ⇒ sin t = 0 ONLY is fine for the first M1, and potentially the following M1A1A0A0.
dx
dy
Note: = 0 ⇒ cos t = 0 ONLY is fine for the first M1 and potentially the following M1A1A0A0.
dx
dy
Note: = 0 ⇒ sin t = 0 & cos t = 0 has the potential to achieve all five marks.
dx
Note: It is possible for a candidate to gain full marks in part (b) if they make sign errors in part (a).

(b) An alternative method for finding the coordinates of the two maximum points.
Some candidates may use y = 3cos 2t to write down that the y-coordinate of a maximum point is 3.
They will then deduce that t = 0 or π and proceed to find the x-coordinate of their maximum point. These
candidates will receive no credit until they attempt to find one of the x-coordinates for the maximum point.
 π
M1M1: Candidate states y = 3 and attempts to substitute t = 0 or π into x = 4sin  t +  .
 6
M1M1 can be implied by candidate stating either (2, 3) or (2, − 3).
Note: these marks can only be awarded together for a candidate using this method.
A1: For both (2, 3) or (− 2, 3).
A0A0: Candidate cannot achieve the final two marks by using this method. They can, however, achieve these
marks by subsequently solving their numerator equal to 0.
PhysicsAndMathsTutor.com

Question
Scheme Marks
Number
6. (a) 0.73508 B1 cao
[1]
1 π
(b) Area ≈ × ;× 0 + 2 ( their 0.73508 + 1.17157 + 1.02280 ) + 0  B1 M1
2 8
π
= × 5.8589... = 1.150392325... = 1.1504 (4 dp) awrt 1.1504 A1 [3]
16
du
(c) {u = 1 + cos x} ⇒ = − sin x B1
dx
 
∫ ∫
2sin 2 x 2(2sin x cos x)
 dx =  dx sin 2 x = 2sin x cos x B1
 (1 + cos x)  (1 + cos x)
4(u − 1)  (1 − u ) 
=
∫ u
.(−1) du = 4
 ∫u
du 

M1


1 
= 4  − 1 du = 4 ( ln u − u ) + c dM1
u 
= 4ln (1 + cos x ) − 4 (1 + cos x ) + c = 4ln (1 + cos x ) − 4cos x + k AG A1 cso [5]
π
  π π Applying limits x = and
(d) =  4ln  1 + cos  − 4cos  −  4ln (1 + cos 0 ) − 4cos 0  2 M1
  2 2 x = 0 either way round.
= [ 4 ln1 − 0] − [ 4ln 2 − 4]
±4(1− ln 2) or
= 4 − 4ln 2 {= 1.227411278...} ± (4 − 4ln 2) or awrt ± 1.2, A1
however found.
Error = ( 4 − 4ln 2 ) − 1.1504... awrt ± 0.077
A1 cso [3]
= 0.0770112776... = 0.077 (2sf ) or awrt ± 6.3(%)
12
(a) B1: 0.73508 correct answer only. Look for this on the table or in the candidate’s working.
1 π π
(b) B1: Outside brackets × or or awrt 0.196
2 8 16
M1: For structure of trapezium rule [ ............. ] ; (0 can be implied).
A1: anything that rounds to 1.1504
Bracketing mistake: Unless the final answer implies that the calculation has been done correctly
1 π
Award B1M0A0 for × + 2 ( their 0.73508 + 1.17157 + 1.02280 ) (nb: answer of 6.0552).
2 8
1 π
Award B1M0A0 for × (0 + 0) + 2 ( their 0.73508 + 1.17157 + 1.02280 ) (nb: answer of 5.8589).
2 8
Alternative method for part (b): Adding individual trapezia
π  0 + 0.73508 0.73508 +1.17157 1.17157 + 1.02280 1.02280 + 0 
Area ≈ × + + +  = 1.150392325...
8  2 2 2 2 
π
B1: and a divisor of 2 on all terms inside brackets.
8
M1: One of first and last ordinates, two of the middle ordinates inside brackets ignoring the 2.
A1: anything that rounds to 1.1504
PhysicsAndMathsTutor.com

du dx 1
6. (c) B1: = − sin x or du = − sin x dx or = oe.
dx du − sin x
B1: For seeing, applying or implying sin 2 x = 2sin x cos x.
(u − 1) (1 − u )
M1: After applying substitution candidate achieves ± k
∫ u
( du ) or ± k
∫ u
( du ) .
(λ u − 1) ( −λ + u )
Allow M1 for “invisible” brackets here, eg: ±
∫ u
( du ) or ±
∫ u
( du ) , where λ is a
positive constant.


1 
dM1: An attempt to divide through each term by u and ± k  − 1 du → ± k ( ln u − u ) with/without
 u 
+ c . Note that this mark is dependent on the previous M1 mark being awarded.
Alternative method: Candidate can also gain this mark for applying integration by parts followed by a
correct method for integrating ln u. (See below).
A1: Correctly combines their + c and " − 4 " together to give 4ln (1 + cos x ) − 4cos x + k
As a minimum candidate must write either 4ln (1 + cos x ) − 4 (1 + cos x ) + c → 4ln (1 + cos x ) − 4cos x + k
or 4ln (1 + cos x ) − 4 (1 + cos x ) + k → 4ln (1 + cos x ) − 4cos x + k
Note: that this mark is also for a correct solution only.
Note: those candidates who attempt to find the value of k will usually achieve A0.
(d) π
M1: Substitutes limits of x = and x = 0 into {4ln (1 + cos x ) − 4cos x} or their answer from part (c) and
2
  π π
subtracts the either way round. Note that:  4ln 1 + cos  − 4cos  − [ 0] is M0.
  2  2
A1: 4(1 − ln 2) or 4 − 4ln 2 or awrt 1.2, however found.
This mark can be implied by the final answer of either awrt ± 0.077 or awrt ± 6.3
A1: For either awrt ± 0.077 or awrt ± 6.3 (for percentage error). Note this mark is for a correct solution
only. Therefore if there if a candidate substitutes limits the incorrect way round and final achieves (usually
fudges) the final correct answer then this mark can be withheld. Note that awrt 6.7 (for percentage error) is
A0.
Alternative method for dM1 in part (c)
(1 − u )    
∫ ∫ ∫
u
du =  (1 − u ) ln u − − ln u du  =  (1 − u ) ln u + u ln u − du  = ( (1 − u ) ln u + u ln u − u )
u    u 
(u − 1)     
∫ ∫ ∫
u
or du =  (u − 1) ln u − ln u du  =  (u − 1) ln u −  u ln u − du   = ( (u − 1) ln u − u ln u + u )
u     u 
(1 − u )
So dM1 is for
∫ u
du going to ( (1 − u ) ln u + u ln u − u ) or ( (u − 1)ln u − u ln u + u ) oe.

Alternative method for part (d)


 1


1 
M1A1 for 4  − 1 du =  4 [ ln u − u ]2 = 4 ( ln1 − 1) − ( ln 2 − 2 )  = 4 (1 − ln 2 )
1

 2  u  
Alternative method for part (d): Using an extra constant λ from their integration.
  π π 
 4ln 1 + cos 2  − 4cos 2 + λ  −  4ln (1 + cos 0 ) − 4cos 0 + λ 
   
λ is usually − 4 , but can be a value of k that the candidate has found in part (d).
Note: The extra constant λ should cancel out and so the candidate can gain all three marks using this
method, even the final A1 cso.
PhysicsAndMathsTutor.com

Question
Scheme Marks
Number
{ }
uuur uuur uuur uuur
7. OA = 2i − j + 5k , OB = 5i + 2 j + 10k , OC = 2i + 4 j + 9k & OD = − i + j + 4k
uuur
(a) AB = = ± ( (5i + 2 j + 10k ) − (2i − j + 5k ) ) ; = 3i + 3 j + 5k M1; A1
[2]
 2  3  5  3
       
(b) l : r =  −1  + λ  3  or r =  2  + λ  3 See notes M1 A1ft
 5  5  10  5
       
C [2]
D
Let θ = BAD
ˆ
14
d Let d be the shortest
°
distance from C to l.
71° 109
l
A 43 B
 −1   2   − 3   3
uuur uuur uuur       uuur  
(c) AD = OD − OA =  1  −  −1  =  2  or DA =  − 2  M1
 4   5   −1   
       1
 3  −3 Applies dot product
    formula between
 3 •  2
( )
uuur uuur uuur uuur
AB • AD  5   −1  their AB or BA M1
cos θ = uuur uuur =    
( )
uuur uuur
AB . AD (3) + (3) + (5)2 . (− 3) 2 + (2) 2 + (−1) 2
2 2
and their AD or DA .

 −9 + 6 −5  Correct followed through


cos θ = ±  
 (3)2 + (3)2 + (5)2 . (− 3) 2 + (2)2 + (−1) 2  expression or equation. A1
 
−8
cos θ = ⇒ θ = 109.029544... = 109 (nearest °) awrt 109
A1 cso AG
43 . 14
[4]
uuur uuur uuur uuur uuur
(d) OC = OD + DC = OD + AB = ( −i + j + 4k ) + ( 3i + 3j + 5k )
uuur uuur uuur uuur uuur M1
OC = OB + BC = OB + AD = ( 5i + 2 j + 10k ) + ( −3i + 2 j − k )
uuur
So, OC = 2i + 4 j + 9k A1
[2]
(e) Area ABCD = ( 1
2
°
)
( 43)( 14)sin109 ; × 2 = 23.19894905 awrt 23.2 M1; dM1 A1
[3]
d
(f) = sin 71 or 43 d = 23.19894905... M1
14
∴ d = 14 sin 71° = 3.537806563... awrt 3.54 A1
[2]
15
PhysicsAndMathsTutor.com

uuur uuur
7. (a) M1: Finding the difference between OB and OA .
Can be implied by two out of three components correct in 3i + 3j + 5k or − 3i − 3 j − 5k
A1: 3i + 3 j + 5k
(b) M1: An expression of the form ( 3 component vector ) ± λ ( 3 component vector )
( ) ( )
uuur uuur uuur uuur
A1ft: r = OA + λ their ± AB or r = OB + λ their ± AB .
 x
 
Note: Candidate must begin writing their line as r = or l = ... or  y  = ... So, Line = ... would be A0.
z
 
uuur uuur
(c) M1: An attempt to find either the vector AD or DA.
Can be implied by two out of three components correct in − 3i + 2 j − k or 3i − 2 j + k , respectively.
( ) ( )
uuur uuur uuur uuur
M1: Applies dot product formula between their AB or BA and their AD or DA .
A1ft: Correct followed through expression or equation. The dot product must be correctly followed through
correctly and the square roots although they can be un-simplified must be followed through correctly.
A1: Obtains an angle of awrt 109 by correct solution only.
Award the final A1 mark if candidate achieves awrt 109 by either taking the dot product between:
 3  −3  − 3  3
       
(i)  3  and  2  or (ii)  − 3  and  − 2  . Ignore if any of these vectors are labelled incorrectly.
 5  −1   − 5  1
       
Award A0, cso for those candidates who take the dot product between:
 −3  −3  3  3
       
(iii)  − 3  and  2  or (iv)  3  and  − 2  .
 −5  −1   5  1
       
They will usually find awrt 71 and apply 180 – awrt 71 to give awrt 109. If these candidates give a
convincing detailed explanation which must include reference to the direction of their vectors then this can be
given A1 cso. If still in doubt, here, send to review.
uuur uuur uuur uuur
(d) M1: Applies either OD + their AB or OB + their AD .
uuur
This mark can be implied by two out of three correctly followed through components in their OD .
A1: For 2i + 4 j + 9k .
(e) M1: 1
2 ( their AB )( their CB ) sin ( their 109° or 71° from (b) ) . Awrt 11.6 will usually imply this mark.
dM1: Multiplies this by 2 for the parallelogram. Can be implied.
Note: 12 ( ( their AB + their AB ) ) ( their CB ) sin ( their 109° or 71° from (b) )
A1: awrt 23.2

= sin ( their 109° or 71° from (b) ) or (their AB ) d = (their Area ABCD)
(f) d
M1:
their AD
Award M0 for ( their AB ) in part (f), if the area of their parallelogram in part (e) is
( their AB )( their CB ) .
Award M0 for
d
their 43
= sin 71 or ( their )
14 d = 23.19894905...

A1: awrt 3.54


Note: Some candidates will use their answer to part (f) in order to answer part (e).
PhysicsAndMathsTutor.com

7. Alternative method for part (c): Applying the cosine rule:


 −1   2   − 3   3
uuur uuur uuur       uuur  
AD = OD − OA =  1  −  −1  =  2  or DA =  − 2  M1: as above.
 4   5   −1   
       1
 5   −1   6   − 6
uuur uuur uuur       uuur  
DB = OD − OA =  2  −  1  =  1  or BD =  −1 
 10   4   6   −6 
       
uuur uuur uuur
So AB = 43 , AD = 14 and DB = 73
a2 + b2 − c2
M1: Cosine rule structure of cosθ = assigned
( ) +( ) −( ) 2ab
2 2 2
43 14 73 uuur uuur uuur
cos θ = each of AB , AD and DB in any order as their a, b and c.
2 43 . 14
A1: Correct application of cosine rule.
 − 16 
cos θ = ⇒ θ = 109.029544...  = 109 (nearest °) A1: awrt 109 (no errors seen). AG
 2 43. 14 

Alternative method for part (d):


 2  3
uuur    
OE =  −1  + λ  3 
 5 5
   
 2 + 3λ   −1   3 + 3λ 
uuur      
DE =  −1 + 3λ  −  1  =  − 2 + 3λ 
 5 + 5λ   4   1 + 5λ 
     
 3 + 3λ   3 
uuur uuur    
DE • AB = 0 ⇒  − 2 + 3λ  •  3  = 0 uuur uuur
 1 + 5λ   5  M1: Takes the dot product between DE and AB and
    progresses to find a value of λ
8
9 + 9λ − 6 + 9λ + 5 + 3λ = 0 ⇒ λ = −
43
 2 + 3λ   −1   103
43 
uuur      110  uuur
DE =  −1 + 3λ  −  1  =  − 43  dM1: Uses their value of λ to find DE
 5 + 5λ   4   3 
     43 
Length DE = 3.537806563... A1: awrt 3.54
PhysicsAndMathsTutor.com

Question
Scheme Marks
Number
8. (a) 1 = A(5 − P ) + B P Can be implied. M1
1 1
A= ,B= Either one. A1
5 5
1 1
giving 5
+ 5
See notes. A1 cao, aef
P (5 − P )
[3]

∫ ∫
1 1
(b) dP = dt B1
P (5 − P) 15
1 1 1 M1*
ln P − ln(5 − P ) = t ( + c )
5 5 15 A1ft
1 1  1 
{t = 0, P = 1 ⇒} ln1 − ln(4) = 0 + c ⇒ c = − ln 4  dM1*
5 5  5 
Using any of the
1  P  1 1 subtraction (or addition)
eg: ln   = t − ln 4 dM1*
5  5 − P  15 5 laws for logarithms
CORRECTLY
 4P  1
ln   = t
5 − P 3
4P 5−P −1t
=e 3
1t
eg: = e3 or eg: Eliminate ln’s correctly. dM1*
5−P 4P
gives 4 P = 5e 3 − Pe 3 ⇒ P (4 + e 3 ) = 5e 3
1t 1t 1t 1t

 (÷ e 3 t ) 
1t 1
5e 3
P = 1t  1t  Make P the subject. dM1*
(4 + e 3 )  (÷ e ) 
3

5 25
P = − 13 t
or P = − 13 t
etc. A1
(1 + 4e ) (5 + 20e )
[8]
− 13 t
(c) 1 + 4e >1 ⇒ P < 5. So population cannot exceed 5000. B1
[1]
12
(a) M1: Forming a correct identity. For example, 1 = A(5 − P ) + B P . Note A and B not referred to in question.
1 1
A1: Either one of A = or B = .
5 5
1 1
1 1
A1: 5 + 5
or any equivalent form, eg: + , etc. Ignore subsequent working.
P (5 − P ) 5P 25 − 5 P
This answer must be stated in part (a) only.
1 1 A B
A1 can also be given for a candidate who finds both A = and B = and + is seen in their
5 5 P 5− P
working.
1 1
Candidate can use ‘cover-up’ rule to write down 5
+ 5
, as so gain all three marks.
P (5 − P )
Candidate cannot gain the marks for part (a) in part (b).
PhysicsAndMathsTutor.com

8. (b) B1: Separates variables as shown. dP and dt should be in the correct positions, though this mark can be
implied by later working. Ignore the integral signs.
M1*: Both ± λ ln P and ± µ ln(± 5 ± P ) , where λ and µ are constants.
Or ± λ ln mP and ± µ ln(n(± 5 ± P )) , where λ , µ , m and n are constants.
λ µ
A1ft: Correct follow through integration of both sides from their
∫ P
+
(5 − P )
dP =
∫ K dt

with or without + c
dM1*: Use of t = 0 and P = 1 in an integrated equation containing c
dM1*: Using ANY of the subtraction (or addition) laws for logarithms CORRECTLY.
dM1*: Apply logarithms (or take exponentials) to eliminate ln’s CORRECTLY from their equation.
dM1*: A full ACCEPTABLE method of rearranging to make P the subject. (See below for examples!)
5
A1: P = − 1t
{where a = 5, b = 1, c = 4} .
(1 + 4e 3 )
25
Also allow any “integer” multiples of this expression. For example: P = − 1t
(5 + 20e 3 )
Note: If the first method mark (M1*) is not awarded then the candidate cannot gain any of the six
remaining marks for this part of the question.

∫ ∫ ∫ ∫
1 1
1
Note: dP = 15 dt ⇒ 5
+ 5
dP = 15 dt ⇒ ln P − ln(5 − P ) = 15t is B0M1A1ft.
P(5 − P ) P (5 − P )
dM1* for making P the subject
Note there are three type of manipulations here which are considered acceptable to make P the subject.
P 5
= e 3 ⇒ P = 5e 3 − Pe 3 ⇒ P (1 + e 3 ) = 5e 3 ⇒ P =
1t 1t 1t 1t 1t
(1) M1 for
5−P − 1t
(1 + e 3 )
P 1t 5−P 1t 5 1t 5 1t 5
(2) M1 for = e3 ⇒ = e3 ⇒ − 1 = e3 ⇒ = e3 + 1 ⇒ P =
5−P
1t
P P P (1 + e 3 )
2
1t 1t  5 25 1t
(3) M1 for P (5 − P ) = 4e 3 ⇒ P 2 − 5P = − 4e 3 ⇒  P −  − = − 4e 3 leading to P = ...
 2 4
P P
Note: The incorrect manipulation of = − 1 or equivalent is awarded this dM0*.
5−P 5
1
t 1
Note: ( P ) − (5 − P ) = e 3
⇒ 2 P − 5 = t leading to P = ... or equivalent is awarded this dM0*
3
− 13 t
(c) B1: 1 + 4e > 1 and P < 5 and a conclusion relating population (or even P) or meerkats to 5000.
25 − 1t
For P = − 13 t
, B1 can be awarded for 5 + 20e 3 > 5 and P < 5 and a conclusion relating
(5 + 20e )
population (or even P) or meerkats to 5000.
a
B1 can only be obtained if candidates have correct values of a and b in their P = − 1t
.
(b + c e 3 )
− 1t 5
Award B0 for: As t → ∞ , e 3 → 0 . So P → = 5 , so population cannot exceed 5000,
(1 + 0)
5
unless the candidate also proves that P = − 13 t
oe. is an increasing function.
(1 + 4e )
If unsure here, then send to review!
PhysicsAndMathsTutor.com

8. Alternative method for part (b)


1 1 1
B1M1*A1: as before for ln P − ln(5 − P ) = t ( + c )
5 5 15
 P  1
Award 3rd M1for ln   = t+c
5 − P 3
P 1t
Award 4th M1 for = Ae 3
5−P
1  1
Award 2nd M1 for t = 0, P = 1 ⇒ = Ae0 ⇒ A = 
5 −1  4
P 1 t1
= e3
5−P 4
then award the final M1A1 in the same way.
PhysicsAndMathsTutor.com

Mark Scheme (Results)

Summer 2012

GCE Core Mathematics C4


(6666) Paper 1
PhysicsAndMathsTutor.com

June 2012
6666 Core Mathematics C4
Mark Scheme

Question
Scheme Marks
Number

1 = A ( 3 x − 1) + Bx ( 3 x − 1) + Cx
2
1. (a) B1
x→0 (1 = A ) M1
x→ 1
3 1= C ⇒ C = 3
1
3 any two constants correct A1
2
Coefficients of x
0 = 9 A + 3 B ⇒ B = −3 all three constants correct A1 (4)

⌠⎛1 3 3 ⎞
(b)(i) ⎮ ⎜ − + ⎟ dx
⎮ ⎜ x 3 x − 1 ( 3x − 1) ⎟ 2
⌡⎝ ⎠
3 3
= ln x − ln ( 3x − 1) + ( 3x − 1) ( +C )
−1

3 ( −1) 3 M1 A1ft A1ft


( +C ) ⎞⎟
1
⎜ = ln x − ln ( 3 x − 1) −
⎝ 3x − 1 ⎠

2
⎡ 1 ⎤
f ( x ) dx = ⎢ln x − ln ( 3 x − 1) −
2
(ii) ∫ 1
⎣ 3x − 1 ⎥⎦ 1
⎛ 1⎞ ⎛ 1⎞
= ⎜ ln 2 − ln 5 − ⎟ − ⎜ ln1 − ln 2 − ⎟ M1
⎝ 5⎠ ⎝ 2⎠
2× 2
= ln + ... M1
5
3 ⎛4⎞
= + ln ⎜ ⎟ A1 (6)
10 ⎝5⎠
[10]
PhysicsAndMathsTutor.com

Question
Scheme Marks
Number

dV
2. (a) V = x 3 ⇒ = 3x2 ¿ cso B1 (1)
dx

dx dx dV 0.048
(b) = × = M1
dt dV dt 3x 2
At x = 8
dx 0.048
=
dt 3 ( 82 )
= 0.00025 ( cm s )
−1
2.5 × 10−4 A1 (2)

dS
(c) S = 6 x 2 ⇒ = 12 x B1
dx
dS dS dx ⎛ 0.048 ⎞
= × = 12 x ⎜ 2 ⎟ M1
dt dx dt ⎝ 3x ⎠
At x = 8
= 0.024 ( cm 2 s −1 )
dS
A1 (3)
dt
[6]
PhysicsAndMathsTutor.com

Question
Scheme Marks
Number

(a) f ( x ) = ... (... − ... x )


− 12
3. M1
1

6 6
= 6×9 ( ... )
2
1 , , 2 or equivalent B1
9 2 3

⎛ ( − 1 ) ( − 32 ) kx 2 + ( − 12 ) ( − 32 )( − 52 ) kx 3 + ... ⎞
= ... ⎜1 + ( − 12 ) ( kx ) ; + 2 ( ) ( ) ⎟
M1; A1ft
⎝ 2 3! ⎠
⎛ 2 ⎞ 4
= 2 ⎜1 + x + ... ⎟ or 2 + x A1
⎝ 9 ⎠ 9
4 4 2 40 3
= 2+ x+ x + x + ... A1 (6)
9 27 729

4 4 2 40 3
(b) g ( x ) = 2 − x+ x − x + ... B1ft (1)
9 27 729

4 4 40
(c) h ( x ) = 2 + ( 2 x ) + ( 2 x ) + ( 2 x ) + ...
2 3
M1 A1 (2)
9 27 729
⎛ 8 16 320 ⎞
⎜= 2+ x + x2 + x 3 + ... ⎟ [9]
⎝ 9 27 729 ⎠
PhysicsAndMathsTutor.com

Question
Scheme Marks
Number

⌠ 3
4. ∫ y d y = ⎮⌡ cos x dx
2
Can be implied. Ignore integral signs B1

= ∫ 3sec x dx
2

1 2
y = 3 tan x ( +C ) M1 A1
2

π
y = 2, x =
4
1 2 π
2 = 3 tan + C M1
2 4
Leading to
C = −1
1 2
y = 3 tan x − 1 or equivalent A1 (5)
2
[5]
PhysicsAndMathsTutor.com

Question
Scheme Marks
Number

dy dy
5. (a) Differentiating implicitly to obtain ± ay 2 and/or ±bx 2 M1
dx dx
dy
48 y 2 + ... − 54 ... A1
dx
dy
9x2 y → 9x2 + 18 xy or equivalent B1
dx

( 48 y 2 + 9 x 2 ) ddxy + 18 xy − 54 = 0 M1

d y 54 − 18 xy ⎛ 18 − 6 xy ⎞
= ⎜= 2 ⎟ A1 (5)
dx 48 y 2 + 9 x 2 ⎝ 16 y + 3 x ⎠
2

(b) 18 − 6 xy = 0 M1
3 3
Using x = or y=
y x
2
⎛3⎞ ⎛3⎞
3
⎛3⎞ ⎛3⎞
16 y + 9 ⎜ ⎟ y − 54 ⎜ ⎟ = 0 or
3
16 ⎜ ⎟ + 9 x 2 ⎜ ⎟ − 54 x = 0 M1
⎝ y⎠ ⎝ y⎠ ⎝ x⎠ ⎝ x⎠
Leading to

16 y 4 + 81 − 162 = 0 or 16 + x 4 − 2 x 4 = 0 M1
81
y4 = or x 4 = 16
16
3 3
y= ,− or x = 2, − 2 A1 A1
2 2

Substituting either of their values into xy = 3 to obtain a value of the


M1
other variable.
⎛ 3⎞ ⎛ 3⎞
⎜ 2, ⎟ , ⎜ −2, − ⎟ both A1 (7)
⎝ 2⎠ ⎝ 2⎠
[12]
PhysicsAndMathsTutor.com

Question
Scheme Marks
Number

dx
6. (a) = 2 3 cos 2t B1
dt
dy
= −8cos t sin t M1 A1
dt
d y −8cos t sin t
= M1
dx 2 3 cos 2t
4sin 2t
=−
2 3 cos 2t
dy 2 ⎛ 2⎞
=− 3 tan 2t ⎜k = − ⎟ A1 (5)
dx 3 ⎝ 3⎠

π 3
(b) When t = x = , y =1 can be implied B1
3 2
2 ⎛ 2π ⎞
m=− 3 tan ⎜ ⎟ ( = 2) M1
3 ⎝ 3 ⎠
⎛ 3⎞
y −1 = 2 ⎜ x − ⎟ M1
⎝ 2⎠
y = 2x − 2 A1 (4)

(c) x = 3 sin 2t = 3 × 2sin t cos t M1


x 2 = 12sin 2 t cos 2 t = 12 (1 − cos 2 t ) cos 2 t
⎛ y⎞ y
x 2 = 12 ⎜ 1 − ⎟ or equivalent M1 A1 (3)
⎝ 4⎠4
[12]
Alternative to (c)
y = 2 cos 2t + 2 M1
sin 2t + cos 2t = 1
2 2

x2 ( y − 2)
2

+ =1 M1 A1 (3)
3 4
PhysicsAndMathsTutor.com

Question
Scheme Marks
Number

(a)
x 1 2 3 4
7. M1
y ln2 2 ln 4 3 ln 6 2ln8
0.6931 1.9605 3.1034 4.1589

1
Area = × 1( ...) B1
2
≈ ... ( 0.6931 + 2 (1.9605 + 3.1034 ) + 4.1589 ) M1
1
≈ × 14.97989 ... ≈ 7.49 7.49 cao A1 (4)
2

x ln 2 x − ⌠
2 32 2 3 1
∫x
1
(b) 2
ln 2 x dx = ⎮ x 2 × dx M1 A1
3 ⌡3 x
= x 2 ln 2 x − ⌠
2 3 2 1
⎮ x 2 dx
3 ⌡3
2 3 4 3
= x 2 ln 2 x − x 2 ( +C ) M1 A1 (4)
3 9

4
⎡ 2 32 4 32 ⎤ ⎛ 2 32 4 32 ⎞ ⎛ 2 4⎞
(c) ⎢⎣ 3 x ln 2 x − x = ⎜ 4 ln 8 − 4 ⎟ − ⎜ ln 2 − ⎟ M1
9 ⎥⎦ 1 ⎝ 3 9 ⎠ ⎝3 9⎠
= (16 ln 2 − ...) − ... Using or implying ln 2n = n ln 2 M1
46 28
= ln 2 − A1 (3)
3 9
[11]
PhysicsAndMathsTutor.com

Question
Scheme Marks
Number

⎛ 8 ⎞ ⎛ 10 ⎞ ⎛ −2 ⎞
uuur ⎜ ⎟ ⎜ ⎟ ⎜ ⎟
8. (a) AB = ⎜ 3 ⎟ − ⎜ 2 ⎟ = ⎜ 1 ⎟ M1 A1 (2)
⎜ 4⎟ ⎜ 3 ⎟ ⎜ 1⎟
⎝ ⎠ ⎝ ⎠ ⎝ ⎠

⎛10 ⎞ ⎛ −2 ⎞ ⎛ 8 ⎞ ⎛ −2 ⎞
⎜ ⎟ ⎜ ⎟ ⎜ ⎟ ⎜ ⎟
(b) r = ⎜ 2 ⎟ + t ⎜ 1 ⎟ r = ⎜ 3⎟ + t ⎜ 1⎟ M1 A1ft (2)
⎜ 3 ⎟ ⎜ 1⎟ ⎜ 4⎟ ⎜ 1⎟
⎝ ⎠ ⎝ ⎠ ⎝ ⎠ ⎝ ⎠

⎛10 − 2t ⎞ ⎛ 3 ⎞ ⎛ 7 − 2t ⎞
uur ⎜ ⎟ ⎜ ⎟ ⎜ ⎟
(c) CP = ⎜ 2 + t ⎟ − ⎜12 ⎟ = ⎜ t − 10 ⎟ M1 A1
⎜ 3+t ⎟ ⎜ 3 ⎟ ⎜ t ⎟
⎝ ⎠ ⎝ ⎠ ⎝ ⎠

⎛ 7 − 2t ⎞ ⎛ −2 ⎞
⎜ ⎟⎜ ⎟
⎜ t − 10 ⎟ . ⎜ 1 ⎟ = −14 + 4t + t − 10 + t = 0 M1
⎜ t ⎟ ⎜ 1⎟
⎝ ⎠⎝ ⎠
Leading to t=4 A1
⎛10 − 8 ⎞ ⎛ 2 ⎞
⎜ ⎟ ⎜ ⎟
Position vector of P is ⎜ 2 + 4 ⎟ = ⎜ 6 ⎟ M1 A1 (6)
⎜ 3+ 4 ⎟ ⎜7⎟
⎝ ⎠ ⎝ ⎠
[10]

Alternative working for (c)

⎛ 8 − 2t ⎞ ⎛ 3 ⎞ ⎛ 5 − 2t ⎞
uur ⎜ ⎟ ⎜ ⎟ ⎜ ⎟
CP = ⎜ 3 + t ⎟ − ⎜12 ⎟ = ⎜ t − 9 ⎟ M1 A1
⎜ 4 + t ⎟ ⎜ 3 ⎟ ⎜ t +1 ⎟
⎝ ⎠ ⎝ ⎠ ⎝ ⎠
⎛ 5 − 2t ⎞ ⎛ −2 ⎞
⎜ ⎟⎜ ⎟
⎜ t − 9 ⎟ . ⎜ 1 ⎟ = −10 + 4t + t − 9 + t + 1 = 0 M1
⎜ t +1 ⎟ ⎜ 1 ⎟
⎝ ⎠⎝ ⎠
Leading to t =3 A1
⎛8 − 6⎞ ⎛ 2⎞
⎜ ⎟ ⎜ ⎟
Position vector of P is ⎜ 3 + 3 ⎟ = ⎜ 6 ⎟ M1 A1 (6)
⎜ 4 + 3⎟ ⎜ 7 ⎟
⎝ ⎠ ⎝ ⎠
PhysicsAndMathsTutor.com

Mark Scheme (Results)

January 2013

GCE Mathematics
6666 Core Mathematics 4
PhysicsAndMathsTutor.com

January 2013
6666 Core Mathematics C4
Mark Scheme
Question
Scheme Marks
Number
−3 −3
−3 ⎛ 3x ⎞ 1 ⎛ 3x ⎞ 1
1. (2 + 3 x) − 3 = ( 2 ) ⎜ 1 + ⎟ = ⎜1 + ⎟ (2) − 3 or B1
⎝ 2 ⎠ 8⎝ 2 ⎠ 8
⎧1 ⎫ ⎡ (− 3)(− 4) (− 3)(− 4)(−5) ⎤
= ⎨ ⎬ ⎢ 1 + (− 3)(k x) + (k x) 2 + (k x)3 + ... ⎥ see notes M1 A1
⎩ ⎭⎣
8 2! 3! ⎦
⎧1 ⎫ ⎡ ⎛ 3x ⎞ (− 3)( − 4) ⎛ 3x ⎞ (− 3)(− 4)(−5) ⎛ 3 x ⎞ ⎤
2 3

= ⎨ ⎬ ⎢ 1 + (− 3) ⎜ ⎟ + ⎜ ⎟ + ⎜ ⎟ + ... ⎥
⎩ 8 ⎭ ⎣⎢ ⎝ 2 ⎠ 2! ⎝ 2 ⎠ 3! ⎝ 2 ⎠ ⎦⎥
1⎡ 9 27 2 135 3 ⎤
= ⎢ 1 − x; + x − x + ... ⎥ See notes below!
8⎣ 2 2 4 ⎦
1 9 27 2 135 3
= − x; + x − x + ... A1; A1
8 16 16 32
[5]
5
1 1
B1: (2) − 3 or outside brackets or as constant term in the binomial expansion.
8 8
M1: Expands (... + kx )
−3
to give any 2 terms out of 4 terms simplified or un-simplified,
(− 3)( − 4) (− 3)(− 4)
1 + (− 3)(k x) or (− 3) ( k x ) + ( k x ) or 1 + ...... + (k x)
2 2
Eg:
2! 2!
(− 3)(− 4) ( − 3)( − 4)( − 5)
or (k x)2 + (k x)3 where k ≠ 1 are ok for M1.
2! 3!
(− 3)(− 4) (− 3)(− 4)( −5)
A1: A correct simplified or un-simplified 1 + (− 3)(k x) + (k x) 2 + ( k x )3
2! 3!
expansion with consistent ( k x ) where k ≠ 1 .

⎧1 ⎫ ⎡ ⎛ 3x ⎞ (− 3)(− 4) ⎛ 3x ⎞ (− 3)(− 4)(−5) ⎛ 3x ⎞ ⎤


2 3
“Incorrect bracketing” ⎨ ⎬ ⎢ 1 + (− 3) ⎜ ⎟ + ⎜ ⎟ + ⎜ ⎟ + ... ⎥ is M1A0
⎩ 8 ⎭ ⎣⎢ ⎝ 2 ⎠ 2! ⎝ 2 ⎠ 3! ⎝ 2 ⎠ ⎥⎦
unless recovered.
1 9
A1: For − x (simplified fractions) or also allow 0.125 − 0.5625 x.
8 16
1⎡ 9 ⎤ ⎡ 9 27 2 135 3 ⎤
Allow Special Case A1 for either SC: ⎢ 1 − x ; ... ⎥ or SC: K ⎢ 1 − x + x − x + ... ⎥
8⎣ 2 ⎦ ⎣ 2 2 4 ⎦
(where K can be 1 or omitted), with each term in the [.........] either a simplified fraction or a decimal.
27 2 135 3 11 7
A1: Accept only x − x or 1 x 2 − 4 x 3 or 1.6875 x 2 − 4.21875 x 3
16 32 16 32
PhysicsAndMathsTutor.com

1. ctd 1⎡ ⎛ 3 x ⎞ (− 3)(− 4) ⎛ 3 x ⎞ ( − 3)(− 4)(−5) ⎛ 3 x ⎞ ⎤


2 3

Candidates who write = ⎢ 1 + (− 3) ⎜ − ⎟ + ⎜ − ⎟ + ⎜ − ⎟ + ... ⎥ where


8 ⎣⎢ ⎝ 2 ⎠ 2! ⎝ 2 ⎠ 3! ⎝ 2 ⎠ ⎦⎥
3 3 1 9 27 2 135 3
k = − and not and achieve + x+ x + x + ... will get B1M1A1A0A0.
2 2 8 16 16 32
Alternative method: Candidates can apply an alternative form of the binomial expansion.
(− 3)(− 4) − 5 (− 3)(− 4)(−5) − 6
(2 + 3 x) − 3 = (2) − 3 + (− 3)(2) −4 (3 x) + (2) (3 x) 2 + (2) (3 x)3
2! 3!
1
B1: or (2) − 3
8
M1: Any two of four (un-simplified) terms correct.
A1: All four (un-simplified) terms correct.
1 9
A1: − x
8 16
27 2 135 3
A1: + x − x
16 32
Note: The terms in C need to be evaluated, so −3C0 (2) − 3 + −3C1 (2) −4 (3 x) + −3C2 (2) − 5 (3 x) 2 + −3C3 (2) − 6 (3 x)3
without further working is B0M0A0.
PhysicsAndMathsTutor.com

Question
Scheme
Number
⎧ du 1 ⎫
⎪⎪u = ln x ⇒ dx = x ⎪⎪
∫ x ln x dx ,
1
2. (a) ⎨ −2 ⎬
⎪ dv = x −3 ⇒ v = x = −1
3

⎪⎩ dx −2 2 x 2 ⎪⎭


±λ 1 1
In the form 2
ln x ± µ 2 . M1
x x x


−1 −1 1 −1
= ln x − . dx ln x simplified or un-simplified. A1
2 x2 2 x2 x 2x2


−1 1
− . simplified or un-simplified. A1
2x2 x

∫x
⎧ −1 1 1 ⎫
⎨= 2
ln x + 3
dx ⎬
⎩ 2 x 2 ⎭


1 1
1 1⎛ 1 ⎞ ± µ . → ± β x −2 . dM1
= − 2 ln x + ⎜ − 2 ⎟ {+ c} 2
x x
2x 2 ⎝ 2x ⎠
Correct answer, with/without + c A1
[5]
Applies limits of 2 and
⎪⎧ ⎡ 1 1 ⎤ ⎪⎫ ⎛ 1 ⎞ ⎛ 1 ⎞
2
1 1 1 to their part (a)
(b) ⎨ ⎢ − 2 ln x − 2 ⎥ ⎬ = ⎜ − ln 2 − 2 ⎟
− ⎜− ln1 − ⎟ M1
⎩⎪ ⎣ 2 x 4 x ⎦1 ⎭⎪ ⎝ 2(2) 2 2
4(2) ⎠ ⎝ 2(1) 4(1) 2 ⎠ answer and subtracts
the correct way round.
1
3 1 3 1
= − ln 2 or − ln 2 8 or ( 3 − 2ln 2 ) , etc, or awrt 0.1 or equivalent. A1
16 8 16 16
[2]
7


±λ 1 1
(a) M1: Integration by parts is applied in the form 2
ln x ± µ 2 . or equivalent.
x x x
−1
A1: ln x simplified or un-simplified.
2x2


−1 1
A1 : − . or equivalent. You can ignore the dx.
2x2 x


1 1
dM1: Depends on the previous M1. ± µ . → ± β x −2 .
x2 x
1 1⎛ 1 ⎞ 1 1 x− 2 x− 2
A1: − 2 ln x + ⎜ − 2 ⎟ {+ c} or = − 2 ln x − 2 {+ c} or ln x − {+ c}
2x 2 ⎝ 2x ⎠ 2x 4x −2 4
−1 − 2ln x
or {+ c} or equivalent.
4x2
You can ignore subsequent working after a correct stated answer.
(b) M1: Some evidence of applying limits of 2 and 1 to their part (a) answer and subtracts the correct way round.
3 1 3 1
1 ln ( 1 ) + 3
A1: Two term exact answer of either − ln 2 or − ln 2 8 or ( 3 − 2ln 2 ) or 4
16 8 16 16 16
or 0.1875 − 0.125ln 2. Also allow awrt 0.1. Also note the fraction terms must be combined.
Note: Award the final A0 in part (b) for a candidate who achieves awrt 0.1 in part (b), when their answer to
part (a) is incorrect.
PhysicsAndMathsTutor.com

2. (b) ctd Note: Decimal answer is 0.100856... in part (b).

Alternative Solution
⎧ du ⎫
⎪ u = x −3 ⇒ = − 3x −4 ⎪


1 ⎪ dx ⎪
3
ln x dx , ⎨ ⎬
x ⎪ dv = ln x ⇒ v = x ln x − x ⎪
⎪⎩ dx ⎪⎭

∫ ∫
1 1 −3
3
ln x dx = 3 ( x ln x − x) − ( x ln x − x) 4 dx
x x x

∫ ∫
1 1 λ
k 3
ln x dx = 3 ( x ln x − x) ± dx
x x x3 M1
where k ≠ 1

∫ ∫
1 1 3


−2 ln x dx = 3 ( x ln x − x) − dx 1 3
x3 x x3 Any one of 3 ( x ln x − x) or − dx A1
x x3


1 3
( x ln x − x) − dx and k = − 2 A1
x3 x3

∫ ∫
1 1 3 1
−2 3
ln x dx = 3 ( x ln x − x) + {+ c} ± µ 3 → ± β x −2 . dM1
x x 2 x2 x
1 3


1 1 3 − 3 ( x ln x − x) − or equivalent
3
ln x dx = − 3 ( x ln x − x) − {+ c} 2x 4 x2 A1
x 2x 4 x2
with/without + c .
1 1
=− 2
ln x − 2 {+ c}
2x 4x
PhysicsAndMathsTutor.com

Question
Scheme Marks
Number
3. Method 1: Using one identity
9 x 2 + 20 x − 10 B C
≡ A + +
( x + 2)(3 x − 1) ( x + 2) (3x − 1)
A=3 their constant term = 3B1
9 x + 20 x − 10 ≡ A( x + 2)(3 x − 1) + B (3 x − 1) + C ( x + 2)
2
Forming a correct identity. B1
x2 : 9 = 3 A , x : 20 = 5 A + 3B + C Attempts to find the value of
Either
constant: − 10 = − 2 A − B + 2C either one of their B or their C M1
or from their identity.
x = − 2 ⇒ 36 − 40 − 10 = − 7 B ⇒ −14 = − 7 B ⇒ B = 2
Correct values for
1 20 7 7 7 their B and their C, which are A1
x= ⇒1+ − 10 = C ⇒ − = C ⇒ C = − 1 found using a correct identity.
3 3 3 3 3
[4]
Method 2: Long Division
9 x 2 + 20 x − 10 5x − 4
≡ 3+ their constant term = 3 B1
( x + 2)(3 x − 1) ( x + 2)(3 x − 1)
5x − 4 B C
So, ≡ +
( x + 2)(3 x − 1) ( x + 2) (3 x − 1)
5 x − 4 ≡ B(3x − 1) + C ( x + 2) Forming a correct identity. B1

Either x : 5 = 3B + C , constant: − 4 = − B + 2C Attempts to find the value of


either one of their B or their C M1
or from their identity.
x = − 2 ⇒ − 10 − 4 = − 7 B ⇒ − 14 = − 7 B ⇒ B = 2
Correct values for
1 5 7 7 7 their B and their C, which are
x = ⇒ − 4 = C ⇒ − = C ⇒ C = −1
3 3 3 3 3 found using A1
5 x − 4 ≡ B (3 x − 1) + C ( x + 2)
9 x 2 + 20 x − 10 2 1 [4]
So, ≡ 3 + −
( x + 2)(3x − 1) ( x + 2) (3 x − 1)
4

1st B1: Their constant term must be equal to 3 for this mark.
2nd B1 (M1 on epen): Forming a correct identity. This can be implied by later working.
M1 (A1 on epen): Attempts to find the value of either one of their B or their C from their identity. This can
be achieved by either substituting values into their identity or comparing coefficients and solving the
resulting equations simultaneously.
A1: Correct values for their B and their C, which are found using a correct identity.
9 x 2 + 20 x − 10 A B
Note : ≡ + , leading to 9 x 2 + 20 x − 10 ≡ A(3x − 1) + B ( x + 2) , leading to
( x + 2)(3x − 1) ( x + 2) (3x − 1)
A = 2 and B = −1 will gain a maximum of B0B0M1A0

.
PhysicsAndMathsTutor.com

3. ctd 9 x 2 + 20 x − 10 A( x + 2)(3x − 1) + B (3x − 1) + C ( x + 2)


Note: You can imply the 2nd B1 from either ≡
( x + 2)(3x − 1) ( x + 2)(3 x − 1)
5x − 4 B (3 x − 1) + C ( x + 2)
or ≡
( x + 2)(3x − 1) ( x + 2)(3 x − 1)

Alternative Method 1: Initially dividing by (x + 2)


9 x 2 + 20 x − 10 9x + 2 14
≡ −
"( x + 2)"(3 x − 1) (3 x − 1) ( x + 2)(3 x − 1)
5 14
≡ 3+ − B1: their constant term = 3
(3 x − 1) ( x + 2)(3 x − 1)
− 14 B C
So, ≡ +
( x + 2)(3x − 1) ( x + 2) (3x − 1)
−14 ≡ B(3x − 1) + C ( x + 2) B1: Forming a correct identity.
⇒ B = 2, C = − 6 M1: Attempts to find either one of their
B or their C from their identity.
9 x 2 + 20 x − 10 5 2 6
So, ≡ 3+ + −
( x + 2)(3 x − 1) (3x − 1) ( x + 2) (3 x − 1)
9 x 2 + 20 x − 10 2 1
and ≡ 3 + − A1: Correct answer in partial fractions.
( x + 2)(3x − 1) ( x + 2) (3x − 1)

Alternative Method 2: Initially dividing by (3x - 1)


9 x 2 + 20 x − 10 3x + 233 7
≡ − 3
( x + 2)"(3x − 1)" ( x + 2) ( x + 2)(3x − 1)
5 7
≡ 3+ 3
− 3
B1: their constant term = 3
( x + 2) ( x + 2)(3x − 1)
− 73 B C
So, ≡ +
( x + 2)(3x − 1) ( x + 2) (3x − 1)
− 73 ≡ B(3 x − 1) + C ( x + 2) B1: Forming a correct identity.
1 M1: Attempts to find either one of their B or their C
⇒ B = , C = −1
3 from their identity.
9 x + 20 x − 10
2 5 1
1
So, ≡ 3+ 3
+ 3

( x + 2)(3 x − 1) ( x + 2) ( x + 2) (3 x − 1)
9 x 2 + 20 x − 10 2 1
and ≡ 3 + − A1: Correct answer in partial fractions.
( x + 2)(3x − 1) ( x + 2) (3x − 1)
PhysicsAndMathsTutor.com

Question
Scheme Marks
Number
4. (a) 1.0981 B1 cao
[1]
1
(b) Area ≈ × 1 ;×⎡⎣ 0.5 + 2 ( 0.8284 + their 1.0981) + 1.3333⎤⎦ B1; M1
2
1
= × 5.6863 = 2.84315 = 2.843 (3 dp) 2.843 or awrt 2.843 A1
2
[3]

{u = 1 + x} ⇒ ddux = 12 x
1
− dx
(c) 2
or = 2(u − 1) B1
du


(u − 1) 2
...... M1

∫ ∫
⎧ x ⎫ (u − 1) 2 u
⎨ dx = ⎬ . 2(u − 1) du
⎩ 1+ x

⎭ u (u − 1) 2
. 2(u − 1) A1
u

∫ ∫
(u − 1)
3
(u − 3u + 3u − 1)
3 2
Expands to give a “four term” cubic in u.
= 2 du = {2} du M1
u u Eg: ± Au 3 ± Bu 2 ± Cu ± D


⎛ 2 1 ⎞
= {2}
An attempt to divide at least three terms in
⎜ u − 3u + 3 − ⎟ du M1
⎝ u ⎠ their cubic by u. See notes.


⎛u 3
3u 2
⎞ (u − 1)3 ⎛ u 3 3u 2 ⎞
= {2} ⎜ − + 3u − ln u ⎟ → ⎜ − + 3u − ln u ⎟ A1
⎝ 3 2 ⎠ u ⎝ 3 2 ⎠
3
⎡ 2u 3 ⎤
Area( R ) = ⎢ − 3u 2 + 6u − 2ln u ⎥
⎣ 3 ⎦2
⎛ 2(3)3 ⎞ ⎛ 2(2)3 ⎞ Applies limits of 3 and 2 in
= ⎜ − 3(3) 2 + 6(3) − 2ln 3 ⎟ − ⎜ − 3(2) 2 + 6(2) − 2ln 2 ⎟ u or 4 and 1 in x and M1
⎝ 3 ⎠ ⎝ 3 ⎠ subtracts either way round.
11 11 ⎛2⎞ 11 ⎛9⎞ Correct exact answer
= + 2ln 2 − 2ln 3 or + 2ln ⎜ ⎟ or − ln ⎜ ⎟ , etc A1
3 3 ⎝3⎠ 3 ⎝4⎠ or equivalent.
[8]
12
(a) B1: 1.0981 correct answer only. Look for this on the table or in the candidate’s working.
1 1
(b) B1: Outside brackets ×1 or
2 2
M1: For structure of trapezium rule [ ............. ]
A1: anything that rounds to 2.843
Note: Working must be seen to demonstrate the use of the trapezium rule. Note: actual area is 2.85573645…
1
Note: Award B1M1 A1 for (0.5 + 1.3333) + ( 0.8284 + their 1.0981) = 2.84315
2
Bracketing mistake: Unless the final answer implies that the calculation has been done correctly
1
Award B1M0A0 for × 1 + 0.5 + 2 ( 0.8284 + their 1.0981) + 1.3333 (nb: answer of 6.1863).
2
1
Award B1M0A0 for × 1 (0.5 + 1.3333) + 2 ( 0.8284 + their 1.0981) (nb: answer of 4.76965).
2
PhysicsAndMathsTutor.com

4. (b) ctd Alternative method for part (b): Adding individual trapezia
⎡ 0.5 + 0.8284 0.8284 +1.0981 1.0981 +1.3333 ⎤
Area ≈ 1× ⎢ + + ⎥⎦ = 2.84315
⎣ 2 2 2
B1: 1 and a divisor of 2 on all terms inside brackets.
M1: First and last ordinates once and two of the middle ordinates twice inside brackets ignoring the 2.
A1: anything that rounds to 2.843
(c)
du 1 − 12 1 dx
B1: = x or du = dx or 2 x du = dx or dx = 2(u − 1)du or = 2(u − 1) oe.
dx 2 2 x du
x (u − 1) 2
1st M1: becoming (Ignore integral sign).
1+ x u
x (u − 1) 2 (u − 1) 2 2
1st A1 (B1 on epen): dx becoming . 2(u − 1) {du} or . {du} .
1+ x u u (u − 1) −1
You can ignore the integral sign and the du .
2nd M1: Expands to give a “four term” cubic in u, ± Au 3 ± Bu 2 ± Cu ± D
where A ≠ 0, B ≠ 0, C ≠ 0 and D ≠ 0 The cubic does not need to be simplified for this mark.
rd
3 M1: An attempt to divide at least three terms in their cubic by u.
(u 3 − 3u 2 + 3u − 1) 1
Ie. → u 2 − 3u + 3 −
u u


(u − 1) 3
⎛u 3
3u 2

2nd A1: du → ⎜ − + 3u − ln u ⎟
u ⎝ 3 2 ⎠
th
4 M1: Some evidence of limits of 3 and 2 in u and subtracting either way round.

11 11 ⎛2⎞ 11 ⎛9⎞ ⎛ 11 ⎞
3rd A1: Exact answer of + 2ln 2 − 2ln 3 or + 2ln ⎜ ⎟ or − ln ⎜ ⎟ or 2 ⎜ + ln 2 − ln 3 ⎟
3 3 ⎝3⎠ 3 ⎝4⎠ ⎝6 ⎠
22 ⎛2⎞ 11 22 2
or + 2ln ⎜ ⎟ , etc . Note: that fractions must be combined to give either or or 3
6 ⎝3⎠ 3 6 3

Alternative method for 2nd M1 and 3rd M1 mark

∫ ∫
(u − 1) 2 (u 2 − 2u + 1)
{2} . (u − 1) du = {2} . (u − 1) du
u u An attempt to expand (u − 1) 2 , then
divide the result by u and then go on to 2nd M1

∫ ∫
⎛ 1⎞ multiply by (u − 1) .
= {2} {} (
⎜ u − 2 + ⎟ . (u − 1) du = 2 u 2 − ...) du
⎝ u⎠
to give three out of four of

∫⎝
⎛ 1⎞
= {2} ⎜ u − 2u + 1 − u + 2 − ⎟ du
2
D 3rd M1
u⎠ ± Au 2 , ± Bu , ± C or ±
u

∫⎝
⎛ 1⎞
= {2} ⎜ u − 3u + 3 − ⎟ du
2

u⎠
PhysicsAndMathsTutor.com

4. (c) ctd Final two marks in part (c): u = 1 + x

( )
4
⎡ 2 1+ x 3

Area( R ) = ⎢ ( ) ( ) ( )
x ⎥
2
− 3 1+ x + 6 1+ x − 2ln 1 +
⎢ 3 ⎥
⎣⎢ ⎦⎥ 1

(
⎛ 2 1+ 4 3
) ⎞

(
− 3 1 + 4 + 6 1 + 4 − 2ln 1 + 4 ⎟) ( ) ( )
2
=
⎜ ⎟
⎜ 3 ⎟
⎝ ⎠ M1: Applies limits of 4 and 1 in x
⎛ 2 1+ 1
( ) ⎞ and subtracts either way round.
3

− ⎜ (
− 3 1 + 1 + 6 1 + 1 − 2ln 1 + 1 ⎟ ) ( ) ( )
2

⎜ ⎟
⎜ 3 ⎟
⎝ ⎠
⎛ 16 ⎞
= (18 − 27 + 18 − 2ln 3) − ⎜ − 12 + 12 − 2ln 2 ⎟
⎝ 3 ⎠
11 11 ⎛2⎞ 11 ⎛9⎞
= + 2ln 2 − 2ln 3 or + 2ln ⎜ ⎟ or − ln ⎜ ⎟ , etc A1: Correct exact answer or equivalent.
3 3 ⎝3⎠ 3 ⎝4⎠

Alternative method for the final 5 marks in part (b)


⎧ d"u " ⎫
⎪ "u " = u −1 ⇒ = − u −2 ⎪⎪

(u − 1) 3
⎪ dx
du , ⎨ ⎬
u ⎪ dv = (u − 1)3 (u − 1) 4 ⎪
⇒ v=
⎪⎩ dx 4 ⎪⎭


(u − 1) 4
1 (u − 1) 4
= −− du
4u 4 u2


(u − 1) 4 1 u 4 − 4u 3 + 6u 2 − 4u + 1 M1: Applies integration by parts and
= + du
4u 4 u2 expands to give a five term quartic.


(u − 1) 4 1 4 1
= + u 2 − 4u + 6 − + 2 du M1: Dividing at least 4 terms.
4u 4 u u
(u − 1) 4 1 ⎛ u3 1⎞
= + ⎜ − 2u 2 + 6u − 4ln u − ⎟ A1: Correct Integration.
4u 4⎝ 3 u⎠
3


⎡ (u − 1) 4 1⎤
3
(u − 1)3 u 3 u 2 3u
du = ⎢ + − + − ln u − ⎥
2 u ⎣ 4u 12 2 2 4u ⎦ 2
⎛ 16 27 9 9 1 ⎞ ⎛1 8 4 6 1⎞
= ⎜ + − + − ln 3 − ⎟ − ⎜ + − + − ln 2 − ⎟ M1
⎝ 12 12 2 2 12 ⎠ ⎝ 8 12 2 2 8⎠
⎛5 ⎞
= ( 7 − ln 3) − ⎜ − ln 2 ⎟
⎝3 ⎠
11 2
= + ln
6 3
(u − 1)3 ⎛ 11 2⎞

3
Area( R ) = 2 du = 2 ⎜ + ln ⎟ A1
2 u ⎝6 3⎠
PhysicsAndMathsTutor.com

Question
Scheme Marks
Number
5. Working parametrically:
1
x = 1 − t , y = 2t − 1 or y = et ln 2 − 1
2
1
(a) { x = 0 ⇒} 0 = 1 − t ⇒ t = 2 Applies x = 0 to obtain a value for t. M1
2
When t = 2 , y = 22 − 1 = 3 Correct value for y. A1
[2]
Applies y = 0 to obtain a value for t.
(b) {y = 0 ⇒} 0 = 2t − 1 ⇒ t = 0 M1
(Must be seen in part (b)).
1
When t = 0 , x = 1 − (0) = 1 x =1 A1
2
[2]
dx 1 dy dy
(c) =− and either = 2t ln 2 or = et ln 2 ln 2 B1
dt 2 dt dt
dy 2t ln 2
= dy dx
dx 1 Attempts their divided by their . M1
− dt dt
2
1 −1
At A, t = "2", so m(T) = − 8ln 2 ⇒ m(N) = Applies t = "2" and m(N) = M1
8ln 2 m ( T)
1 1 M1 A1 oe
y −3= ( x − 0) or y = 3 + x or equivalent. See notes.
8ln 2 8ln 2 cso
[5]
⎛ 1⎞
∫ ( 2 − 1).⎜⎝ − 2 ⎟⎠ dt
Complete substitution
(d) Area( R ) = t
for both y and dx M1
x = − 1 → t = 4 and x = 1 → t = 0 B1
t
2
Either 2t →
ln 2
(2t )
⎧ 1 ⎫⎛ 2
t
⎞ or ( 2t − 1) → −t M1*
= ⎨− ⎬ ⎜ − t⎟ ± α (ln 2)
⎩ 2 ⎭ ⎝ ln 2 ⎠ or (2 t
− 1) → ± α (ln 2)(2t ) − t
2t
(2 t

ln 2
− 1) →
− t A1

⎪⎧ 1 ⎡ 2t ⎤ ⎪⎫
0
1 ⎛ ⎛ 1 ⎞ ⎛ 16 ⎞⎞ Depends on the previous method mark.
⎨− ⎢ − t⎥ ⎬ = − ⎜⎜ ⎟− ⎜ − 4⎟⎟ Substitutes their changed limits in t and dM1*
⎪⎩ 2 ⎣ ln 2 ⎦ 4 ⎪⎭ 2 ⎝ ⎝ ln 2 ⎠ ⎝ ln 2 ⎠⎠ subtracts either way round.
15 15
= −2 − 2 or equivalent. A1
2ln 2 2ln 2
[6]
15
PhysicsAndMathsTutor.com

5. (a) M1: Applies x = 0 and obtains a value of t.


A1: For y = 22 − 1 = 3 or y = 4 − 1 = 3
Alternative Solution 1:
M1: For substituting t = 2 into either x or y.
1
A1: x = 1 − (2) = 0 and y = 22 − 1 = 3
2
Alternative Solution 2:
M1: Applies y = 3 and obtains a value of t.
1
A1: For x = 1 − (2) = 0 or x = 1 − 1 = 0 .
2
Alternative Solution 3:
M1: Applies y = 3 or x = 0 and obtains a value of t.
A1: Shows that t = 2 for both y = 3 and x = 0 .
(b) M1: Applies y = 0 and obtains a value of t. Working must be seen in part (b).
A1: For finding x = 1.
Note: Award M1A1 for x = 1.
dx dy
(c) B1: Both and correct. This mark can be implied by later working.
dt dt
dy dx dy 1 dy
M1: Their divided by their or their × . Note: their must be a function of t.
dt dt dt ⎛ dx ⎞ dt
their ⎜ ⎟
⎝ dt ⎠
−1
M1: Uses their value of t found in part (a) and applies m( N) = .
m ( T)
M1: y − 3 = (their normal gradient) x or y = (their normal gradient) x + 3 or equivalent.

1 1 1
A1: y−3= ( x − 0) or y = 3 + x or y − 3 = ( x − 0) or ( 8ln 2 ) y − 24ln 2 = x
8ln 2 8ln 2 ln 256
y−3 1
or = . You can apply isw here.
( x − 0) 8ln 2
Working in decimals is ok for the three method marks. B1, A1 require exact values.
⎛ dx ⎞
M1: Complete substitution for both y and dx . So candidate should write down ( 2t − 1) . ⎜ their
(d) ∫⎝

dt ⎠
B1: Changes limits from x → t. x = − 1 → t = 4 and x = 1 → t = 0 . Note t = 4 and t = 0 seen is B1.
2t
M1*: Integrates 2t correctly to give
ln 2
(2t )
... or integrates ( 2t − 1) to give either −t or ±α (ln 2)(2t ) − t .
± α (ln 2)
2t
A1: Correct integration of ( 2t − 1) with respect to t to give − t.
ln 2
dM1*: Depends upon the previous method mark.
Substitutes their limits in t and subtracts either way round.
15 15 15 − 4ln 2 7.5 15
A1: Exact answer of − 2 or − 2 or or − 2 or log 2 e − 2 or equivalent.
2ln 2 ln 4 2ln 2 ln 2 2
PhysicsAndMathsTutor.com

Questio
n Scheme Marks
Number
5. Alternative: Converting to a Cartesian equation:
t = 2 − 2x ⇒ y = 2 2− 2 x − 1
Applies x = 0 in their Cartesian
(a) {x = 0 ⇒} y = 22 − 1 M1
equation...
y=3 ... to arrive at a correct answer of 3. A1
[2]
Applies y = 0 to obtain a value for
(b) {y = 0 ⇒} 0 = 22−2 x − 1 ⇒ 0 = 2 − 2 x ⇒ x = ... x. M1
(Must be seen in part (b)).
x =1 x = 1 A1
[2]
± λ 22− 2 x , λ ≠ 1 M1
= − 2 ( 22−2 x ) ln 2
dy
−2 ( 22−2 x ) ln 2 or equivalent A1
(c)
dx

1 −1
At A, x = 0, so m(T) = − 8ln 2 ⇒ m(N) = Applies x = 0 and m(N) = M1
8ln 2 m ( T)
1 1
y −3= ( x − 0) or y = 3 + x or
8ln 2 8ln 2 As in the original scheme. M1 A1 oe
equivalent.
[5]

∫(2 − 1) dx
Area( R ) = 2− 2 x Form the integral of their Cartesian
(d) M1
equation of C.
For 22−2 x − 1 with limits of x = − 1 and
∫ (2 − 1) dx
1
2− 2 x
=
( 22−2 x − 1)
B1

1
−1 x = 1 . Ie.
−1

22 − 2 x
Either 22− 2 x →
− 2ln 2
22 − 2 x
⎛ 2 ⎞
2− 2 x or (2 2− 2 x
− 1) →
± α (ln 2)
−x M1*
=⎜ − x⎟
⎝ − 2ln 2 ⎠ or ( 22− 2 x − 1) → ± α (ln 2)(22− 2 x ) − x
2 2− 2 x
( 22−2 x − 1) → − 2ln 2
−x A1

⎧⎪ ⎡ 22−2 x ⎤ ⎫⎪ ⎛ ⎛ 1
1
⎞ ⎛ 16 ⎞⎞
Depends on the previous method
⎨⎢ − x⎥ ⎬ = ⎜ ⎜ − 1⎟ − ⎜ + 1⎟ ⎟ mark.
− 2ln 2 ⎦ −1 ⎪⎭ ⎝ ⎝ − 2ln 2 ⎠ ⎝ −2ln 2 ⎠⎠
⎩⎪ ⎣ Substitutes limits of -1 and their xB dM1*
and subtracts either way round.
15 15
= −2 − 2 or equivalent. A1
2ln 2 2ln 2
[6]
15
(d) Alternative method: In Cartesian and applying u = 2 – 2x
PhysicsAndMathsTutor.com

∫ ( 2 − 1){dx} , where u = 2 − 2 x ∫ (2 − 1) {dx}


2− 2 x
Area( R) = u
M0: Unless a candidate writes

= ( 2 − 1) ( − ){du}

0
u 1
Then apply the “working parametrically” mark scheme.
2
4
PhysicsAndMathsTutor.com

Questio
n Scheme Marks
Number
5. (d) Alternative method: For substitution u = 2t
⎛ 1⎞
∫ ( 2 − 1).⎜⎝ − 2 ⎟⎠ dt
Complete substitution
Area( R ) = t
for both y and dx M1
du du
where u = 2t ⇒ = 2t ln 2 ⇒ = u ln 2
dt dt
x = − 1 → t = 4 → u = 16 and x = 1 → t = 0 → u = 1 Both correct limits in t or
B1
both correct limits in u.


1u −1 If not awarded above, you can
So area( R ) = − du
2u ln 2 award M1 for this integral


1 1 1
= − − du
2 ln 2 u ln 2
u
Either 2t →
ln 2
or ( 2t − 1) →
u ln u
− M1*
⎧ 1 ⎫⎛ u ln u ⎞ ± α (ln 2) ln 2
= ⎨− ⎬ ⎜ − ⎟
⎩ 2 ⎭⎝ ln 2 ln 2 ⎠
or ( 2t − 1) → ± α (ln 2)(u ) −
ln u
ln 2

( 2t − 1) → ln 2 − ln u2
u ln
A1
⎧⎪ 1 ⎡ u ln u ⎤
1
⎫⎪ 1 ⎛ ⎛ 1 ⎞ ⎛ 16 ln16 ⎞ ⎞ Depends on the previous
⎨− ⎢ − ⎥ ⎬ = − ⎜⎜ ⎟− ⎜ − ⎟⎟ method mark.
⎪⎩ 2 ⎣ ln 2 ln 2 ⎦ 16 ⎪⎭ 2 ⎝ ⎝ ln 2 ⎠ ⎝ ln 2 ln 2 ⎠ ⎠
Substitutes their changed limits in
dM1*
u and subtracts either way round.
15 ln16 15 15 ln16 15
= − or −2 − or −2
2ln 2 2ln 2 2ln 2 2ln 2 2ln 2 2ln 2 A1
or equivalent.
[6]
PhysicsAndMathsTutor.com

Questio
n Scheme Marks
Number
6. (a) { y = 0 ⇒} 1 − 2cos x = 0 1 − 2cos x = 0 , seen or implied. M1
At least one correct value of x. (See notes). A1
5π π
⇒x= , π 5π
3 3 Both and A1 cso
3 3
[3]



For π (1 − 2cos x) .

2
V =π (1 − 2cos x) 2 dx
3
(b) π B1
3 Ignore limits and dx
{∫ (1 − 2cos x) dx } = ∫ (1 − 4cos x + 4cos x)dx
2 2

⎛ 1 + cos 2 x ⎞ cos 2 x = 2cos 2 x − 1


=
∫ 1 − 4cos x + 4 ⎜
⎝ 2
⎟ dx
⎠ See notes.
M1


= ( 3 − 4cos x + 2cos 2 x ) dx

Attempts ∫ y 2 to give any two of


± A → ± Ax , ± B cos x → ± B sin x or M1
2sin 2 x
= 3 x − 4sin x + ±λ cos 2 x → ± µ sin 2 x .
2
Correct integration. A1
⎛⎛ 2sin ( 3 ) ⎞ ⎛ π
10 π
2sin ( 3 ) ⎞ ⎞
2 π Applying limits
V = {π } ⎜ ⎜ 3 ( 53π ) − 4sin ( 53π ) + ⎟ − ⎜ 3 ( 3 ) − 4sin ( π3 ) + ⎟ ⎟
⎜ ⎟ the correct way
⎝⎝ 2 ⎠ ⎝ 2 ⎠⎠
round. Ignore
ddM1
π.
⎛⎛ 3⎞ ⎛ 3 ⎞⎞
= π ⎜ ⎜⎜ 5π + 2 3 − ⎟ − ⎜ π − 2 3 + ⎟⎟

⎝⎝ 2 ⎟⎠ ⎜⎝ 2 ⎟⎠ ⎟⎠
= π ( (18.3060...) − ( 0.5435...) ) = 17.7625π = 55.80

(
= π 4π + 3 3 ) or 4π 2 + 3π 3 Two term exact answer. A1
[6]
9
PhysicsAndMathsTutor.com

6. (a) M1: 1 − 2cos x = 0 .


1
This can be implied by either cos x = or any one of the correct values for x in radians or in
2
degrees.
π 5π
1st A1: Any one of either or or 60 or 300 or awrt 1.05 or 5.23 or awrt 5.24 .
3 3
nd π 5π
2 A1: Both and .
3 3

(b)


B1: (M1 on epen) For π (1 − 2cos x)2 . Ignore limits and dx .
1 M1: Any correct form of cos 2 x = 2cos 2 x − 1 used or written down in the same variable.
st

1 + cos 2 x
This can be implied by cos 2 x = or 4cos 2 x → 2 + 2cos 2 x or cos 2 A = 2cos 2 A − 1 .
2
2nd M1: Attempts ∫ y 2 to give any two of ± A → ± Ax , ± B cos x → ± B sin x or ±λ cos 2 x → ± µ sin 2 x .
Do not worry about the signs when integrating cos x or cos 2 x for this mark.

∫ (1 − 2cos x) = ∫1 + 4cos x is ok for an attempt at ∫ y .


2 2 2
Note:
2sin 2 x 2sin 2 x
1st A1: Correct integration. Eg. 3x − 4sin x + or x − 4sin x + + 2 x oe.
2 2
3rd ddM1: Depends on both of the two previous method marks. (Ignore π ).
5π π
Some evidence of substituting their x = and their x = and subtracting the correct
3 3
way round.
You will need to use your calculator to check for correct substitution of their limits into their integrand
if a candidate does not explicitly give some evidence.
Note: For correct integral and limits decimals gives: π ( (18.3060...) − ( 0.5435...) ) = 17.7625π = 55.80
( )
2nd A1: Two term exact answer of either π 4π + 3 3 or 4π 2 + 3π 3 or equivalent.

Note: The π in the volume formula is only required for the B1 mark and the final A1 mark.
Note: Decimal answer of 58.802... without correct exact answer is A0.
Note: Applying ∫ (1 − 2cos x ) dx will usually be given no marks in this part.
PhysicsAndMathsTutor.com

Questio
n Scheme Marks
Number
i: 9 + λ = 2 + 2µ (1)
j : 13 + 4λ = − 1 + µ (2) Any two equations.
7. (a) M1
(Allow one slip).
k : −3 − 2λ = 1 + µ (3)
Eg: (2) − (3) : 16 + 6λ = − 2 or An attempt to eliminate
dM1
(2) − 4(1) : − 23 = − 9 − 7 µ one of the parameters.
Leading to λ = − 3 or µ = 2 Either λ = − 3 or µ = 2 A1
⎛ 9⎞ ⎛ 1⎞ ⎛ 6⎞ ⎛ 2⎞ ⎛ 2⎞ ⎛ 6⎞
⎜ ⎟ ⎜ ⎟ ⎜ ⎟
l1 : r = ⎜ 13 ⎟ − 3 ⎜ 4 ⎟ = ⎜ 1 ⎟ or l2 : r = ⎜ −1 ⎟ + 2 ⎜⎜ 1 ⎟⎟ = ⎜⎜ 1 ⎟⎟
⎜ ⎟
See notes ddM1 A1
⎜ − 3⎟ ⎜ − 2⎟ ⎜ 3⎟ ⎜ 1⎟ ⎜ 1 ⎟ ⎜ 3⎟
⎝ ⎠ ⎝ ⎠ ⎝ ⎠ ⎝ ⎠ ⎝ ⎠ ⎝ ⎠
[5]
⎛ 1⎞ ⎛ 2⎞ ⎛ 1⎞ ⎛ 2⎞
Realisation that the dot
⎜ ⎟ ⎜ ⎟ ⎜ ⎟ ⎜ ⎟ product is required
(b) d1 = ⎜ 4 ⎟ , d 2 = ⎜ 1 ⎟ ⇒ ⎜ 4 ⎟ • ⎜ 1 ⎟
between ± Ad1 and M1
⎜ − 2⎟ ⎜1⎟ ⎜ − 2⎟ ⎜1⎟
⎝ ⎠ ⎝ ⎠ ⎝ ⎠ ⎝ ⎠ ± Bd 2 .
⎛ 2+ 4−2 ⎞
cos θ = ± ⎜ ⎟ Correct equation. A1
⎜ (1) 2 + (4) 2 + (− 2) 2 . (2) 2 + (1) 2 + (1) 2 ⎟
⎝ ⎠
4
cos θ = ⇒ θ = 69.1238974... = 69.1 (1 dp) awrt 69.1 A1
21. 6
[3]
⎛ 4⎞ ⎛ 9⎞ ⎛ 1⎞ ⎛ 9 + λ ⎞
uuur ⎜ ⎟ uuur ⎜ ⎟ ⎜ ⎟ ⎜ ⎟
(c) OA = ⎜ 16 ⎟ , OP = ⎜ 13 ⎟ + λ ⎜ 4 ⎟ = ⎜ 13 + 4λ ⎟
⎜ − 3⎟ ⎜ − 3⎟ ⎜ − 2 ⎟ ⎜ − 3 − 2λ ⎟
⎝ ⎠ ⎝ ⎠ ⎝ ⎠ ⎝ ⎠
⎛ 9+λ ⎞ ⎛ 4⎞ ⎛λ +5 ⎞
uuur ⎜ ⎟ ⎜ ⎟ ⎜ ⎟
AP = ⎜ 13 + 4λ ⎟ − ⎜ 16 ⎟ = ⎜ 4λ − 3 ⎟ M1 A1
⎜ − 3 − 2λ ⎟ ⎜ − 3 ⎟ ⎜ − 2λ ⎟
⎝ ⎠ ⎝ ⎠ ⎝ ⎠
⎛ λ + 5 ⎞ ⎛ 1⎞
uuur ⎜ ⎟ ⎜ ⎟
AP • d1 = 0 ⇒ ⎜ 4λ − 3 ⎟ • ⎜ 4 ⎟ = λ + 5 + 16λ − 12 + 4λ = 0 dM1
⎜ − 2λ ⎟ ⎜ − 2 ⎟
⎝ ⎠ ⎝ ⎠
1 1
leading to {21λ − 7 = 0 ⇒} λ = λ= A1
3 3
⎛ 1 ⎞ ⎛ 28 ⎞
⎜93 ⎟ ⎜ 3 ⎟
⎛ 9⎞ ⎛ 1⎞ ⎜ ⎟ ⎜ ⎟
uuur
Position vector OP = ⎜⎜ 13 ⎟⎟ + ⎜⎜ 4 ⎟⎟ = ⎜⎜ 14 ⎟⎟ or ⎜⎜
1 1 43 ⎟
ddM1 A1
⎜ − 3⎟ 3 ⎜ − 2 ⎟ ⎜ 3 ⎟ 3 ⎟
⎝ ⎠ ⎝ ⎠ ⎜ ⎟
⎜ −3 2 ⎟ ⎜ − 11 ⎟
⎜ ⎟ ⎜ ⎟
⎝ 3⎠ ⎝ 3 ⎠
[6]
14
PhysicsAndMathsTutor.com

7. (a) M1: Writes down any two equations. Allow one slip.
dM1: Attempts to eliminate either λ or µ to form an equation in one parameter only.
A1: For either λ = − 3 or µ = 2 . Note: candidates only need to find one of the parameters.
ddM1: For either substituting their value of λ into l1 or their µ into l2 .
⎛ 6⎞
2nd
A1: For either ⎜⎜ 1 ⎟⎟ or 6i + j + 3k or ( 6 1 3) .
⎜ 3⎟
⎝ ⎠
Note: Each of the method marks in this part are dependent upon the previous method marks.
M1: Realisation that the dot product is required between ± Ad1 and ± Bd 2 . Allow one slip in
(b)
d1 = i + 4 j − 2k.
⎛ d •d ⎞
A1: Correct application of the dot product formula d1 • d 2 = ± d1 d 2 cosθ or cosθ = ± ⎜⎜ 1 2 ⎟⎟
⎝ d1 d 2 ⎠
The dot product must be correctly applied and the square roots although they can be un-simplified must
be correctly applied.
A1: awrt 69.1 . This can be also be achieved by 180 − 110.876 = awrt 69.1 . θ = 1.2064...c is A0.
⎛ −12 − 24 + 12 ⎞ − 24
Common response: cos θ = ⎜ ⎟= is M1A1...
⎜ (−3) 2 + (−12) 2 + (6) 2 . (4) 2 + (2) 2 + (2) 2 ⎟ 189. 24
⎝ ⎠
Alternative Method: Vector Cross Product
Only apply this scheme if it is clear that a candidate is applying a vector cross product method.
⎛ 1⎞ ⎛ 2⎞ ⎧ i j k ⎫ M1: Realisation that the vector cross
⎜ ⎟ ⎜ ⎟ ⎪ ⎪
d1 × d 2 = ⎜ 4 ⎟ × ⎜ 1 ⎟ = ⎨ 1 4 − 2 = 6i − 5 j − 7k ⎬ product is required between ± Ad1 and
⎜ − 2⎟ ⎜1⎟ ⎪ 2 1 ⎪ ± Bd 2 . Allow one slip in d1 = i + 4 j − 2k.
⎝ ⎠ ⎝ ⎠ ⎩ 1 ⎭
(6) 2 + (5) 2 + (−7) 2
sin θ = A1: Correct applied equation.
(1) 2 + (4) 2 + (− 2) 2 . (2) 2 + (1) 2 + (1) 2
110
sin θ = ⇒ θ = 69.1238974... = 69.1 (1 dp) A1: awrt 69.1
21. 6
(c)
uuur uuur
M1: Attempts to find AP in terms of the parameter by subtracting the components of OP from l1 and
uuur uuur uuur uuur
OA . Ignore the direction of subtraction and ignore any confusion between OP and PO or between OA
uuur
and AO . The correct subtraction of two components is enough to establish that subtraction is intended.
The coordinates or position vector of P must be given in terms of a parameter. Taking P : ( x, y, z ) gains
no marks although this can be recovered later.uuurSee Additional Solutions.
A1: (M1 on epen) A correct expression for AP . Again accept the reverse direction. uuur
dM1: Depends on the previous M. Taking the scalar product of their expression for AP with
d1 or a multiple of d1 and equating to 0 and obtaining an equation for λ . The equation must derive from
an expression of the form x1 x2 + y1 y2 + z1 z2 = 0 . Differentiation can be used. See Additional Solutions.
A1: Solving to find λ = 13 .
ddM1: Depends on both previous Ms. Substitutes their value of the parameter into their expression for
uuur uuur
OP . Substituting into AP is a common error which loses the mark.
Note: Needs 2 correct co-ordinates if λ = 13 found and then P stated without method to gain ddM1.
PhysicsAndMathsTutor.com

1 1 2
A1: 9 i + 14 j − 3 k . Accept vector notation or coordinates. Must be exact.
3 3 3
PhysicsAndMathsTutor.com

7. (c) Additional Solution 1:


⎛ x⎞
uuur ⎜ ⎟
Taking OP = ⎜ y ⎟ , in itself, can gain no marks but this may be converted to a parameter at a later
⎜z⎟
⎝ ⎠
stage in the solution and, at that stage, any relevant marks can be awarded.
⎛ x⎞ ⎛ 4 ⎞ ⎛ x−4 ⎞
uuur ⎜ ⎟ ⎜ ⎟ ⎜ ⎟
For example, AP = ⎜ y ⎟ − ⎜ 16 ⎟ = ⎜ y − 16 ⎟
⎜ z ⎟ ⎜ − 3⎟ ⎜ z + 3 ⎟
⎝ ⎠ ⎝ ⎠ ⎝ ⎠
⎛ x − 4 ⎞ ⎛ 1⎞
leading to: ⎜⎜ y − 16 ⎟⎟ ⎜⎜ 4 ⎟⎟ = x − 4 + 4 y − 64 − 2 z − 6 = 0 No marks gained at this stage.
⎜ z + 3 ⎟ ⎜ − 2⎟
⎝ ⎠⎝ ⎠
⎛ 9⎞ ⎛ 1⎞ ⎛ 9 + λ ⎞
uuur ⎜ ⎟
Using, OP = ⎜ 13 ⎟ + λ ⎜⎜ 4 ⎟⎟ = ⎜⎜ 13 + 4λ ⎟⎟ on x + 4 y − 2 z = 74
⎜ − 3⎟ ⎜ − 2 ⎟ ⎜ − 3 − 2λ ⎟
⎝ ⎠ ⎝ ⎠ ⎝ ⎠
which gives: 9 + λ + 4(13 + 4λ ) − 2(− 3 − 2λ ) = 74 At this stage award M1A1 and dM1
(which is implied by an equation)
1 1
⇒ 21λ + 67 = 74 ⇒ λ = A1: Solving to find λ = .
3 3
Position vector
⎛ 1 ⎞ ⎛ 28 ⎞
⎜93 ⎟ ⎜ 3 ⎟
⎛ 9⎞ ⎛ 1⎞ ⎜ ⎟ ⎜ ⎟
uuur ⎜ ⎟ 1 ⎜ ⎟ ⎜ 1⎟ ⎜ 43 ⎟ ddM1 A1
OP = ⎜ 13 ⎟ + ⎜ 4 ⎟ = 14 or
⎜ ⎟ ⎜ 3 ⎟
⎜ − 3⎟ 3 ⎜ − 2 ⎟ ⎜ 3 ⎟ ⎜ ⎟
⎝ ⎠ ⎝ ⎠
⎜ −3 2 ⎟ ⎜ − 11 ⎟
⎜ ⎟ ⎜ ⎟
⎝ 3⎠ ⎝ 3 ⎠
Additional Solution 2: Using Differentiation
⎛ 9+λ ⎞ ⎛ 4⎞ ⎛λ +5 ⎞
uuur ⎜ ⎟ ⎜ ⎟ ⎜ ⎟
AP = ⎜ 13 + 4λ ⎟ − ⎜ 16 ⎟ = ⎜ 4λ − 3 ⎟ M1A1: As main scheme
⎜ − 3 − 2λ ⎟ ⎜ − 3 ⎟ ⎜ − 2λ ⎟
⎝ ⎠ ⎝ ⎠ ⎝ ⎠
AP 2 = (λ + 5) 2 + (4λ − 3) 2 + (−2λ ) 2 = {21λ 2 − 14λ + 34}
d

( AP 2 ) = 42λ − 14 = 0 M1
1 1
leading to λ = A1: Solving to find λ = .
3 3
... then apply the main scheme.
PhysicsAndMathsTutor.com

Question
Scheme Marks
Number

∫ ∫ ∫ ∫
⎧ dθ (3 − θ ) ⎫ 1 1 125
8. (a) ⎨ = ⎬ ⇒ dθ = dt or dθ = dt B1
⎩ dt 125 ⎭ 3 −θ 125 3 −θ
1 1
− ln (θ − 3) = t {+ c} or − ln ( 3 − θ ) = t {+ c} See notes. M1 A1
125 125
1
ln (θ − 3) = − t +c
125

1
t +c −
1
t Correct completion
θ − 3 = e 125 or e 125 c
e to θ = Ae −0.008t + 3 .
θ = Ae −0.008 t
+3 * A1
[4]
(b) {t = 0 , θ = 16 ⇒} 16 = Ae −0.008(0) + 3 ; ⇒ A = 13 See notes. M1; A1
Substitutes θ = 10 into an equation
10 = 13e −0.008t + 3 of the form θ = Ae−0.008t + 3, M1
or equivalent. See notes.
Correct algebra to −0.008t = ln k ,
7 ⎛7⎞
e −0.008t = ⇒ −0.008t = ln ⎜ ⎟ where k is a positive value. See M1
13 ⎝ 13 ⎠
notes.
⎧ ⎛7⎞ ⎫
⎪⎪ ln ⎜ ⎟ ⎪⎪
⎝ 13 ⎠
⎨t = ⎬ = 77.3799... = 77 ( nearest minute ) awrt 77 A1
⎪ ( − 0.008 ) ⎪
⎪⎩ ⎪⎭
[5]
9
8. (a)
B1: (M1 on epen) Separates variables as shown. dθ and dt should be in the correct positions,
though this mark can be implied by later working. Ignore the integral signs.
M1: Both ± λ ln ( 3 − θ ) or ± λ ln (θ − 3) and ± µ t where λ and µ are constants.
1 1
A1: For − ln (θ − 3) = t or − ln ( 3 − θ ) = t or −125ln (θ − 3) = t or −125ln ( 3 − θ ) = t
125 125
Note: + c is not needed for this mark.
A1: Correct completion to θ = Ae −0.008t + 3 . Note: + c is needed for this mark.
1 1
1 − −
Note: ln (θ − 3) = −
t t
t + c leading to θ − 3 = e 125 + ec or θ − 3 = e 125 + A , would be final
125
A0.
1 1
Note: From − ln (θ − 3) = t + c , then ln (θ − 3) = − t +c
125 125
1 1
− t +c − t
⇒θ − 3 = e or θ − 3 = e 125 ec ⇒ θ = Ae −0.008t + 3 is required for A1.
125

1 1
Note: From − ln ( 3 − θ ) = t + c , then ln ( 3 − θ ) = − t +c
125 125
1 1
− t +c − t
⇒ 3−θ = e 125
or 3 − θ = e 125 c
e ⇒ θ = Ae −0.008t + 3 is sufficient for A1.
1
− t
Note: The jump from 3 − θ = Ae 125
to θ = Ae −0.008t + 3 is fine.
PhysicsAndMathsTutor.com

1
1 −
ln (θ − 3) = −
t
Note: t + c ⇒ θ − 3 = Ae 125 , where candidate writes A = ec is also
125
acceptable.
PhysicsAndMathsTutor.com

8. (b)

M1: (B1 on epen) Substitutes θ = 16, t = 0 , into either their equation containing an unknown
constant or the printed
equation. Note: You can imply this method mark.
A1: (M1 on epen) A = 13 . Note: θ = 13e−0.008t + 3 without any working implies the first two marks,
M1A1.
M1: Substitutes θ = 10 into an equation of the form θ = Ae−0.008t + 3, or equivalent.
where A is a positive or negative numerical value and A can be equal to 1 or -1.
M1: Uses correct algebra to rearrange their equation into the form −0.008t = ln k ,
where k is a positive numerical value.
A1: awrt 77 or awrt 1 hour 17 minutes.
Alternative Method 1 for part (b)

∫ ∫
1 1 1
dθ = dt ⇒ − ln (θ − 3) = t +c
3 −θ 125 125
M1: Substitutes t = 0,θ = 16,
1
− ln (16 − 3) = (0) + c
{t = 0 , θ = 16 ⇒} 125 into − ln (θ − 3) =
1
t +c
⇒ c = − ln13 125
A1: c = − ln13
1 1
− ln (θ − 3) = t − ln13 or ln (θ − 3) = − t + ln13
125 125
M1: Substitutes θ = 10 into an equation of the
1 1
− ln (10 − 3) = t − ln13 form ±λ ln (θ − 3) = ± t ±µ
125 125
where λ , µ are numerical values.
M1: Uses correct algebra to rearrange their
1
ln13 − ln 7 = t equation into the form ± 0.008t = ln C − ln D ,
125
where C, D are positive numerical values.
t = 77.3799... = 77 ( nearest minute ) A1: awrt 77.
Alternative Method 2 for part (b)

∫ ∫
1 1 1
dθ = dt ⇒ − ln 3 − θ = t +c
3 −θ 125 125
M1: Substitutes t = 0,θ = 16,
1
− ln 3 − 16 = (0) + c
{t = 0 , θ = 16 ⇒} 125 into − ln ( 3 − θ ) =
1
t +c
⇒ c = − ln13 125
A1: c = − ln13
1 1
− ln 3 − θ = t − ln13 or ln 3 − θ = − t + ln13
125 125
M1: Substitutes θ = 10 into an equation of the
1 1
− ln ( 3 − 10 ) = t − ln13 form ±λ ln ( 3 − θ ) = ± t ±µ
125 125
where λ , µ are numerical values.
1 M1: Uses correct algebra to rearrange their
ln13 − ln 7 = t
125 equation into the form ± 0.008t = ln C − ln D ,
PhysicsAndMathsTutor.com

where C, D are positive numerical values.


t = 77.3799... = 77 ( nearest minute ) A1: awrt 77.

8. (b) Alternative Method 3 for part (b)

∫ ∫
10 t
1 1
dθ = dt
16 3 −θ 0 125
t
10 ⎡ 1 ⎤
= ⎡⎣ − ln 3 − θ ⎤⎦16 = ⎢ t
⎣125 ⎥⎦ 0
M1A1: ln13
M1: Substitutes limit of θ = 10 correctly.
1 M1: Uses correct algebra to rearrange their
− ln 7 − − ln13 = t
125 own equation into the form
± 0.008t = ln C − ln D ,
where C, D are positive numerical values.
t = 77.3799... = 77 ( nearest minute ) A1: awrt 77.

Alternative Method 4 for part (b)


M1*: Writes down a pair of equations in A and t
{θ = 16 ⇒} 16 = Ae −0.008 t
+3 , for θ = 16 and θ = 10 with either A unknown or
A being a positive or negative value.
{θ = 10 ⇒} 10 = Ae −0.008t + 3
A1: Two equations with an unknown A.

M1: Uses correct algebra to solve both of


⎛ 13 ⎞ ⎛7⎞ their equations leading to answers of the form
− 0.008t = ln ⎜ ⎟ or − 0.008t = ln ⎜ ⎟
⎝ A⎠ ⎝ A⎠ −0.008t = ln k , where k is a positive numerical
value.
⎛ 13 ⎞ ⎛7⎞
ln ⎜ ⎟ ln ⎜ ⎟
= ⎝ ⎠ = ⎝ ⎠
A A
t(1) and t(2)
− 0.008 − 0.008
⎛ 13 ⎞ ⎛7⎞
ln ⎜ ⎟ ln ⎜ ⎟ M1: Finds difference between the two times.
= ⎝ ⎠ − ⎝ A⎠
A
t = t(1) − t(2) (either way round).
− 0.008 − 0.008
⎧ ⎛7⎞ ⎫
⎪⎪ ln ⎜ ⎟ ⎪⎪
⎝ 13 ⎠
⎨t = ⎬ = 77.3799... = 77 ( nearest minute ) A1: awrt 77. Correct solution only.
⎪ ( −0.008) ⎪
⎪⎩ ⎪⎭
PhysicsAndMathsTutor.com

Mark Scheme (Results)

Summer 2013

GCE Core Mathematics 4 (6666/01R)


PhysicsAndMathsTutor.com

Question
Scheme Marks
Number
1. At least one of “A”
5x + 3 A B C B1
≡ + + or “C” are correct.
(2 x + 1)( x + 1) 2
(2 x + 1) ( x + 1) ( x + 1) 2
Breaks up their partial fraction
correctly into three terms and B1 cso
A = 2, C = 2
both " A" = 2 and "C " = 2 .

5 x + 3 ≡ A( x + 1) 2 + B (2 x + 1)( x + 1) + C (2 x + 1)
x = −1 ⇒ − 2 = −C ⇒ C = 2 Writes down a correct identity and
attempts to find the value of either M1
1 5 1 1 1
x=− ⇒ − +3 = A ⇒ = A ⇒ A=2 one “A” or “ B” or “C”.
2 2 4 2 4

Either x 2 : 0 = A + 2 B , constant : 3 = A + B + C
x : 5 = 2 A + 3B + 2C
Correct value for “B” which is found
leading to B = − 1 using a correct identity and follows
A1 cso
from their partial fraction
decomposition.
[4]
5x + 3 2 1 2 4
So, ≡ − +
(2 x + 1)( x + 1) 2
(2 x + 1) ( x + 1) ( x + 1) 2

Notes for Question 1


BE CAREFUL! Candidates will assign their own “A, B and C” for this question.
B1: At least one of “A” or “C” are correct.
B1: Breaks up their partial fraction correctly into three terms and both " A" = 2 and "C " = 2 .
M1: Writes down a correct identity (although this can be implied) and attempts to find the value of either
one of “A” or “B” or “C”.
This can be achieved by either substituting values into their identity or
comparing coefficients and solving the resulting equations simultaneously.
A1: Correct value for “B” which is found using a correct identity and follows from their partial fraction
decomposition.
Note: If a candidate does not give partial fraction decomposition then:
• the 2nd B1 mark can follow from a correct identity.
• the final A1 mark can be awarded for a correct “B” if a candidate goes writes out their partial
fractions at the end.
Note: The correct partial fraction from no working scores B1B1M1A1.
Note: A number of candidates will start this problem by writing out the correct identity and then attempt to
find “A” or “ B” or “C”. Therefore the B1 marks can be awarded from this method.
PhysicsAndMathsTutor.com

Question
Scheme Marks
Number
2. 3x −1 + xy − y 2 + 5 = 0
3x − 1 → 3x − 1 ln 3 B1 oe
Differentiates implicitly to include either
dy dy M1*
⎧ dy ⎫ ⎛ dy ⎞ dy ±λ x or ± ky .
⎨ =⎬ 3x − 1 ln 3 + ⎜ y + x ⎟− 2y = 0 dx dx
⎩ dx ⎭ ⎝ dx ⎠ dx
dy
(ignore) xy → + y + x B1
dx
dy dy A1
... + y + x − 2y = 0
dx dx
Substitutes x = 1, y = 3 into their
{(1, 3) ⇒} 3 (1 − 1) dy dy
ln 3 + 3 + (1)
− 2(3) = 0 dM1*
dx dx differentiated equation or expression.
dy dy dy
ln 3 + 3 + −6 = 0 ⇒ 3 + ln 3 = 5
dx dx dx
dy 3 + ln 3
= dM1*
dx 5
= ( ln e3 + ln 3 ) = ln ( 3e3 ) = ln ( 3e3 )
dy 1 1 dy 1
Uses 3 = ln e3 to achieve A1 cso
dx 5 5 dx 5
[7]
7
Notes for Question 2

B1: Correct differentiation of 3x − 1 . I.e. 3x − 1 → 3x − 1 ln 3 or 3x − 1 =


3
(
1 x
3 ) → ( 3x ) ln 3
1
3
(
1 x
3 ) = e x ln 3 → (ln 3)e x ln 3
or 3x − 1 = e( x − 1) ln 3 → ln 3 e( x − 1) ln 3 or 3x − 1 =
3
1
3
1
3
dy dy ⎛ dy ⎞
M1: Differentiates implicitly to include either ± λ x or ± ky . (Ignore ⎜ = ⎟ ).
dx dx ⎝ dx ⎠
dy
B1: xy → + y + x
dx
dy dy
1st A1: ... + y + x − 2y = 0 Note: The 1st A0 follows from an award of the 2nd B0.
dx dx
Note: The " = 0" can be implied by rearrangement of their equation.
dy dy dy dy
ie: 3x − 1 ln 3 + y + x − 2y leading to 3x − 1 ln 3 + y = 2 y − x will get A1 (implied).
dx dx dx dx
2nd M1: Note: This method mark is dependent upon the 1st M1* mark being awarded.
Substitutes x = 1, y = 3 into their differentiated equation or expression. Allow one slip.
3 M1: Note: This method mark is dependent upon the 1st M1* mark being awarded.
rd

dy dy
Candidate has two differentiated terms in and rearranges to make the subject.
dx dx
Note: It is possible to gain the 3rd M1 mark before the 2nd M1 mark.
dy y + 3x −1 ln 3
Eg: Candidate may write = before substituting in x = 1 and y = 3
dx 2y − x
⎛ 1 ⎞
= ln ( 3e3 ) , ⎜ = ln ( µ e ) , λ = 5 and µ = 3 ⎟
dy 1
2nd A1: cso. Uses 3 = ln e3 to achieve 3

dx 5 ⎝ λ ⎠
Note: 3 = ln e needs to be seen in their proof.
3

Notes for Question 2 Continued


PhysicsAndMathsTutor.com
2. Alternative Method: Multiplying both sides by 3
3x −1 + xy − y 2 + 5 = 0
3x + 3 xy − 3 y 2 + 15 = 0
3 x → 3x l n 3 B1
Differentiates implicitly to include either
dy dy M1*
⎧ dy ⎫ x ⎛ dy ⎞ dy ±λ x or ± ky .
Aliter ⎨ = ⎬ 3 ln3 + ⎜ 3 y + 3x ⎟ − 6 y = 0 dx dx
Way 2 ⎩ dx ⎭ ⎝ dx ⎠ dx
dy
(ignore) 3xy → + 3 y + 3 x B1
dx
dy dy A1
... + 3 y + 3 x − 6y = 0
dx dx
Substitutes x = 1, y = 3 into their
{(1, 3) ⇒} 3 ln 3 + 3(3) + (3)(1) ddyx − 6(3) ddyx =
1
0
differentiated equation or expression.
dM1*
dy dy dy
3ln 3 + 9 + 3 − 18 = 0 ⇒ 9 + 3ln3 = 15
dx dx dx
dy 9 + 3ln 3 ⎧ 3 + ln 3 ⎫
= ⎨= ⎬ dM1*
dx 15 ⎩ 5 ⎭

= ( ln e3 + ln 3 )
dy 1
dx 5
= ( ln e3 + ln 3 ) = ln ( 3e3 ) = ln ( 3e3 )
dy 1 1 dy 1
Uses 3 = ln e3 to achieve A1 cso
dx 5 5 dx 5
[7]
7

NOTE: Only apply this scheme if the candidate has multiplied both sides of their equation by 3.
dy 3 y + 3x ln 3
NOTE: For reference, =
dx 6 y − 3x
dy
NOTE: If the candidate applies this method then 3xy → + 3 y + 3x must be seen for the 2nd B1 mark.
dx
PhysicsAndMathsTutor.com

Question
Scheme Marks
Number


4
1
3. dx , u = 2 + (2 x + 1)
0 2 + (2 x + 1)
1
du − dx M1
1
Either = ± K (2 x + 1) 2 or = ± λ (u − 2)
du − dx dx du
= (2 x + 1) 2
or =u−2 1
dx du du − dx A1
Either = (2 x + 1) 2 or = (u − 2)
dx du
⎧⎪ ⎫⎪
∫ ∫
1 1 Correct substitution A1
⎨ dx ⎬ = (u − 2) du
⎪⎩ 2 + (2 x + 1) ⎪⎭ u (Ignore integral sign and du ).


⎛ 2⎞
= ⎜ 1 − ⎟ du An attempt to divide each term by u. dM1
⎝ u⎠
± Au ± B ln u ddM1
= u − 2ln u
u − 2ln u A1 ft

Applies limits of 5 and 3 in u


{So [u − 2ln u ] } = ( 5 − 2ln 5) − ( 3 − 2ln 3)
5
3
or 4 and 0 in x in their integrated function M1
and subtracts the correct way round.
⎛ 3⎞ ⎛ 3 ⎞ A1
= 2 + 2ln ⎜ ⎟ 2 + 2ln ⎜ ⎟
⎝5⎠ ⎝ 5 ⎠ cao cso
[8]
8
Notes for Question 3

1
M1: Also allow du = ± λ dx or (u − 2)du = ± λ dx
(u − 2)
Note: The expressions must contain du and dx . They can be simplified or un-simplified.
1
A1: Also allow du = dx or (u − 2)du = ± λ dx
(u − 2)
Note: The expressions must contain du and dx . They can be simplified or un-simplified.


1
A1: (u − 2) du . (Ignore integral sign and du ).
u
dM1: An attempt to divide each term by u.
Note that this mark is dependent on the previous M1 mark being awarded.
Note that this mark can be implied by later working.
ddM1: ± Au ± B ln u , A ≠ 0, B ≠ 0
Note that this mark is dependent on the two previous M1 marks being awarded.
A1ft: u − 2ln u or ± Au ± B ln u being correctly followed through, A ≠ 0, B ≠ 0
M1: Applies limits of 5 and 3 in u or 4 and 0 in x in their integrated function and subtracts the correct
way round.
⎛3⎞ ⎛ 3⎞
A1: cso and cao. 2 + 2ln ⎜ ⎟ or 2 + 2ln ( 0.6 ) , ⎜ = A + 2ln B , so A = 2, B = ⎟
⎝5⎠ ⎝ 5⎠
⎛3⎞
Note: 2 − 2ln ⎜ ⎟ is A0.
⎝5⎠
PhysicsAndMathsTutor.com

Notes for Question 3 Continued


3. ctd 1
Note: (u − 2) du = u − 2ln u with no working is 2nd M1, 3rd M1, 3rd A1.
u


1
but Note: (u − 2) du = (u − 2) ln u with no working is 2nd M0, 3rd M0, 3rd A0.
u
PhysicsAndMathsTutor.com

Question
Scheme Marks
Number

{ }
1 1
4. (a) 3 (8 − 9 x) = (8 − 9 x ) 3 Power of M1
3
1 1
⎛ 1
9x ⎞3 ⎛ 9x ⎞3 1
= (8) ⎜1 −3
⎟ = 2 ⎜1 − ⎟ ( 8) 3 or 2 B1
⎝ 8 ⎠ ⎝ 8 ⎠
⎡ ⎛1⎞ ( 1 )(− 32 ) ( 1 )(− 32 )(− 53 ) ⎤
= {2} ⎢ 1 + ⎜ ⎟ (k x) + 3 (k x) 2 + 3 (k x)3 + ... ⎥ see notes M1 A1
⎣ ⎝ 3⎠ 2! 3! ⎦
⎡ ⎛ 1 ⎞⎛ − 9 x ⎞ ( 3 )(− 3 ) ⎛ − 9 x ⎞
1 2 2
( 3 )(− 3 )(− 3 ) ⎛ − 9 x ⎞
1 2 5 3

= {2} ⎢ 1 + ⎜ ⎟⎜ ⎟ + ⎜ ⎟ + ⎜ ⎟ + ... ⎥
⎣⎢ ⎝ 3 ⎠⎝ 8 ⎠ 2! ⎝ 8 ⎠ 3! ⎝ 8 ⎠ ⎦⎥
⎡ 3 9 2 45 3 ⎤
= 2 ⎢ 1 − x; − x − x + ... ⎥ See notes below!
⎣ 8 64 512 ⎦
3 9 2 45 3
= 2 − x; − x − x + ... A1; A1
4 32 256
[6]
(b) { 3
7100 = 10 71 =10 3 (8 − 9 x) , so x = 0.1
3
} Writes down
or uses x = 0.1 B1
3 9 45
When x = 0.1, 3 (8 − 9 x) ≈ 2 − (0.1) − (0.1) 2 − (0.1)3 + ... M1
4 32 256
= 2 − 0.075 − 0.0028125 − 0.00017578125
= 1.922011719
So, 3
7100 = 19.220117919... = 19.2201 (4 dp) 19.2201 cso A1 cao
[3]
9
Notes for Question 4
1
1
(a) M1: Writes or uses . This mark can be implied by a constant term of 8 3 or 2.
3
1
B1: ( 8) 3 or 2 outside brackets or 2 as candidate’s constant term in their binomial expansion.
1
M1: Expands (... + kx ) 3 to give any 2 terms out of 4 terms simplified or un-simplified,
⎛1⎞ ( 1 )(− 23 ) ( 1 )(− 32 )(− 53 ) ( 1 )( − 23 )
Eg: 1 + ⎜ ⎟ (k x) or 3 (k x) 2 + 3 ( k x)3 or 1 + ...... + 3 (k x) 2
⎝ ⎠
3 2! 3! 2!
( 1 )(− 23 ) ( 1 )(− 32 )(− 53 )
or 3 (k x) 2 + 3 ( k x)3 where k ≠ 1 are fine for M1.
2! 3!
⎛1⎞ ( 1 )(− 23 ) ( 1 )(− 32 )(− 53 )
A1: A correct simplified or un-simplified 1 + ⎜ ⎟ (k x) + 3 ( k x) 2 + 3 ( k x )3
⎝ ⎠ 3 2! 3!
expansion with consistent ( k x ) . Note that ( k x ) must be consistent (on the RHS, not necessarily the LHS)
in a candidate’s expansion. Note that k ≠ 1 .
⎡ ⎛ 1 ⎞⎛ − 9 x ⎞ ( 13 )(− 23 ) ( 13 )(− 23 )(− 53 ) ⎛ − 9 x ⎞
3

You would award B1M1A0 for 2 ⎢ 1 + ⎜ ⎟⎜ + ( )
− + ⎟ + ...
2
⎟ 9 x ⎜ ⎥
⎢⎣ ⎝ 3 ⎠⎝ 8 ⎠ 2! 3! ⎝ 8 ⎠ ⎥⎦
because ( k x ) is not consistent.
PhysicsAndMathsTutor.com

Notes for Question 4 Continued


4. (a) ctd ⎡ ⎛ 1 ⎞⎛ − 9 x ⎞ ( 13 )(− 23 ) ⎛ − 9 x
2
⎞ ( 13 )(− 23 )(− 53 ) ⎛ − 9 x3 ⎞ ⎤
“Incorrect bracketing” = {2} ⎢ 1 + ⎜ ⎟⎜ ⎟ + ⎜ ⎟ + ⎜ ⎟ + ... ⎥
⎣⎢ ⎝ 3 ⎠⎝ 8 ⎠ 2! ⎝ 8 ⎠ 3! ⎝ 8 ⎠ ⎥⎦
is M1A0 unless recovered.
3
A1: For 2 − x (simplified please) or also allow 2 − 0.75 x.
4
⎡ 3 ⎤ ⎡ 3 9 2 45 3 ⎤
Allow Special Case A1A0 for either SC: = 2 ⎢ 1 − x ; ... ⎥ or SC: K ⎢ 1 − x − x − x + ... ⎥
⎣ 8 ⎦ ⎣ 8 64 512 ⎦
(where K can be 1 or omitted), with each term in the [.........] either a simplified fraction or a decimal.
9 2 45 3
A1: Accept only − x − x or − 0.28125 x 2 − 0.17578125 x 3
32 256
⎡ ⎛ 1 ⎞⎛ 9 x ⎞ ( 13 )(− 23 ) ⎛ 9 x ⎞
2
( 13 )(− 23 )(− 53 ) ⎛ 9 x ⎞
3
⎤ 9
Candidates who write = 2 ⎢ 1 + ⎜ ⎟⎜ ⎟ + ⎜ ⎟ + ⎜ ⎟ + ... ⎥ where k =
⎣⎢ ⎝ 3 ⎠⎝ 8 ⎠ 2! ⎝ 8 ⎠ 3! ⎝ 8 ⎠ ⎦⎥ 8
9 3 9 2 45 3
and not − and achieve 2 + x ; − x + x + ... will get B1M1A1A0A0.
8 4 32 256
Note for final two marks:
⎡ 3 9 2 45 3 ⎤ 3 9 2 45 3
2 ⎢ 1 − x; − x − x + ... ⎥ = 2 + x − x − x + ... scores final A0A1.
⎣ 8 64 512 ⎦ 4 32 256
⎡ 3 9 2 45 3 ⎤ 3 9 2 45 3
2 ⎢ 1 − x; − x − x + ... ⎥ = 2 − − x − x + ... scores final A0A1
⎣ 8 64 512 ⎦ 4 32 256

Alternative method: Candidates can apply an alternative form of the binomial expansion.
( 1 )(− 23 ) − 53 ( 1 )(− 23 )(− 53 ) − 83
{ } ⎛1⎞ −
1 1 2
3 (8 − 9 x ) = ( 8 − 9 x ) 3 = (8) 3 + ⎜ ⎟ (8) 3 (− 9 x) + 3 (8) (− 9 x) 2 + 3 (8) (− 9 x)3
⎝3⎠ 2! 3!
1
B1: ( 8 ) 3 or 2
M1: Any two of four (un-simplified or simplified) terms correct.
A1: All four (un-simplified or simplified) terms correct.
3
A1: 2 − x
4
9 2 45 3
A1: − x − x
32 256
Note: The terms in C need to be evaluated,
1 1 1 2 1 5 1 8
− − −
so 3C0 (8) 3 + 3C1 (8) 3 (− 9 x) + 3 C2 (8) 3 (− 9 x) 2 + 3C3 (8) 3 (− 9 x)3 without further working is B0M0A0.

(b) B1: Writes down or uses x = 0.1


8
M1: Substitutes their x, where x < into at least two terms of their binomial expansion.
9
A1: 19.2201 cao

Be Careful! The binomial answer is 19.22011719


and the calculated 3 7100 is 19.21997343... which is 19.2200 to 4 decimal places.
PhysicsAndMathsTutor.com

Question
Scheme Marks
Number
5. (a) 6.248046798... = 6.248 (3dp) 6.248 or awrt 6.248 B1
[1]
1
(b) Area ≈ × 2 ; ×⎡⎣3 + 2 ( 7.107 + 7.218 + their 6.248 ) + 5.223⎤⎦ B1; M1
2
= 49.369 = 49.37 (2 dp) 49.37 or awrt 49.37 A1
[3]
1 1


− t − t

∫ ∫ {dt} ,
⎧ 1
− t ⎫ 1
− t
1
− t ± At e ±B e A ≠ 0, B ≠ 0
{dt} M1
3 3
⎨ (4t e 3
+ 3) d t ⎬ = − 12t e 3
− −12e 3
(c) ⎩ ⎭ See notes. A1
+ 3t 3 → 3t B1
1 1 1 1
− t − t − t − t
= −12t e 3
− 36e 3
{+ 3t} −12t e 3
− 36e 3 A1
8
⎡ 1
− t
1
− t ⎤
⎢ −12t e − 36e + 3t ⎥ =
3 3

⎣ ⎦0
Substitutes limits of 8 and
0 into an integrated
function of the form of
⎛ 1
− (8)
1
− (8) ⎞ ⎛ 1
− (0)
1
− (0) ⎞ 1 1
= ⎜ −12(8) e 3 − 36e 3 + 3(8) ⎟ − ⎜ −12(0) e 3 − 36e 3 + 3(0) ⎟ either ± λ te
− t
3
±µe
− t
3
or dM1
⎝ ⎠ ⎝ ⎠ 1 1
− t − t
± λ t e ± µ e + Bt and
3 3

subtracts the correct way


round.
⎛ −
8

8

= ⎜ − 96e 3 − 36e 3 + 24 ⎟ − ( 0 − 36 + 0 )
⎝ ⎠
8 8
− −
= 60 − 132e 3
60 − 132e 3 A1
[6]
8

(d) Difference = 60 − 132e 3
− 49.37 = 1.458184439... = 1.46 (2 dp) 1.46 or awrt 1.46 B1

[1]
11
Notes for Question 5
(a) B1: 6.248 or awrt 6.248. Look for this on the table or in the candidate’s working.
1
(b) B1: Outside brackets × 2 or 1
2
M1: For structure of trapezium rule [ ............. ] . Allow one miscopy of their values.
A1: 49.37 or anything that rounds to 49.37
Note: It can be possible to award : (a) B0 (b) B1M1A1 (awrt 49.37)
Note: Working must be seen to demonstrate the use of the trapezium rule. Note: actual area is 50.828…
Bracketing mistake: Unless the final answer implies that the calculation has been done correctly,
Award B1M0A0 for 1 + 3 + 2 ( 7.107 + 7.218 + their 6.248 ) + 5.223 (nb: answer of 50.369).
PhysicsAndMathsTutor.com

Notes for Question 5 Continued


5. (b) ctd Alternative method for part (b): Adding individual trapezia
⎡ 3 + 7.107 7.107 + 7.218 7.218 + 6.248 6.248 + 5.223 ⎤
Area ≈ 2 × ⎢ + + + ⎥ = 49.369
⎣ 2 2 2 2 ⎦
B1: 2 and a divisor of 2 on all terms inside brackets.
M1: First and last ordinates once and two of the middle ordinates twice inside brackets ignoring the 2.
A1: anything that rounds to 49.37
1 1 1


− t − t − t
(c) M1: For 4t e 3
→ ± At e 3
±B e 3
{dt} , A ≠ 0, B ≠ 0
1
⎛ 1
− t ⎞
1


− t − t
A1: For t e → ⎜ − 3t e 3 − −3e 3 ⎟ (some candidates lose the 4 and this is fine for the first A1 mark).
3

⎝ ⎠
1
⎛ 1
− t ⎞
1 1 1
⎛ 1
− t ⎞
1

∫ ∫ ∫
− t − t − t − t − t
or 4t e 3 → 4 ⎜ − 3t e 3 − − 3e 3 ⎟ or −12t e 3 − −12e 3 or 12 ⎜ − t e 3 − − e 3 ⎟
⎝ ⎠ ⎝ ⎠
These results can be implied. They can be simplified or un-simplified.
B1: 3 → 3t or 3 → 3 x (bod) .
Note: Award B0 for 3 integrating to 12t (implied), which is a common error when taking out a factor of 4.
⎛ 3⎞ ⎛ 3 ⎞
Be careful some candidates will factorise out 4 and have 4 ⎜ .... + ⎟ → 4 ⎜ .... + t ⎟
⎝ 4⎠ ⎝ 4 ⎠
which would then be fine for B1.
8
Note: Allow B1 for ∫
0
3dt = 24
1
− t
1
− t
1
− t ⎛ 1
− t − t ⎞
1
A1: For correct integration of 4t e 3
to give −12t e 3
− 36e 3
or 4 ⎜ − 3t e 3 − 9e 3 ⎟ or equivalent.
⎝ ⎠
This can be simplified or un-simplified.
1 1
− t − t
dM1: Substitutes limits of 8 and 0 into an integrated function of the form of either ± λ te 3
± µe 3
or
1 1
− t − t
± λ te ± µ e + Bt and subtracts the correct way round.
3 3

Note: Evidence of a proper consideration of the limit of 0 (as detailed in the scheme) is needed for dM1.
So, just subtracting zero is M0.
8

A1: An exact answer of 60 − 132e 3 . A decimal answer of 50.82818444... without a correct answer is A0.
Note: A decimal answer of 50.82818444... without a correct exact answer is A0.
Note: If a candidate gains M1A1B1A1 and then writes down 50.8 or awrt 50.8 with no method for
substituting limits of 8 and 0, then award the final M1A0.
IMPORTANT: that is fine for candidates to work in terms of x rather than t in part (c).
Note: The "3t " is needed for B1 and the final A1 mark.
(d) B1: 1.46 or awrt 1.46 or -1.46 or awrt -1.46.
Candidates may give correct decimal answers of 1.458184439... or 1.459184439...
Note: You can award this mark whether or not the candidate has answered part (c) correctly.
PhysicsAndMathsTutor.com

Question
Scheme Marks
Number
⎛ a⎞ ⎛ 6⎞ ⎛ 21 ⎞ ⎛ 25 ⎞
⎜ ⎟ ⎜ ⎟ JJJG ⎜ ⎟ JJJG ⎜ ⎟
6. l : r = ⎜ b ⎟ + λ ⎜ c ⎟ , OA = ⎜ −17 ⎟ , OB = ⎜ −14 ⎟
⎜ 10 ⎟ ⎜ −1 ⎟ ⎜ 6⎟ ⎜ 18 ⎟
⎝ ⎠ ⎝ ⎠ ⎝ ⎠ ⎝ ⎠
⎛ 21 ⎞ ⎛ a ⎞ ⎛ 6⎞
⎜ ⎟ ⎜ ⎟ ⎜ ⎟
(a) A is on l, so ⎜ −17 ⎟ = ⎜ b ⎟ + λ ⎜ c ⎟
⎜ 6 ⎟ ⎜10 ⎟ ⎜ −1 ⎟
⎝ ⎠ ⎝ ⎠ ⎝ ⎠

{k : 10 − λ = 6 ⇒} λ = 4 λ=4 B1
Substitutes their value of
{i : a + 6λ = 21 ⇒} a + 6(4) = 21
λ into a + 6λ = 21 M1
a = −3 a = − 3 A1 cao
[3]
⎛ 25 ⎞ ⎛ 21 ⎞ ⎛ 21 ⎞ ⎛ 25 ⎞ Finds the difference
JJJG JJJG JJJG JJJG
(b) { } AB
⎜ ⎟ ⎜ ⎟
= ⎜ −14 ⎟ − ⎜ −17 ⎟ { }
BA
⎜ ⎟ ⎜ ⎟
= ⎜ −17 ⎟ − ⎜ −14 ⎟ between OA and OB . M1
⎜ 18 ⎟ ⎜ 6 ⎟ ⎜ 6 ⎟ ⎜ 18 ⎟
⎝ ⎠ ⎝ ⎠ ⎝ ⎠ ⎝ ⎠ Ignore labelling.
⎛ 4⎞ ⎛− 4⎞
JJJG JJJG
{ AB } ⎜ ⎟
= ⎜ 3⎟ {BA } ⎜
= ⎜ −3⎟

⎜ 12 ⎟ ⎜ −12 ⎟
⎝ ⎠ ⎝ ⎠
⎛ 4 ⎞ ⎛ 6⎞
JJJG JJJG
{ }
⎜ ⎟ ⎜ ⎟
AB ⊥ l ⇒ AB • d = 0 ⇒ ⎜ 3 ⎟ • ⎜ c ⎟ = 24 + 3c − 12 = 0; ⇒ c = − 4 See notes. M1; A1 ft
⎜12 ⎟ ⎜ −1⎟
⎝ ⎠ ⎝ ⎠
{j : b + cλ = − 17 ⇒} b + (− 4)(4) = − 17 ; ⇒ b = −1 See notes.
ddM1;
A1 cso cao
[5]
(c) AB = 42 + 32 + 122 or AB = (− 4) 2 + (−3) 2 + (−12) 2 See notes. M1
So, AB = 13 A1 cao
[2]
⎛ 21 ⎞ ⎛ − 4 ⎞ ⎛ 17 ⎞
JJJJG JJJG JJJG See notes for
(d) { ⎜ ⎟
}⎜ ⎟ ⎜ ⎟
OB′ = OA + BA = ⎜ −17 ⎟ + ⎜ − 3 ⎟ ; = ⎜ − 20 ⎟ alternative M1;A1 cao
⎜ 6 ⎟ ⎜ −12 ⎟ ⎜ −6 ⎟ methods.
⎝ ⎠ ⎝ ⎠ ⎝ ⎠
[2]
12
Notes for Question 6
(a) B1: λ = 4 seen or implied.
M1: Substitutes their value of λ into a + 6λ = 21
A1: a = − 3 .
Note: Award B1M1A1 if the candidate states a = − 3 from no working.
Alternative Method Using Simultaneous equations for part (a).
B1: For 60 − 6λ = 36
M1: 60 − 6λ = 36 and a + 6λ = 21 solved simultaneously to give a = ...
A1: a = − 3 , cao.
PhysicsAndMathsTutor.com

Notes for Question 6 Continued


6. (b)
ctd
JJJG JJJG
M1: Finds the difference between OA and OB . Ignore labelling.
If no “subtraction” seen, you can award M1 for 2 out of 3 correct components of the difference.
⎛ 6⎞ ⎛ 6⎞
JJJG ⎜ ⎟ JJJG ⎜ ⎟
M1: Applies the formula AB • ⎜ c ⎟ or BA • ⎜ c ⎟ correctly to give a linear equation in c which is set equal
⎜ −1 ⎟ ⎜ −1 ⎟
⎝ ⎠ ⎝ ⎠
⎛ 6⎞
⎜ ⎟
to zero. Note: The dot product can also be with ± k ⎜ c ⎟ .
⎜ −1 ⎟
⎝ ⎠
A1ft: c = − 4 or for finding a correct follow through c.
ddM1: Substitutes their value of λ and their value of c into b + cλ = − 17
Note that this mark is dependent on the two previous method marks being awarded.
A1: b = −1
M1: An attempt to apply a three term Pythagoras in order to find AB ,
(c)
so taking the square root is required here.
A1: 13 cao
Note: Don’t recover work for part (b) in part (c).
(d) M1: For a full applied method of finding the coordinates of B′.
Note: You can give M1 for 2 out of 3 correct components of B′.
⎛ 17 ⎞
⎜ ⎟
A1: For either ⎜ − 20 ⎟ or 17 i − 20 j − 6k or (17, − 20, − 6 ) cao.
⎜ −6 ⎟
⎝ ⎠
Helpful diagram!
⎛ 25 ⎞
⎜ ⎟
B ⎜ −14 ⎟
⎜ 18 ⎟
⎝ ⎠

⎛− 4⎞
JJJG ⎜ ⎟
⎛ 21 ⎞ BA = ⎜ − 3 ⎟
⎜ ⎟ ⎜ −12 ⎟
A ⎜ −17 ⎟ ⎝ ⎠
⎜ 6⎟
⎝ ⎠

l
⎛− 4⎞
JJJG ⎜ ⎟
BA = ⎜ − 3 ⎟
⎜ −12 ⎟
⎛ p⎞ ⎝ ⎠
⎜ ⎟
B′ ⎜ q ⎟
⎜r⎟
⎝ ⎠
PhysicsAndMathsTutor.com

Notes for Question 6 Continued


Acceptable Methods for the Method mark in part (d)
⎛ 21 ⎞ ⎛ − 4 ⎞
JJJJG JJJG JJJG JJJG
Way 1 { ⎜
}⎟ ⎜ ⎟
OB′ = OA + BA = ⎜ −17 ⎟ + ⎜ − 3 ⎟ (using their BA )
⎜ 6 ⎟ ⎜ −12 ⎟
⎝ ⎠ ⎝ ⎠
⎛ 21 ⎞ ⎛ 4 ⎞
JJJJG JJJG JJJG JJJG
Way 2 { ⎜
}
⎟ ⎜ ⎟
OB′ = OA − AB = ⎜ −17 ⎟ − ⎜ 3 ⎟ (using their AB )
⎜ 6 ⎟ ⎜12 ⎟
⎝ ⎠ ⎝ ⎠
⎛ 25 ⎞ ⎛− 4⎞
JJJJG JJJG JJJG JJJG
Way 3 { ⎜ ⎟
} ⎜ ⎟
OB′ = OB + 2 BA = ⎜ −14 ⎟ + 2 ⎜ − 3 ⎟ (using their BA )
⎜ 18 ⎟ ⎜ −12 ⎟
⎝ ⎠ ⎝ ⎠
⎛ 25 ⎞ ⎛ 4⎞
JJJJG JJJG JJJG JJJG
Way 4 { ⎜ ⎟
} ⎜ ⎟
OB′ = OB − 2 AB = ⎜ −14 ⎟ − 2 ⎜ 3 ⎟ (using their AB )
⎜ 18 ⎟ ⎜12 ⎟
⎝ ⎠ ⎝ ⎠
⎛ 25 ⎞ ⎛ Minus 4 ⎞ ⎛ 21 ⎞ ⎛ Minus 4 ⎞ ⎧ ⎛ 17 ⎞ ⎫
⎜ ⎟ ⎜ ⎟ ⎜ ⎟ ⎜ ⎟ ⎪ ⎜ ⎟⎪ JJJG JJJG
Way 5 ⎜ −14 ⎟ → ⎜ Minus 3 ⎟ → ⎜ −17 ⎟ → ⎜ Minus 3 ⎟ ⎨→ ⎜ − 20 ⎟ ⎬ , so OA + their BA
⎜ 18 ⎟ ⎜ Minus 12 ⎟ ⎜ 6 ⎟ ⎜ Minus 12 ⎟ ⎪ ⎜ − 6 ⎟⎪
⎝ ⎠ ⎝ ⎠ ⎝ ⎠ ⎝ ⎠ ⎩ ⎝ ⎠⎭
⎛ 21 ⎞ ⎛ 25 ⎞
JJJJG JJJG JJJG
Way 6 { ⎜
}
OB′ = 2OA − OB = 2 ⎜ −17 ⎟ −
⎟ ⎜ ⎟
⎜ −14 ⎟
⎜ 6⎟ ⎜ 18 ⎟
⎝ ⎠ ⎝ ⎠
JJJG JJJG JJJJG
Way 7 OB = 25 i − 14 j + 18k , OA = 21i − 17 j + 6k and OB′ = p i + qj + rk ,
25 + p −14 + q 18 + r ⎞
( 21, − 17, 6 ) = ⎛⎜ , , ⎟
⎝ 2 2 2 ⎠

p = 21(2) − 25 = 17 M1: Writing down any two equations correctly and


q = − 17(2) + 14 = − 20 an attempt to find at least two of p, q or r.
r = 6(2) − 18 = − 6
PhysicsAndMathsTutor.com

Question
Scheme Marks
Number
π
7. x = 27sec3 t , y = 3tan t , 0-t -
3
dx dy
At least one of or correct. B1
dx dy dt dt
(a) = 81sec 2 t sec t tan t , = 3sec 2 t
dt dt dx dy
Both and are correct. B1
dt dt
dy 3sec 2 t ⎧ 1 cos t cos 2 t ⎫ dy dx
= ⎨ = = = ⎬ Applies their divided by their M1;
dx 81sec3 t tan t ⎩ 27sec t tan t 27 tan t 27sin t ⎭ dt dt
π dy 3sec 2 ( π6 ) 4 ⎧ 3 1⎫ 4
At t = , = = ⎨= = ⎬ A1 cao cso
6 dx 81sec ( 6 ) tan ( 6 ) 72 ⎩ 54 18 ⎭
3 π π 72
[4]
2 2
⎛ y⎞ ⎛ ⎛ x ⎞⎞
2
⎛ x ⎞
{1 + tan t = sec 2 t } ⇒ 1 + ⎜ ⎟ = ⎜ 3 ⎜ ⎟ ⎟ = ⎜ ⎟
3
2
(b) M1
⎝ 3 ⎠ ⎜⎝ ⎝ 27 ⎠ ⎟⎠ ⎝ 27 ⎠

( )
2 1
y2 x 3 2 2
⇒1+ = ⇒ 9 + y2 = x 3 ⇒ y = x 3 − 9 2
* A1 * cso
9 9
a = 27 and b = 216 or 27 - x - 216 a = 27 and b = 216 B1
[3]
2
⎛ 2 ⎞
∫ ( ) ∫( )
1
2
⎛ 2 ⎞
∫ ( ) ∫ (x ) For π ⎜ x 3 − 9 ⎟ or π x 3 − 9
125 1 125 2
2

(c) V =π ⎜ x3 − 9 2
⎟ dx or π
2
3
− 9 dx ⎜ ⎟ B1
⎜ ⎟ ⎝ ⎠
27 ⎝ ⎠ 27
Ignore limits and dx . Can be implied.
5
3 5
125 Either ± Ax 3 ± Bx or x 3 oe M1
⎡3 5 ⎤
= {π } ⎢ x 3 − 9 x ⎥
5
5
⎣5 ⎦ 27 3 3
x − 9 x oe A1
5
⎛⎛ 3 5
⎞ ⎛3 5
⎞⎞ Substitutes limits of 125 and 27 into an
= {π } ⎜ ⎜ (125) 3 − 9(125) ⎟ − ⎜ (27) 3 − 9(27) ⎟ ⎟ integrated function and subtracts the dM1
⎜ 5 ⎟
⎝⎝ ⎠ ⎝5 ⎠⎠ correct way round.
= {π } ( (1875 − 1125 ) − (145.8 − 243) )
4236 π 4236 π
= or 847.2π or 847.2π A1
5 5
[5]
12
Notes for Question 7
dx dy
(a) B1: At least one of or correct. Note: that this mark can be implied from their working.
dt dt
dx dy
B1: Both and are correct. Note: that this mark can be implied from their working.
dt dt
dy dx dy dx
M1: Applies their divided by their , where both and are trigonometric functions of t.
dt dt dt dt
4
A1: or any equivalent correct rational answer not involving surds.
72

Allow 0.05 with the recurring symbol.
PhysicsAndMathsTutor.com

Notes for Question 7 Continued


dx
Note: Please check that their is differentiated correctly.
dt
dx
Eg. Note that x = 27sec3 t = 27 ( cos t ) ⇒ = − 81( cos t ) (− sin t ) is correct.
−3 −2

dt
(b) M1: Either:
• Applying a correct trigonometric identity (usually 1 + tan 2 t = sec 2 t ) to give a Cartesian equation in
x and y only.
• Starting from the RHS and goes on to achieve 9 tan 2 t by using a correct trigonometric identity.
• Starts from the LHS and goes on to achieve 9sec2 t − 9 by using a correct trigonometric identity.

( )
1
2
A1*: For a correct proof of y = x 3 − 9 .
2

Note this result is printed on the Question Paper, so no incorrect working is allowed.
B1: Both a = 27 and b = 216 . Note that 27 - x - 216 is also fine for B1.
(c)
2
⎛ 2 ⎞
∫ ( ) ∫( x )
1

B1: For a correct statement of π ⎜ x 3 − 9 or π


2
2
⎟ 3
− 9 . Ignore limits and dx . Can be implied.
⎜ ⎟
⎝ ⎠
5 2
3 53
M1: Either integrates to give ± Ax 3 ± Bx , A ≠ 0, B ≠ 0 or integrates x 3 correctly to give x oe
5
5
3 5 x3
A1: x 3 − 9 x or. − 9 x oe.
5 ⎛5⎞
⎜ ⎟
⎝3⎠
dM1: Substitutes limits of 125 and 27 into an integrated function and subtracts the correct way round.
Note: that this mark is dependent upon the previous method mark being awarded.
4236 π
A1: A correct exact answer of or 847.2π .
5
Note: The π in the volume formula is only required for the B1 mark and the final A1 mark.
Note: A decimal answer of 2661.557... without a correct exact answer is A0.
Note: If a candidate gains the first B1M1A1 and then writes down 2661 or awrt 2662 with no method for
substituting limits of 125 and 27, then award the final M1A0.
(a) Alternative response using the Cartesian equation in part (a)

( )
1 1
dy − −
2 M1
⎧ 1
⎫ −
1 = ± K x 3
x 3
− 9 2

⎪ ⎛ 2
⎞ 2
⎪ d y 1 ⎛ 2
⎞ 2 ⎛
2 −
1
⎞ d x
Way 2 ⎨ y = ⎜ x 3 − 9 ⎟ ⇒⎬ = ⎜ x3 − 9⎟ ⎜ x 3 ⎟
− ⎛2 −1 ⎞
( )
1
⎪ ⎝ ⎠ ⎪ d x 2⎝ ⎠ ⎝3 ⎠ dy
=
1 23
− 2 A1
⎩ ⎭ x 9 ⎜ x 3 ⎟ oe
dx 2 ⎝ 3 ⎠
π ⎛π ⎞
At t = , x = 27sec3 ⎜ ⎟ = 24 3 π
6 ⎝6⎠ Uses t = to find x and substitutes
6

1 dM1
⎛ ⎞ 2 ⎛ ⎞
( ) ( )
dy 1
2
2 −
1 dy
⇒ = ⎜ 24 3 3 − 9 ⎟ ⎜ 24 3 3 ⎟ their x into an expression for .
dx 2 ⎜⎝ ⎟ ⎜3
⎠ ⎝


dx

dy 1 ⎛ 1 ⎞⎛ 1 ⎞ 1 1
So, ⇒ = ⎜ ⎟ ⎜⎜ ⎟⎟ = A1 cao cso
dx 2 ⎝ 3 ⎠ ⎝ 3 3 ⎠ 18 18
Note: Way 2 is marked as M1 A1 dM1 A1
Note: For way 2 the second M1 mark is dependent on the first M1 being gained.
PhysicsAndMathsTutor.com

Notes for Question 7 Continued


7. (b) Alternative responses for M1A1 in part (b): STARTING FROM THE RHS
For applying 1 + tan 2 t = sec 2 t oe
( )
1

{RHS =} x − 9 = ( 27sec t ) − 9 = 9sec t − 9 = 9 tan t


2 2
Way 2 3 2 3 3 2 2 M1
to achieve 9 tan 2 t

( )
1
= 3tan t = y {= LHS} cso Correct proof from x − 9
2
3 2
to y. A1*

M1: Starts from the RHS and goes on to achieve 9 tan 2 t by using a correct trigonometric identity.
7. (b) Alternative responses for M1A1 in part (b): STARTING FROM THE LHS
Way 3 For applying 1 + tan 2 t = sec 2 t oe
{LHS =} y = 3tan t = ( 9 tan 2 t ) = 9sec2 t − 9 M1
to achieve 9sec2 t − 9
2
⎛ x3 ⎞
( )
2 1
⎛ x ⎞3
( )
1
2
= 9⎜ ⎟ − 9 = 9⎜ ⎟⎟ − 9 = x 3 − 9
2
Correct proof from y to x − 9 . A1*
2 2
cso 3

⎝ 27 ⎠ ⎜ 9
⎝ ⎠
M1: Starts from the LHS and goes on to achieve 9sec 2 t − 9 by using a correct trigonometric identity.
7. (c) Alternative response for part (c) using parametric integration


Way 2
π 3tan t ( 81sec 2 t sec t tan t ) dt

V = π 9 tan 2 t ( 81sec 2 t sec t tan t ) dt
Ignore limits and dx . Can be implied.
B1


= {π } 729sec 2 t tan 2 t sec t tan t dt


= {π } 729sec t ( sec t − 1) sec t tan t dt
2 2


= {π } 729 ( sec − sec ) sec tan d
4
t 2
t t t t


= {π } 729 ( sec t − sec t ) sec t tan t dt
4 2

± A sec5 t ± B sec3 t M1
⎡ ⎛1 1 ⎞⎤
= {π } ⎢ 729 ⎜ sec5 t − sec3 t ⎟ ⎥ ⎛1 1 ⎞
⎣ ⎝ 5 3 ⎠⎦ 729 ⎜ sec5 t − sec3 t ⎟ A1
⎝5 3 ⎠
5
⎡ ⎛ 1 ⎛ 5 ⎞5 1 ⎛ 5 ⎞3 ⎞ 1 ⎞⎤ Substitutes sec t = and sec t = 1 into an
⎛1
V = {π } ⎢729 ⎜ ⎜ ⎟ − ⎜ ⎟ ⎟ − 729 ⎜ 15 − 13 ⎟ ⎥ 3 dM1
⎢⎣ ⎜ 5⎝ 3⎠ 3 ⎝ 3 ⎠ ⎟⎠ ⎝5 3 ⎠ ⎥⎦ integrated function and subtracts the correct

way round.
⎡⎛ 250 ⎞ ⎛ 2 ⎞ ⎤
= 729π ⎢⎜ ⎟ − ⎜ − ⎟⎥
⎣⎝ 243 ⎠ ⎝ 15 ⎠ ⎦
4236 π 4236 π
= or 847.2π or 847.2π A1
5 5
[5]
PhysicsAndMathsTutor.com

Question
Scheme Marks
Number
dx
8. = k ( M − x) , where M is a constant
dt
(a) dx
is the rate of increase of the mass of waste products. Any one correct explanation. B1
dt

M is the total mass of unburned fuel and waste fuel Both explanations are correct. B1
(or the initial mass of unburned fuel)
[2]

∫ ∫ ∫ ∫
1 1
(b) dx = k dt or dx = dt B1
M −x k ( M − x)
1
− ln ( M − x ) = kt {+ c} or − ln ( M − x ) = t {+ c} See notes M1 A1
k
{t = 0 , x = 0 ⇒} − ln ( M − 0 ) = k (0) + c See notes M1
c = − ln M ⇒ − ln ( M − x ) = kt − ln M
then either... or...
− kt = ln ( M − x ) − ln M kt = ln M − ln ( M − x )
⎛M − x⎞ ⎛ M ⎞
− kt = ln ⎜ ⎟ kt = ln ⎜ ⎟
⎝ M ⎠ ⎝M − x⎠
M −x ekt =
M
e− kt = ddM1
M M −x
− kt
(M − x ) ekt = M
Me =M −x
M − x = Me − kt
A1 * cso
leading to x = M − Me − kt or x = M (1 − e − kt ) oe
[6]
⎧ 1 ⎫ 1
(c) ⎨ x = M , t = ln 4 ⇒ ⎬ M = M (1 − e− k ln 4 ) M1
⎩ 2 ⎭ 2
1 1
⇒ = 1 − e − k ln 4 ⇒ e − k ln 4 = ⇒ − k ln 4 = − ln 2
2 2
1
So k = A1
2
⎛ − ln 9 ⎞
1
x = M ⎜1 − e 2 ⎟ dM1
⎝ ⎠
2 2
x= M x= M A1 cso
3 3
[4]
12
PhysicsAndMathsTutor.com

Notes for Question 8 Continued


8. (a) B1: At least one explanation correct.
B1: Both explanations are correct.
dx
is the rate of increase of the mass of waste products.
dt
or the rate of change of the mass of waste products.

M is the total mass of unburned fuel and waste fuel


or the initial mass of unburned fuel
or the total mass of rocket fuel and waste fuel
or the initial mass of rocket fuel
or the initial mass of fuel
or the total mass of waste and unburned products.
(b)
B1: Separates variables as shown. dx and dt should be in the correct positions, though this mark can be
implied by later working. Ignore the integral signs.
M1: Both ± λ ln ( M − x ) or ± λ ln ( x − M ) and ± µ t where λ and µ are any constants.
1 1
A1: For − ln ( M − x ) = kt or − ln ( x − M ) = kt or − ln ( M − x ) = t or − ln ( x − M ) = t
k k
1 1
or − ln ( kM − kx ) = t or − ln ( kx − kM ) = t
k k
Note: + c is not needed for this mark.
IMPORTANT: + c can be on either side of their equation for the 1st A1 mark.
M1: Substitutes t = 0 AND x = 0 in an integrated or changed equation containing c (or A or ln A , etc.)
Note that this mark can be implied by the correct value of c.
ddM1: Uses their value of c which must be a ln term, and uses fully correct method to eliminate their
logarithms. Note: This mark is dependent on both previous method marks being awarded.
M (e kt − 1)
A1: x = M − Me − kt or x = M (1 − e − kt ) or x = or equivalent where x is the subject.
ekt
Note: Please check their working as incorrect working can lead to a correct answer.
⎧ dx dx 1 ⎫ 1
Note: ⎨ = k ( M − x) ⇒ = ⇒ ⎬ x = − ln(kM − kx) {+ c} is B1(Implied) M1A1.
⎩ dt d t kM − kx ⎭ k
1
(c) M1: Substitutes x = M and t = ln 4 into one of their earlier equations connecting x and t.
2
1 ln 2
A1: k = , which can be an un-simplified equivalent numerical value. i.e. k = is fine for A1.
2 ln 4
dM1: Substitutes t = ln 4 and their evaluated k (which must be a numerical value) into one of their earlier
equations connecting x and t.
Note: that the 2nd Method mark is dependent on the 1st Method mark being awarded in part (c).
2
A1: x = M cso.
3
Note: Please check their working as incorrect working can lead to a correct answer.
PhysicsAndMathsTutor.com

Notes for Question 8 Continued


Aliter
∫ ∫
1
8. (b) dx = k dt B1
Way 2 M −x

− ln ( M − x ) = kt {+ c} See notes M1 A1

ln ( M − x ) = − kt + c
M − x = Ae − kt
{t = 0 , x = 0 ⇒} M − 0 = Ae− k (0) M1
⇒M =A
M − x = Me− kt ddM1
− kt
So, x = M − Me A1
[6]
(b) B1M1A1: Mark as in the original scheme.
M1: Substitutes t = 0 AND x = 0 in an integrated equation containing their constant of integration which
could be c or A. Note that this mark can be implied by the correct value of c or A.
ddM1: Uses a fully correct method to eliminate their logarithms and writes down an equation containing
their evaluated constant of integration.
Note: This mark is dependent on both previous method marks being awarded.
Note: ln ( M − x ) = − kt + c leading to ln ( M − x ) = e − kt + ec or ln ( M − x ) = e − kt + A would be dddM0.
A1: Same as the original scheme.

Aliter
∫ ∫ k dt
x t
1
8. (b) dx = B1
Way 3 0 M −x 0

⎣ ln ( M − x ) ⎤⎦ 0 = [ kt ]0
x
⎡−
t
M1 A1

− ln ( M − x ) − ( − ln M ) = kt Applies limits of M1
− ln ( M − x ) + ln M = kt
and then follows the original scheme.

(a) B1M1A1: Mark as in the original scheme (ignoring the limits).


ddM1: Applies limits 0 and x on their integrated LHS and limits of 0 and t .
M1A1: Same as the original scheme.
PhysicsAndMathsTutor.com

Notes for Question 8 Continued

∫ ∫ ∫ ∫
Aliter 1 ⎧ −1 ⎫
8. (b) dx = k dt ⎨⇒ dx = k dt ⎬ B1
Way 4 M −x ⎩ x−M ⎭

− ln x − M = kt + c Modulus not required


M1 A1
for 1st A1.
{t = 0 , x = 0 ⇒} − ln 0 − M = k (0) + c Modulus
not required here!
M1
⇒ c = − ln M ⇒ − ln x − M = kt − ln M
then either... or...
− kt = ln x − M − ln M kt = ln M − ln x − M
x−M M
− kt = ln kt = ln
M x−M
As x < M
⎛M − x⎞ ⎛ M ⎞
− kt = ln ⎜ ⎟ kt = ln ⎜ ⎟ Understanding of
⎝ M ⎠ ⎝M − x⎠
modulus is required ddM1
M −x e kt =
M here!
e − kt =
M M −x
(M − x ) ekt = M
Me − kt = M − x
M − x = Me − kt
A1 * cso
leading to x = M − Me− kt or x = M (1 − e − kt ) oe
[6]
B1: Mark as in the original scheme.
M1A1M1: Mark as in the original scheme ignoring the modulus.
ddM1: Mark as in the original scheme AND the candidate must demonstrate that they have converted
ln x − M to ln ( M − x ) in their working.
Note: This mark is dependent on both the previous method marks being awarded.
A1: Mark as in the original scheme.
Aliter
Use of an integrating factor (I.F.)
8. (b)
Way 5 dx dx
= k (M − x) ⇒ + kx = kM
dt dt
I.F. = e kt B1

( e x ) = kMekt ,
d kt
dt
ekt x = Me kt + c M1A1
− kt
x = M + ce
{t = 0 , x = 0 ⇒} 0 = M + ce− k (0) M1
⇒ c = −M
x = M − Me − kt ddM1A1
PhysicsAndMathsTutor.com

Mark Scheme (Results)

June 2013

GCE Core Mathematics 4 (6666/01)


PhysicsAndMathsTutor.com

General Principles for Core Mathematics Marking

(But note that specific mark schemes may sometimes override these general principles).

Method mark for solving 3 term quadratic:


1. Factorisation
( x 2 + bx + c) = ( x + p)( x + q), where pq = c , leading to x =
(ax 2 + bx + c) = (mx + p)(nx + q), where pq = c and mn = a , leading to x =

2. Formula
Attempt to use correct formula (with values for a, b and c).

3. Completing the square


2
⎛ b⎞
Solving x 2 + bx + c = 0 : ⎜ x ± ⎟ ± q ± c, q ≠ 0, leading to x =...
⎝ 2⎠

Method marks for differentiation and integration:


1. Differentiation
Power of at least one term decreased by 1. ( x n → x n −1 )

2. Integration
Power of at least one term increased by 1. ( x n → x n +1 )

Use of a formula
Where a method involves using a formula that has been learnt, the advice given in recent
examiners’ reports is that the formula should be quoted first.
Normal marking procedure is as follows:
Method mark for quoting a correct formula and attempting to use it, even if there are mistakes
in the substitution of values.
Where the formula is not quoted, the method mark can be gained by implication from correct
working with values, but may be lost if there is any mistake in the working.

Exact answers
Examiners’ reports have emphasised that where, for example, an exact answer is asked for, or
working with surds is clearly required, marks will normally be lost if the candidate resorts to
using rounded decimals.

Answers without working


The rubric says that these may not gain full credit. Individual mark schemes will give details of
what happens in particular cases. General policy is that if it could be done “in your head”,
detailed working would not be required.
PhysicsAndMathsTutor.com

Question
Scheme Marks
Number
⎧ du ⎫ ⎧ du ⎫
⎪ u = x2 ⇒ = 2x⎪ ⎪⎪ u = x ⇒ =1 ⎪
⎪ ⎪ ⎪

dx dx
1. (a) x 2 e x dx , 1st Application: ⎨ nd
⎬ , 2 Application: ⎨ ⎬
⎪ dv = e x ⇒ v = e x ⎪ ⎪ dv = e x ⇒ v = ex ⎪
⎪⎩ dx ⎪⎭ ⎪⎩ dx ⎭⎪


x 2 e x − λ xe x {dx} , λ > 0 M1

= x 2 e x − 2 xe x dx

x 2 e x − 2 xe x {dx} A1 oe


Either ± Ax 2 e x ± Bxe x ± C e x {dx}
( ∫ )
= x 2 e x − 2 xe x − e x dx
or for ± K xe {dx} → ± K ( xe − e {dx} )
M1
∫ ∫
x x x

± Ax 2 e x ± Bxe x ± C e x M1
= x 2 e x − 2( xe x − e x ) {+ c}
Correct answer, with/without + c A1
[5]

(b)
{⎡⎣ x e
2 x
− 2( xe − e ) ⎤⎦
x x
0
1
} Applies limits of 1 and 0 to an expression of the
form ± Ax 2 e x ± Bxe x ± Ce x , A ≠ 0 , B ≠ 0 and M1
= (12 e1 − 2(1e1 − e1 ) ) − ( 02 e0 − 2(0e0 − e0 ) ) C ≠ 0 and subtracts the correct way round.
=e−2 e − 2 cso A1 oe
[2]
7
Notes for Question 1
(a)


M1: Integration by parts is applied in the form x 2 e x − λ xe x {dx} , where λ > 0 . (must be in this form).

A1:

x 2 e x − 2 xe x {dx} or equivalent.


M1: Either achieving a result in the form ± Ax 2 e x ± Bxe x ± C e x {dx} (can be implied)


(where A ≠ 0 , B ≠ 0 and C ≠ 0) or for ± K xe x {dx} → ± K xe x − e x {dx} ( ∫ )
M1: ± Ax e ± Bxe ± C e (where A ≠ 0 , B ≠ 0 and C ≠ 0)
2 x x x

A1: x 2 e x − 2( x e x − e x ) or x 2 e x − 2 x e x + 2e x or ( x 2 − 2 x + 2)e x or equivalent with/without + c .


(b) M1: Complete method of applying limits of 1 and 0 to their part (a) answer in the form ± Ax 2 e x ± Bxe x ± Ce x ,
(where A ≠ 0 , B ≠ 0 and C ≠ 0 ) and subtracting the correct way round.
Evidence of a proper consideration of the limit of 0 (as detailed above) is needed for M1.
So, just subtracting zero is M0.
A1: e − 2 or e1 − 2 or − 2 + e . Do not allow e − 2e0 unless simplified to give e − 2.
Note: that 0.718... without seeing e − 2 or equivalent is A0.
WARNING: Please note that this A1 mark is for correct solution only.
So incorrect [.......]0 leading to e − 2 is A0.
1

Note: If their part (a) is correct candidates can get M1A1 in part (b) for e − 2 from no working.
Note: 0.718... from no working is M0A0
PhysicsAndMathsTutor.com

Question
Scheme Marks
Number
⎧⎪ ⎛ 1 + ⎫⎪
x⎞ 1

1
1 1
2. (a) ⎨ ⎜ ⎬
⎟ = (1 + x ) 2
(1 − x ) 2
(1 + x) 2 (1 − x)

2 B1
⎪⎩ ⎝ 1 − x⎠
⎪⎭
⎛ ⎛ 1 ⎞ ( 1 )(− 12 ) 2 ⎞ ⎛ ⎛ 1⎞ (− 1 )(− 32 ) ⎞
= ⎜1 + ⎜ ⎟ x + 2 x + ... ⎟ × ⎜ 1 + ⎜ − ⎟ (− x) + 2 (− x) 2 + ... ⎟ See notes M1 A1 A1
⎝ ⎝2⎠ 2! ⎠ ⎝ ⎝ 2⎠ 2! ⎠
⎛ 1 1 ⎞ ⎛ 1 3 ⎞
= ⎜ 1 + x − x 2 + ... ⎟ × ⎜1 + x + x 2 + ... ⎟
⎝ 2 8 ⎠ ⎝ 2 8 ⎠
1 3 1 1 1
= 1 + x + x 2 + x + x 2 − x 2 + ... See notes M1
2 8 2 4 8
1 2 Answer is given in
= 1+ x + x A1 *
2 the question.
[6]
⎛ 1 + ( 261 ) ⎞ ⎛ 1 ⎞ 1⎛ 1 ⎞
2

(b) ⎜⎜ ⎟ = 1+ ⎜ ⎟ + ⎜ ⎟ M1
⎝ 1 − ( 26 ) ⎠
1 ⎟
⎝ 26 ⎠ 2 ⎝ 26 ⎠
3 3 1405
ie: = B1
5 1352
7025 7025
so, 3= A1 cao
4056 4056
[3]
9
Notes for Question 2
1 1 1 1
seen or implied. (Also allow ( (1 + x)(1 − x) − 1 ) 2 ).
− −
(a) B1: (1 + x) (1 − x)
2 2
or (1 + x)(1 − x) 2

1
M1: Expands (1 + x) 2 to give any 2 out of 3 terms simplified or un-simplified,
1 ⎛ 1 ⎞ ( 1 )(− 12 ) 2 ( 1 )(− 12 ) 2
Eg: 1+ x or + ⎜ ⎟ x + 2 x or 1 + .... + 2 x
2 ⎝2⎠ 2! 2!
1

or expands (1 − x) 2
to give any 2 out of 3 terms simplified or un-simplified,
⎛ 1⎞ ⎛ 1⎞ (− 1 )(− 32 ) ( − 1 )( − 23 )
Eg: 1 + ⎜ − ⎟(− x) or + ⎜ − ⎟ (− x) + 2 (− x) 2 or 1 + .... + 2 (− x) 2
⎝ 2 ⎠ ⎝ 2 ⎠ 2! 2!

(− 12 )(− 23 ) 2
Also allow: 1 + .... + ( x) for M1.
2!
A1: At least one binomial expansion correct (either un-simplified or simplified). (ignore x 3 and x 4 terms)
A1: Two binomial expansions are correct (either un-simplified or simplified). (ignore x 3 and x 4 terms)
Note: Candidates can give decimal equivalents when expanding out their binomial expansions.
M1: Multiplies out to give 1, exactly two terms in x and exactly three terms in x 2 .
A1: Candidate achieves the result on the exam paper. Make sure that their working is sound.
⎛ 1 1 ⎞ ⎛ 1 3 ⎞
Special Case: Award SC FINAL M1A1 for a correct ⎜ 1 + x − x 2 + ... ⎟ × ⎜ 1 + x + x 2 + ... ⎟
⎝ 2 8 ⎠ ⎝ 2 8 ⎠
3 2 1 2 1 2 1 3 2 1 1 2
multiplied out with no errors to give either 1 + x + x + x − x or 1 + x + x + x + x or
8 4 8 2 8 2 8
1 1 2 1 1 2 1 5 2 1 1 2 1
1+ x + x + x + x or 1 + x + x + x − x leading to the correct answer of 1 + x + x 2 .
2 4 2 4 2 8 2 8 2
PhysicsAndMathsTutor.com

Notes for Question 2 Continued


2. (a) ctd 1
1 1 −
1
1 3
Note: If a candidate writes down either (1 + x) 2 = 1 + x − x 2 + ... or (1 − x) 2 = 1 + x + x 2 + ...
2 8 2 8
with no working then you can award 1st M1, 1st A1.
Note: If a candidate writes down both correct binomial expansions with no working, then you can award
1st M1, 1st A1, 2nd A1.
1 ⎛1 + x⎞ 1 2
(b) M1: Substitutes x = into both sides of ⎜ ⎟ and 1 + x + x
26 ⎝1 − x⎠ 2
27 1405
B1: For sight of (or better) and or equivalent fraction
25 1352
3 3 1405 1405 3 3 53 5 ⎛ 1405 ⎞
Eg: and or 0.6 3 and or and 1 or 3 and ⎜ ⎟
5 1352 1352 5 1352 3 ⎝ 1352 ⎠
are fine for B1.
7025 14050 182650
A1: or any equivalent fraction, eg: or etc.
4056 8112 105456
Special Case: Award SC: M1B1A0 for 3 ≈ 1.732001972.. or truncated 1.732001 or awrt 1.732002.
7025
Note that = 1.732001972... and 3 = 1.732050808...
4056

Aliter ⎧⎪ ⎛ 1 + x⎞ (1 + x)(1 − x) (1 − x 2 ) ⎫⎪ 1
1
2. (a) ⎨ ⎜ ⎟= = = ⎬ = (1 − x 2 2
) (1 − x) − 1 (1 − x 2 ) 2 (1 − x) − 1 B1
Way 2 ⎪⎩ ⎝ 1 − x⎠ (1 + x)(1 − x) (1 − x) 2 ⎪⎭
⎛ ⎛1⎞ ⎞ ⎛ (−1)(− 2) ⎞
= ⎜ 1 + ⎜ ⎟ ( − x 2 ) + ... ⎟ × ⎜1 + ( −1) (− x) + (− x) 2 + ... ⎟ See notes M1A1A1
⎝ ⎝2⎠ ⎠ ⎝ 2! ⎠
⎛ ⎞
= ⎜ 1 − x 2 + ... ⎟ × (1 + x + x 2 + ... )
1
⎝ 2 ⎠
1
= 1 + x + x2 − x2 See notes M1
2
1 2 Answer is given in the
= 1+ x + x A1 *
2 question.
[6]
Aliter 1

2. (a) B1: (1 − x ) (1 − x) − 1 seen or implied.


2 2

Way 2 1
⎛1⎞
M1: Expands (1 − x 2 ) 2 to give both terms simplified or un-simplified, 1 + ⎜ ⎟ ( − x 2 )
⎝2⎠
−1
or expands (1 − x) to give any 2 out of 3 terms simplified or un-simplified,
(−1)(− 2) (−1)(− 2)
Eg: 1 + ( −1) (− x) or ... + ( −1) (− x) + (− x) 2 or 1 + .... + (− x) 2
2! 2!
A1: At least one binomial expansion correct (either un-simplified or simplified). (ignore x 3 and x 4 terms)
A1: Two binomial expansions are correct (either un-simplified or simplified). (ignore x 3 and x 4 terms)
M1: Multiplies out to give 1, exactly one term in x and exactly two terms in x 2 .
A1: Candidate achieves the result on the exam paper. Make sure that their working is sound.
PhysicsAndMathsTutor.com

Notes for Question 2 Continued


Aliter ⎧⎪ ⎛ 1 + x ⎞ (1 + x)(1 + x) ⎫⎪ −
1
1
2. (a) ⎨ ⎜ ⎟ = = ⎬ = (1 + x )(1 − x 2
) 2
(1 + x )(1 − x 2
)

2 B1
Way 3 ⎪⎩ ⎝ 1 − x ⎠ (1 − x)(1 + x) ⎪⎭
⎛ 1 ⎞ Must follow
= (1 + x) ⎜ 1 + x 2 + ... ⎟ M1A1A1
⎝ 2 ⎠ on from above.
1
= 1 + x + x2 dM1A1
2
Note: The final M1 mark is dependent on the previous method mark for Way 3.
Aliter
Assuming the result on the Question Paper. (You need to be convinced that a candidate is
2. (a)
applying this method before you apply the Mark Scheme for Way 4).
Way 4
⎧⎪ ⎛ 1 + x⎞ (1 + x) 1 2 ⎫⎪ 1
⎛ 1 2⎞ 1

⎨ ⎜ ⎟ = = 1+ x + ⎬ ⇒ + = ⎜ + + ⎟ −
2 2
x (1 x ) 1 x x (1 x ) B1
⎪⎩ ⎝ 1 − x⎠ (1 − x) 2 ⎪⎭ ⎝ 2 ⎠
⎛ 1 ⎞ ( 1 )( − 12 ) 2 ⎧ ⎫
1
1 1 2
(1 + x) 2 = 1 + ⎜ ⎟ x + 2 x + ... ⎨= 1 + x − x + ...⎬ ,
⎝2⎠ 2! ⎩ 2 8 ⎭
M1A1A1
1
⎛1⎞ ( 1 )(− 12 ) ⎧ 1 1 ⎫
(1 − x) 2 = 1 + ⎜ ⎟ (− x) + 2 (− x) 2 + ... ⎨= 1 − x − x 2 + ...⎬
⎝2⎠ 2! ⎩ 2 8 ⎭
⎛ 1 ⎞ ⎛ 1 ⎞⎛ ⎞
1
1 1
RHS = ⎜1 + x + x 2 ⎟ (1 − x) 2 = ⎜1 + x + x 2 ⎟⎜ 1 − x − x 2 + ... ⎟
⎝ 2 ⎠ ⎝ 2 ⎠⎝ 2 8 ⎠
1 1 2 1 2 1 2
=1− x − x + x − x + x See notes M1
2 8 2 2
1 1 2
=1+ x − x
2 8
1 1
So, LHS = 1 + x − x 2 = RHS A1 *
2 8
[6]
⎛ 1 ⎞
1 1
B1: (1 + x) = ⎜1 + x + x 2 ⎟ (1 − x) seen or implied.
2 2

⎝ 2 ⎠
M1: For Way 4, this M1 mark is dependent on the first B1 mark.
1
Expands (1 + x) 2 to give any 2 out of 3 terms simplified or un-simplified,
1 ⎛ 1 ⎞ ( 1 )(− 12 ) 2 ( 1 )(− 12 ) 2
Eg: 1+ x or + ⎜ ⎟ x + 2 x or 1 + .... + 2 x
2 ⎝2⎠ 2! 2!
1
or expands (1 − x) 2 to give any 2 out of 3 terms simplified or un-simplified,
⎛1⎞ ⎛1⎞ ( 1 )(− 12 ) ( 1 )(− 12 )
Eg: 1 + ⎜ ⎟(− x) or + ⎜ ⎟ (− x) + 2 (− x) 2 or 1 + .... + 2 (− x) 2
⎝ ⎠
2 ⎝ ⎠
2 2! 2!
A1: At least one binomial expansion correct (either un-simplified or simplified). (ignore x 3 and x 4 terms)
A1: Two binomial expansions are correct (either un-simplified or simplified). (ignore x 3 and x 4 terms)
M1: For Way 4, this M1 mark is dependent on the first B1 mark.
Multiplies out RHS to give 1, exactly two terms in x and exactly three terms in x 2 .
A1: Candidate achieves the result on the exam paper. Candidate needs to have correctly processed both
⎛ 1 ⎞
1 1
the LHS and RHS of (1 + x) 2 = ⎜1 + x + x 2 ⎟ (1 − x) 2 .
⎝ 2 ⎠
PhysicsAndMathsTutor.com

Question
Scheme Marks
Number
3. (a) 1.154701 B1 cao
[1]
1 π
(b) Area ≈ × ;×⎡⎣1 + 2 (1.035276 + their 1.154701) + 1.414214 ⎤⎦ B1; M1
2 6
π
= × 6.794168 = 1.778709023... = 1.7787 (4 dp) 1.7787 or awrt 1.7787 A1
12
[3]


2
⎛ ⎛ x ⎞⎞
π For π ⎜ sec ⎜ ⎟ ⎟ .


2
2 ⎛ ⎛ x ⎞⎞ ⎝ ⎝ 2 ⎠⎠ B1
(c) V =π ⎜ sec ⎜ ⎟ ⎟ dx
0 ⎝ ⎝ 2 ⎠⎠ Ignore limits and dx .
Can be implied.
⎛ x⎞
π ± λ tan ⎜ ⎟ M1
⎡ ⎛ x ⎞⎤ 2 ⎝2⎠
= {π } ⎢ 2 tan ⎜ ⎟ ⎥
⎣ ⎝ 2 ⎠⎦ 0 ⎛ x⎞ A1
2 tan ⎜ ⎟ or equivalent
⎝2⎠
= 2π 2π A1 cao cso
[4]
8
Notes for Question 3
(a) B1: 1.154701 correct answer only. Look for this on the table or in the candidate’s working.
1 π π
(b) B1: Outside brackets × or or awrt 0.262
2 6 12
M1: For structure of trapezium rule [ ............. ]
A1: anything that rounds to 1.7787
Note: It can be possible to award : (a) B0 (b) B1M1A1 (awrt 1.7787)
Note: Working must be seen to demonstrate the use of the trapezium rule. Note: actual area is 1.762747174…
π π
Note: Award B1M1A1 for (1.035276 + their 1.154701) = 1.778709023...
(1 + 1.414214) +
12 6
Bracketing mistake: Unless the final answer implies that the calculation has been done correctly,
1 π
Award B1M0A0 for × + 1 + 2 (1.035276 + their 1.154701) + 1.414214 (nb: answer of 7.05596...).
2 6
1 π
Award B1M0A0 for × (1 + 1.414214) + 2 (1.035276 + their 1.154701) (nb: answer of 5.01199...).
2 6
Alternative method for part (b): Adding individual trapezia
π ⎡ 1 +1.035276 1.035276 +1.154701 1.154701+ 1.414214 ⎤
Area ≈ × ⎢ + + ⎥ = 1.778709023...
6 ⎣ 2 2 2 ⎦
π
B1: and a divisor of 2 on all terms inside brackets.
6
M1: First and last ordinates once and two of the middle ordinates twice inside brackets ignoring the 2.
A1: anything that rounds to 1.7787
PhysicsAndMathsTutor.com

Notes for Question 3 Continued

∫ ∫ ∫(
2
⎛ ⎛ x ⎞⎞ ⎛ x⎞ 1
B1: For a correct statement of π ⎜ sec ⎜ ⎟ ⎟ or π sec 2 ⎜ ⎟ or π {dx} .
⎝ ⎝ 2 ⎠⎠ ⎝2⎠ cos ( 2x ) )
2
3. (c)
Ignore limits and dx . Can be implied.


⎛ x2 ⎞
Note: Unless a correct expression stated π sec ⎜ ⎟ would be B0.
⎝ 4 ⎠
⎛ x⎞
M1: ± λ tan ⎜ ⎟ from any working.
⎝2⎠
⎛ x⎞ 1 ⎛ x⎞
A1: 2 tan ⎜ ⎟ or 1 tan ⎜ ⎟ from any working.
⎝2⎠ (2) ⎝ 2⎠
A1: 2π from a correct solution only.

Note: The π in the volume formula is only required for the B1 mark and the final A1 mark.
Note: Decimal answer of 6.283... without correct exact answer is A0.


Note: The B1 mark can be implied by later working – as long as it is clear that the candidate has applied π y 2
in their working.


Note: Writing the correct formula of V = π y 2 {dx} , but incorrectly applying it is B0.
PhysicsAndMathsTutor.com

Question
Scheme Marks
Number
π π
4. x = 2sin t , y = 1 − cos 2t {= 2sin t} ,
2

2
-t-
2
dx dy
At least one of or correct. B1
dx dy dy dt dt
(a) = 2cos t , = 2sin 2t or = 4sin t cos t
dt dt dt dx dy B1
Both and are correct.
dt dt
dy 2sin 2t ⎧ 4cos t sin t ⎫ dy dx
So, = ⎨= = 2sin t ⎬ Applies their divided by their
dx 2cos t ⎩ 2cos t ⎭ dt dt
M1;
⎛ 2π ⎞ π dy
2sin ⎜ and substitutes t = into their .
π dy ⎟ 6 dx
At t = , = ⎝ 6 ⎠ ; =1
6 dx ⎛π ⎞ dy
2cos ⎜ ⎟ Correct value for of 1 A1 cao cso
⎝6⎠ dx
[4]
(b) y = 1 − cos 2t = 1 − (1 − 2sin 2 t ) M1
= 2sin 2 t
⎛ x⎞
2
x2 ⎛ ⎛ x⎞ ⎞
2
x2
So, y = 2 ⎜ ⎟ or y = or y = 2 − 2 ⎜1 − ⎜ ⎟ ⎟ y= or equivalent. A1 cso isw
⎝2⎠ 2 ⎜ ⎝ 2 ⎠ ⎟⎠ 2

Either k = 2 or − 2 - x - 2 B1
[3]
(c) Range: 0 - f ( x) - 2 or 0 - y - 2 or 0 - f - 2 See notes B1 B1
[2]
9
Notes for Question 4
(a)
dx dy
B1: At least one of or correct. Note: that this mark can be implied from their working.
dt dt
dx dy
B1: Both and are correct. Note: that this mark can be implied from their working.
dt dt
dy dx π dy
M1: Applies their divided by their and attempts to substitute t = into their expression for .
dt dt 6 dx
This mark may be implied by their final answer.
dy sin 2t 1
Ie. = followed by an answer of would be M1 (implied).
dx 2cos t 2
A1: For an answer of 1 by correct solution only.
dy
Note: Don’t just look at the answer! A number of candidates are finding = 1 from incorrect methods.
dx
dx dy dy dy dx
Note: Applying divided by their is M0, even if they state = ÷ .
dt dt dx dt dt
dx dy dy − 2sin 2t
Special Case: Award SC: B0B0M1A1 for = − 2cos t , = − 2sin 2t leading to =
dt dt dx − 2cos t
π dy
which after substitution of t = , yields =1
6 dx
Note: It is possible for you to mark part(a), part (b) and part (c) together. Ignore labelling!
PhysicsAndMathsTutor.com

Notes for Question 4 Continued


4. (b) M1: Uses the correct double angle formula cos 2t = 1 − 2sin 2 t or cos 2t = 2cos 2 t − 1 or
cos 2t = cos 2 t − sin 2 t in an attempt to get y in terms of sin 2 t or get y in terms of cos 2 t
or get y in terms of sin 2 t and cos 2 t . Writing down y = 2sin 2 t is fine for M1.
x2
A1: Achieves y = or un-simplified equivalents in the form y = f(x). For example:
2
2x2 ⎛ x⎞
2
⎛ ⎛ x⎞ ⎞
2
4 − x2 x2
y= or y = 2 ⎜ ⎟ or y = 2 − 2 ⎜ 1 − ⎜ ⎟ ⎟ or y = 1 − +
4 ⎝2⎠ ⎜ ⎝ 2 ⎠ ⎠⎟ 4 4

and you can ignore subsequent working if a candidate states a correct version of the Cartesian equation.
IMPORTANT: Please check working as this result can be fluked from an incorrect method.
Award A0 if there is a + c added to their answer.
B1: Either k = 2 or a candidate writes down − 2 - x - 2 . Note: − 2 - k - 2 unless k stated as 2 is B0.
(c) Note: The values of 0 and/or 2 need to be evaluated in this part
B1: Achieves an inclusive upper or lower limit, using acceptable notation. Eg: f ( x ) . 0 or f ( x) - 2
B1: 0 - f ( x) - 2 or 0 - y - 2 or 0 - f - 2
Special Case: SC: B1B0 for either 0 < f ( x) < 2 or 0 < f < 2 or 0 < y < 2 or (0, 2)
Special Case: SC: B1B0 for 0 - x - 2 .
IMPORTANT: Note that: Therefore candidates can use either y or f in place of f ( x)
Examples: 0 - x - 2 is SC: B1B0 0 < x < 2 is B0B0
x . 0 is B0B0 x - 2 is B0B0
f ( x) > 0 is B0B0 f ( x) < 2 is B0B0
x > 0 is B0B0 x < 2 is B0B0
0 . f ( x) . 2 is B0B0 0 < f ( x) - 2 is B1B0
0 - f ( x) < 2 is B1B0. f ( x) . 0 is B1B0
f ( x) - 2 is B1B0 f ( x) . 0 and f ( x) - 2 is B1B1. Must state AND {or} ∩
2 - f ( x) - 2 is B0B0 f ( x) . 0 or f ( x) - 2 is B1B0.
f ( x) - 2 is B1B0 f ( x) . 2 is B0B0
1 - f ( x) - 2 is B1B0 1 < f ( x) < 2 is B0B0
0 - f ( x) - 4 is B1B0 0 < f ( x) < 4 is B0B0
0 - Range - 2 is B1B0 Range is in between 0 and 2 is B1B0
0 < Range < 2 is B0B0. Range . 0 is B1B0
Range - 2 is B1B0 Range . 0 and Range - 2 is B1B0.
[0, 2] is B1B1 (0, 2) is SC B1B0

Aliter dx dy
= 2cos t , = 2sin 2t , So B1, B1.
4. (a) dt dt
Way 2 π dx ⎛π ⎞ dy ⎛ 2π ⎞
At t = , = 2cos ⎜ ⎟ = 3 , = 2sin ⎜ ⎟= 3
6 dt ⎝6⎠ dt ⎝ 6 ⎠
dy
Hence =1 So implied M1, A1.
dx
PhysicsAndMathsTutor.com

Notes for Question 4 Continued

Aliter Correct differentiation of their Cartesian equation. B1ft


1 2 dy
4. (a) y= x ⇒ =x dy
Way 3 2 dx Finds = x , using the correct Cartesian equation only. B1
dx
π
Finds the value of “x” when t =
πdy ⎛π ⎞ 6
At t = , = 2sin ⎜ ⎟ M1
6 dx ⎝6⎠ and substitutes this into their
dy
dx
dy
= 1 Correct value for of 1 A1
dx
Aliter
y = 1 − cos 2t = 1 − (2cos 2 t − 1) M1
4. (b)
Way 2 2− y 2− y
y = 2 − 2cos 2 t ⇒ cos 2 t = ⇒ 1 − sin 2 t =
2 2
2− y
2
⎛x⎞ (Must be in the form y = f ( x) ).
1−⎜ ⎟ =
⎝2⎠ 2
⎛ ⎛ x⎞ ⎞
2

y = 2 − 2 ⎜1 − ⎜ ⎟ ⎟ A1
⎜ ⎝ 2 ⎠ ⎟⎠

Aliter ⎛ x⎞
x = 2sin t ⇒ t = sin −1 ⎜ ⎟
4. (b) ⎝2⎠
Way 3 Rearranges to make t the subject
M1
⎛ ⎛ x ⎞⎞ and substitutes the result into y.
So, y = 1 − cos ⎜ 2sin −1 ⎜ ⎟ ⎟
⎝ ⎝ 2 ⎠⎠ ⎛ ⎛ x ⎞⎞
y = 1 − cos ⎜ 2sin −1 ⎜ ⎟ ⎟ A1 oe
⎝ ⎝ 2 ⎠⎠
Aliter 1
y = 1 − cos 2t ⇒ cos 2t = 1 − y ⇒ t = cos −1 (1 − y )
4. (b) 2
Way 4 ⎛1 ⎞ Rearranges to make t the subject
So, x = ± 2sin ⎜ cos −1 (1 − y ) ⎟ M1
⎝2 ⎠ and substitutes the result into y.
⎛ ⎛ x ⎞⎞ ⎛ ⎛ x ⎞⎞
So, y = 1 − cos ⎜ 2sin −1 ⎜ ⎟ ⎟ y = 1 − cos ⎜ 2sin −1 ⎜ ⎟ ⎟ A1 oe
⎝ ⎝ 2 ⎠⎠ ⎝ ⎝ 2 ⎠⎠
Aliter dy 1 dy 1
= 2sin t = x ⇒ y = x 2 + c = x ⇒ y = x2 + c M1
4. (b) dx 2 dx 2
Way 5 Eg: when eg: t = 0 (nb : − π2 - t - π2 ), 1
Full method of finding y = x 2
1 2 2 A1
x = 0, y = 1 − 1 = 0 ⇒ c=0 ⇒ y= x using a value of t: − 2 - t - π2
π
2

dy 1
Note: = 2sin t = x ⇒ y = x 2 , with no attempt to find c is M1A0.
dx 2
PhysicsAndMathsTutor.com

Question
Scheme Marks
Number
du 1 − 12
5. (a) {x = u 2 ⇒ } dx
du
= 2u or = x
dx 2
or
du
=
1
dx 2 x
B1

⎧⎪ ⎫⎪
∫ ∫ u (2u − 1) 2u du
1 1
⎨ dx ⎬ = M1
⎩⎪ x(2 x − 1) ⎪⎭
2

∫ u (2u − 1) du
2
= A1 * cso

[3]
(b) 2 A B
≡ + ⇒ 2 ≡ A(2u − 1) + Bu
u (2u − 1) u (2u − 1)
See notes M1 A1
u = 0 ⇒ 2 = − A ⇒ A= − 2
u = 12 ⇒ 2 = 12 B ⇒ B = 4
M N
+ , M ≠ 0 , N ≠ 0 to
∫ ∫
2 −2 4 Integrates
(2u − 1)
So du = + du u M1
u (2u − 1) u (2u − 1)
obtain any one of ± λ ln u or ± µ ln(2u − 1)
At least one term correctly followed through A1 ft
= − 2ln u + 2ln(2u − 1)
−2ln u + 2ln(2u − 1) . A1 cao
So, [ −2ln u + 2ln(2u − 1) ] 1
3

Applies limits of 3 and 1 in u or 9


= ( −2ln 3 + 2ln(2(3) − 1) ) − ( −2ln1+ 2ln(2(1) − 1) ) and 1 in x in their integrated function M1
and subtracts the correct way round.
= − 2ln 3 + 2ln 5 − (0)
⎛5⎞ ⎛5⎞
= 2ln ⎜ ⎟ 2ln ⎜ ⎟ A1 cso cao
⎝3⎠ ⎝ 3⎠
[7]
10
Notes for Question 5
dx du 1 − 12 du 1 dx
(a) B1: = 2u or dx = 2u du or = x or = or du =
du dx 2 dx 2 x 2 x
M1: A full substitution producing an integral in u only (including the du ) (Integral sign not necessary).
The candidate needs to deal with the “ x ”, the “ (2 x − 1) ” and the “ dx ” and converts from an
integral term in x to an integral in u. (Remember the integral sign is not necessary for M1).
A1*: leading to the result printed on the question paper (including the du ). (Integral sign is needed).
(b)
2 A B 1 P Q
M1: Writing ≡ + or writing ≡ + and a complete method for
u (2u − 1) u (2u − 1) u (2u − 1) u (2u − 1)
finding the value of at least one of their A or their B (or their P or their Q).
A1: Both their A = − 2 and their B = 4 . (Or their P = −1 and their Q = 2 with the multiplying factor of
2 in front of the integral sign).
M N
M1: Integrates + , M ≠ 0 , N ≠ 0 (i.e. a two term partial fraction) to obtain any one of
u (2u − 1)
± λ ln u or ± µ ln(2u − 1) or ± µ ln ( u − 1
2 )
A1ft: At least one term correctly followed through from their A or from their B (or their P and their Q).
A1: −2ln u + 2ln(2u − 1)
Notes for Question 5 Continued
5. (b) ctd M1: Applies limits of 3 and 1 in u or 9 and 1 in x in their (i.e. any) changed function and subtracts the
PhysicsAndMathsTutor.com
correct way round.
Note: If a candidate just writes ( −2ln 3 + 2ln(2(3) − 1) ) oe , this is ok for M1.
⎛5⎞
A1: 2ln ⎜ ⎟ correct answer only. (Note: a = 5, b = 3 ).
⎝3⎠
Important note: Award M0A0M1A1A0 for a candidate who writes

∫ ∫
2 2 2
du = + du = 2ln u + ln(2u − 1)
u (2u − 1) u (2u − 1)
2
AS EVIDENCE OF WRITING AS PARTIAL FRACTIONS IS GIVEN.
u (2u − 1)
Important note: Award M0A0M0A0A0 for a candidate who writes down either

∫ ∫
2 2
du = 2ln u + 2ln(2u − 1) or du = 2ln u + ln(2u − 1)
u (2u − 1) u (2u − 1)
2
WITHOUT ANY EVIDENCE OF WRITING as partial fractions.
u (2u − 1)
Important note: Award M1A1M1A1A1 for a candidate who writes down


2
du = − 2ln u + 2ln(2u − 1)
u (2u − 1)
2
WITHOUT ANY EVIDENCE OF WRITING as partial fractions.
u (2u − 1)

∫ u (2u − 1) du we can allow a maximum of


1
Note: In part (b) if they lose the “2” and find

M1A0 M1A1ftA0 M1A0.


PhysicsAndMathsTutor.com

Question
Scheme Marks
Number

6. = λ (120 − θ ) , θ - 100
dt

∫ ∫ ∫ λ (120 − θ ) dθ = ∫ dt
1 1
(a) dθ = λ dt or B1
120 − θ
1
− ln (120 − θ ) ; = λ t + c − ln (120 − θ ) ; = t + c
M1 A1;
or See notes
λ M1 A1
{t = 0 , θ = 20 ⇒} − ln (120 − 20 ) = λ (0) + c See notes M1
c = − ln100 ⇒ − ln (120 − θ ) = λ t − ln100
then either... or...
− λt = ln (120 − θ ) − ln100 λt = ln100 − ln (120 − θ )
⎛ 120 − θ ⎞ ⎛ 100 ⎞
− λ t = ln ⎜ ⎟ λ t = ln ⎜ ⎟
⎝ 100 ⎠ ⎝ 120 − θ ⎠
120 − θ eλt =
100
e− λt = dddM1
100 120 − θ
(120 − θ ) eλt = 100
100e − λt = 120 − θ
⇒ 120 − θ = 100e − λt
A1 *
leading to θ = 120 − 100e− λt
[8]
(b) {λ = 0.01 , θ = 100 ⇒} 100 = 120 − 100e − 0.01t
M1
⎛ 120 − 100 ⎞ Uses correct order of operations by
⇒ 100e− 0.01t = 120 − 100 ⇒ − 0.01t = ln ⎜ ⎟
⎝ 100 ⎠ moving from 100 = 120 − 100e − 0.01t
1 ⎛ 120 − 100 ⎞ to give t = ... and t = A ln B ,
t= ln ⎜ ⎟ dM1
− 0.01 ⎝ 100 ⎠ where B > 0
⎧ 1 ⎛1⎞ ⎫
⎨t = ln ⎜ ⎟ = 100ln 5⎬
⎩ − 0.01 ⎝ 5 ⎠ ⎭
t = 160.94379... = 161 (s) (nearest second) awrt 161 A1
[3]
11
PhysicsAndMathsTutor.com

Notes for Question 6


(a) B1: Separates variables as shown. dθ and dt should be in the correct positions, though this mark can be
implied by later working. Ignore the integral signs.
Either or

∫ ∫
1 1
M1: dθ → ± A ln (120 − θ ) dθ → ± A ln (120 − θ ) , A is a constant.
120 − θ λ (120 − θ )

∫ ∫λ
1 1 1 1
A1: dθ → − ln (120 − θ ) dθ → − ln (120 − θ ) or − ln (120λ − λθ ) ,
120 − θ (120 − θ ) λ λ
M1: ∫ λ dt → λt ∫1 dt → t
A1: ∫ λ dt → λt + c or ∫1 dt → t + c The + c can appear on either side of the equation.
IMPORTANT: + c can be on either side of their equation for the 2nd A1 mark.
M1: Substitutes t = 0 AND θ = 20 in an integrated or changed equation containing c (or A or ln A ).
Note that this mark can be implied by the correct value of c. { Note that − ln100 = − 4.60517... }.

dddM1: Uses their value of c which must be a ln term, and uses fully correct method to eliminate their
logarithms. Note: This mark is dependent on all three previous method marks being awarded.
A1*: This is a given answer. All previous marks must have been scored and there must not be any errors in
the candidate’s working. Do not accept huge leaps in working at the end. So a minimum of either:
120 − θ
(1): e − λt = ⇒ 100e − λt = 120 − θ ⇒ θ = 120 − 100e − λt
100
100
or (2): eλt = ⇒ (120 − θ ) eλt = 100 ⇒ 120 − θ = 100e − λt ⇒ θ = 120 − 100e − λt
120 − θ
is required for A1.


1 1
Note: dθ → − ln (120λ − λθ ) is ok for the first M1A1 in part (a).
(120λ − λθ ) λ

(b) M1: Substitutes λ = 0.01 and θ = 100 into the printed equation or one of their earlier equations connecting
θ and t. This mark can be implied by subsequent working.
dM1: Candidate uses correct order of operations by moving from 100 = 120 − 100e − 0.01t to t = ...
Note: that the 2nd Method mark is dependent on the 1st Method mark being awarded in part (b).
A1: awrt 161 or “awrt” 2 minutes 41 seconds. (Ignore incorrect units).
Aliter
∫ ∫λ
1
6. (a) dθ = dt B1
Way 2 120 − θ

− ln (120 − θ ) = λ t + c
M1 A1;
See notes
M1 A1
− ln (120 − θ ) = λ t + c
ln (120 − θ ) = − λ t + c
120 − θ = Ae − λt
θ = 120 − Ae− λt
{t = 0 , θ = 20 ⇒} 20 = 120 − Ae0 M1
A = 120 − 20 = 100
So, θ = 120 − 100e − λt dddM1 A1 *
[8]
PhysicsAndMathsTutor.com

Notes for Question 6 Continued


(a) B1M1A1M1A1: Mark as in the original scheme.
M1: Substitutes t = 0 AND θ = 20 in an integrated equation containing their constant of integration which
could be c or A. Note that this mark can be implied by the correct value of c or A.
dddM1: Uses a fully correct method to eliminate their logarithms and writes down an equation containing
their evaluated constant of integration.
Note: This mark is dependent on all three previous method marks being awarded.
Note: ln (120 − θ ) = − λ t + c leading to 120 − θ = e − λt + ec or 120 − θ = e − λt + A , would be dddM0.
A1*: Same as the original scheme.
Note: The jump from ln (120 − θ ) = − λ t + c to 120 − θ = Ae − λt with no incorrect working is condoned
in part (a).

∫ ∫ ∫ ∫
Aliter 1 ⎧ −1 ⎫
6. (a) dθ = λ dt ⎨⇒ dθ = λ dt ⎬ B1
Way 3 120 − θ ⎩ θ − 120 ⎭

− ln θ − 120 = λ t + c Modulus required M1 A1


for 1st A1. M1 A1
{t = 0 , θ = 20 ⇒} − ln 20 − 120 = λ (0) + c Modulus
not required here!
M1
⇒ c = − ln100 ⇒ − ln θ − 120 = λ t − ln100
then either... or...
− λ t = ln θ − 120 − ln100 λt = ln100 − ln θ − 120
θ − 120 100
− λ t = ln λt = ln
100 θ − 120
As θ - 100
⎛ 120 − θ ⎞ ⎛ 100 ⎞
− λ t = ln ⎜ ⎟ λt = ln ⎜ ⎟
⎝ 120 − θ ⎠
⎝ 100 ⎠ Understanding of
modulus is required dddM1
120 − θ eλ t =
100 here!
e− λt =
100 120 − θ

− λt
(120 − θ ) eλt = 100
100e = 120 − θ
⇒ 120 − θ = 100e− λt
A1 *
leading to θ = 120 − 100e− λt
[8]
B1: Mark as in the original scheme.
M1: Mark as in the original scheme ignoring the modulus.


1
A1: dθ → − ln θ − 120 . (The modulus is required here).
120 − θ
M1A1: Mark as in the original scheme.
M1: Substitutes t = 0 AND θ = 20 in an integrated equation containing their constant of integration which
could be c or A. Mark as in the original scheme ignoring the modulus.
dddM1: Mark as in the original scheme AND the candidate must demonstrate that they have converted
ln θ − 120 to ln (120 − θ ) in their working. Note: This mark is dependent on all three previous method
marks being awarded.
A1: Mark as in the original scheme.
PhysicsAndMathsTutor.com

Notes for Question 6 Continued


Aliter
Use of an integrating factor
6. (a)
Way 4 dθ dθ
= λ (120 − θ ) ⇒ + λθ = 120λ
dt dt
IF = eλt B1

dt
( e θ ) = 120λ eλt ,
d λt
M1A1

eλtθ = 120λ eλt + k M1A1


− λt
θ = 120 + Ke M1
{t = 0 , θ = 20 ⇒} − 100 = K
θ = 120 − 100e − λt M1A1
PhysicsAndMathsTutor.com

Question
Scheme Marks
Number
7. x 2 + 4 xy + y 2 + 27 = 0
⎧ dy ⎫ ⎛ dy ⎞ dy
(a) ⎨ = ⎬ 2x + ⎜ 4 y + 4x ⎟ + 2 y = 0 M1 A1 B1
⎩ dx ⎭ ⎜ ⎟
dx ⎠ dx

dy
2 x + 4 y + (4 x + 2 y ) =0 dM1
dx
dy − 2x − 4 y ⎧ − x − 2 y ⎫
= ⎨= ⎬ A1 cso oe
dx 4x + 2 y ⎩ 2x + y ⎭
[5]
(b) 4x + 2 y = 0 M1
1
y = − 2x x=− y A1
2
2
⎛ 1 ⎞ ⎛ 1 ⎞
x 2 + 4 x(−2 x) + (−2 x) 2 + 27 = 0 ⎜ − y ⎟ + 4 ⎜ − y ⎟ y + y + 27 = 0
2
M1*
⎝ 2 ⎠ ⎝ 2 ⎠
3 2
− 3x 2 + 27 = 0 − y + 27 = 0
4
x2 = 9 y 2 = 36 dM1*
x = −3 y=6 A1
1
When x = − 3 , y = − 2(− 3) When y = 6 , x = − (6) ddM1*
2
y=6 x = −3 A1 cso
[7]
12
Notes for Question 7
dy dy ⎛ dy ⎞
(a) M1: Differentiates implicitly to include either 4 x or ± ky . (Ignore ⎜ = ⎟ ).
dx dx ⎝ dx ⎠
⎛ ⎞
A1: ( x ) → ( 2 x ) and
2
⎜ ... + y + 27 = 0 → + 2 y
2 dy
= 0 ⎟.
⎝ d x ⎠
Note: If an extra term appears then award A0.
Note: The " = 0" can be implied by rearrangement of their equation.
dy dy dy dy
i.e.: 2 x + 4 y + 4 x + 2y leading to 4 x + 2y = − 2 x − 4 y will get A1 (implied).
dx dx dx dx
dy ⎛ dy ⎞
B1: 4 y + 4 x or 4 ⎜ y + x ⎟ or equivalent
dx ⎝ dx ⎠
dy dy
dM1: An attempt to factorise out as long as there are at least two terms in .
dx dx
dy dy
ie. ... + (4 x + 2 y ) = ... or ... + 2(2 x + y ) = ...
dx dx
Note: This mark is dependent on the previous method mark being awarded.
− 2x − 4 y + 2x + 4 y − 2( x + 2 y ) − x − 2y
A1: For or equivalent. Eg: or or
4x + 2 y − 4x − 2 y 4x + 2 y 2x + y
cso: If the candidate’s solution is not completely correct, then do not give this mark.
PhysicsAndMathsTutor.com

Notes for Question 7 Continued


dy dx
(b) M1: Sets the denominator of their equal to zero (or the numerator of their equal to zero) oe.
dx dy
1
A1: Rearranges to give either y = − 2 x or x = − y . (correct solution only).
2
The first two marks can be implied from later working, i.e. for a correct substitution of either y = − 2 x
1
into y 2 or for x = − y into 4xy .
2
M1*: Substitutes y = ± λ x or or x = ± µ y or y = ± λ x ± a or x = ± µ y ± b (λ ≠ 0, µ ≠ 0) into
x 2 + 4 xy + y 2 + 27 = 0 to form an equation in one variable.
dM1*: leading to at least either x 2 = A , A > 0 or y 2 = B , B > 0
Note: This mark is dependent on the previous method mark (M1*) being awarded.
A1: For x = − 3 (ignore x = 3) or if y was found first, y = 6 (ignore y = − 6) (correct solution only).
ddM1* Substitutes their value of x into y = ± λ x to give y = value
or substitutes their value of x into x 2 + 4 xy + y 2 + 27 = 0 to give y = value .
Alternatively, substitutes their value of y into x = ± µ y to give x = value
or substitutes their value of y into x 2 + 4 xy + y 2 + 27 = 0 to give x = value
Note: This mark is dependent on the two previous method marks (M1* and dM1*) being awarded.
A1: (− 3, 6) cso.
Note: If a candidate offers two sets of coordinates without either rejecting the incorrect set or accepting the
correct set then award A0. DO NOT APPLY ISW ON THIS OCCASION.
Note: x = − 3 followed later in working by y = 6 is fine for A1.
Note: y = 6 followed later in working by x = − 3 is fine for A1.
Note: x = − 3 , 3 followed later in working by y = 6 is A0, unless candidate indicates that they
are rejecting x = 3

dy dx
Note: Candidates who set the numerator of equal to 0 (or the denominator of their equal to zero) can
dx dy
only achieve a maximum of 3 marks in this part. They can only achieve the 2nd, 3rd and 4th Method marks to
give a maximum marking profile of M0A0M1M1A0M1A0. They will usually find (− 6, 3) { or even
(6, − 3) }.

dy
Note: Candidates who set the numerator or the denominator of equal to ± k (usually k = 1 ) can only
dx
achieve a maximum of 3 marks in this part. They can only achieve the 2nd, 3rd and 4th Method marks to give a
marking profile of M0A0M1M1A0M1A0.

Special Case: It is possible for a candidate who does not achieve full marks in part (a), (but has a correct
dy
denominator for ) to gain all 7 marks in part (b).
dx
dy 2x − 4 y
Eg: An incorrect part (a) answer of = can lead to a correct (− 3, 6) in part (b) and 7 marks.
dx 4x + 2 y
PhysicsAndMathsTutor.com

Question
Scheme Marks
Number
⎛ 13 ⎞ ⎛ 2⎞ ⎛−p⎞
⎜ ⎟ ⎜ ⎟ JJJG ⎜ ⎟
8. l : r = ⎜ 8 ⎟ + λ ⎜ 2 ⎟ , A ( 3, − 2, 6 ) , OP = ⎜ 0 ⎟
⎜1⎟ ⎜ −1 ⎟ ⎜ 2p⎟
⎝ ⎠ ⎝ ⎠ ⎝ ⎠
⎛ 3⎞ ⎛ − p ⎞ ⎛ − p ⎞ ⎛ 3⎞ Finds the difference
JJJG JJJG JJJG JJJG
(a) { } ⎜ ⎟ ⎜
PA = ⎜ − 2 ⎟ − ⎜ 0 ⎟

{ } ⎜ ⎟ ⎜
AP = ⎜ 0 ⎟ − ⎜ − 2 ⎟

between OA and OP . M1
⎜ 6 ⎟ ⎜ 2p⎟ ⎜ 2p⎟ ⎜ 6 ⎟
⎝ ⎠ ⎝ ⎠ ⎝ ⎠ ⎝ ⎠ Ignore labelling.
⎛ 3 + p ⎞ ⎛ − 3 − p ⎞
⎜ ⎟ ⎜ ⎟
= ⎜ −2 ⎟ =⎜ 2 ⎟ Correct difference. A1
⎜6 − 2p⎟ ⎜ 2p − 6⎟
⎝ ⎠ ⎝ ⎠
⎛ 3 + p ⎞ ⎛ 2⎞
⎜ ⎟ ⎜ ⎟
⎜ −2 ⎟ • ⎜ 2⎟ = 6 + 2 p − 4 − 6 + 2 p = 0 See notes. M1
⎜ 6 − 2 p ⎟ ⎜ −1 ⎟
⎝ ⎠ ⎝ ⎠
p =1 A1 cso
[4]
(b) AP = 4 + (−2) + 4
2 2 2
or AP = (− 4) + 2 + (−4)
2 2 2
See notes. M1
So, PA or AP = 36 or 6 cao A1 cao
It follows that, AB = "6" {= PA } or PB = "6 2 " = 2 PA{ } See notes. B1 ft
{Note that AB = "6" = 2(the modulus of the direction vector of l ) }
⎛ 3⎞ ⎛ 2⎞
JJJG ⎜ ⎟ ⎜ ⎟
OB = ⎜ − 2 ⎟ ± 2⎜ 2⎟ or
⎜ 6⎟ ⎜ − 1⎟ Uses a correct method in order
⎝ ⎠ ⎝ ⎠
to find both possible sets of M1
⎛ 13 ⎞ ⎛ 2⎞ ⎛ 13 ⎞ ⎛ 2⎞
JJJG ⎜ ⎟ ⎜ ⎟ JJJG ⎜ ⎟ ⎜ ⎟
coordinates of B.
OB = ⎜ 8 ⎟ − 3 ⎜ 2 ⎟ and OB = ⎜ 8 ⎟ − 7 ⎜ 2 ⎟
⎜1⎟ ⎜ − 1⎟ ⎜1⎟ ⎜ − 1⎟
⎝ ⎠ ⎝ ⎠ ⎝ ⎠ ⎝ ⎠
⎛7 ⎞ ⎛ −1 ⎞
⎜ ⎟ ⎜ ⎟
= ⎜ 2 ⎟ and ⎜ −6 ⎟ Both coordinates are correct. A1 cao
⎜4⎟ ⎜ 8⎟
⎝ ⎠ ⎝ ⎠
[5]
9
Notes for Question 8
JJJG JJJG
8. (a) M1: Finds the difference between OA and OP. Ignore labelling.
If no “subtraction” seen, you can award M1 for 2 out of 3 correct components of the difference.
⎛ 3+ p ⎞ ⎛ −3 − p ⎞
⎜ ⎟ ⎜ ⎟
A1: Accept any of ⎜ − 2 ⎟ or (3 + p) i − 2 j + (6 − 2 p)k or ⎜ 2 ⎟ or (− 3 − p ) i + 2 j + (2 p − 6)k
⎜6 − 2p⎟ ⎜ 2 p − 6⎟
⎝ ⎠ ⎝ ⎠
PhysicsAndMathsTutor.com

Notes for Question 8 Continued


8. (a) ⎛ 2⎞ ⎛ 2⎞
JJJG ⎜ ⎟ JJJG ⎜ ⎟
M1: Applies the formula PA • ⎜ 2 ⎟ or AP • ⎜ 2 ⎟ correctly to give a linear equation in p which is set equal to
⎜ −1 ⎟ ⎜ −1 ⎟
⎝ ⎠ ⎝ ⎠
⎛ 2⎞
⎜ ⎟
zero. Note: The dot product can also be with ± k ⎜ 2 ⎟ . Eg: Some candidates may find
⎜ −1 ⎟
⎝ ⎠
⎛ 13 ⎞ ⎛ 3 ⎞ ⎛ 10 ⎞
⎜ ⎟ ⎜ ⎟ ⎜ ⎟
⎜8⎟ − ⎜ − 2 ⎟ = ⎜ 10 ⎟ , for instance, and use this in their dot product which is fine for M1.
⎜1⎟ ⎜ 6 ⎟ ⎜ −5⎟
⎝ ⎠ ⎝ ⎠ ⎝ ⎠
A1: Finds p = 1 from a correct solution only.
Note: The direction of subtraction is not important in part (a).
(b) M1: Uses their value of p and Pythagoras to obtain a numerical expression for either AP or PA or AP 2 or
PA2 . Eg: PA or AP = 42 + (−2) 2 + 42 or (− 4) 2 + 22 + (− 4) 2 or 42 + 22 + 42
or PA2 or AP 2 = 42 + (−2) 2 + 42 or (− 4) 2 + 22 + (− 4) 2 or 42 + 22 + 42
A1: AP or PA = 36 or 6 cao or AP 2 = 36 cao
B1ft: States or it is clear from their working that AB = "6" {= their evaluated PA } or
{
PB = "6" 2 = 2 (their evaluated PA) }.
Note: So a correct follow length is required here for either AB or PB using their evaluated PA.
Note: This mark may be found on a diagram.
JJJG JJJG JJJG
Note: If a candidate states that AP = AB and then goes on to find AP = 6 then the B1 mark can be implied.
IMPORTANT: This mark may be implied as part of expressions such as:

{ AB =} (10 + 2λ ) 2 + (10 + 2λ ) 2 + (− 5 − λ ) 2 = 6 or { AB = } (10 + 2λ ) + (10 + 2λ ) + (− 5 − λ ) = 36


2 2 2 2

or { PB =} (14 + 2λ ) 2 + (8 + 2λ ) 2 + (−1− λ ) 2 = 6 2 or { PB = } (14 + 2λ ) + (8 + 2λ ) + (−1− λ ) = 72


2 2 2 2

M1: Uses a full method in order to find both possible sets of coordinates of B:
⎛ 3⎞ ⎛ 2⎞ ⎛ 13 ⎞ ⎛ 2⎞ ⎛ 13 ⎞ ⎛ 2⎞
JJJG ⎜ ⎟ ⎜ ⎟ JJJG ⎜ ⎟ ⎜ ⎟ JJJG ⎜ ⎟ ⎜ ⎟
Eg 1: OB = ⎜ − 2 ⎟ ± 2 ⎜ 2 ⎟ Eg 2: OB = ⎜ 8 ⎟ − 3 ⎜ 2 ⎟ and OB = ⎜ 8 ⎟ − 7 ⎜ 2 ⎟
⎜ 6⎟ ⎜ − 1⎟ ⎜1⎟ ⎜ − 1⎟ ⎜1⎟ ⎜ − 1⎟
⎝ ⎠ ⎝ ⎠ ⎝ ⎠ ⎝ ⎠ ⎝ ⎠ ⎝ ⎠
Note: If a candidate achieves at least one of the correct (7, 2, 4) or (−1, − 6, 8) then award SC M1 here.
⎛ 3⎞ ⎛ 2⎞ ⎛ 3⎞ ⎛ 2⎞
JJJG ⎜ ⎟ ⎜ ⎟ JJJG ⎜ ⎟ ⎜ ⎟
Note: OB = ⎜ − 2 ⎟ − 3 ⎜ 2 ⎟ and OB = ⎜ − 2 ⎟ − 7 ⎜ 2 ⎟ is M0.
⎜ 6⎟ ⎜ − 1⎟ ⎜ 6⎟ ⎜ − 1⎟
⎝ ⎠ ⎝ ⎠ ⎝ ⎠ ⎝ ⎠
A1: For both (7, 2, 4) and (−1, − 6, 8). Accept vector notation or i , j , k notation.
Note: All the marks are accessible in part (b) if p = 1 is found from incorrect working in part (a).
⎛ 3⎞ ⎛ 2⎞
JJJG ⎜ ⎟ ⎜ ⎟
Note: Imply M1A1B1 and award M1 for candidates who write: OB = ⎜ − 2 ⎟ ± 2 ⎜ 2 ⎟ , with little or no
⎜ 6⎟ ⎜ − 1⎟
⎝ ⎠ ⎝ ⎠
earlier working.
PhysicsAndMathsTutor.com

Notes for Question 8 Continued


8. Helpful Diagram!

JJJG 2 λ = −3
AB = 9λ 2 + 90λ + 225
⎛13 + 2λ ⎞
⎜ ⎟ l
⎛10 + 2λ ⎞ B ⎜ 8 + 2λ ⎟
JJJG ⎜ ⎟ ⎜ 1− λ ⎟
AB = ⎜ 10 + 2λ ⎟ ⎝ ⎠
⎜ −5 − λ ⎟
⎝ ⎠
λ = −5

⎛ 3⎞
⎜ ⎟
A⎜ − 2 ⎟
⎜ 6⎟
⎝ ⎠
λ = −7

6
⎛13 + 2λ ⎞ ⎛ 4⎞
JJJG ⎜ ⎟
⎜ ⎟ PA = ⎜ − 2 ⎟
B ⎜ 8 + 2λ ⎟
⎜ 1− λ ⎟ ⎜ 4⎟
⎝ ⎠ 6 ⎝ ⎠
45°

6 2
45°
⎛ −1 ⎞ ⎛−p⎞
⎜ ⎟ ⎜ ⎟
⎛ 14 + 2λ ⎞ P⎜ 0⎟ = ⎜ 0 ⎟
JJJG ⎜ ⎟ ⎜ 2⎟ ⎜ 2p⎟
PB = ⎜ 8 + 2λ ⎟ ⎝ ⎠ ⎝ ⎠
⎜ −1 − λ ⎟
⎝ ⎠
JJJG 2
PB = 9λ 2 + 90λ + 261

8. (b) Way 2: Setting AB = "6" or AB 2 = "36" Note: It is possible for you to apply the main scheme for Way 2.
{ AB = "6" ⇒ AB 2
= "36" ⇒ } (10 + 2λ ) + (10 + 2λ ) 2 + (− 5 − λ ) 2 = "36"
2
B1ft could be implied here.
9λ 2 + 90λ + 225 = 36 ⇒ 9λ 2 + 90λ + 189 = 0
λ 2 + 10λ + 21 = 0 ⇒ (λ + 3)(λ + 7) = 0
λ = − 3, − 7
Then apply final M1 A1 as in the original scheme. ... M1 A1

8. (b) Way 3: Setting PB = "6 2 " or PB 2 = "72" Note: It is possible for you to apply the main scheme for Way 3.
{PB = "6" 2 ⇒ PB 2 = "72" ⇒ } (14 + 2λ ) 2 + (8 + 2λ ) 2 + (−1− λ ) 2 = "72" B1ft could be implied here.
9λ 2 + 90λ + 261 = 72 ⇒ 9λ 2 + 90λ + 189 = 0
λ 2 + 10λ + 21 = 0 ⇒ (λ + 3)(λ + 7) = 0
λ = − 3, − 7
Then apply final M1 A1 as in the original scheme. ... M1 A1
PhysicsAndMathsTutor.com

Notes for Question 8 Continued


(You need to be convinced that a candidate is applying this method before you apply the Mark Scheme for
8. (b)
Way 4).
JJJG JJJG
JJJG JJJG ° PA • PB
Way 4: Using the dot product formula between PA and PB , ie: cos 45 = JJJG JJJG .
PA . PB
⎛ 4 ⎞ ⎛ 14 + 2λ ⎞
JJJG JJJG ⎜ ⎟ ⎜ ⎟
PA • PB = ⎜ − 2 ⎟ • ⎜ 8 + 2λ ⎟ = 56 + 8λ − 16 − 4λ − 4 − 4λ = 36
⎜ 4 ⎟ ⎜ −1 − λ ⎟
⎝ ⎠ ⎝ ⎠
JJJG
For finding PA as before. M1
{cos 45 =}
° 1
=
36
36 or 6 A1 cao
2 6 9λ + 90λ + 261
2
JJJG
PB = 9λ 2 + 90λ + 261 B1 oe
1 36
=
2 9λ + 90λ + 261
2

9λ 2 + 90λ + 261 = 72 ⇒ 9λ 2 + 90λ + 189 = 0


λ 2 + 10λ + 21 = 0 ⇒ (λ + 3)(λ + 7) = 0
λ = − 3, − 7
Then apply final M1 A1 as in the original scheme. ... M1 A1

(You need to be convinced that a candidate is applying this method before you apply the Mark Scheme for
8. (b)
Way 5).
JJJG JJJG
JJJG JJJG ° AB • PB
Way 5: Using the dot product formula between AB and PB , ie: cos 45 = JJJG JJJG
AB . PB
Attempts the dot product formula
JJJG JJJG M1
between AB and PB .
⎛10 + 2λ ⎞ ⎛ 14 + 2λ ⎞ JJJG JJJG
⎜ ⎟ ⎜ ⎟ Correct statement with AB and PB
⎜ 10 + 2λ ⎟ • ⎜ 8 + 2λ ⎟ A1
⎜ − 5 − λ ⎟ ⎜ −1 − λ ⎟ simplified as shown.
cos 45° =
1
= ⎝ ⎠ ⎝ ⎠ JJJG
2 9λ + 90λ + 225 9λ + 90λ + 261
2 2 Either AB = 9λ 2 + 90λ + 225 or
JJJG B1
PB = 9λ 2 + 90λ + 261

140 + 20λ + 28λ + 4λ 2 + 80 + 20λ + 16λ + 4λ 2 + 5 + 5λ + λ + λ 2


{cos 45 =}
° 1
=
2 9λ 2 + 90λ + 225 9λ 2 + 90λ + 261
9λ 2 + 90λ + 225
{cos 45 =}
° 1
=
2 9λ 2 + 90λ + 225 9λ 2 + 90λ + 261
1 (9λ 2 + 90λ + 225) 2
=
2 (9λ 2 + 90λ + 225) (9λ 2 + 90λ + 261)
1 (9λ 2 + 90λ + 225)
=
2 (9λ 2 + 90λ + 261)
9λ 2 + 90λ + 261 = 2(9λ 2 + 90λ + 225) ⇒ 9λ 2 + 90λ + 189 = 0
λ 2 + 10λ + 21 = 0 ⇒ (λ + 3)(λ + 7) = 0
λ = − 3, − 7
Then apply final M1 A1 as in the original scheme. ... M1 A1
PhysicsAndMathsTutor.com

Notes for Question 8 Continued


8. (b) Way 6:
⎛ 4⎞ ⎛ 2⎞ ⎛ 2⎞
JJJG ⎜ ⎟ ⎜ ⎟ ⎜ ⎟
PA = ⎜ − 2 ⎟ = 2 ⎜ −1 ⎟ and direction vector of l is d = ⎜ 2 ⎟
⎜ 4⎟ ⎜ 2⎟ ⎜ −1 ⎟
⎝ ⎠ ⎝ ⎠ ⎝ ⎠
JJJG A correct statement relating these
So, PA = 2 d or PA = 2 d M1 A1 B1
distances (and not vectors)

Apply final M1 A1 as in the original scheme. ... M1 A1


JJJG
Note: PA = 2d with no other creditable working is M0A0B0...
⎛ 3⎞ ⎛ 2⎞
JJJG JJJG ⎜ ⎟ ⎜ ⎟
Note: PA = 2d , followed by OB = ⎜ − 2 ⎟ ± 2 ⎜ 2 ⎟ is M1A1B1M1 and the final A1 mark is for both sets of
⎜ 6⎟ ⎜ − 1⎟
⎝ ⎠ ⎝ ⎠
correct coordinates.

You might also like